Sie sind auf Seite 1von 305

ffiemist

Ya
II
Section V
Carbonyls and Alcohols

Organic
Sections V-VilI
Section VI Section VII
Nitrogen Compounds

Section VIil

'

'

Organic Chemistry Laboratory Techniques

Kr E. V. I. E.IilI @ Specializing in, MCAT Preparation

Section V
Carbonyls
and

Oxygen Containing Compounds a) Alcohol Properties b) Alcohol Reactivity c) Alcohol Spectroscopy


,
,

Alcohols
by Todd Bennett

o
OH'
-l-

t-\ .,/ - \

il JI
CH3

il

-c\" CH,I
O

ii. esters iii. Lactones iv. Acid Anhydrides v. Acid flalides vi. Amides Carbon5rl Reactivity
,

d)AldehydeandKetrrnePro,p.ertie.s e) Aldehyde and Ketone Kactivity, f) Aldehyde and Ketone spectroscopy g) Ketals and Acetals i. Protecting Oroups: h) Carboxylic Acids and Derivatives i. Carboxylic Acids
:
,r,

il ll lr (R,r'- \oCHr -\ oo

R-

./ c-..
ll

ll

+I

a) Attack at Carbonyi Carbon b) Deprotonation of cr-Protons c) Oxidation,Reduction ReactionC

CHrI

ll

2 more

times

ll. Name Reactions a) QrignardReaction " I,' b) Aldol Condensation


c) Claisen Condensation
d) Transesterification e) Wittig Reaction ' '
'

.,

R-t\ctr, +
o
II

R-c\cI.
o

R-C\
o

OH
CIr

--L,/\

-Cr o
(-

ll
-ott
O

Cl3

f) Pinacol Rearrangement g lodoform Reaction ,r, $ynthetic r.o'gic


,

h) n/olff-Kishner Reaction 1

-c\

-/-\ OHRO -\ Iodoform Test

, rrl . -,:

oll , __

HCI3

yeliow oil

a) Reactions of Acetoacetic ,Ester b) Reactions of Malonic Ester

c) Decarboxylation d) Protecting Croups

"

' Carbonyl Biocheqistry a) Biological Oxidation-ReduCtion

b) Biochemical Reagents

REKI{ELEY I)R.E V.l.-E-w'


Speci altztng

in MCAT Preparation

Carbonyls & Alcohols Section Goals


oB
Kecoqnize the carbonyl functional qrouDs and frpes of comoounds.
You must be able to recognize functional grouFS sr-i;h as ar,lles, anhvdrides, acid halides, and

especiallr'ketonesandaldehydes. Youmustkn('\\', 1-.:'-.:.:..'.lrdsaremostreactivetowards subsbitution (which is based cin the leaving groun :lrc: i:: .'.- * r-*.ra:t electrophilic (which is based

on the electron withclrawing or -d-ona:i:.: :.-::..:-. ,': the functional group).

a?

Be able to identify infrared peaks for carbont-l compounds. Carbonrl compounds will have a peak in the inirar=i =:=:::- .:: r: ::.e area of 1700+ cm-1. This will m.rsr Lleh be useful when compaiing two carbonr'. . --:': . -_ -: . : --i:rtiJr ing an unknown carbonvl
.r\Itl'--r'LLlld.

\oumaywishtoknowr6ughlr'rr'hercci:=:i - -i:i'.-r::.rJketjnesfallinthelRrang'e. carbon ds.

-:-. Lrcluded in this group - -i:r-:- -; :- - Claisen reactioil. Theshould : Aldol r tg nlu(rr :: :--.*', :-:'.- a role in select biochemical
1

Ee a,ble to the aciditv of al carbons. Ti.:"::t-onalphato,thecarbonylc_an!edeprotrr:.:::-t - - .-:: i::-,:.-,nbondedtoit. ThepK6of a -:::"j:riketoneistherangelT!2. Oncedepr..:::.::..-i::.r:i-,::.:=:ormed. Theenolat'ehisan ;: *].-t::lum of its own with the enol structurE. I:.. - - r :,-: - . -: :, r:iorr into an enol is referred :: :s :f -i:omerization. Ee able to identifv ketals, hemiketals, acetals and hemiacetals.

*?
sw
@p

L'nd

erstand the difference between thermo"dr-na

ic and kinetic enolates.

Iinos the mechanisms for acidic and basic calbonrl reactions. . ' = : - -- - --.:l for transesteriJication is preser.: _-: n*:r ,_- _" --:la:iic chemistry so it important -- i:r-.: :' : .*::=.-.:r- to carry "out the procesS. - -.-'.grize the steps. Also recogniz.
Recoqnire ommon oxidizing and reducilq aqents.

,:
=

---'.ude LiAlHa and NaBFIa.

he loss of bonds to hydrogen. -i -r.- the loss of bonds to oxvsen

up

fi.nolr common reactions by both name and re,ageots. : -' - - . ::.)\\ by name. Ttis a good idea
j
-

- -'- : - -: :s.rcfion conditions. Hishliehts --- '.'. itting . reaction, the io"dof6rm =

:,1-,

Organic Chemistry

Carbonyls and Alcohols

Introduction

The carbon-oxygen bond is a major part of organic and biological chemistry.

significant part of organic chemistry on the MCAT involves compounds that contain carbon-oxygen bonds. In the case of carbonyl compounds, the carbonoxygen n-bond is easily broken to form new bonds to the carbonyl carbon and subsequently form a new compound. The carbon-oxygen o'-bond found in alcohols and sugars can undergo several reactions, but it is generally not as reactive as the carbon-oxygen n-bond. Our goal is to organize the vast multitude of reactions involving carbonyl compounds and alcohols. Figure 5-1 shows several types of carbonyl compounds and carbonyl derivatives with which you should be familiar.
Types of Carbonyl Compounds

o
il
R

o
il
R

,ZC\
Ketone

,zc:. H
Aldehyde

R'O \/

OR'

,zc:. RH
Acetal

R'O OH \/ ,z c:. RH
Hemiacetal

R'O \/ -,.\
Ketal

OR'

R'O \/

OH

,ZC\ RR
Hemiketal

o
il

o
il

Carboxylic Acid

,zc:. ROH
o
il

.zc:. oR'
Ester

oo lr il ,zc:. .zc\ ROR


Acid anhydride

o
il
R

o
il

-zc\ RX
Acid halide

,zc:. NHz
Amide NHPh

./\ RN
I

,/ C\
H
Imide

ll

R'

N
lt

,ZC\ RR
Phenylhydrzine derivative

1--L_/
Lactam

o
I

.+

il

o/\c*r,
Figure 5-1 Copyright
@

.zc\ R

CHz

Enolate Resonance Forms

by The Berkeley Review

Exclusive MCAT Preparation

Organic Chemistry

Carbonyls and

Alcohols
I

Oxygen Containing Compounds


!,:.i -,,i,

ioxy fi |cu tfii

ing Gbffid,uffi$],

'',

r,i

r,

i iLr;'

."'

oxygen containing compounds, because of the highly electronegative nature of oxygen/ are very reactive. Much of organic chemistry revolves around alcohols and carbonyls, so it is imperative to get a fundamental understanding of their properties, reactivity, and spectroscopic evidence that supports their existence. Alcohol Properties Because of their ability to form hydrogen bonds, alcohols typically have high boiling points and are generally miscible in water. Alcohols mike good solverits as they are often liquids at room temperature and they have u lurg" range between their melting and boiling points. Alcohols are hydrophilic, poiar molecttles that become less hydrophilic (more lipophilic) as tireir iarbon chain iength increases. The smaller alcohols (three carbons or less) are highly water soluble, but as the size of the alkyl group increases, their wateisoiubitity decreases. As with all compounds, their physical properties vary with mass and branching, as well as the position of the hydroxyl gtonp. As themolecular mass increases, the boiling point increases, but the effect on the melting point is less clear. As the branchrng increases, the boiling point decreases. Tatle s-1 shows the physical properties of several alcohols, from which the effects of mass, branching, and positioning of the hydroxyl group on the physical properties can
be ascertained.

Isomer

IUPAC Name (Common Name) Methanol


Ethanol 1-Propanol (n-Propanol) 2-Propanol (l-Propanoi) 1-Butanol (n-Butanol) 2-Butanol (sec-Butanol) 2-Methyl- 1 -propanol (r-Butanol) 2-MethyI-2-prop anol
(/

Boiling Point
64.6'C
78.4'C
97.2"C 82.3"C 117,3C 99.6"C

Melting Point
-98"C -115"C

Density

(g/rnL)
0.797

Water Solubility (g/100 mL)

CH3OH H3CCH2OH H3CCH2CH2OH

High High
High

0.789
0.804 0.786 0.810 0.806 0.802

-727'C

(H3C)2CHoH
H3C(CH2)3OH

-90"c -90"c
-115'C -122'C 24'C -79'C

High
8.2
12.8

H3CCH(oH)CH2CH3
(H3C)2CHCH2OH

707.7'C

11.3

(H3C)3CoH
H3C(CH2)aOH (H3CCH2)2CHOH
H3C(CH2)aCH3 C6H11OH

-Butanol)

82.0'C
r37.6"C
119.3"C

0.789
0.814 0.809 0.815 0.814 0.956

High
2.7

1-Pentanol (n-Pentanol)

H3CCH(OH)CH2CH2CH3 2-Pentanol
3-Pentanol

5.0 5.6 0.8


2.7

115.9'C
157.5"C 161.5"C

l-Hexanol (n-Hexanol)
Cyclohexanol n-Octanol

H3C(CH2)6CH2OH

194.7'C

0.817

0.3

Table

5-1

Copyright O by The Berkeley Review

The Berkeley Review

Organic Chemistry
Example 5.1

Carbonyls and Alcohols

Oxygen Containing Compounds

\Alhat type of alcohol is the following molecule?

A. B. C. D.

Primary alcohol
Secondary alcohol

Tertiary alcohol
Phenol

Solution
The compound has the alcohol functional group attached to a carbon that is bonded to two other carbons. This is defined as a secondary alcohol, so the best
answer is choice B.

Alcohol Reactivity Alcohols are nucleophilic reagents in organic chemistry. Th"y are not good nucleophiles in their protonated (neutral) state, but they can be deprotonated and converted into their anion (alkoxide) form under basic conditions. Because alkoxides (the deprotonated form of the alcohol) are strong bases, they are not the ideal nucleophile, but they are generally better than alcohols. Alcohol chemistry also involves oxidation into a carbonyl as we shall see later in this section. Alcohols are commonly formed from the reduction of carbonyls, which we shall also postpone for the moment. The common reactions to form and consume alcohols that do not involve carbonyl compounds center around nucleophilic substitution. Figure 5-2 shows nucleophilic substitution reactions that convert alkyl halides into alcohols. Figure 5-3 shows nucleophilic substitution reactions that convert alcohols into alkyl halides.

Alkyl halides to alcohols


R R

OI{\

',.'F*
H
R'

+X
H
R'

o,.Fu,
H
R"

1. RCO?-

2. OH-(aq)

HOJ.,,,.
H
R''

---+
H"O

\.,,R

+ Br- + RCO2H

ot,,F*
R'

-z

acetone --

ou,r o
R' Figure 5-2

+ HX

- -:-, right

by The Berkeley Review

Exclusive MCAT Preparation

Organic Chemistry

Carbonyls and Alcohols


Alcohols to alkyl halides

Orygen Containing Compounds

o
R

,ruro
H
R'

cll"\cl PBr"

XR

'r*F., H
R'

Retention

ouro
H
R"

---L

Ut
R"

frrO
H

Inversion

ou,|o*,
R'

HBr __-+

RuFu,
R'

Racemization

Figure 5,3
Spectroscopic Evidence for Alcohols Alcohols can be detected using either infrared or NMR spectroscopy. In IR spec_tra, hydroxyl groups present a distinct absorbance between 3200 and 3500 cm-l that is medium in intensity and broad due to hydrogen bonding. In ,lHNvn spectra, hydroxyl gronpr present a signal betweln l ind 5 ppm i'hat is broad due to hydrogen bonding, although the broadness varies with ihe solvent. They have no definite 6-value (it varies with concentration and solvent). The peak slowly disappears with the addition of D2o to the NMR tube. The oH group does not couple-well, so we rarely consider splitting patterns for alcohols. Figure 5-4 shows the lHNMR spectrum for 2-propanol in carbon tetrachloride solvent. Figure 5-5 shows the IR spectrum for 2-propanol obtained neat on salt
plates.

HOH HsC

CHs

Figure 5-4

Copyright

by The Berkeley Review

The Berkeley Review

Organic Chemistry

Carbonyls and

Alcohols

Oxygen Containing Compounds

1392 cm'l

3422 cm-l 2894 cml 2984 cm'l

HOH

l4J) cm ' &

l-165 cm-l

"r.X.*r,
Figure 5-5

Aldehyde and Ketone Properties


Because aldehydes and ketones do not form hydrogen bonds, they typically have boiling points only slightly higher than alkanes of equal mass. Because of the polarity of the carbonyl bond, they are slightly miscible in water. Aldehydes and

aldehydes and ketones, from which the effects of mass, branching, and positioning of the carbonyl group on the physical properties can be ascertained.
[somer

ketones are aprotic, polar molecules that become less hydrophilic as their carbon chain length increases. The smaller aldehydes and ketones (three carbons or less) are generally water soluble but as the size of the alkyl group increases, their water solubility decreases. Table 5-2 shows the physical properties of several

IUPAC Name (CommonName) Methanal (Formaldehyde)


Ethanal (Acetaldehyde)

Boiling Melting Point Point


-27'C
21"C

Water

Solubility (g/100 mL) High Infinite


76.3 6.8
J.J

HCHO
H3CCHO H3CCH2CHO H3C(CH2)2CHo HgC(CHz)gCHO H3C(CH2)aCHO C6H5CHO
H3CCOCH3 H3CCOCH2CH3

-92"C

-127'C -81'C -99'C -92'C -56'C


-26"C

Propanal (Propionaldehyde) Butanal (n-Butyraldehyde)


Pentanal

49'C
76"C 103"C

Hexanal
Benzaldehyde Propanone (Acetone) Butanone (Ethyl methyl ketone) 2-Pentanone 3-Pentanone 2-Hexanone
3-F{exanone

728'C
778"C

2.1

0.3

56'C

-94'C
-86"C -78"C

Infinite
25.6 5.7 4.9

80"c
102"C

H3CCO(CHz)zC}{s
(H3CCH2)2CO

101'C
T28"C

-47'C
-55"C

H3CCo(CHz)eCHe
H3CCH2CO(CH2)2CH3 H3CCOCH2CH(CH3)2
C5H19O

r.6
1..3

I24'C
T79'C
156"C

4-Methyl-2-Pentanone Cyclohexanone Acetophenone

-85"C

r.9
2.2

.C 27'C

C6H5COCH3

202'C

Insoluble

Table 5-2

Copyright

by The Berkeley Review

Exclusive MCAT Preparation

Organic Chemistry
Example 5.2

Carbonyls and Alcohols

Orygen Containing Compounds

What is the IUPAC name for the following compound?


OH
I

./\,/\'--H o
il

C. D.

A. 1-A1do-4-pentanol B. 4-Hydroxypentanal
5-Oxo-2-pentanol

2-Hydroxypentaldehyde

Solution
The longest chain is five carbons and the highest priority functional group is the aldehyde. The functional group with the most oxidized carbon receives the highest priority according to IUpAC convention. For naming aldehydes, the ',e,' is dropped from the alkane chain of the same length and an ial', suffix is added. This makes the compound pentanal, which makei choice B correct. The oH is

:9""'*

a hydrogen bonded to the carbonyl carbon along with either an alkr-l group or in the-case of formaldehyde, a second hydrogen. Ketones consrst of a carbonyl group with two alkyl substituents attached. The cher,-rrstn- occurs primarily at the electrophilic carbonyl center. Aldehydes and ketones are reactive with most nucleophiles, but not by a traditional nucleoph-L: 'u'nstirr-rtion mechanism. once a nucleophile attacks a carbonyl carbon, il io::--. a fcur-ligand intermediate with u ,-r"gutirru charge on oxygen known as a:::.,;.,'..-;.--;.. :'::ermediate. This intermediatu iuitt Uu shown in the mechanism oi :na:r'! :a:r.::., I reactions in this section. The chemistry of aldehydes is similar r- '-r= -i=rr.--.::r-of ketones except that an aldehyde can be oxidized into a ca:'r-.,-.'--: ::-.:-,.,-hil" k"tones cannot be oxidized easily. oxidation in carbcr.-" r ::.e::---::-. :a:, be r.iewed as either the gain of bonds io oxygen or the loss -.: : ::: :- ... j:--:er. \ve shall thoroughly idd."ss carbonyl reactions throughc-: --i -: ::-i .l

Aldehyde and Ketone Reactivity Aldehydes consist of a carbonvl with

Eile

jea
.q..

M.

D.
Smffi

Ailt
iTrr;: -iiinn
u[1iJ]11

*d
.n
$lPi"u

i1

lnixf

Cffi
Berkeley Review

The Berkeley Review

fil,f

Organic Chemistry

Carbonyls and Alcohols

Oxygen Containing Compounds

HsC

cH2cH3

2ppm

Figure 5-5

o rAcH2cH2cH3
1117 cm'1

cm'1
1ZZA

1457

cm-r

1388 cm I

and
1362 cm'1

"^-t

Figure 5-7
Example 5.3 Pentanal can be distinguished from 3-pentanone by all of the following EXCEpT: A. a signal at9 - 1,0 ppm in the IHNMR. B. fir'e signals in the lHNMR rather than two signals. C. an IR absorbance at L826 cm-1. D. an ultraviolet absorbance at 230 nm instead of 240 nm.

Solution -rn aldehyde hydrogen is found between 9 ppm and 10 ppm in the 1HNMR, so ::-oice A is a valid way to distinguish an aldehyde from a ketone. choice A is --minated. Pentanal has five unique hydrogens while 3-pentanone has two --nique hydrogens, so the two compounds can be distinguished by their :espective number of signals in the 1HNMR. Choice B is etiminated. Aldehydes =C ketones have different n-bonds, so they have different carbonyl absorbances -:- 'he ultraviolet absorbance region, This eliminates choice n. Ataenyaes and Aetones have different IR absorbances, but they are observed around 1TO0 cm-l , :.;'t at 1826 .m-1. This makes choice C an invalid technique, which makes choice
C

'ie

best answer.

-rr"'-right

by The Berkeley Review

Exclusive MCAT Preparation

Organic Chemistry

Carbonyls and Atcohols

Orygen Containing Compounds

Ketals, Hemiketals, Acetals, and Hemiacetals Ketals and hemiketals are derivatives of ketones while acetals and hemiacetals are derivatives of aldehydes. Ketals occur when a ketone roses the carbonyl grgup and gains two alkoxy functional groups (R-o). The oxidation state of carbon does not change, because the carb-on ,till hus two bonds to oxygen, but now-it is,two sigma-bonds to two different oxygen atoms rather thanJsigmabond and pi-bond to the same oxygen. A hemiketal occurs when the ketone has its carbonyl group converted into ahydroxyl group and gains one arkoxy group. Acetals are similar to ketals, except it is ihe" ataenyae"tnat loses its carbonyl group to gain the two groups. Hemiacetals are similar to hemiketais, f]<o1y except it again is an aidehyde, rather than a ketone, that converts its carbonyl qtoYp into a hydroxyl group while gaining an arkoxy group. Figure 5,g shows the formation of the four compounds.

RR'

A
Ketone

R"O =-=.
xs R"OH/OH-

RR'

X
Ketal

OH

Hemiketal

RR'

A
Ketone

xs

R"oH/H'

R"O OR" \ / RR'


R

RH

A
o

'o

xs

R'OH/OH
R

X
Acetal

oH
H

Aldehyde

Hemiacetal

ll nAs

R'O OR' R'oH/Ff -\ /-" -EnAn Aldehyde


,*,

Figure 5-g Acetals and ketals are useful as protecting groups in organic synthesis. Acetals and ketals can be formed and removea onty ,t d", aciiic .orrditio.rr, where as hemiacetals and hemiketals are formed oniy under basic conditions but removed under any conditions.

ul

,[

fi

,d

Copyright O by The Berkeley Review

The Berkeley Review

l{!

Organic Chemistry

Carbonyls and

Alcohols

Oxygen Containing Compounds

Figure 5-9 shows a general mechanism for ketal formation under acidic conditions. It is the same mechanism for the formation of an acetal from an
aldehyde, except that an aldehyde is the reactant, rather than a ketone.

H*

+ 2 R'OH

HzO

Or"r.^"r"'lf

/t 'll

tl

deprotonate

"\*.

n'-

oH

break

l
t
'll,-*"
I

\6..
.l

"o7o

R'-

OH

.:+.
n'o+r

:o/

,""41,

11,**H

\. C)' +

R'

deprotonate
-iiA

protonate

Figure 5-9

of the mechanism are labeled to emphasize the predictable nature of acid-catalyzed mechanisms. When you draw a mechanism for an acid catalyzed reaction, the intermediates must carry positive charges and no molecule should er-er carry a negative charge. With the exception of rearrangement steps in
T'he steps

selected cases, acid catalyzed mechanisms follow this same pattern of: 1) p'rotonate (making the leaving group a better leaving group), 2) break (the leaving group leaves), 3) make (the nucleophile attacks the carbocation), and 4) Ceprotonate (returning the molecule to a neutral state). Base catalyzed rrechanisms follow the exact opposite pattern of: 1) deprotonate (to make a s'Tong nucleophile),2) make/break (where the nucleophile attacks and dislodges 5e leaving group), and 3) protonate (returning the molecule to a neutral state).

Copgight

by The Berkeley Review

tt

Exclusive MCAT Preparation

Organic Chemistry

Carbonyls and Alcohols

Orygen Containing Compounds

o
P:,
s1:i

These mechanisms should be kept as simplistic as possible. The mechanism for the formation of a hemiacetal from an aldehyde and alcohol in the presence of strong base is shown in Figure 5-10.

Ac

A
.o.

:-. ..1r1 f
.t\\,, !!

'Base

-tE-::i

a"or.-ranua"

\r
s.E

\
make/break

:.:

R,onH< )
H

/vrotonate

- -:E_

{;

Figure 5-10

when you draw the mechanism for a base catalyzed reaction, be sure that the intermediates carry negative charges ancl no molecule ever carries a positive charge. Ketals and acetais serve as protecting groups for carbonyl groups in synthesis involving ketones and aldehydes. Hemiacetals and hemiketals are not useful as protecting groups, but they are important in sugar chemistry.
Example
5.4

-:
-: ,lL
:',r,,

Addition of ethanol at a pH of 4 to propanal yields which organic product?

A.

tY_"'' C.'oyou' "oyo' "'oy-o"


B.
D.

Er

;.r
_4

lr:rt

EtHEIOHEt-HEiH
Solution Addition of an alcohol to an aldehyde under acidic conditions (pH = 4 is acidic) yields an acetal. The only acetal in the choices is choice A. Choice B has too many ethoxy groups, choice c is a hemiacetal, and choice D is a geminal diol.
Example
5.5

5o
lrl

Addition of sodium methoxide in methanol to acetone vields: A. isopropyl alcohol.

rr

B. C. D.

acetaldehyde.
a ketal.
a

ilnt

hemiketal.

!l

Solution Addition of an alcohol to a ketone under basic conditions yields a hemiketal. The methoxide anion attacks the carbonyi carbon of acetone to generate the tetrahedral anion intermediate, which then deprotonates the methanol to form the hemiketal and regenerate the methoxide anion. The best answer is choice D.

flx l; ur
dflni

H$

ilhn

Copyright O by The Berkeley Review

t2

The Berkeley Review

Organic Chemistry

Carbonyls and Alcohols

Oxygen Containing Compounds

groups for aldehydes and ketones at this time. Both of these carbonyl compounds employ the same reaction to add the protecting group, forming either an acetal or ketal. As we have seen, aldehydes and ketones, in the presence of alcohols and acid, form acetals (from aldehydes) and ketals (from ketones). Because acetals and ketals are less reactive than aldehydes and ketones, they are an ideal protecting group. The alcohol that is typically used to form the protected carbonyl compound is a vicinal diol (a l,2-diol), such as ethylene glycol (HOCH2CH2OH). Figure 5-11 shows the protecting of cvclohexanone using ethylene glycol.

Acetals and Ketals as Protecting Groups Protecting groups are used in synthesis to prevent a reagent from reacting at a site where it is undesirable to have a reaction. We shall only discuss protecting

Cvclohexanone

5+U
o
Figure 5-11

Hzo

Protected as a Ketal

The mechanism for forming the protected ketone is the same generic acidcatalyzed mechanism shown in Figure 5-9, only instead of using two molecules of alcohol, a vicinal diol is used, so the second "mske" step involves the second hvdroxyl group of the vicinal diol rather than a new alcohol.
Example 5.6

The addition of ethylene glycol (HOCH2CH2OH) in the presence of acid orovides a protecting group for ketones. \Alhich of the following is the protected
form of 2-pentanone?

.{./

u'l-f

C.

o'n
oo

xr^-- \><,,.
Solution

oo

The original ketone (2-pentanone) has two bonds from carbon 2 to oxygen, so the :roduct must also have two bonds from carbon 2 to oxygen. This eliminates all

:f

the answer choices except choice

B.

The product of a diol and a ketone in

:i:r,hr-drous, acidic conditions is a cyclic ketal.

There is not much to using an acetal or ketal protecting group. The last thing to :rnsider is when to use a protecting group in synthesis. As a guideline, any time --j-.at vou have a molecule with more than one reactive site, you must protect the sr:es at which you wish to have no reaction. The exception to this rule is when --:e site vou wish to react at is significantly more reactive than any other sites on
:::e mo1ecule.

lc'pr-right

by The Berkeley Review

t3

Exclusive MCAT Preparation

Organic Chemistry

Carbonyls and

Alcohols

Orygen Containing Compounds

Carboxylic Acids and Their Derivatives Carboxylic acid derivatives are different from aldehydes and ketones in that the carbonyl carbon has a functional group that possibly can act as a leaving group. Much of the chemistry of carboxylic acids and acid derivatives centers around changing the group on the carbonyl carbon. We shall address each functional group starting with the carboxylic acid and look at their chemical reactions. Carboxylic Acids Carboxylic acids are weak acids with a pKu between 2 and 5. They are readily converted into esters, anhydrides, or acid halides. They carry out similar orgurri. reactions as esters, but are less reactive than esters. Carboxylic acids cin be formed by saponification (treating an ester with strong base in water), by treating

alcohols and aldehydes in water, or by hydroryzing a nitriie o, un uniide ,rirrg strong acid at high temperatures. These reactionr ut" shown in Figure s-12.

a methyl ketone (RCocH3) with 12 and strong bise, by oxidizing primaiy

saponification:

R,Aoo,-Ester
O

3 r

xs H.O/Olr _______::*

OAO
Carboxylate

HOR,

Ester

hydrolysis:

xs

HrOlFf

oAoo
Ester

A ROH
Carboxylic acid

HoR,

r :

Iodoform reaction: O

Methyl ketone
Oxidation:

o4.", o

12/oH

A RO_

+ I,CH
Iodoform

-/\ RH
Oxidation:

ll

xrranonZorr :-

Carboxylate o

Aldehyde

ROH
Krc,gr/.L

A
o
lt

s
,tru

Carboxylic acid

",S{

HH
OH Primary alcohol

-/\ R-

\/

--

frfi

-AROH
Carboxylic acid

Amide hydrolysis: O
R

xs

H2OlFf

o
il il

NHR'

ROH
Carboxylic acid

R'NH3}

ffi ffi dl
cn

Secondarv amide

Nitrile hydrolysis:

It- c: N -

.t H'zolFI.
A

m ilf'

pd

il

Lm

ROH
Carboxylic acid

NHn

tf

rtilJlli

Figure 5-12

Copyright

by The Berkeley Review

The Berkeley Review

Organic Chemistry

Carbonyls and Alcohols

Oxygen Containing Compounds

Carboxylic acids can be reduced into primary alcohols or converted into other compounds such as acid halides, acid anhydrides, or esters. Figure 5-13 shows four reactions of carboxylic acids with which you are expected to be familiar.

HH

oA.'
Acid halide

ROH

Primary alcohol

A
Ester

"f
OR'

ROR
Figure 5-13

AA
Acid anhydride
D.

oo

Example 5.7 Treatment of benzoic acid with ethanol and acid yields which of the following
compounds?

A'oB'

c.

d*oa"*o,oao'
oEt
Solution Addition of an alcohol to a carboxylic acid under acidic conditions with enough heat to overcome the activation barrier yields a new ester by way of a transesterification reaction. The final product is an ester with an ethoxy group in place of the hydroxyl group of the carboxylic acid, along with a water molecuie side product. Choice B is the best answer.
Esters

Esters are carbonyl compounds

with an alkyl group and an alkoxy group

attached to the carbonyl carbon. Esters have a leaving group (the alkoxy group),

so they undergo more reactions than ketones or aldehydes. Their reactivity correlates with the pKu of the conjugate acid of the leaving group. Leaving groups that are more stable (are less basic and their conjugate acids have a lower pKu value) are better leaving groups. Esters can easily exchange their alkoxy group in the presence of acid and an alcohoi in what is referred to as a t r ans e s t er ifi ca t i on rcaction.

Copyright

by The Berkeley Review

l5

Exclusive MCAT Preparation

Organic Chemistry

Carbonyls and Alcohols

Oxygen Containing Compounds

j
I

.d

Example 5.8 What is the major organic product for the following reaction?

H3CH2C
A.
B.

A+
o

H3coH/Ff

OCH2CH3

1+
D.

.nr

:l

.te

:
o

C.

;r

,,Ao.ru
Solution

EtO

A OCH3 ;X::"' :X;

new ester with the methoxy group attached instead of the ethoxy grorp. Ethanol

At high temperature under acidic conditions, an alcohol can undergo a transesterification reaction when mixed with an ester. The final product ii the

jt"

ut""t"

rta"

In Example 5.8, a C-o bond and an o-H bond were both broken in the reactants and- formed in the products. This implies that the enthalpy (AH) for the reaction is close to 0. Going from an ester and non-cyciic alcohoi to an ester and noncyclic alcohol generates a change in entropy (AS) of roughly 0, because the reaction starts and finishes with roughly the same degreei of freedom. This ilPlies that the change in free energy for transesterification is around 0 (AG = AH - TAS). This means that the equilibrium constant for a transesterification reaction is_ approximately 1. The reaction can be driven to products by focusing on Le Chatelier's Principle. It will proceed in high yield if ihe producis are removed or a reactant is constantly added. This also has biological significance in that transesterification can be used for shuttles in the cell membrane. For the formation of a lactone (cyclic ester), there is a ring formed and an alcohol molecule produced from a linear system. Depending on the reaction, there can be either a gain or a loss in entropy, so lactonelormati,on is less predictable.
Lactones

-{t

&

C ri

5!
iilMi;

ffi
.u'f
rfud

in Figure 5-1. Lactones undergo the same chemistry as esters, only entropy is a factor in the reaction,s flvorability. Illgtones can be synthesized by treating cyclic ketones with peroxyacijs (RCO3H) in what is known as the Baeyer-Villiger reaction. Lactone chemistry is easier to understand when you recognize that the compound is an ester. Figure 5-14 shows the formation of a lactone from an intramolecular transesterificulion
reaction of a hydroxyester.

T,actones are cyclic esters, as shown

ury

:-

Jle Au

OHA

H* __+

dru

ROH

dfrM

,ffi
mtu

mfl

Figure 5-14

W
LiS[M

Copyright

by The Berkeley Review

The Berkeley Review

l{lLr!][

Organic Chemistry

Carbonyls and Alcohols

Orygen Containing Compounds

Acid Anhydrides Acid anhydrides are formed by a condensation reaction (dehydration) of two carboxylic acids at elevated temperatures (as shown in Figure 5-13). Therefore, when you add water to an acid anhydride, it breaks into two carboxylic acids. It is easiest to predict the product if you focus on the inorganic side product (H2O in this case) more than the organic product. If you circle and remove an H from one reactant and an OH from the second reactant and then connect the two leftover fragments, then you can derive the organic product rather effortlessly. This is shown in Figure 5-15 as a short-cut method for deducing the product of dehydration of two carboxylic acids.

A
Find the water

^*A

o
,,.11....

ROR'

ROR'

AA
o o

Connect the atoms

Viva la anhydride

Figure 5-15
Example 5.9 What is the major product from the following reaction?

OH

HsC

--OH

A. B. C. D.

Diketone Acid anhydride


Ester

Aldehyde

Solution

Addition of heat to carboxylic acids yields an acid anhydride by driving off


water. The final products are an acid anhydride and a water molecule. Choice B is the best answer There are three possible acid anhydrides that can form. One of the three possible acid anhydrides that may form is drawn below. The other two acid anhydrides that can form result from the condensation of each acid upon itself. Probability says that the product below is the most likely.

HrrCo

ooo
Atoms are

oo
connected Anhydrides are good

.;.,4o,'L.";,AoA.*,,

Water is found

Acid Halides Acid halides are similar to esters, but with a halide (Cl, Br and I) in place of the alkoxy group. They are the most reactive of all the carbonyl compounds, because the halide is a great leaving group. They undergo the same substitution reactions as other carbonyl compounds that have a leaving group, but they react faster. For some reactions, acid halides can be too reactive. They are a useful intermediate product in many synthetic pathways, such as the conversion of a carboxylic acid into an amide for instance, where the carboxylic acid is first converted to the more reactive acid halide which then goes on to form the amide.
Copyright
@

by The Berkeley Review

t7

Dxclusive MCAT Preparation

Organic Chemistry

Carbonyls and Alcohols

Orygen Containing Compounds

Or

Example 5.10 what are the two products formed when propanoyr chroride, H3CCH2CoCI, is treated with methyl amine, H3CNH2?

Cr
ll"p
-- "1>uL

Ca:

A. B. C. D.

N-methyl propanamide and hydrochloric acid 1-amino-2-butanone and hydrochloric acid Propanal and chloromethylamine Carbon dioxide and N,N-ethylmethylamine

:eai raei
!dl

Solution A' amine is a good nucleophile capable of attacking the carbonyl carbon and displacing the chloride anion to form an amide, choicl A. Amides are biological structures that you are required to know according to the MCAT student Manuril. This reaction can be used as a precursor to synthJsizing an amine. Amides can be reduced to an amine using LiAlH4 in ether followediy neutralization with a weak acid. There are problems with direct synthesis of primary amines using ammonia and an alkyl halide due to multiple additions. The point here is thai amides can be products themselves, or intermediates in amine synthesis. Amides Amides are carbonyls with-an amine group bonded to the carbonyl carbon. Amides form the backbone of proteins, ut-ra tn"y are found in most of the bases of nucleic acids (i.e., DNA and RNA). An amide bond that links amino acids together is referred to as a peptide bond. Amides can be reduced into amines using a strong reducing agent such as lithium aluminum hydride, LiAlHa.
Example 5.11
\A/hich of the following compounds does

IAi

thri
kr.

-{rh

A,>L '2+-q

E',:E
a-L'l

-,f

::i

Jt

Nor

A. B. C. D.

contarn an amide bond?

Guanine

Uracil
Isoleucine
j-*

Cytochrome

-t-ld i:: E:-

Solution Most amino acids do not contain an amide bond, although polymers of amino acid (polypeptide chains) is in i: bonds proteins that amiio icids form peptide (amide) bonds. The peptide J, of proteins are broken down in acidic aqueous environments to regenerate the individual amino acids, Guanine and uracil are bases in RNA, and they contain amide bonds (as drawn below). It is the amide functional group that forms the hydrogen bonding in base pairing. Cytochrome is an enzyme (protein), so it contains amide troids in its peptide linkages. The best answer is choice C, isoleucine, an amino acid.

fiT -*Ao
H
Uracil

o /\

o .H
H

(pyrimidine base)

Guanine

(purine base)

Copyright

by The Berkeley Review

la

The Berkeley Review

Organic Chemistry
rcffiH$ffiiiiiiffiH ftiffiffi

Carbonyls and Alcohols

Carbonyl Reactivity

Carbonyl reactions can be categorized in one of three ways. The fist type of reaction involves a nucleophile attacking the electrophilic carbon. The second type of reaction involves the deprotonation of an alpha proton and the subsequent nucleophilicity of the anion that is generated. The last type of reaction falls into the realm of oxidation-reduction chemistry, although most mechanisms for carbonyl redox reactions involve a nucleophile attacking the carbonyl carbon. If you keep things simple in terms of the three types of reactions, you should be able to summarize all of carbonyl chemistry and work through any questions they may present. Attack at Carbonyl Carbon
Because carbonyl compounds contain a C=O bond, they are good electrophiles.

We shall consider ketones and aldehydes first, but other carbonyl compounds also act as electrophiles. The difference in reactivity between a ketone and a carboxylic acid derivative, such as an ester, centers around the presence of a leaving group on the carbonyl carbon. The C=O bond is polar with a partial positive charge on the carbon atom and a partial negative charge on the oxygen atom. It is the partial positive charge on the carbon that makes a carbonyl such a wonderful electrophile. Figure 5-16 shows a generic carbonyl reaction, where a carbonyl compound is attacked by a nucleophile to form a tetrahedral intermediste.

o
il

---------+

Nuc:-

Nuc \/

O-

Carbonyl

Tetrahedral Intermediate

Figure 5-16
seen aldehydes and ketones serving as electrophiles in the formation of acetals, hemiacetals, ketals and hemiketals. As far as electrophilic chemistry of carbonyls is concerned, there is no major variation between reactions. In other words, it is best to view carbonyl reactions as all basically the same with slight variations of the nucleophile. The tetrahedral intermediate in Figure 5-16 represents every generic intermediate in carbonyl addition reactions. For carbonyl compounds that have leaving groups, the reactivity of the carbonyl

\{e have already

compound is based on the strength of the leaving group. Stronger leaving groups make for a more reactive (more electrophilic) carbonyl compound. The strength of a leaving group can be inferred from the pKu of its conjugate acid. Leaving groups are considered to be good when they form a stable compound upon leaving. So, as the leaving group gets stronger, it gets more stable, which makes it less basic, and thus makes its conjugate acid stronger. This means that good ieaving groups generally have conjugate acids with low pKu values. Figure 5-17 shows a carbonyl reactivity chart (relative reactivity of substituted carbonyls) for an acid halide, an acid anhydride, an ester, and an amide. The conjugate acid of each leaving group is shown in the same diagram.

Copyright

by The Berkeley Review

t9

Exclusive MCAT Preparation

organic chemistry

carbonyrs and Alcohors

Carbonyl Reactivity

The better X is at leaving from H, the better it will be at leaving from C, so the best leaving group has a conjugate acid with the lowest pKu.

very good PKu = -1.0 to -7 very strong acid leaving group

fr -_,ezt-oii o-tr- oa. R.,,t\x)


o o

I t (

ii

^d

fl

f,

to \_,
o

PKu=3to5
average acid strength

good leaving group

o H-oRi

R
PKu= 14to 17 weak acid

-[o
seml-poor

*r_4rro,
pKu = 33 to

il

OR'

R
very weak

--CC NHR'

n n

iT

leaving group

35 acid

very poor leaving group

Relative Reactivity:

o
lt

-zC:. x

-ll
)r

oo
il Figure 5-17

o
il

.il
OR'

o
NHR'

&
d

o-'- o/'\R

o/c\

-zL\ R-

t el h

Figure 5-18.

This technique is a good predictor of the reactivity of all carboxylic acid derivatives. The generic reaction and its tetrahedral intermediate are shown in

The relative reactivity implies that an acid halide can easily be converted into an anhydride, ester, or amide. An acid anhydride can easily be converted into an ester or amide, but it is difficult to convert the anhydride into an acid halide.

o\

R-zY:.L.G. T L:Nuc

il)

op L.G. ( \ //
R7'i

o
il

Nuc

t =:: /'c \oNuc


R

L.C.

Figure 5-18 Carbonyl substitution reactions proceed via a tetrahedral intermediate as shown in Figure 5-18. If the leaving group is not a good one, then the reaction cannot go further than the tetrahedral intermediate and will ultimately shift back to tf,e carbonyl reactant. This results in an equilibrium between the original carbonyl and the tetrahedral intermediate. This is observed with ketoner u"na uta"nya"t when they are present in a solvent that has nucleophilic capability.

Copyright O by The Berkeley Review

20

The Berkeley Keview

Organic Chemistry
Example 5.12

Carbonyls and Alcohols

Carbonyl Reactivity

\Mhich of the following is the MOST electrophilic carbonyl compound?

A. B. C. D.

Amide
Ester

Aldehyde Acid anhydride

Solution
The reactivity of a carbonyl is dictated by the leaving group on the carbon of the carbonyl. As the acidity of the conjugate acid of the ieaving group increases, so does the reactivity of that particular carbonyl. In the choices above, the conjugate acids of the leaving groups are an amine, an alcohol, H2, and a carboxylic acid respectively. The most acidic is the carboxylic acid, so the anhydride is the most

reactive carbonyl.

Deprotonation of cr-Protons
The hydrogen on the alpha carbon (the carbon adjacent to the carbonyl carbon) is acidic (its pKu is approximately 19), so it can be removed using a strong base. Enolates are formed when a hydrogen on the alpha carbon is deprotonated. The enolate can regain a proton at either the carbon or the oxygen. If it is protonated at the oxygen, an enol is formed. There is an equilibrium between the ketone and =nol. The conversion from a ketone into an enol is known as tautomerizotion, iecause a ketone and its enol are tautomers, structural isomers that vary in the :osition of a n-bond and a hydrogen. The tautornerization of acetone is shown in

F:zure 5-19. Ketone Carbanion

Enolate
,

Enol

rr

-,_f) CH: uru ./t\

.u. il ^

.u.\
._ Q.z-

ll)

o;o,-r.sf|r"

.e
CHe

tl

:ot

urc,zc-

-cHr._=

,c2c\ CHs

ilA:

uuru

Figure 5-19
-- :

- carbanion that forms is a good nucleophile. When an alkyl halide is added can attack the alkyl halide in a nucleophilic ; -:siitution reaction to form a new carbon-carbon bond. This results in a longer

:: :.!e solution, the carbanion


.
1

)
e
.1

=::i-,e. The halide can be any halide, but the reaction works best with an alkyl ,'--re compound. Alkyl bromides and chlorides yield more O-alkylation side :: :.i:cts than alkyl iodides. The generic reaction is shown in Figure 5-20.

Ketone

Carbanion

Longer Ketone

.u.

,/ -\ lJ:F\ CH: l) HA:base

ilil

.u.

:o:
il

Hzg. -\ \

CH:

H"C

'l

.zC\

CH" J

o4
2l

Figure 5-20
w

::sht

O by The Berkeley Review

Exclusive MCAT Preparation

Organic Chemistry

Carbonyls and Alcohols

Carbonyl Reactivity

l
,ll

temperature and base size. At low temperature with a bulky base, the less hindered (less substituted) alpha carbon gels deprotonated to form the so called kinetic enolate (lower energy transition staie). This is referred to as kinetic control. At a higher temperature with a smallbase, the more hindered (more substituted) a-lpha carbon gets deprotonated to form the so called theimodynnmic enolate (leading to the more stable product). This is referred to as thermodynamic control We shall apply this concept to aldol condensation reactions of asvmmetric ketones later on in this section.
Example 5.13 iodoethane, and subsequently followed by workup?

Whgn the carbonyl is asymmetric, the possibility of two different enolates arises (each formed by deprotonating a different arpha hydrogen). If one of the two alpha carbons is more substituted, then it is both It"ri.ully hindered and more stable. This creates a situation where the-or" reaction can be dictated by

what is the final product after acetone is treated first with NaH, followed by

A. 2-Methyl-2-butanol B. 3-Methyl-2-butanone C. 2-Pentanone D. 2-Butanone


Solution
This reaction is simiiar to the generic reaction in Figure 5-20. The carbon chain le1sfh is_ increased by two carbons (from three to five) when the electrophile is ethyl iodide. This eliminates choice D. The product is a ketone, so choice A is eliminated. The final product is 2-pentanone, as shown below, which makes

choice C the best answer.

!o!
-- ^./ HzGf

HAtbase

---CH.. l)1'r -

ilil ^ t\

ioi
ozL\ H,C- CH,

. (_).

-->(-

ll

,/-\ H,C 'l


H3CH2C

CH:

B,A

ru!
lllm

"\+
HsC

2-Pentanone

{l$r.l

.Frul

rtu.
d@,ri

crosslink, they undergo dehydrogenation (1oss of hydrogen), an oxidative process. when the n-bond in a fatty acid is hydrogenated to form an aliphatic chain, it has undergone a reductive process.
Copyright
@

aldehydes, which can be further oxidized into caiboxylic acids. secondary alcohols are oxidized into ketones. Tertiary alcohols cinnot be oxidized (the alcohol carbon has no hydrogen to lose). Reduction is defined as the opposit! of oxidation, so the reverse of each reaction just mentioned represents reduction. To make the processes more clear, we shall define oxidation and reduction in terms of bonds to oxygen and bonds to hydroge'. More than just oxygen containing compounds do redox chemistry. when two cysteine residues form a

oxidation and reduction are recurring in organic chemistry, so working from a logic-based foundation is key. If the oxophilic carbon (carbtn containin[ a bond t9 gxygen) has hydrogens, it can be oxidized. Primary alcohols are oxidized into

frnm
SMrm

(fill!$l

ffm

Jlimm

$Mf,

by The Berkeley

The Berkeley Review

Organic Chemistry

Carbonyls and Alcohols

Carbonyl Reactivity

Oxidation is defined as an increase in oxidation state, which is caused by either losing a bond to a less electronegative atom (in most cases hydrogen) or gaining a

bond to a more electronegative atom (in most cases oxygen). Reduction is


defined as a decrease in oxidation state, which is caused by either gaining a bond io a less electronegative atom (in most cases hydrogen) or losing a bond to a nore electronegative atom (in most cases oxygen). Reduction Oxidation Gain of bonds to O Loss of bonds to O
Loss of bonds to H Increase in oxidation state Gain of bonds to H
Decrease in oxidation state

)etermining oxidation states using the method learned in general chemistry is a of assigning oxygen a -2 (because it is more electronegative than the atoms =tatter :c rvhich it bonded, and it makes two bonds) and a +1 to hydrogen (because it is ,ess electronegative than the atoms to which it bonded, and it makes one bond). ll:re oxidation state of any remaining atoms is found by difference. In organic ::.emistry, oxidation states for specific atoms can easily be found by considering =iectron sharing in each bond. If the bond is between two atoms of unequal
=-ectronegativity, then the more electronegative atom is assigned a -1 and the less =-ectronegative atom is assigned a +1. The oxidation state of an atom is found rv summing the numbers from all of the bonds and any formal charge it may har-e. Figure 5-21 shows this method as it applies to the oxidation state of carbon I n 2-propanol and acetone.

l+

lol

>

-o

CHs

CHs

lost a bond to H gained a bond to O

Figure 5-21

::-:. oi the four bonds to carbon is analyzed for its relative electronegativity : ::'.pared to the atoms to which it is bonded. Bonds to hydrogen give a negative
:

,-"'jfl, because carbon is less electronegative than oxygen. Both carbons in a :,:::r-carbon bond get zero, because there is no difference in electronegativity. -: r .econdary alcohol, the oxophilic carbon has an oxidation state of 0 while in a ,-:r--r., the oxophilic carbon has an oxidation state of +2. This means that the :,:::::r rvas oxidized by two electrons, which is predictable, because it has lost a : -: ' to hvdrogen (oxidizing it by one electron) and has gained a bond to oxygen , '':-zrng it by another electron). Figure 5-22 shows that oxygen and hydrogen

- :arbon and a positive to hydrogen, because carbon is more electronegative - ::. hvdrogen. Bonds to oxygen give a positive to carbon and a negative to

: :.-t change oxidation


H

state when going from 2-pentanol to acetone.

A-,
Hrclo ocfu cH,
Figure 5-22

\11

CHs

r:.

::n
as

has a -2 oxidation state, as is expected. Hydrogen has a +1 oxidation expected. Oxidation states should be made this simple.

23

Exclusive MCAT Preparation

Organic Chemistry

Carbonyls and Alcohots

Carbonyl Reactivity

Oxidizing and Reducing Agents

reaction. The coupled half-reactions are shown in Figure 5-23.

(known as PCC) or by NAD+. in the reduction half-reaction, NAD+ is redlced to NADH, because for every oxidation half-reaction, there is a reduction half-

oxidizing agents are rich in oxygen (given that they deliver oxygen to the reactant) and reducing agents are rich in hydrogen (given that itrey deliver hydrogen to the reactant). This perspective is appfcable in both organic chemistry and biochemistry. It should be noted that NAD+ acts like Cro3-with pyridine, NADH acts like NaBH4, and FADH2 acts like H2/pd. Ethanol can be oxidized into ethanal using either Cro3 in the presence of HCI and pyridine

NAD*
Figure 5-23

NADH

oxidizing agents cause oxidation (and in doing so, they get reduced). oxidizing agents are rich in oxygen, poor in hydrogen, and have an atom in a high oxidation state with high electron affinity. Reducing agents cause reduction (aid in doing so, they get oxidized). Reducing agents are poor in oxygen, rich in hydrogen, and have an atom in a low oxidation state with low ionizai=ion energy. Some common oxidizing and reducing agents are listed below. oxidizing Agents (Rich in o; Poor in H) Reducing Agents (poor in o; Rich in H)

-;_E
:

.l

:rl

l':

;ler'ff"tS

KMnO4 CrO3 03 RCO3H ROOR Clz NAD+ NADP+ FAD

LiAlHa NaBH4 H2NNH2 Hz/Pd HCI/Zn HOCH2CH2SH

lmruils

i,;.m

NADH

NADPH

FADH2

acid and 2' alcohol -+ ketone -- 3" alcohols undergo no oxidation reactionsj.

Typical oxidation and reduction reactions involve the conversion between alcohols and carbonyl compounds. Figure s-24 is a summary of common oxidation-reduction reactions in organic chemistry (1' alcohol + aldehyde -+
Primary alcohols have two hydrogens to lose, so they are oxidized twice (first into an aldehyde and then into a carboxylic acid.) The carboxylic acid is reduced back into a primary alcohol using LiAlHn.

loo ntou
Lr

o,*'ot'5,lt"nt<-

o ll

,_,

oxidizingagent-

o ll o'
arboxytic acid

r.

ullof,ot
reducing agent

Secondary alcohols have one hydrogen to lose, so they are oxidized once.

n-f ou
R'

oxidizing agent
reducing agent Figure 5-24

oll

o
o'

2" aicohol

ketone

Copyright

by The Berkeley Review

24

The Berkeley Review

. "lmtn:

Organic Chemistry

Carbonyls and Alcohols

Carbonyl Reactivity

Oxidation From an organic chemistry perspective, oxidation is the process of gaining oxygen (and/or losing hydrogen) at the carbon which has an electronegative atom attached. The first rule is to count the number of carbon-oxygen bonds to the carbon of interest in the reactant. As the number of carbon-oxygen bonds increases, the oxidation level (and oxidation state) of the compound increases. In terms of oxidation levels, every carbon-oxygen bond counts as one. If you lose a carbon-hydrogen bond and replace it with a carbon-oxygen bond, the oxidation level has increased by one while the oxidation state of the carbon has increased by two. The oxidation of a primary alcohol into an aldehyde and then into a
carboxylic acid is shown in Figure 5-25.

HOH
(-

H.C

]li ,/^-\

o
CrO"/HCl H
Pyridine -+

o *t
H
KeCrrOz HrSOa(at)

*r ll

(-

*r ll *r

,/^-\ HsC

,/atr\ HsC

OH

oiidation

0+1-1-1=-1

State'

Oxidation State:

0+1+1-1=+1
Figure 5-25

0+1+1+1=+3

Oxidation State:

You should be able to recognize reagents, the reaction type, and correctly predict the product for redox reactions. As a point of interest, Cro4 with Hct in pvridine is known as pyridinium chlorochromate, or PCC for short. Most organic compounds lose carbon-hydrogen bonds and gain carbon-oxygen bonds rlhen oxidized, so the reagent (oxidizing agent) must be rich in oxygen. The two :nost common oxidizing agents in organic chemistry are Cro42- and Mno4-. Some common oxidation reactions of alcohols are shown in Figure 5-26.

o\
ttJ "\rr'"H Primary alcohol

''PCC''

Ott

CrOeCl-

A ----9+

\-.r*

-t

a\
RH
Aidehyde

il

Pyridinium
K"CrnOo
H2SOa(aq)

o\ oH ,t1 -\rurcH
Primary alcohol

o
(-

ll

,/-\ ROH
Carboxylic acid

t\

o
KMnOa
KOH(aq)
tl
(

oYt-ot
R

Secondary alcohol

---+
tot
Figure 5-26

,/-\ RR
Ketone

Tertiary alcohol

No Reaction (No Hs to lose)

lrpvright

by The Berkeley Review

25

Exclusive MCAT Preparation

Organic Chemistry

Carbonyls and Alcohols

Carbonyl Reactivity

( :

These reactions should be known well enough that you can recognize them in a

group of many reactions. For other oxidation reactions, you should simply

s''

recognize the oxidizing agent and know that the product is more oxidized than the reactant. Figure 5-27 shows some oxidation reactions of compounds other than alcohols. The common theme is that one of the reactants is rich in oxygen.

iilli

,4.

RH HR

\:J /\

& c.

o. -----+
(H"C)2S

\ F /

03 has excessive oxygen; alkene reactant is being


oxidized

Disubstituted alkene

H Aldehyde
R

5l
t;r

t,r

.['
............--------'H3COH/H2O HIO4

HIO* has

Jd excessive

oxygen; diol reactant


is being oxidized

Al&

H R Vicinal diol

H Aldehvde

l'"r

{L
Ifr"

o
c6HscoqH
-------lC6H5CO3H has excessive

c
m

oxygen; ketone reactant


is being oxidized

Sut

Ketone

Lactone (Ester)

Figure 5-27
The three oxidation reactions shown in Figure 5-27 are ozonoiysis of an alkene, oxidative cleavage of a vicinal diol using periodic acid, and the Baeyer-Viiliger oxidation of a ketone respectively. These reactions are rare enough that they don't need to be memortzed,but you should recognize that each is an oxidation. More important than memorizing organic chemistry reactions is the ability to predict results and explain observations based on the data given about these reactions in the passages associated with them. For instance, excessive oxygen in the reactant should provide a ciue that the organic molecule is being oxidized. Example 5.14 What is the major organic product for the following reaction?

fri
EE

tu
.{'-

lrildh

8fill

,c
mi

S.iil
i(Mm
mMm

milm

,.\-)
B'o

OH I

OH

KMnOa

t-

oHlfi*

ffim
imM@u

\)

oH

ruoHAA""
The Berkeley Review

ili. n"
LI]L-

@.

Solution
The secondary alcohol is oxidized into a ketone, which eliminates choice D. The primary alcohol is oxidized all the u'ay to a carboxyiic acid, which makes choice C the best answer.

$ild!!
fihs
fum-il

frum'Jl

Copyright O by The Berkeley Review

llllluOWII

Organic Chemistry

Carbonyls and Alcohols

Carbonyl Reactivity

Example 5.15 \Ahich of the following compounds is NOT an oxidizing agent?

A. Peroxybenzoic acid (C6H5CO3H) B. Ozone (O3) C. Diisobutylaluminum hydride [((H3C)2CHCHz)zAlH] D. Chromic acid (H2CrOa)
Solution organic oxidizing agents generally contain oxygen atoms, like choices A, B, and D' Diisobutylaluminum hydride (DIBAH) contains no oxygens, and in fact contains hydrogens. DIBAH is a selective reducing agent that is weaker than Lithium aluminum hydride (LiAtH4). The best answer is choice C.
Example 5.16

C. a strong base. D. a strong acid.


Solution

oxidation of an aldehyde into a carboxylic acid requires the addition of: A. an oxidizing agent. B. a reducing agent.

Oxidation requires the addition of an oxidizing agent to carry out the oxidation. Oxidation can be carried out in either acidic or basic conditions, so choices C and D are eliminated. The best answer for this question is choice A.
Example 5.17

i\'hen a primary alcohol reacts with potassium permanganate, the primary


alcohol is acting as which of the following in the reaction?

{, An oxidizing agent B. A reducing agent C. A strong base D. A strong acid


Solution

it is losing electrons. This means that it is :ausing the reduction of Mno4- into Mno2. Reducing agents get oxidized, so the :rimary alcohol is the reducing agent. The best answer is choice B.
Erample 5.18 Cridation of a secondary alcohol leads to:

Because the alcohol is being oxidized,

{. an aldehyde. B. a carboxylic acid. C. a ketone. D. a tertiary alcohol.


Solution
-{s shown in Figure 5-26, a secondary alcohol is oxidized into a ketone and no --.rrther, because it has only one hydrogen to lose to oxidation. The best answer :or this question is choice C.

'V

lopyright

by The Berkeley Review

27

Dxclusive MCAT Preparation

Organic Chemistry

Carbonyls and Alcohols

Carbonyl Reactivity

Example 5.19 \Alhich of the following compounds CANNOT be oxidized by chromic acid?

A. 2-Methyl-3-pentanol B. Buteraldehyde
C. D.
3-Pentanol Phenol

Solution Primary and secondary alcohols can be oxidized while tertiary alcohols and phenols cannot be oxidized. Choices A and C are secondary alcohols, so they can
be oxidized into ketones. Aldehydes are oxidized into carboxylic acids, so choice B is eliminated. This leaves choice D, phenol, as the best answer. The carbon bonded to oxygen in phenol has no hydrogen atoms, so it cannot be oxidized.

Example 5.20 To oxidize a primary alcohol into an aldehyde without further oxidation into a carboxylic acid, what reagent should you add in conjunction with chromic acid?

A. Pyridine and hydrochloric acid B. Toluene and hydrochloric acid C. Pyridine and sulfuric acid D. Toluene and sulfuric acid
Solution A primary alcohol is oxidized into an aldehyde and no further when pyridinium chlorochromate is added to the solution. This is known as PCC or Collin's reagent. The absence of water from solution prevents the aldehyde from further oxidation into a carboxylic acid. The best answer for this question is choice A.
Reduction

Reduction is the process of losing oxygen (and/or gaining hydrogen) at the carbon which has an electronegative atom attached. As with oxidation, the first step is to count the number of carbon-oxygen bonds in the reactant to the carbon of interest. As the number of carbon-oxygen bonds decreases, the oxidation level decreases (as does the oxidation state). If you lose a carbon-oxygen bond and replace it with a carbon-hydrogen bond, the oxidation level decreases by one and the oxidation state of carbon decreases by two. From a ketone or aldehyde to an alcohol, the oxidation state decreases by two and the level decreases by one. The reduction of an ester into a primary alcohoi is shown in Figure 5-28.

ll*r 1. LiArH,(tho I i] ,/^\ ocH2cH3,-\rHrci*)'' - HSC v-r H + HoCH'CH' ,z^c\ + HsC vf'|
*r
OxidationState:0+
L + L -+1 =

OHOH

+3

OxidationState:0+1- 1- 1=-1 Figure 5-28

The two most common reducing agents in carbonyl chemistry are LiAlH4 and NaBH4. Aluminum is less electronegative than boron, so LiAlH4 is more apt to donate an H- to the carbonyl carbon, and thus is more reactive than NaBH4. As such, NaBH4 reduces only ketones and aldehydes while LiAlHa reduces all carbonyl compounds. Figures 5-29a and 5-29b show these carbonyl reductions.

Copyright

by The Berkeley Review

2A

The Berkeley Review

Organic Chemistry
o
il

Carbonyls and Alcohols

Carbonyl Reactivity

HOH ----+
1. LiA1H4(thfl

o/t\o*u
Carboxylic acid

2. NHaCI(aq)

RH

\r -zc:.
\s
(-

+ H'o

Primary alcohol

o
il
1.

HOH
LiAIH4(thf)
2. NHaCI(aq)

o/t\oo'
Ester

-zc:. R'H
Primary alcohol

+ HoR'

---+
1. NaBHa(et2O)

o
il

HOH
2. NHaCI(aq)

\s

o/t\r,
Aldehyde

n/-\n
Primary alcohol

o
il
1. NaBHa(et2O)

HOH
o,
2. NHaCI(aq)

o/t\
Ketone
1
S

-zc:. RR'
Secondary alcohol

\s

Figure 5-29a

o
lt

HCI/Zn(Hg) ____=>
or
1. HS(CH2)2SH

H
s-

o/t\*,
e rt

Aldehyde

2. Raney

Ni

o/t\r,
Alkane

o
li -/._\
Ketone

n
,t

(.

il

H2NNH2 \ r --.-->
R'

HH

d d

KOHlaq)

_. ^,ZC: R' RAlkane

+ N, + HrO

n
.e

lr';ulfide linkage

it-t

2 equivalents

HSCH2CH2OH Reduced Linkage

+l

S_ CH"CH"OH LL

s-

cH2cH2oH

F.H \zH2 :

Pd

oI

FADH2

..
rd

cH2co2H

--,>>

\J ou7 Y""
H

HH

CH2CO2H

-':<aturated fatty acid


to

Saturated fatty acid

Figure 5-29b

\s
rl1

:;;lion,

= aldehyde-to-alkane reactions are Clemmensen reduction and Raney nickel respectively. They reduce either an aldehyde or ketone to an alkane.

:w

29

Exclusive MCAT Preparation

OfganiC ChemiStry

carbonyrs and Arcohors

Carbonyl Reactivity

j
ts

Example 5.21

what is the major organic product after pentanal is treated with sodium
borohydride (NaBHa)?

A. B. C. D.

A primary alcohol A secondary alcohol A hemiacetal A carboxylic acid

Solution Aldehydes can be oxidized into a carboxylic acid or reduced into a primary alcohol. sodium borohydride is a reducing agent, so an alcohol is formed. This eliminates choices C and D. The carbonyl in an aldehyde is on the C-1 carbon, so the hydroxyl group forms on C-1 carbon, resulting in the formation of a primary alcohol. Choice A is the best answer.
Example 5.22

:l :

Si

Et

Acetone, when reacted with lithium aluminum hydride and quenched with weakly acidic water, yields which of the foliowing products?

C. D.

A. Propane B. 1-Propanol
2-Propanol Propene

q.

Solution
Acetone is a ketone, thus when it is reduced, it forms a secondary alcohol. The only secondary alcohol listed is choice C, so choice C is the best answer.
Example 5.23
Stfu,l

Ss{

,Jtffift
"@nilI

To carry out the following synthetic transformation, what reagent should be


added?

H A. B. C. D.

--+

cH2oH

LiAlHa
NaBH4

H2N=NH2 FADH2

Solution

In this reaction, an aldehyde has been reduced into a primary alcohol on

compound where an ester functional group is unaffected. FADH2 caries out hydrogenation of a n-bond, so choice D is eliminated. Hydrazine, H2NNH2 reduces aldehydes and ketones to alkanes, so choice c is eliminated. Both LiAlHa and NaBH4 reduce aldehydes to primary alcohols, but LiAlH4 would also reduce the ester. This eliminates choice A and makes choice B the best answer. NaBH4 is selective for aldehydes and ketones.

Copyright

by The Berkeley Review

30

The Berkeley Review

'ffiW

Organic Chemistry
Example 5.24

Carbonyls and Alcohols

Carbonyl Reactivity

\A/hat is the oxidation state of aluminum in LiAlHa?

A. +3 B. +1

c.

-3

D. -5
Solution
The term hydride implies that hydrogen has a negative charge associated with it. Lithium is +1, so the Al must have an oxidation state of +3 to have the charge on the compound be zero. The best answer is choice A. Example 5.25
\'Vhat is the major organic product for the following reaction?

A-.'Jl-"Hffi
OH
OH
B.

oH c'o

uo" uoH

D'oH

Solution
Sodium borohydride, NaBH4, reduces aldehydes and ketones only. Both the <etone and the aldehyde functional groups are reduced to alcohols (secondary and primary respectively). This yields a 1,3-diol, so the best answer is choice A.

t
2

I
t

-opyright

by The Berkeley Review

3t

Exclusive MCAT Preparation

Organic Chemistry

Carbonyls and Alcohols

Name Reactions

:
ffil
wrfri

NaulG,,,Keactions

,,

&
,

Claisen condensation, transesterification, the wittig reaction, pinacol


rearrangement, the iodoform reaction, and the Wolff-Kishner reaction that you should know. There are other name reactions in organic chemistry, but these are the name reactions you must know according to the MCAT student Msnual. we will address each reaction according to their frequency on previous MCAT exams. Where appropriate, we will emphasize the reaction mechanism or its regioselectivity and stereoselectivity. of most importance to the MCAT, we shall address any biological applications of the reactions or analogous reactions.

Name Reactions There are name reactions such as the Grignard reaction, aldol condensation,

cb

mi

ffiw

sm

irssr

frW @

Grignard Reaction Grignard reactions involve the addition of a carbanion (carbanion metal halide) to an electrophilic carbon center in a carbonyl to form a new carbon-carbon bond. when alkyl magnesium bromide, the Grignard nucleophile, attacks the carbonyl carbon, a tetrahedral intermediate forms. Almost all Grignard reactions g"r,"tui" alcohols. No matter what the reactant, Grignard reactions generate a hydroxyl group in the product where the carbonyl group originated. Figure 5 -30 shows three such Grignard reactions, the third of which is a double addition to an ester (an acid anhydride or acid halide yield the same alcohol product as an ester). Grignard Reaction
No L.G. .'. one alkyl group

h
ufim

uw

cl,

l,

G.

n.

adds:

one C

+
R'

two C (probably chiral)


HO
R'

-+".-i'NIgBr --+
Aldehyde

lO

BrMs'-O

& M m

ffi

oXn
C ->

work-up-

oXr_,
2" Alcohol

# ft
@

No L.G. .'. one alkyl group adds: two

-O R' BrMg+

"-Xo4
-+

three C (possibly chiral)

,qp

Ho

R'

-S-.'MgBr --->
Ketone Has a L.G.
.',

oXo
3"

Alcohol

two alkyl groups add: one C

three C (never chiral)

____+
Ester

RR'

ketone

(or anhydride or acid halide)

HO R

X RR'
3" Alcohol

work-up

BrMg*

-/

-X:RMgBr
Figure 5-30

Copyright

by The Berkeley Review

32

The Berkeley Review

Organic Chemistry

Carbonyls and Alcohols

Name Reactions

I,
,1

u e

r
.S

t1

Crignard nucleophiles add once to aldehydes, which have one R-group, to form secondary alcohols. Ketones have two R-groups, so they form tertiary alcohols. Carbonyls with a leaving group (esters, acid anhydrides, and acid halides) start rr-ith one R-group, but because they have a leaving group, they add a new Rgroup twice. Because the Grignard reagent adds twice to esters, acid halides and acid anhydrides, the products are achiral tertiary aicohols. Grignard reagents are strong bases, so it is critical that no protic hydrogens are present. This is why the reaction uses anhydrous ether solvent. The second step in the Grignard reaction iequires a weak acid, as opposed to a strong acid, to avoid protonation of the l'droxyl group, which can lead to an E1 elimination reaction. The Grignard reaction can also take place with carbon dioxide to form a carboxylic acid.
Example 5.26 ;\-hen 2-butanone is added to 1-propylmagnesiumbromide in ether then worked '-:c rvith ammonium chloride in water, it yields a major organic product of:

B, 3-ethyl-2-pentanol. C. 3-methyl-3-hexanol. D. 3-bromo-2-butanone.


Solution
Tr.e Grignard reagent attacks the carbonyl carbon to form a new carbon-carbon :-.nd. The reactant is a ketone, so the product is a tertiary alcohol, eliminating ::.cice D. The IUPAC name in choice A is not possible, because a2-propyl group '". -',r-rld actually be part of the longest chain. The reaction is drawn below:

4.. 2-propyl-2-l:utanol.

H3CH2CH2CMgBT

BrMsO H:C

"XL

CH"CH.CH
CH2CH3

L'T:''L

HO + CH'CH"CH,

z r ;]C- CH2CH? H:C

\/

'5'o

-:.e longest chain in the product is six carbons, so the product is hexanol. The

-lir {C name is 3-methyl-3-hexanol, so choice C is the best answer.


Erample 5.27 :::.'"'1 magnesium bromide, when added to 3-pentanone, yields which of the --..rving organic products following weak acid workup?

\. Triethyl ether F. 3-Ethyl-3-pentanone t-. 3-Ethyl-3-pentanol


D.
S

,l-Heptanol

rtrution *^-. reactant is a ketone, so the product is a tertiary alcohol. This eliminates : :,:es A and B. The reaction is drawn below:

tll ' -n.aU


= ,ongest

/ O

HTCH2CMgBT

---->

!L ,, ).r.._." ,' rr,,arrra- -cHrcHr-HrcH2c' -cH2cH3

BrMsO CH.CH' "X L r

HO

CH"CH"

--

\C name

chain in the product is five carbons, so the product is pentanol. The is 3-ethyl-3-pentanol, so choice C is the best answer.

rew

- -:'" right

by The Berkeley Review

33

Exclusive MCAT Preparation

Organic Chemistry

Carbonyls and Alcohols

Name Reactions

Aldol Condensation Aldol reactions start with deprotonation of an alpha hydrogen from a carbonyl
(ketone or aldehyde). The pKu of most o-hydrogens that are adjacent to only one carbonyl is between 17 and 19. This means that they are weakly acidic and can be removed by using a strong base. The resulting anionic compound (enolate) nucleophilically attacks a neutral carbonyl compound (electrophile) to form a Ghydroxy carbonyl compound. This species is rarely isolated, because under the

reaction conditions, it undergoes elimination to yield an cr,G-unsaturated carbonyl. Regioselectivity is involved, because the final product can have either E or Z structural geometry. The anionic species can also be protonated on the oxygen, resulting in the formation of an enol, but that is in equilibrium with the carbonyl. Conversion from a ketone (or aldehyde) into an enol is referred to as tautomerization, which was shown in Figure 5-19. Figure 5-3L shows the aldol
condensation reaction of acetone.

HsC

A 5A.
:o:

Aldol Condensation

:o:
<+l
-.QHz
(

Or

:o:
I

CH3 withOH-, HeCketone-to-anionratio-

zAHrC- - CH,

is about 1000: 1 "3'\f

CHs

/ zo: \-7

\ ",.

:o:

9Hz
I

HaC

A
HgC

:o:

CH

elimination I
CHs

HeC

9Hz
I

-P
CH,

,/t\ HsC I

CHs

:.o.:o

,(

,'f\ H:C I
Figure 5-31

cr,B-Unsaturatedketone

:9lr

B-Hydroxyketone

It is often easier to predict a reaction by focusing on the side product. In an aldol condensation that goes all the way to an cx,,B-unsaturated ketone, water is the side product. Finding the atoms in the reactants that make up the water makes predicting the product easy. Figure 5-32 shows how to align two acetone molecules for the aldol condensation.

*.A.* >Hur-

oo
I

H:C

:{
rr o

'nl

orient

th; iater,

connect the

r' "..4.r,
u
HrC

-CH:
Figure 5-32

Jt

Copyright O by The Berkeley Review

34

The Berkeley Review

ns

Organic Chemistry

Carbonyls and Alcohols

Name Reactions

ry1

ne

aI
te) B-

he ed
rer

he he
AS

to1

The aldol condensation offers the potential for thermodynamic control versus kinetic control. when the ketone is asymmetric, then the choice of which alpha hydrogen is deprotonated is dictated by reaction conditions. The reaction conditions that are most significant are steric hindrance of the transition state and thermodynamic stability of the intermediates and product. When the reaction )'ields a major product that was selected to minimize steric hindrance in the transition state, this product is said to be the kinetic product. This term refers to the lower activation energy required to go through a less sterically hindered transition state. When the reaction yields a major product that was selected to maximize stability of the intermediates or product, this product is said to be the llrcrmodynamic product. This term refers to the greater amount of free energy released when forming the more stable compound. Figure 5-33 shows the kinetic and thermodynamic products for the aldol condensation of butanone. Kinetic Control
less steric hindrance lower transition state

Thermodvnamic Control

t.t.4
Hti

(lower Eactivation) prefered at low temp

.-tnu
/ \
H,t.,

with strong, bulky base

/\

Hn Htn

more substituted intermediate greater energy released (more negative AG .*r. ) prefered at high temp with strong, small base

H\A
{3

Krnetic Control

CH'CH]
t-butOK ___
t-butOH, -78'

t-gAcH2cH3
H3CH2C

t-.AcH,c
HsC

P H3CH2C
o

CH:

Jt+
CH.:

It
minor

H3

CH2CH3

major Thermodynamic Control

*r"cA
:( H:C
1o1

c 'rffil

.c.''
KH. -*
50"c
CH2CH3

.r"cAc 'cHt
HsC
major
CH2CH3

,A

.r.cAc

,CHz

H3CH2C CH:
minor

Jt

ide
(es !ne

Figure 5-33 Erample 5.28

lhe following reaction is best described

as:

')u.
o
{" l

----+

1. NaFlthfl 2. workup

\. B. C. D.

via the kinetic enolate. via the thermodynamic enolate. an aldol condensation via the kinetic enolate. an aldol condensation via the thermodynamic enolate.
@

a Claisen condensation a Ciaisen condensation

ew

-rpyright

by The Berkeley Review

35

Exclusive MCAT Preparation

Organic Chemistry
Solution

Carbonyls and Alcohols

Name Reactions

T
rur
f,mfi
t!fism

The reaction involves the condensation of an asymmetric ketone upon itself,

which eliminates choices A and B, because the Claisen condensation requires the reaction of esters with esters, not aldehydes or ketones. Because the more hindered alpha carbon is involved in the addition, the reaction must have been carried out by way of the thermodynamic enolate (more substituted enolate). This makes choice D the best answer.
\Arhen a reaction has a biological example, its likelihood of appearing on the MCAT is increased. The biological application of the aldol reaction is the fourth step of glycolysis, which is a retro-aldol reaction. In the retro-aldol step, a sixcarbon B-hydroxyketone breaks into two three-carbon fragments. The two threecarbon species, after neutralization, are dihydroxyacetone phosphate (DHAP) and glyceraldehyde-3-phosphate (G-3-P). It is not necessary to know the mechanism, but you should know that the enzyme responsible for this step is aldolase. Figure 5-34 shows the fourth step of glycolysis.
Step

dq
rfo

!m

ah

M
mffi @

do

IV

GlYcolYsis

0-m

ft'@
HO

cH"cm)

H H

ff"
cH2@

Aldolase-

LIr\-IIrrv

il-' +
U rr

t;

1",o,,
DHAP

ffi h ffi

o
OH

_T-OH
I
-n*

cH2@
Figure 5-34

B-hydroxyketone

cH2@
G-3-P

Claisen Condensation Reaction The Claisen condensation reaction is essentially the aldol reaction with an ester. Esters have a leaving group, so the product is not an c[,8-unsaturated carbonyl, as
condensation reaction. Note that without acidic workup, the carbon between the two carbonyl species would be deprotonated by the ethoxide in solution. Claisen Condensation

is seen with aldol condensation reactions. Figure 5-35 shows the Claisen

EtO

:o:

EtO-Na*
EtOH CHs EtO

\o

CH" L

EtO

:o:

-OEt

9Hz
I

The base is chosen to match H

the leaving group so that a tranesterification reaction won't generate a new ester.

,cto,
[:oz \-tz

/ll

"r.Ag. B-Ketoester
The G-ketoester exists in equilibrium

with the deprotonated species until


the solution is neutralized.

Figure 5-35 Copyright


@

by The Berkeley Review

36

The Berkeley Review

Organic Chemistry
3

Carbonyls and Alcohols

Name Reactions

The B-ketoester product has a pair of hydrogens that are now conjugated to two

:
1

carbonyl groups. As such, their acidity is enhanced, so the sodium ethoxide deprotonates the B-ketoester product preferentially over the ester reactant. Once reutralized, a B-ketoester is formed. A B-ketoester when treated with strong acid in water can hydrolyze to form a B-ketoacid, which when heated readily r-rLdergoes decarboxylation into a ketone. This ultimately leads to a ketone with a longer alkyl chain than the starting ester.
Fatty acid synthesis is a biological example of a Claisen condensation reaction, -.r-here the ester is actually a thioester. Synthesis of a fatty acid involves Claisen condensation of acetyl-CoA onto another thioester. The chain gl'ows two carbons at a time as shown in Figure 5-36.
Fatty Acid Synthesis (Claisen Condensation with a Thiosester)

:o:

:o:

:o:

c"ASA.";-T.orA?'r,
fhe enzyme acts as
--ase by -*re

T"orAcH2
Hrc-f{scoa :9.:A
:o:
dehvdration CH (elimination )

coorA.r,,
HrcA
g_

.. ^ ieprotonating H3c t scoA thioester

/-]A: \-:7

c-.*o,n,o"r,".

:o:
| 1

:o:

hydrogenation A CoAS 9Hz FADH2 CoAS

JI

A1 CoAS
I

reduction

9Hz

NADH

H,cf
H
S

r-

trtA
Figure 5-36

",ctSi.u H

:t
e

Transesterification

lransesterification involves the exchange of one alkoxy group for another on an rster. Because the equilibrium constant is roughly one, the reaction is driven by Le ChAtelier's Principle. Either a product is removed or a reactant is added. An :cid-catalyzed transesterification reaction is shown in Figure 5-37.

R/\

ll

oR,

+ HoR" --

cat.

H'

HoR'

J(

OR''

Figure 5-37

Transesterification can be carried out in either acid or base. When acid is employed, the mechanism is a typical acid-catalyzed process. In base, the alkoxide nucleophile causes the reactivity. If the base is hydroxide, a carboxylate and alcohol. are formed as the two products. When a fatty acid triglyceride is ,ieated with strong base, glycerol and fatty acid carboxylates are formed from a hvdrolysis reaction. Figure 5-38 shows the acid-catalyzed mechanism for
:ransesterification.

Copyright O by The Berkeley Review

6/

Exclusive MCAT Preparation

Organic Chemistry
Fi+

Carbonyls and Alcohols


Transesterification mechanism

Name Keactions

RA

+
OR'

R,,oH

;H--

R'oH

io!
ll

RAoR.,
'll a"oroto,",ut"

o-.^",.1i
H

nAon, l
break

\o. o: il)
I I

fi

n"

- ijs

HV

A
i

,:6tn
l l-rk"
OR''

o.

ofo*
o"-Ht
-"uJll
I

nAon"
nti0.""0 n"
1

,:n/n

")

.)t'-.

deprotorlate

n'ci:

.'iin,'

R'protonate

../i..
O-@

'./\.
OH Figure 5-38

\/

OR"

The biological applications of transesterification involves membrane transport and the conversion of fatty acids into triglycerides. Fatty acids are found ilcell membranes, where they are part of a phospholipid (comprised of glyceror, two fatty acids, and a phosphate group). when glycerol binds three fatty acids, triglycerides are formed. Fatty acids that have been esterified with glycerol are broken down in basic water in what is known as saponification, the based catalyzed cleavage of the ester bonds of the triglyceride, as shown in Figure 5-39.

Saponification

o4*
o4-*
oAo
Copyright
@

OH

oH--

OH +3
oo
OH
Figure 5-39

by The Berkeley Review

5B

The Berkeley Review

ns

Organic Chemistry

Carbonyls and Alcohots

Name Reactions

Wittig reaction Another useful carbonyl reaction for synthesis is the Wittig reaction. The Wittig reaction essentially undoes an ozonolysis reaction. It is yet another way to make a carbon-carbon bond. The Wittig reaction converts a carbonyl into an alkene, rvith stereoselectivity. You can get either the E or Z product depending what reagents you choose. The double bond you form can be added to by electrophilic addition reactions, with high stereoselectivity. What this means is that there are now synthetic routes to get high yietds for stereoselective synthesis. we are going to see it in its most simplistic form however. It goes through a fourmembered intermediate, similar to the hydroborane addition reaction with an alkene. Figure 5-40 shows the synthesis of a phosphonium ylide (the reactive
species in a Wittig reaction).

GP:+

o o

R2cHBr--+

o G o
Figure 5-40

uq a*ro, + Li-but ->

o oo G o
P-

CR^

it is the final product here that is the reactive species in a Wittig reaction. There rs a resonance form which has a double bond between the carbon and :hosphorus. You should recall that the phosphorus is able to make more than :our bonds due to the availability of 3d-orbitals. The overlap between a 3p,rrbital (from P) and a 2p-orbital (from C) is not as strong as the overlap between :''r'o 2p-orbitals however, so the n-bond between phosphorus and carbon is not =at strong. Figure 5-41 shows the Wittig reaction and various intermediates in re mechanism in an aprotic environment.

rrt
oll
'\'o

ls,
lre
,ed
\9.

o G o

Wittig reaction

P:CH" .. g{ z\

o G o

p:O +

H"C=={ .\

/R
R

a 3P:=

CH2

\ o:

cR2

*''ls::
Figure 5-41

.--.>

azP

ll+ o

il'
CRz

CH.

b. an aprotic environment, the transition state is a four-centered transition state. .re phosphonium ylide @3P=CH2 can be synthesized from @gP: and H3CBI ;nder basic conditions, as shown in Figure 5-40. The wittig reaction prefers the E-alkene product whenever there is asymmetry in the reactants.

ew

-opyright O by The Berkeley Review

39

Exclusive MCAT Preparation

Organic Chemistry

Crtoryts ad lllcohols

ttime Eeactions

o
!ilr

Pinacol Rearrangement The pinacol rearrangement is a topic in the MCAT Student Manual, so we shall cover it. It is a classic example of rearrangement with some applications in synthesis. The pinacol rearrangement converts a vicinal diol into a ketone. The reaction proceeds with an alkyl migration, which converts a tertiary carbocation

into a resonance stabilized carbocation. Figure 5-42 shows the pinacol


rearrangement of pinacol into pinacolone.

il m il e C m

p
F F u

HO: \/

\ '\< .. OU

Pinacol rearrangement

:O
\ /

CH" J

r&
@ ffi

ffi

tuto"f \"'t* CHr H:lC

\ tu.*'f
HeC

ff

fI p,otonat"

H"C CFL r o"o.o,o.u,"\ H-A.'

f- \,,.*
(c).
CHs

m m
d

if+ \"'.*
CFI3 ..,.rrl

v\/

H:C
HO

i-\"tcbrs CH:
/t"rt"
7

V** HO:
..
H

,,*"?3 \att,
HsC

\@

CF{,

t"uttu$"

---

@'r-

i)

CFL

HsC
Figure 5-42

/-

'("'a*r,
\
CF{3

/J

Iodoform Reaction
The iodoform reaction falls in a general class of reactions known as the haloform reactions, which convert a methyl ketone into a carboxylic acid and a haloform, HCX3. The iodoform reaction is used as a chemical test for a methyl ketone, because iodoform is yellow in color and insoluble in water. Methyl ketones react

with iodine,

12, in the presence of base to generate a carboxylic acid and a deprotonated iodoform molecule. The carboxylic acid is deprotonated to yield a carboxylate and iodoform. The iodoform reaction is shown in Figure 5-43.

Iodoform reaction

o
o

/c\.rr,
o

il

o
il

OH *

R--C-....o CHz

I-I *

-O r

lt

R--C\

CH^

'/-\

ll

+ HCr.J -_ oo

ROH

ll -zL\

ot

-Cr-o&

R-zc-. CI:

l]

l"

'-*""1:l

Figure 5-43

Copyright

by The Berkeley Review

40

The Berkeley Review

Organic Chemistry
"r', ii1 f

Carbonyls and Alcohols

Name Reactions

nall

sin
Ihe ion
rcol

.::e are three common methods for reducing a ketone or aldehyde into an .-.:ne. There is the Clemmensen reduction, which is carried out under acidic : :.:ritions using HCl and Zn(Hg). There is the Wolff-Kishner reduction, which is
,.:::ed out under basic conditions using hydrazine (H2NNH2) and basic , -:(up. And last there is the treatment of the carbonyl with a 1,2 dithiol , -SCH2CH2SH) followed by treatment with Raney nickel (fine nickel in the
r:.sence of aluminum oxide). The good thing about having three reactions is ,, :.: each is carried out at different pH values. This allows for many different .,:::onvl compounds to be reduced independent of any pH sensitive functional l:- jps elsewhere on the molecule. For the MCAT, it is important to know the :::.hanism for the Wolff-Kishner reaction. The Clemmensen reaction - -:hanism is not thoroughly understood, so for the Clemmensen you really only =:i to know that most transition metals have a positive oxidation potential and .:. 'leerefore strong reducing agents. When added in their zero oxidation state, -.=ials donate electrons to the organic molecule, causing the molecule to get :=-ruced. Figure 5-44 shows the three reactions carried out on cyclopentanone.

f-Kishner Reduction

Wiilff-Kishner reduction HzN:.


F3

d
o
L_,/

N -N

HH -->
Raney nickel reduction

+Nz

f-f
RanevNi-

I A r-^g" X
cat HCr

ti

+ HSCH'GH'sH

tj
HH
+ HrO + ZnCI2

)rm

Clemmensen reduction

rm/
trIr

ract
Jd

)^

da
Figure 5-44 must apply the right reaction under the right conditions, which depends on of other functional groups on the molecule. The Wolff-Kishner :=action is carried out in two steps. The first step of the Wolff-Kishner reaction -...'olves the substitution of hydrazine for the carbonyl oxygen, resulting in the . , -nation of a hydrazone. Treating the hydrazone with strong base in water :=nerates nitrogen gas. The first step of the reaction is reversible with the :,rlition of water and acid. That is to say that you can regenerate the carbonyl by
.--e presence

.:u

f2

:--:ding water and a catalytic amount of acid to the hydrazine deril'ative. -:ibonyls can also react with secondary imines, but because there is only one ::cton to lose from the amine, the reaction generates an enamile rather than an -::'.ine. We shall consider such reactions in the nitrogen compound section. - rzure 5-45 shows the mechanism for Wolff-Kishner reduction. -op;rright
@

by The Berkeley Review

4t

Exclusive MCAT Preparation

Organic Chemistry
Step 1:

Carbonyls and Alcohols

Name Reactions

:o:
lt

.zc:. RR
Step 2:

HH \/ :N-N: /\ HH

sril-..
N

ffi fil

-->

lt

.zC:. R

&

M
G

fiJ

.zC:.

il)*\'. il
R

A. -:.gtt
^-

H.r-.

..O

RR

V ,/\

HH \/

ilt
J..t
I

I ..o *-r,
R

H_o.ll, \\
- H.h. \
H

't/

F1

!5:1,; * ..fE Rln


:OH

il

..o

t
';r9S
Rln
./;\ ^

'i,7*-r, l^_\l-Et

'i-z\!
--I

n {r
"iS

*)'

./:.\ .t' R- I -R .-O-H \"/

.29:. Rln
H rrgure 5-45

4
H

t"
:4
,,,,1
.1*s,,

Copyright O by The Berkeley Review

The Berkeley Review

)ns

Organic Chemistry
StftthGtiru:::::D$giC
Synthetic Logic

Carbonyls and Alcohols

Synthetic Logic

Svnthesis is a recurring topic throughout organic chemistry. The method for :esting synthesis in most lecture courses is not feasible on the MCAT, because the \ICAT is a multiple choice exam. However, synthesis appears on the MCAT. Svnthetic pathways are shown in a step-wise road-map fashion similar to how a :athway appears in biochemistry. The passage will present the steps, which may

rr may not include the reagents. Questions

can focus on stereochemical

:onsequences, reaction types, and the logic behind a step in the overall synthesis. lhe strategy behind a synthesis is based on the idea that it is easier to solve a raze backwards than forwards. The steps to analyze a synthetic pathway are:

1) l) 3)
-

Identify the new bonds (and functional groups) formed by counting the atoms and noting the changes in bonds from reactant to product.
Break the product into fragments that match the reactant skeleton. Reconnect the fragments with the proper chemical reagents.

et's consider the conversion of 1-propanol into 2-pentanol. Figures 5-46 through -:-f8 show the stepwise analysis of the synthesis and retrosynthetic analysis.

1) A five carbon secondary alcohol is formed

from a three carbon primary OH

alcohol, so two carbons must be added to the reactant.

?"':r?frr

9o., ----+ -^--IFigure 5-46

;"':rfi:,

Using retrosynthetic logic, the product is broken into two fragments. The fragment with the oxygen must have come from the carbonyl.

,.-J,!
Svnthesis requires:

OH

E+
a-.\r,
nucleophile

OH

"\>
retron

and

!^retron

carbonyl electrophile

Figure 5-47
= :ctrophile). The more oxidized fragment (oxygen containing fragment, or :' ::-t fragment in this case) comes from the carbonyl. The oxygen is bonded to = first carbon, so it requires an aldehyde. The rest of the problem requires

--:

reaction calls for a three carbon nucleophile adding to a two carbon carbonyl

.'

.tresizing the Grignard reagent from 1-propanol and choosing the solvents.

,-

This Grignard reaction requires propyl magnesium bromide and ethanal.

3.o

\-^""ffiyrgu.
Figure 5-48

OH

rew

-:vright

by The Berkeley Review

43

Exclusive MCAT Preparation

Organic Chemistry

Carbonyls and Alcohols

Synthetic Logic

shows the conversion of an alcohol into a silyl ether and then back into an alcohol' The ketal protecting group has already been shown in Figure 5-11.
Protection and deprotection of an alcohol

Protecting Groups Protecting groups are used when a reactant has multipre reactive sites, but you only wish to react at one of them. you must determine which functional groups can react under the reaction conditions. In cases where more than orru ,It" react, the functional group you wish to not have react is converted "un into a less reactive functional group. Aldehydes are converted into acetals, ketones are converted into ketals, and alcohols are converted into silyl ethers. you must be able-to remove the protecting group at the conclusion of the reaction without invoking extreme conditions which may cause the molecule to react. Figure 5_49

f,c

Ma

8r0

{-rrtl

rut
ffIr

pm cilt

add

tof

HOH RR'

protecting

ffi

(H3C)3SiCl
group

(H3C)3Siq H

-r .. ffi R'

\ /

HOH
NaF or H"O*
protecting group

RR'

E|

Alcohol

Protected alcohol (sityl ether)

Alcohol

Figure 5-49
Reactions of Acetoacetic Ester Acetoacetic ester has a carbon that is alpha to two carbonyl groups. As a consequence, its protons are more acidic thin typical alpha proto*. rr-,u pKu of the protons alpha to both carbonyl groups in acetoacetic estei is roughly ri. as a consequence, it can be deprotonated in mildly basic conditions, rresulting in a carbanion. The carbanion is a strong nucleophiie, capable of displacing a telving group to form a new carbon-carbon bond. Fig.rte 5-t0 shows the addition of an alkyl group to acetoacetic ester.

,rhn

hfr k

o
1.

NaOEt (HoEt)

''''lt+ ry tus ,,t"&


.;

2.

R'-X

ffit

Figure 5-50

base of choice matches the alkoxy group of the ester, so that any transesterification reaction that takes place does not result in any change. ThL solvent is the conjugate acid of the akbxy group, an alcohol, beciuse it is being formed by the reaction anyway. The reactint ii a B-keto ester, which is formed from a Claisen condensation reaction of two esters. The product can be treated with acidic water and heat to hydrolyze the ester into a B-ketoacid, which can undergo decarboxylation into a ketone if the temperature is high enough.

The

Copyright

by The Berkeley Review

44

The Berkeley Review

rc

Organic Chemistry

Carbonyls and Alcohols

Synthetic Logic

lu
ps

tn
SS

Reactions of Malonic Ester \4aionic ester, like acetoacetic ester, has a carbon that is alpha to two carbonyl gloups. As a consequence, the same reactions carried out with acetoacetic ester can also be carried out with malonic ester. When malonic ester undergoes the

re
be

ut
JO

1n

reaction shown in Figure 5-50, the same addition takes place, although the product is a diester rather than a ketoester. The significant difference between acetoacetic acid and malonic ester is that a diester has two functional groups that can undergo substitution, resulting in cyclization. Figure 5-51 shows the addition of an alkyl group to malonic ester followed by the addition of a diamine to form a six-membered ring.

o
1.

o
NaOEt (HoE|
2.

R-X

oEr

of

Figure 5-51

ia
a 1g

iarbiturates form when urea is used in place of the diamine in the second step.
Decarboxylation

]n

rs mentioned with the acetoacetic ester synthetic pathway, a B-ketoacid can -:ndergo decarboxylation when heated. This occurs readily, because the rtramolecular hydrogen bonding of a B-ketoacid aligns the molecule for a shift -i. the n-electrons. Figure 5-52 shows decarboxylation, the six-membered ring -:eated by hydrogen bonding, and the decarboxylation products formed from -:.at conformation.
Decarboxylation

---+ A
1V

CO2 +

HsC

1e 1g

:d :d trl

hrtramolecular :{-bonding in B-ketoacid

"X$-.-r[:^.
Carbon dioxide and an enol form

_-\ H:C

ll
R

Enol to ketone

(tautomerization)

Figure 5-52

:w

-r.pyright

by The Berkeley Review

45

Exclusive MCAT Preparation

Organic Chemistry

Carbonyls and Atcohols

Carbonyl Biochemistry

lffio.G

Biological Oxidation-Reduction
There is a great deal of oxidation-reduction chemistry in biology. using an organic chemistry perspective can simprify many biological reictions. L u general rule, you should know that anabolism is-a redultive process and the result is the build up of a molecule. Examples of anabolism lnclude fatty acid synthesis and gluconeogenesis. You should know that catabolism is an oxidative Process and the result is the breakdown of a molecule. Examples of catabolism include glycolysis and B-oxidation. Typically, catabolism rele-ases energy while anabolism requires, and thereby stores, energy. you should know thii in us, oxidations tend to take place in the mitochonaiiat matrix while reductions tend to take place in the cytoplasm, with glycolysis being a notable exception.
Example 5.29

ffii

following step of glycolysis?

which of the following cofactors or coenzymes is most likely required in the

&

o\
tt

H-J-oH A. Biotin B. AMP C. Zn D. NAD+

"

ove-pe^2P6r2-

| HzC- 9-

GAPDIL u*ou 'Ir I "'

HzC- 9-

pgr2-

Solution In this reaction, carbon 1 gains a bond to oxygen, so it has been oxidized. This means that it must be coupled with a reduition half-reaction. The enzyme GAPDH does not undergo reduction, so the cofactor,/coenzyme must undergo reduction. of the choices, only NAD+ (a species poor in H) is an oxidizing age"nt that undergoes reduction. Choice D is the best u.,r*"r.
Example 5.30
In humans, where does fatty acid synthesis take place?

A. B. C. D.

Cytoplasm
Rough endoplasmic reticulum Mitochondrialmatrix Mitochondrial inner membrane

Solution Fatty acid synthesis involves molecular build up, so it is a reductive process. Reductions occur mostly in the cytoplasm, so choice A is the best answer.
There are several examples of oxidation-reduction chemistry in biology, so we shall only focus on a few here and a few more with carbohydrate che"mistry in section 6. We shall consider oxidative decarboxylation, which shows itself in aerobic and anaerobic metabolism. Figure 5-53 shows the decarboxylation of pyruvate, a three-carbon species, into acetyl coenzyme A, where ihe acetyl moiety contains two carbons.

Copyright

by The Berkeley Review

46

The Berkeley Review

;ry

Organic Chemistry
o-

Carbonyls and Alcohols

Carbonyl Biochemistry

an

Q:g:Q

ia
he

+C

o\

_,,,
I

S-CoA

,id
.\.e

CHg Pyruvate

NAD+

NADH + H+
Carbon dioxide

CH:
Acetyl CoA

;m

ile
-1S,

Figure 5-53 Biochemical Reagents


For biological redox chemistry, there are two common reversible reactions. The first reaction is the inter-conversion between NAD+ and NADH, and the second

nd

he

leaction is the inter-conversion between FAD and FADH2. The two molecules and their reduction reactions are shown over the next few pages. Figure 5-54

shows NAD+ (the oxidized form), which acts as an oxidizing agent in :iochemical reactions, and NADH (the reduced form), which acts as a reducing
agent in biochemical reactions.

NHz

-o- P- o-

il

cH2

/l
4l

z
.)

9-

11S

o
Adenine N
N

o
N

ne
3o

nt
N

z
.)

o o a. o

H
OH

H HO OH

NAD+
(Nicotinamide adenine dinucleotide) Figure 5-54
iq

NADH
(Nicotinamide adenine dinucleotide hydride)

te in in
of r'l

\-{D+ serves as an oxidizing agent by picking up a hydride anion on its pyridine :-rg to yield NADH. The species being oxidized gives off H+ as a side product. ,ris biological redox reaction plays a major role in many cycles including colysis. NADH carries out the reduction of aldehydes and ketones, much like "-lr \aBH4 does in organic chemistry reductions. A second biological reducing :qent is flavin adenine dinucleotide dihydride (FADH2), which carries out r',-drogenation of n-bonds, much like H2 and a catalyst such as palladium. ::gure 5-55 shows the oxidized form, FAD, and the reduced form, FADH2.

:w

-opyright

by The Berkeley Review

47

Exclusive MCAT Preparation

Organic Chemistry

Carbonyls and Alcohols

Carbonyl Biochemistry

:..g$.
Flavin
I

HsC

HsC

CHr

s$
HO CH.
I

I
L_

-N"'

tl

Ao

"l o" "I,r" '-f o"


CH.

H-f

OH

,,-lI

o,,

HTOH
CH.

o
I

t'

o
O

t'

-O- P: o
I I

I -O- P:

z
a o
o-

-AcH 2

NHz

-o- PI

o H HO OH

ilL ol\

o-

\A
N

L1-/O

--fI
OH

HO
Figure 5-55

FAD (Flavin adenine dinucleotide)

FADH2 (Reduced form)

'*iil

FAD serves as an oxidizing agent by picking up a hydrogen on the nitrogen of the central pyrimidine ring and a second hydrogen on the imine nitrog"t of th" third ring, to yield FADH2. one hydrogen is gained as a hydride, while the other is gained as a proton, so there is no change in the charge of the compound. Acetyl coenzyme A is also very common in biochemistry, so we shall consider the acylation process of coenzyme A. The acetyl group is transferred from a reactant to a thiol when acetyl CoA forms. The structure of acetyl coenzyme A is shown in Figure 5-56. The biochemical role of coenzyme A is in transfer of an acetyl group to a biological species. Nucleotide

Si

,* ,a

O OH CH?

Acetyl Coenzyme A
-Frgure 5-5b

Copyright O by The Berkeley Review

4a

The Berkeley Review

Organic Chemistry
Oxygen Containing Compounds

Carbonyls and Alcohols


5)

Section Summary

Key Points for Carbonyls and Alcohols (Section

1.

Alcohols (Compounds with hydroxyl groups) a) They have boiling points higher than alkanes of roughly equal mass i. Boiling points of alcohols are: primary > secondary > tertiary ii. Water solubility is high for short chain alcohols

b) c) l.

The hydroxyl group can be converted into a better leaving group

i. ii.

SOC12 and PBr3 convert an aicohol into an alkyl halide They are mesylated or tosylated to form a good leaving group

They have broad IR absorbances between 3300 and 3600 cm-1

Aldehydes and Ketones (Carbonyl compounds with H or R-groups attached) a) They have boiling points higher than alkanes of roughiy equal mass i. Boiling points decrease with branching ii. Water solubility is high for carbonyls up to four carbons

b) c) -r.

They readily form acetals and ketals with alcohol in acid and form
hemiacetals and hemiketals with alcohols in base Aldehydes have strong IR absorbances around 7725 cm-I and. ketones have strong IR absorbances around 1710 cm-1

Carboxylic Acids (Compounds with a COOH group at an end of the chain) a) They have melting and boiling points significantly higher than alkanes of roughly equal mass because they form strong H-bonds. i. They have pKu values in the 2-5 range ii. They represent the end point of oxidation for most molecules

b)
t

l f

They form more reactive derivatives by converting the hydroxyl group into a better leaving group i. SOC12 converts a carboxylic acid into an acyl chloride, heat converts two carboxylic acids into an acid anhydride, and an alcohol in acid converts a carboxylic acid into an ester ii. Carboxylic acids can be made by complete oxidation of a primary

i
I

alcohol or aldehyde or by hydrolysis of acid derivatives such esters, acid anhydrides, acid halides, amides, and nitriles

as

Carboxylic Acid Derivatives (Compounds with a carbonyl group and functional group other than OH at an end of the carbon chain) a) Esters (OH group of a carboxylic acid is replaced by an OR group) b) Lactones (Cyclic esters) c) Acid Anhydrides (forms from the dehydration of two carboxylic acids) d) Acid Halides (OH group of a carboxylic acid is replaced by a halide) e) Amide (OH group of a carboxylic acid is replaced by an amine)

Carbonyl Reactivity

a) b) c)

They are electrophiles that can be attacked at the carbonyl carbon by nucleophile, forming a tetrahedral intermediate Strong bases deprotonate them at the cr-carbon to form a nucleophilic C

Theyundergooxidation-reductionchemistry

i. ii. iii.
Copyright

Oxidation: gain of bonds to O and,i or loss of bonds to H Reduction: loss of bonds to O and/or gain of bonds to H Oxidizing agents are rich in O and reducing agents are rich in H by The Berkeley Review

49

Exclusive MCAT Preparation

Organic Chemistry
Name Reactions

Carbonyls and Alcohols

Section Sumrrary

1.

For the reactions rn the MCAT student Manual, know the basic reaction a) Grignard Reaction (addition of an alkyl magnesium halide to a carbonyl) i. Adds one R-group to aldehydes and ketones, forming alcohols ii. Adds twice to carbonyls with a leaving group, forming a 3" alcohol

b) Aldol Condensation (deprotonation

of the c-carbon of an aldehyde or a ketone and the subsequent addition of the anion to the carbonyi carbon of either a second aldehyde or ketone) i. Forms an c[,8-unsaturated ketone or an cr,B-unsaturated aldehyde ii. Exhibits kinetic versus thermodynamic preference with ketones iii. Has a biological application in the fourth step of glycolysis

c)

Claisen Condensation (deprotonation of the s,-carbon of an ester and the addition of the anion to the carbonyl carbon of a second ester)

i. ii. d)

Forms a B-ketoester Has a biological application in fatty acid synthesis

Transesterification (conversion of one ester into another by exchanging the alkoxy grol;lp, in either acidic or basic conditions)

e) wittig Reaction (conversion of a ketone into an alkene via @3p=CR2) 0 Pinacol Rearrangement (conversion of a vicinal diol into a ketone in acid) g) Iodoform Reaction (conversion of a methyl ketone into a carboxylic acid) h) Wolff-Kishner Reduction (conversion of a ketone into an alkane in base)
Synthesis

1.

b) Protecting groups are used on reactants with multiple functional groups c) -oH group is protected as -orMS and c=o group is protected as C(oR)2 2. 3. 4.
Reactions of Acetoacetic Ester (Atpha-proton has a pKu around 1L) a) Great nucleophile when deprotonated, so it readily attacks C=O b) \rvhen treated with acid water and heat, it hydrolyzes and decarboxylates Reactions of Malonic Ester (Alpha-proton has a pKu around 11) a) Readily adds an R-group at the methylene carbon

synthesis Thought: 1) changes, 2) retrosynthesis, and 3) choosing reagents a) Retrosynthesis (product-to-reactant analysis of changes in a compound)

Can cyclize with a reactant with two nucleophilic sites (i.e., barbiturates) Decarboxylation (observed with B-keto acids or 1,3 diacids)

b)

a) b) 1.

Proceeds through a six-membered ring held together by an H-bond Forms an enol which equilibrates with the carbonyl compound

Carbonyl Biochemistry Oxidation and Reduction in Biochemistry Catabolism (oxidative breakdown of bioiogical molecules) i. Occurs mostly in the mitochondrial matrix (i.e., B-oxidation) b) Anabolism (reductive build up of biological molecules) i. Occurs mostly in the cytoplasm (i.e., gluconeogenesis)

a)

2.

BiochemicalReagents

a) NAD+/NADH are a redox pair nhere NADH is the reduced form b) FAD/FADH 2 ate a redox pair rvhere FADH2 is the reduced form c) Acetyl Coenzyme A (Picks up acetvl group to form thioester)
Copyright
@

by The Berkeley Review

50

The Berkeley Review

Carbonyls
and

Alcohols
Passages
14 Passages
I OO Questions

Suggested schedule: I: After reading this section and attending lecture: Passages I, lV, V, X, & XI Qrade passages immediately after completion and log your mistakes.

II: III:

Following Task I: Passages II, VI, IX, & XIII (26 questions in 34 minutes) Time yourself accurately, grade your answers, and review mistakes.
Review: Passages III, VII, VIII, XII, XIV & Questions 95 - IOO Focus on reviewing the concepts. Do not worry about timing.

Passr

c=ffi6
I.

ffiH

ffiftdHdl#"$tfi 5$ffi
(1 - 6) (7

-{
or_qaLl

the al
iradid
alcoh,:

Alcohols, Acidity, and Nucleophilicity

II.

Alcohol Reactions
Acetals and Ketals Carbonyl Reactivity Study

12)

III.
IV.
V.

(15 - 18)

(te - 25)
(26 - 52) (33 - 3e) (4o - 46) (47 - s3) (54 - 60)

Thermodynamic versus Kinetic Control

VI.

Alcohol Oxidation Unknown OxygenContaining Compounds


Wolff-Kishner versus Clemmensen Reduction

VII.

VIII.
IX. X.

Crignard Reaction

Aldol Condensation and Alpha Hydrogen


Claisen Condensation Reaction

(6t - 67)
(68 - 73) (74 - Bo)

XI.

XII. XIII.
XIV.

Transesterification Reaction
Malonic Ester Synthesis

(Br - 87)
(BB - e4)

Carboxylic Acids
Questions not Based on a Descriptive passage Carbonyls and Alcohols Scoring Scale Raw Score
MCAT Score

(e5 - lOO)

\
m
ttl

85 - IOO

t5-15
LO-12
7

Di

65-82
46 64

-9

JJ-4b

4-6

|-32

t-5

Passage

(Questions 1 - 6)

3.

Alcohols are a common protic solvent used in many rrganic reactions. Alcohols have organic properties (due to ,he alkyl chain), and they exhibit hydrogen bonding (not :aditionally thought of as an organic property). Low-weight .lcohols are the only organic solvents that can be used to

Which of the alkoxides is the BEST nucleophile when undergoing a substitution reaction with methyl
bromide?

,:mulate aqueous conditions while dissolving organic


: rmpounds. They are not an ideal solvent, because they can :.act as a nucleophile, as an acid, or as a base. Alcohols are -rosen in many reactions involving esters as a reactant, '; rere the ester leaving group is the conjugate base of the

A. H3CCH2ONa B. HjCCCI2ONa C. HsCeONa D. para-ClH4C6ONa

.-:ohol. This is

done

to eliminate the chance for

Which of the following phenols is the strongest acid?

:rn sesterification reaction.


In some reactions, alcohols serve as both the reactant and ::.e solvent. For instance, when an alkoxide is used as a base

'. it is in elimination reactions), it is convenient to use the ::njugate acid of the alkoxide as the solvent. This is done, ::Jause the conjugate acid (an alcohol) is formed when the ...oride is protonated. One drawback to alcohol solvents is ::ir relatively high boiling point, due to hydrogen bonding. ,1"1en the solvent has a high boiling point, it is hard to - stil1 the solvent away to isolate the product mixture. Of :anic compounds with equal carbons (and therefore roughly
.:ual molecular mass), alcohols and carboxylic
::rerally have the highest boiling and melting points.
acids

A. para-ClC6HaOH B. para-O2NC6HaOH C. para-H3COC6HaOH D. C6H5OH

The pKu of H3COH is 15.5, and the pKu of CI3COH is 11.2. Which of the following values is the BEST
approximation fbr the pKu of F3COH?

. B.

t9.4
13.8 11.8 10.4

The pKu of methanol (H3COH) is 15.5, while the pKu of phenol (HSCOOH) is 10.0. Which of the following
statements are true?

D.

c.

I. II. m. IV.

Phenol is less acidic than methanol due to the electron donation from the benzene ring through
resonance.

In which of the following reactions would ethanol


the solvent of choice?

be

Phenol is more acidic than methanol due to electron withdraw by the benzene ring through
resonance.

A.

HrC

Methanol is more acidic than phenol, because of


electron withdraw through the inductive effect.

H:N
B.

\ - ,.frt
H

Methanol is less acidic than phenol, because of


electron donation through the inductive effect.

A. I and IV only B. II and III only C. I and III only D . II and IV only

(HjC)2CHMgBr +

H:C
C.

4
CH2CH3

: . Which of the following bases will


methanol completely?

HjCH2CH2C deprotonate

A
ll

CrO3/FI2SOa

->
r. KOEI

D.

A. H3CCO2Na B. NH3 C. NaOH D. H3CCH2Li

ll

H.cgocH2cH3ffi

,-rpyright @ by The Berkeley Review@

53

GO ON TO THE NEXT PAGE

Passage

ll

(Questions7

12)

Which statement is NOT true about alcohols?

nr
@mfr

better nucleophile than a secondary alcohol, which is in turn a better nucleophile than a tertiary alcohol.

Alcohols are useful reagents in a vast number of organic reactions, most often used as a nucleophile. Their ,"u"tiuity as a nucleophile correlates directly to steric hindrance. For instance, because of steric hindrance, a primary alcohol is a

. B.
A
C

Alcohols cannot be oxidized.

. .

exhibit less steric hindrance.

offorming hydrogen bonds. Primary alcohols are better nucleophiles than secondary and tertiary alcohols, because they
capable

Alcohols are among the f'ew organic compounds

iliml

Alcohols do not readily react as electrophiles, because the hydroxyl group is a poor leaving group. To increase the electrophilicity of an alcohol, the hydroxyl group is treated with a Br6nsted-Lowry acid or a Lewis acid to form a better leaving group. Once a hydroxyl group is protonated to form a water molecule, it becomes a stronger leaving group that can be displaced by weak nucleophiles. A hydroxyl group can also be made into a better leaving group by converting it into a tosylate group according to the reaction in Figure 1.

infrared spectroscopy between 3250 and 360b cm-1.

Alcohols can be identified by their broad peak in

,& h
wnmm l1milnr

iM

hm

10. Which

A. B. C. D.
I 1.

of these alcohols can be oxidized to a ketone? H3CCH2CH2OH (H3C)2CHCH2OH

,&sE

H3CCH2CHOHCH3 (H3C)2COHCH2CH3

&/

t[@

o
RoH +

.r-fi-G
o
Figure

CH3

_->

RoSo2c6HacHi

What is the IUPAC name for the following compound?

OH 1 Tosylation of an alcohol

CI

present. The elimination reaction is most favorable with tertiary alcohols under both basic and acidic conditions,
because the alkene product is highly substituted in that case.

dehydration reaction is commonly known as an elimination reaction. Elimination reactions compete with substitution reactions in most cases where an alcohol nucleophile is

at elevated temperatures, which yield an alkene. A

After a hydroxyl group is converted into a better leaving group, the compound readily undergoes dehydration reactions

rfu
CH2CH3

uhl

lh@[ WaT

A. B. C. D.
1

4-chloro-3-ethyl-2-pentanol
2-chloro-3-ethyl-4-pentanol 4-chloro-3-ethy1-sec pentanol 2-chloro-3-ethyl-sec-pentanol

f q d
td

complications with the competing elimination reaction,


which proceeds by an E2 mechanism.
7

with an alkoxrde acting as the nucleophile also

after it is deprotonated. A nucleophilic substitution reaction

The nucleophilicity of alcohols is enhanced by treatment with a strong base, which converts it into an alkoxide anion
has

2.

t q

Which of the following srrucrures represents the product after isopropanol reacts with tosvlchloride?

A.
CH:

"

undergo an E1 elimination reaction?

Which of the fbllowing alcohols would MOST readily

CH:

C.
OH

D.

a
HsC

C.

H3c_il
D.

lt

OH

/' O- CH \

CH.

f,

CH:

"

Which of the following alcohols will MOST readily


undergo transesterification with ethyl acetate and H+?

A. B. C. D.

H3CCH2CH2CH2CH2OH
H3CCH2CH(CH3)CH2OH H3CCH2CH(OH)CH2CH3 (H3C)2COHCH2CH3
54

Copyright @ by The Berkeley Review@

GO ON TO THE NEXT PAGE

Passage
)unds

lll

(Questions 13

18)

13.

What is the product for the following reaction?

than they
ak
1n

cm -l

e?

-:niketals are formed by adding an alcohol to either an :-:ehyde or a ketone in the presence of a base. Acetals and r j:ais do not react at high pH and cannot be formed under ..ic conditions, which makes acetals and ketals ideal :::tecting groups for the aldehyde and ketone functionality in -:;-'tions carried out under basic conditions. Figure 1 shows : :eneric reaction forming a ketal protecting group.

Acetals and ketals are formed by adding excess alcohol to (which forms an acetal) or a ketone (which '..her an aldehyde ,,r:ns a ketal) in the presence of an acid. Hemiacetals and

(X
A. B. C. D.
Acetone Formaldehyde

oH.(aq)

----+

|,2-Ethylene diol No reaction transpires, so no product is formed

A
r3tOh

14.
vicinal diol

What is the product for the following reaction?

ketal

water

Figure

Formation of a ketal from a ketone

x"D
A.

H*Hzo

und?

The ketone can be regenerated by adding water to the as shown by the reverse reaction in Figure 1. The '--ilibrium is shifted back to the ketone by adding water. , -: ketal must be broken under acidic conditions. Figure 2 , '"-. s the acid catalyzed conversion of a ketal into a ketone.

,:..i.

B.

x":.

x": .

11+

:.o

-tx"-

-L-

."><".:"

oduct

.'&)o' ==::- .)<i'OH OH

n
IH

>"."D >".::O
H

OH

\^r

15.

What is the product for the following reaction run under


acidic conditions?

Hov.'VoH *

H.c

A.r,-*

-Xi,-"
J t,,
OH

'-."k:'
o*'H

'

x")

.::x)

'-x: ":oA.:
,H o*o
I :zure

uo&oH
+
HjO+

'x"l
vicinal diol?

'::>o
a

.A* - *A*
2 Acid

16. Which of the following would be added to protect


A

catalyzed conversion of a ketal into a ketone

The ketal protecting group is commonly employed in ';r:: ,ions such as the Grignard reaction, where the Grignard

could attack the carbonyl carbon. Because the - - ::rard reaction is carried out under basic conditions, the ...t remains intact during the course of the Grignard .::::ion, until it is removed at the end. I,GE

'::iint

. B. C. D.

Acid and an ester


Base and an ester

Acid and

ketone

Base and a ketone

:', nght @ by The Berkeley Review@

55

GO ON TO THE NEXT PAGE

17.

The following molecule has what functionality?

Passage lV (Questions 19 - 25)


The reactivity of a carbonyl compound is correlated in a linear fashion to the acidity of the conjugate acid of the leaving group. Generally speaking, the more acidic HX is,

It:

("D
A. Ketal B. Acetal C. Hemiketal D. Hemiacetal

the more reactive RCOX

predict the reactivity of a carbonyl compound using pKu values. Figure 1 shows a generic carbonyl substitution reaction where the X is a variable leaving group.

is.

This relationship allows us to

/.. R
18. Mixing methanol, sodium methoxide, and acetone resuits in the formation of which of the following
compounds?

___+ XR
1

NaOCHT

HOCH3

+XOCH3

!t-

Figure

Generic carbonyl substitution reaction

A. B. C. D.

Ketal
Acetal

Hemiketal
Hemiacetal

groups from halides to carboxylates to alkoxy groups. The relative reactivity can be quantified by comparing reaction rates of various carbonyl compounds. When comparing the relative reaction rates of compounds, a standard is selected. For this
study, a researcher chose the reference reaction in Figure 2.

The leaving group can be a variety

of

I I

NaOCHT

HOCH3

Figure

"raA OCH2CH3 2

HsC

ocH3

Reference reaction for carbonyl substitution study

A reactivity number can be calculated for a given leaving group by comparing the log of the reaction rate with the log of the reaction rate for the reaction with a reference leaving group, such as the ethoxy leaving group. The reactivity number is derived by means of the following equation:

It

B=log

kx
kreference

ffi

Equation

k* is the rate constant for the reaction, and k."1"."n." is the rate constant for the reference reaction with an ethoxy leaving group. Table l shows the pKa values along with reactivity numbers for a
series of X's (leaving groups):

where B is the reactivity number,

o-

cl-

HX
HI
HCI H3CCO2H C6H5SH HCN H3COH

PKa -11.2
-1.O 4.8
1.8
9.1 15.5
1

I
C1

2.8 2.5

H3CCO2C6H5SCNH3CO-

t.l
l
l
0.3

0.2

Table

Copyright @ by The Berkeley Review@

56

GO ON TO THE NEXT PAGE

19. Formic acid has a pKa of 3.78. What b value is


expected for methylethanoate
?

4.

Which of the following types of carbonyl compounds is


the LEAST reactive?

lin
X

'f the
is, us to PKa

a.

2.1
1.8

B. 2.0

c.

D.

0.3

A. Ester B. Anhydride C. Amide D. Acid halide

ution

X-

f 0. In the reference reaction shown in Figure 2, the other


organic product is:

25. Which of the following relationships accurately


describes the relative reactivity
compounds listed?

of the carbonyl

from
lative
les

of

A. B. HOCH2CH3 C. H2O D. H3COCH2CH3


-OCH2CH3

lative n this
:2,

A. Anhydride > Acetoyl iodide > Ester. B. Acetoyl iodide > Anhydride > Ester. C. Ester > Anhydride > Acetoyl iodide. D. Acetoyl iodide > Ester > Anhydride.

I I . What b value
CH:
study

is expected when

X- is bromide?

A. 1.9 B. 2.4 c. 2."7


D.
2.9

aving
re 1og

aving

;tivity
For the last example in Table 1, the leaving group is H3CO-. This is also the nucleophile. How could you
carry out this study?
nstant

or the

rble

-{. By using 24Na in rhe NaoCH3 B. By using a carbonyl with 18O labeling C. By using an R group of CD3 on the carbonyl
compound

for

D.

By using NaOCD3/HOCD3

:J.

AnegativeB-value:

-{. B.
C.

occurs when the observed reaction has a greater equilibrium constant than the reference reaction. occurs when the observed reaction is slower than
the reference reaction.

occurs when the observed reaction is faster than the reference reaction.

D.

is not possible.

AGE

*.m1right @ by The Berkeley Review@

51

GO ON TO THE NEXT PAGE

Passage

(Questions 26 - 32)

6.

Which of the following CANNOT be synrhesized from


2-pentanone using the same reaction sequence as shown in Figure 2 with any alkyl halide?

sCI-

Hydrogens on alpha carbons adjacent to a single carbonyl functional group have pKu values between li and 19.

Hydrogens on an alpha carbon adjacent to two carbonyl functional group have pKu values between 11 and 13. This means that alpha hydrogens can be deprotonated with the
addition of a strong base.

A. 4-Heptanone B. 3-Ethyl-2-pentanone C. 2-Heptanone D. 3-Ethyl-2-hexanone

Upon treating a ketone with one equivalent of a strong base, a small amount of enolate is formed. The enolate exists in the resonance shown in Figure 1 below:

:o:
il

R-zc\..o CHz
Figure

:ii:o

27

What is the percentage of 3-hexanone formed at using the same reactants shown in Figure 2?

.lt-

ffi

R/- \a",

Resonance forms of deprotonated ketone

. 80Vo 3-Hexanone B . 65Vo 3-Hexanone C. 35Vo 3-Hexanone D, 20Vo 3-Hexanone


A

ffi

dt

The resonance structure on the left in Figure 1 (the one carrying a negative charge on the alpha carbon) is a strong nucleophile, capable of undergoing an 5512 reaction with an alkyl halide, resulting in the addition of rhe alkyl group to
the alpha carbon.

c
2

8.

Enolates react in Michael addition reactions as

researcher carried out a study on the effects of temperature on the reaction of an asymmetric ketone. The
researcher chose 2-pentanone and methylated it according to the reaction shown in Figure 2 below:

1.

2.o

LiN(CH3)2

"..4.n , .-_..>

=A*

H.cAcr, o

ll
CH2CH2CH

,. LiN(cH3)2

3. Workup

(A_

H.rC

2-cHrl*A-B

Which of the following products can be formed using the kinetic enolate of 2-butanone in a three-step Michael addition reaction with H2C=CHC(O)CH:?

h ill, 0.

Figure

Methylation of 2-pentanone

The reaction was carried out at five different temperatures and the percentages of the two products, A and B, were recorded for each trial. Table I below shows the percentages ofthe products and the corresponding reaction temperature;

A. B. C. D.

6-Oxo-2-octanone 3-Methyl-6-oxo-2-heptanone 5-Oxo-2-heptanone 6-Oxo-2-heptanone

Temperature
-78'C -33'C

Product A
93.6Vo

Product B
6.4Va

68.1% 4l.3Vo
21.9Vo
8.47a

29.

3I.9Vo
52.17a

0"c
+25"C

bulky. If the experiment were repeated using


LiN(CH2CH3)2 instead of LiN(CHj)2, what would
observed?

The kinetic intermediate is preferable when the base is


be

72.lVo
91.6V0
1

+50'c

The percentage of Product A would be greater at all


temperatures.

Table

At lower temperatures, the base deprotonates the

B.
less

The percentage of Product


temperatures.

A would be less at all

hindered alpha proton, because the transition state is of lower energy. As the temperature of the system is increased, the

C. D.

reaction tends to form the more stable intermediate. An equilibrium exists between the two intermediates. As the
temperature is increased, the percentage of the thermodynamic intermediate increases. This shift in equilibrium explains the change in product distribution with temperature. The more stable intermediate leads to the more substituted enolate.

The percentage of Product A would be greater temperatures below -4"C, and it would be less
temperatures above -4'C.

The percentage of Product

A would be less

temperatures below -4"C, and it would be greater temperatures above -4"C.

Copyright @ by The Berkeley Review@

58

GO ON TO THE NEXT PAGE

rom
own

,i

U. Product A is the result of the methyl iodide reacting


with which intermediate?

Passage Vl (Questions 33 - 39)


Alcohols can be oxidized into a carbonyl (aldehyde or ketone) and/or a carboxylic acid upon the addition of an oxidizing agent. The reaction of alcohols varies with the substitution of the alcohol. For instance, tertiary alcohols do not oxidize. Figure 1 shows a guideline of reactivity for the
alcohols.

A. B. C. D.

The transition state intermediate. The kinetic intermediate. The more stable intermediate. The thermodynamic intermediate.

l'Alcohol
r5"c

RCH2oH [ox] >


R

: n. When treating a ketone with a small base,

the thermodynamic intermediate is preferred. What is the major organic product when 2-methylcyclohexanone is irst treated with potassium hydride at 40'C followed by treatment with ethyl iodide?

R2cHoH [ox] >


2"

Alcohol

A A
o

---------> ,\ HROH

[ox]

.\

. 2-Ethyl-2-methylcyclohexanone B. 2-Ethyl-6-methylcyclohexanone C. 3-Ethyl-2-methylcyclohexanone D . 2-Ethyl-3-methylcyclohexanone

3'Alcohol
Figure

R3coH [ox] >


1

No Reaction

Synopsis of oxidation fbr various alcohols

)ws:

\\hich of the following reactants offers no competition


r,-tween the thermodynamic and kinetic enolate? R

The [Oxl listed in the reactions above represents an oxidizing agent. Because an oxidizing agent gets reduced, it must start in a high oxidation state. Oxidizing agents can be recognized by their abundance of oxygen. In the table below are some common oxidizing agents and their yields in the
oxidation of various alcohols:

rt
)

using
:-step

. B. C. D.
.\

2-methyl-3-heptanone

2,2-dimethyl cyclopentanone
2-butanone

Oxidizing Agent
KMnO4/OHPCC

Alcohol
2-pentanol

Yield
81.]Vo
13.9V0 65.2Va 87.4Vo

2,3-dimethyl cyclopentanone

l-hexanol
2-butanol 2-pentanol
1

CrOj/H2SOa
Na2Cr2O7/H2S04 KMnOa/OHK2Cr2O7|H2SO4

-butanol -butanol -propanol

92.37o
52.3Vo 45.6Va 97.2Vo

l
1

Cu/CuO/H2S04 H2CrO4
rase is

Cyclohexanol

using
uld be

In the absence of water, primary alcohols can be converted into aldehydes. The lower yield in the reactions
involving sulfuric acid is attributed to the formation of
an

r at all
,

alkene side product. The choice of oxidizing reagent can be made based on the sensitivity of the alcohol compound to either acid or base. While the alcohol itself may not be pH-

at all

sensitive, other functionalities on the molecule may be sensitive to the pH of the solution.
3

ater at
less at

3.

What is the change in oxidation state experienced by the reactive carbon in the oxidation of a secondary alcohol?

less at
)ater at

A. From O to +2 B. From -2 to 0 C. From}to-2 D. From +2 to 0 :


:rsht
@

AGE

by The Berkeley Review@

59

GO ON TO THE NEXT PAGE

4.

What is the major organic product and its approximate yield following the treatment of 2-hexanol with aqueous permanganate solution at a pH of 10?

9.

In the following reaction, where R


oxidation and reducing agents?

CH3, what is t

A. B. C. D.

oHoHoo

92.3Vo 2-hexanone

9l.lVo2-hexanone
73.9Vo 2-hexanone 92.3Vo pentanal

*U*+
A.

riQ

-* *A-[*

+ri(oH

Oxidizing agent: C5H1 2O2; reducing agent: TiO2.

5.

Which of the following alcohols


acid?

will NOT undergo oxidation upon treatment with chromic oxide in sulfuric
2-methyl-3-hexanol 2-ethylcyclopentanol 3,3-dimethyl-1-pentanol 2-methyl-2-butanol

B. Oxidizing agent: C5H16O2; reducing agent: TiO2. C. Reducing agent: C5H12O2; oxidizing agent: Ti D. Reducing agent: C5H16O2; oxidizing agent:

A. B. C. D.

6.

Addition of which of the following alcohols to orange


Na2Cr2O7

will turn the solution

green?

A. 2-methylphenol B. 1-methylcyclohexanol C. 3-ethyl-3-heptanol D. 2-rnethyl-1-octanol

7. 'possible? the following oxidation reactions is NOT Which of A. 1-butanol to butanone B. 2-methyl-1-pentanol to a carboxylic C. l-pentanol to an aldehyde D. Cyclohexanol to a ketone
acid

8.

How does the oxidation state of chromium change in the treatment of chromic oxide in sulfuric acid with
ethanol and 2-methyl-2-butanol?

A
B

. . . .

The oxidation state of chromium increases with the addition of both ethanol and 2-methyl-2-butanol. The oxidation state of chromium decreases with the addition of both ethanol and 2-methyl-2-butanol. The oxidation state of chromium increases with the addition of ethanol and remains constant with the

addition of 2-methyl-2-butanol.

The oxidation state of chromium decreases with the addition of ethanol and remains constant with the addition of 2-methyl-2-butano1.

Copyright @ by The Berkeley Review@

60

GO ON TO THE NEXT PA

t is the

Passage

Vll

(Questions 40 - 46)

1.

Which of the following structures is NOT possible for


the formula C6H120?

i(oH)z

A researcher wants to distinguish and identify three -rknown isomers (Compound A, Compound B, and lrmpound C), all of which have the molecular formula, -<H12O. To help distinguish the structures from one
.:l,rther, the researcher performs some standard reactions used

TiO2.

TiO2.

TiO2. TiO2.

qualitative analysis tests for organic compounds. To -,-mpounds A, B, and C, she adds Jones's reagent ,-rO3/H2SO4). Only compound A turns green upon the .:lition of Jones's reagent. Compounds B and C remain r.nse in color for an extended period of time. The researcher - :lcludes that only Compound A is an oxidizable compound. *-, three compounds maintain a brown color when treated .,: Br2 liquid in the presence of CCl4 solvent, indicating - r' the brown bromine liquid did not react with any of the --:.nowns. The bromine liquid tests for the presence of any
-*: -rrjugated alkene n-bonds.

A. Cyclic alcohol B. Linear ketone C. Linearaldehyde D. Cyclic ketone

2.

Which of the compounds summarized NOT correctly identified?

in Table

is

A . Compound A is a primary or secondary alcohol B. Compound B is a methyl ketone C. Compound C is an aldehyde. D . None of the compounds can be a cyclic ether.

- :::e. The precipitate is silver chloride, formed when ' , ide anion is displaced as a leaving group during a
,

- .:pound A, after being treated with strong base, yields a :-:;:pitate when added to AgNO3 in water solvent. This is , : . ,r, n as Tollen's test, which indicates the presence of a 'r,:llv oxidizable carbonyl functional group. Compound A -s a precipitate with H3CCI in the presence of silver

Only Compound B reacts with 12 in basic medium to .:r- a yellow oil at the bottom of the flask (the iodoform ';. used to test for the Z-keto functionality). Only

43. Which of the following compounds CANNOT


oxidized?

be

A. B. C. D.

Primary alcohols
Aldehydes Esters

Hemiacetals

- : rphilic suhsritution

reaction.
44

The product of the reaction of Compound A with Jones's '::irtlr. was isolated, collected, and tested with blue litmus "r.;r The oxidized derivative of Compound A turned the

All of the following are true for a compound that can be used to test for an aldehyde EXCEpT:

lllmus paper red immediately. The information from :,: .;nous tests gave clues as to the type of compounds that

-:

A.

it undergoes
it is reduced.

a color change or phase change when

.-- -nknown was likely to be. The researcher following l::: .istS was able to conclusively identify the functionality
..:h unknown.
.

,,

-,r Compounds

Table 1 summarizes the results of each A, B, and C.


s

B. it is a compound that is rich in hydrogen atoms C. it is a transition metal in a high oxidation srate. D . it is in lower concentration than the aldehvde.
5.

Test -+

Jones's I2lKOH(aq) Tollen


Negative Negative Negative

RX/Ag+
Positive
Negative Negative

:.:ound A Positive

Which of the following is NOT an oxidizing agent?

Positive
Negative Negative

.lund B -:rund C

Positive
Negative

A. KMnO4 B. H3CCO3H
C. Mg(oH)2 D. HIOa

Table

.in
:-;

alcohol, when oxidized, forms a compound that .rrns blue litmus paper red. In terms of substitution,
alcohol is best described as a:

6.

A. primary alcohol. B. secondary alcohol. C. tertiary alcohol. D . primary alcohol or secondary alcohol.

Which of the following reagents can reduce an ester to primary alcohol?

A. H2lPd B. HCllZn C. LiAlHa D. BH3

PAGE

': ,:,,rht O by The Berkeley Review@

6r

GO ON TO THE NEXT PAGE

Passage

Vlll

(Questions 47 - 53)

0.

Wcilff-Kishner reduction converts either a ketone or an aldehyde into an alkane. The two-step process involves the
formation of a hydrazone (the hydrazine derivative) which is reduced under basic aqueous conditions. Figure 1 shows the Wolff-Kishner reduction of a generic ketone.

What accounts for the greater acidity associated with proton on H2N-N=CR2 than a proton on ammonia?

lPass
a

A. B.
C

{a

The hydrazone has greater steric hindrance than


ammonia, which increases the acidity.

11.e,:r
lmE

The hydrazine molecule becomes aromatic upon


losing a proton.

;:I

:tLTe :ri

*A*
ll *A*
*-*t
Figure

ll

H2N-NH2

__-____+.4.
5

*-

n"'

:;:r:e.t;l

Ammonia has less


hydrazone.

of an inductive effect

than

!m0)r

D.
1.

:tffe

-r*,1

The lone pair formed upon deprotonation from the


hydrazone is stabilized by resonance.

Tllrir
Lrna'rm:r,

What product is expected when the following molecule is treated with hydrazine and basic water?

oHHzo

HH

*x*
H2o

+Nz

HO

Wrjlff-Kishner reduction of a ketone

Reduction of a carbonyl into alkane can also be carried out under acidic conditions using Clemmensen reduction.

Zinc metal provides the electrons for the reduction, as shown

in Figure

2.

A. 3-Ethylcyclohexanol B. 3-Hydroxyethylcyclohexane C. Ethylcyclohexane D. 3-Ethylcyclohexanone


52.
Under what conditions is Wrilff-Kishner reduction
prefered over Clemmensen reduction?

oHH

*4.
47

#-*
2

ZznCt2

fu

Im;
r,o,lnl

*X*+
all of the following
compounds

gmm

unr

r(@lmhuMwrl

Figure

Clemmensen reduction of a ketone

When the reactant has an acid sensitive functional


group.

The reactant in a Wrilff-Kishner reduction reaction could

B. When the reactant has an amine sensitive


functional group.

also react with


EXCEPT:

C. When the reactant


group.

has a base sensitive functiond

W.

A. B. C. D.
4

LiAlHa
H3CMgBr(et2o)
O31H2O2

When the reactant has a water sensitive functiond


group.

Wld

HCllZn

3.

What is the product of 2-methylcyclopentanone phenyl hydrazine, which reacts like hydrazine?

m.'
mn

8.

Over the course of a Clemmensen reduction of a ketone, how does the oxidation state of the carbonyl carbon
change?

A.

. B" C. D.
A
49

It increases from -2 to +2. It decreases fiom +2 to -2. It decreases from 0 to -2. It decreases from +2 to 0.
the Wolff-Kishner reduction of a

B. Ph \\ ,zNNN- H
H:C

Ph

-zN-

oH
m

H:C

*
C.

t *
iffilmm

" What is true following


ketone?

*a
*

nn

D.

HH
H:C

(lro{
rltmDnm

. One new chiral center is formed. B . The hybridization of carbon goes from rp2 to sp3. C . Both nitrogen and carbon are reduced.
D

H:C

,dh{

Three net water molecules are formed as a product.

{
GO ON TO THE NEXT PAG

mn$

Copyright O by The Berkeley Review@

62

Passage lX (Questions 54 - 60)


Carbonyl compounds are reactive with nucleophiles. Their electrophilicity can be attributed to the polar nature of :e carbonyl bond which houses a partial positive charge on
::'rrelated to the pKo of the conjugate acid of the best leaving roup on the carbonyl carbon. This is to say that the better :,; leaving group the more reactive the carbonyl compound. l.:le 1 shows the pKu values of conjugate acids of various ..r'ing groups on carbonyl compounds:

54. Which of the following

compounds

is the LEAST

reactive carbonyl compound?

-:e carbon in the carbonyl. Carbonyl reactivity can be

A. B. C. D.

An ester.

A thioester.
An anhydride. An acid chloride.

55.

To make 2-pentanol, it would be best to use which of


the following reaction sequences?

PKa
'74

Acid
HCI H5C6CO2H H3CCO2H HCN H3CH2CSH

4.2
4.8
9.1

A. B. C. D.

H3CMgBr H3CMgBr H3CMgBr H3CMgBr

+ pentanal. + 2-pentanone. + butanal. + butanone.

10.5

t5.1
33.1

H:CHzCOH

NH:

6.

Which of the following Grignard reactions would form


stereoisomers as the product?

Table

h
iir

: -u;leophile is also considered. A Grignard reagent is a :,:,: nucleophile that is selective in its reactivity with " r,,::r1'ls. Three Grignard reactions are shown in Figure l. Reaction I (Grignard reaction with an aldehyde)

addition to the leaving group in a carbonyl reaction,

A. Butanone + ethyl magnesium bromide. B. 3-Pentanone + methyl magnesium bromide. C. Ethyl acetate + 2 equiv. ethyl magnesium bromide. D. Propanal + methyl magnesium bromide.

oR'OH

nAs

ll

n'rvrgxer,g A.l l. NFlcltaqr


l.

57.

nAn

When treated with RMgBr, the following compound would yield which of the following products?

Reaction

OR'OH

II

(Grignard reaction with a ketone)

nAn

ll

r.
2.

n'r,rgx.r,g
NFlaClraqr

o...1

nAn

Reaction

III

(Grignard reaction with an ester)

Requires two equivalents of R'MgBr

R'Mgxer,g U...fn 2. NFIaCkaqr X nA n,


l.
;|unrme

A. A primary alcohol. B. A secondary alcohol. C. A tertiary alcohol. D. A ketone.

o/

H.9

Grignard reactions with three different carbonyls

8.

A Grignard reagent adds how many times to an ester?

,itttll

reagents react most readily with acid halides and i" .-:', drides (to which two equivalents of the Grignard can

::::nard

illrrr,,Li l3n-snard reagents

do not react with amides. Despite

liiil': r:3ngrh
.ri:lr..:

ri:,: jlsplace an amine leaving group. These observations Luru"j i''l :.'rord with the acidity of the conjugate acid of the rLllr:ii. r.: ,5oups. This information can be used to predict the rrltrirl:--:t I product formation and the favorability of a carbonyl
It(llil]ill]l'

of the Grignard reagent as a nucleophile, it

A. B. C. D.

Once.

Twice. Three times.

It depends on the temperature of the mixture.

-i

_ -_

tlli

'rn, 1:111 O

by The Berkeley Review@

63

GO ON TO THE NEXT PAGE

9.

Which of the following conditions is NOT true about a Grignard reaction?

Passage X (Questions 61 - 67)


The Aldol condensation reaction is a main staple in synthetic organic chemistry. The reaction involves the deprotonation of a hydrogen from the alpha carbon (the
carbon adjacent to the carbonyl group) and the subsequent reactions of the carbanion as a nucleophile. The carbanion form is in resonance with the enolate form. In an Aldol condensation, the electrophile to which the carbanion is

62.

$,

. B.
A
C

Ether solvent is required for the Grignard reaction. The Grignard reaction involves the reduction of the carbonyl carbon.

D.

The Grignard reaction can be employed to form either primary, secondary, or tertiary alcohols. For the highest yield, the solution should be acidic
when the Grignard reagent is added.

60. If propanoic
anhydride?

acid has a pKu value of 4.9, what can

be

adding is a neutral carbonyl compound. The preliminary I in Figure 1) is referred to as a ./S hydroxy carbonyl. This B-hydroxy carbonyl can then undergo elimination (shown as Step II in Figure 1) to form a conjugated alkene-carbonyl compound. The conjugated
product (the product of Step
alkene carbonyl compound is referred to as an a,t3-unsaturated

B. C. D.

concluded about the reactivity

of

propanoic acid

carbonyl. The Aldol reaction is shown in Figure


Step

below.

. B. C. D.
A

It is more reactive than benzoyl chloride.

I: Formation of B-hydroxyketone

It is less reactive than N-methylbutamide. It is more reactive than methyl acetate.


It is less reactive than acetic acid anhydride.

R-

il LpAretzol *-.r\* -c- -\ -R /\ R-r"\oH


I

HH

/\ HH
Step

II:

Formation ofu,B-unstaurated ketone

o
HCrraor

*-."\o

*-.4
a

ffi,4

-,iqr:u

:: [r:

R:.\ou
I

*-.An
/\ HH

/\ HH

H
-

Figure

Generic Aldol reaction

The pK6 of an alpha hydrogen falls in the range of 11 19 depending on the number of carbonyls to which it adjacent. The hydrogens of acetone have a pKu of roughly 1 while the central alpha hydrogens of4-oxo-2-pentanone ha a pK2 of roughly 11. The pK2 information can be used predict the most reactive site on a molecule. It can also ! used to predict the degree ofreactivity for a compound.

tr..

61.

Which of the labeled hydrogens on the molecule belc


is the MOST acidic?

!{vI

g.cHucl

+ lii
CHH
I

? oc
I

t,.a-.V
Tllhu
umr'

um

d----+Il3Q
A.
B. C.
a

mdfiu

b d

fl- md l- rfir G-@


lD-"Ifil

D.
Copyright @ by The Berkeley Review@

GO ON TO THE NEXT PA

62. What is the IUPAC


ple in
es the

name for the compound that can undergo an Aldol condensation reaction with itself to form the following compound?

6.

Which of the following bases is NOT strong enough to deprotonate the hydrogen on the alpha-carbon of a
ketone?

n (the
equent banion

ooH
CH:
H

Aldol rion is
minary

A. NaNH2 B. NaH C. Na2CO3 D. NaOCH2CH3

H:C

CHI

CH:

toasa
Ln

then
a

form

iugated

A. B. C. D.

Butanaldehyde

2-Pentanone

2-Methylbutanal 2-Methylpropanal

7.

turated low.
tr
*1

Which of the following compounds CANNOT undergo an Aldol condensation reaction?

Treatment of an aldehyde with excess alcohol in the presence of an acid would lead to which of the lollowing products?

A. A ketone B. An aldehyde C. Acarboxylicacid D . All of the above can undergo an aldol reaction

A. An acetal B. A hemiacetal C. An alkene D. An ester


)

(
R H

4.

\\/hat is the major organic product isolated from the


tbllowing reaction?

of 11 rich it
:

rughly
rone ha

e used

n also
rd.

C.
H:C CH:

D.
H:C

ule be

The chemical inter-conversion from a ketone to an enol :s referred to by which of the following terms?

-\. B. C. D.

Resonance

Conjugate pairing

Hydroxyl exchange Tautomerization by The Berkeley Review@

I PAG

nght

65

GO ON TO THE NEXT PAGE

Passage

Xl

(Questions 68 - 73)

9.

The Claisen condensation involves the dimerization of


an ester. The first step is deprotonation of an ester to form an enolate, which is followed by the subsequent addition of

Which of the following ester reactants would give the LOWEST yield in a Claisen reaction?

the enolate to the carbonyl carbon of a neutral ester. The tetrahedral intermediate that is formed loses an alkoxide leaving group to yield a B-ketoester product.
The base chosen to deprotonate the alpha hydrogen of the ester should match the alkoxide leaving group so as to avoid

A. B. C. D.

H3CCO2CH2CH3

H3CCH2CO2CH2CHj
(H3C)2CHCO2CH2CH3 (H3C)2CHCH2CO2CH2CH3

complications with transesterification. This alkoxide leaving

group is an anion that is also a strong enough base to


deprotonate the alpha hydrogen of the B-ketoester product. To isolate the B-ketoester product, the solution is neutralized with a weak acid. Figure 1 shows the equilibrium mixtures in a Claisen reaction.

0. A Claisen condensation reaction


which of the following esters? A. HCO2CH2CH3

is NOT possible with

A H3CH2C

fr

ot,ct ?)
3c

B. C6H5CH2CO2CH2CH3 C. (C6H5)2CHCO2CH2CH3 D. C6H5CH2CO2CH3

o.n r."

i-"

HC+oc
I

Hzc'H r

71. Which of the tollowing is a product of a Claisen

HjCH2C

A.'A o{*^,
I

o e
CH1-

condensation reaction ofone ester upon itself?

zH'cH'cfRocH2cHj

. B. C. D.
A

H3CCH2COCH2CO2CH2CH3 H3CCOCH(CH3)CO2CH2CH3 (C6H5)CH2COCH2C O2CH2CH3 (C6H5)CH2COCH(C6H5)CO2CH2CH3

OCH2CH-1

B-Ketoestcr

H3CH2C

.A A cO

ocH2cH3

2.

What is the major organic product for the followi


reaction?

cHr
HOCH2CH3

Figure

Claisen reaction scheme

oEt

NaOEt

Without an alpha hydrogen, the last step (equilibrium


step) of the scheme above is not possible. If the B-ketoester that forms has no alpha hydrogen, then the yield is reduced because the alkoxide anion acts as a nucleophile (rather than as a base) and causes the retro-Claisen reaction. To increase the yield (from as low as zero), less alkoxide base is added, and the reaction is run at a lower temperature.
6

--*-->

HOEt

B.o

8. Which proton has the lowest pKu


hydrogens on the following B-ketoester?

value

of the

D.o

&'

oo

HjCH2C OCH2CHj

HH a

H CH'CH. b c-

U"
GO ON TO THE NEXT PA

A. B. C. D.

Hydrogen a.

Hydrogen b.
Hydrogen c. Hydrogen d.

Copyright @ by The Berkeley Review@

73. Counting

stereoisomers, how many products are possible for the following reaction?

Passage Xll (Questions 74 - B0)

EroA \-Anr, ffi


A.
2

oo ll

ll

NaOEt

In transesterification, an alkoxy group of an ester is exchanged by the addition of an alcohol in the presence of either acid or base. The equilibrium constant for the reaction is approximately one, so the reaction must be driven by the
addition of excess alcohol. A researcher carries out a transesterification reaction by adding methyl propanoate to ethanol in the presence of HCI at 25"C for thirty minutes (Reaction I). The product is isolated by neutralization and then purified using fractional distillation. The identical reaction is then repeated under reflux conditions for thirty minutes (Reaction II). The products from Reaction II are then neutralized and isolated by neutralization and then
purified by fractional distillation.

B. J C. 4 D. 6

-'\HTCH2C OCH3 o

ll

+ H3CCH2OH ;Ii

H*-

Reaction

30;,
H+-

-AH3CH2C- -OCH1

ll

+ HjCCH2OH ;ffi

30 min

Reaction

II

Figure 1 ancl Figure 2 show the 1HNMR spectra of the


major components from Reactions I and II respectively. Major component isolated from Reaction
rvrng

Figure

Reaction I's maior component

lUNVR

spectra

Major component isolated from Reaction

II

Figure

Reaction II's major component

lUNVR

spectra

'AG

;riright

by The Berkeley Review@

61

GO ON TO THE NEXT PAGE

the spectrum in Figure

The ratio of the integrals of the peaks are 3 :2 :3 for I and2l.2:3 :3 for the spectrum in

79

Figure
7

2. The 1HNMR spectra were collected

result

The presence of water in the reaction mixture would in which of the following impurities in the

in

product mixture?

deuterochloroform solvent.

4. What is the IUPAC name and formula for


compound in the 1HNMR spectrum in Figure l?

the

A. Methyl propanoate (CH3CH2COZCH) B. Methyl ethanoate (CH3CO2CH3) C. Ethyl ethanoate (CH3CO2CHZCH1) D. Ethyl methanoate (HCO2CH2CH3)
8

A. Carbonic acid (H2CO3) B. Formic acid (HCO2H) C. Acetic acid (CH3CO2H) D. Propanoic acid (CH3CH2CO2H)

0. Why must the NMR be carried out in


solvent?

deuterated

75.

What can be concluded about Reaction

ltnn4R

II from the

spectrum in Figure 2?

A. B. C. D.

To prevent reaction from occurring in the NMR


tube.

A. B. C.

Methyl propanoate, CH3CH2COZCHZ, did not


react with ethanol.

To induce magnetization into solution. To prevent hydrogens from the solvent from being
detected by the NMR spectrophotometer.

in the formation of ethyl


CH3CO2CHzCHz.
from the reulrrangement of a carbocation.

The ethyl and methyl groups exchanged, resulting

acetate,

To decrease the pH of the solution.

Propyl acetate, CH3CO2CH2CH2CH3, formed CH3CH2CO2CH2CH3, was

D. Ethyl propanoate,

generated in a greater than 5OVo yield.

6.

An ester is more reactive

as an electrophile than which

of the following compounds?

. B. C. D.
A

Acid anhydrides
Amides

Acid halides An ester is the most reactive carbonyl compound.

77.

Had ethyl amine (CH3CH2NH2) been used instead of ethanol is Reaction II, the final product would have been which of the following?

A. N-ethylpropyl amine B. l-aminopentane C. N-ethylpropanamide D. N-Hydroxypropanamine

78.

The presence of acid in the reaction mixture which of the following purposes?

A. To protonate the
decrease reactivity. increase reactivity.

ester electrophile helping to


to

B. To protonate the ester electrophile helping C. To protonate the alcohol nucleophile


decrease reactivity.

helping to helping to

D. To protonate the alcohol nucleophile


increase reactivity.

Copyright @ by The Berkeley Review@

68

GO ON TO THE NEXT PAG

aassage

Xlll

(Questions 81 - 87)

o
1.

NaOEr(HoEt)
CH3CH2O

,H3cH2oAr(ocH2cH3

2. RX(et2O)

A-J(
I

I
ocH2cH3

4",
A
o
NH

80 to 90% yield

R
70 to 85Vo yield

\ \_*
Urea

HN

65 to 75Vo yield

'i'

:::l'erent synthetic routes starting from the malonic ester, illr :ading to a carboxylic acid and the other leading to a ,uur. *r:. cvclic amide). The first route leads to a carboxylic i -: specific composition. The carboxylic acid can be an rtilrj jf-:3Ciate product that is subsequently converted into one n ,r":,::al possible carbonyl compounds, such as an ester or irrnr ,:,: The second synthesis route leads to compounds in the rltttur.i -:l-ic acid derivative group. These are useful in the ,r': [:asls of barbiturates.
rirlu,:r

.)Mr

r -:,:sis of many bio-organic compounds. Figure

Figure 1 Synthetic uses of malonic ester lrlalonic ester is a common starting reagent in the 8 2. When (CH3)2C(NH2)2 is used in lieu of urea, what is
1 shows the final product?

A'

B' Hrc.

.cHr

"NAN'
C. D.

"*X.nt
R

first part of both synthetic pathways involves the rlirrLrlr'':-,rnation of the hydrogen alpha that is adjacent to both
:uriiLu:"T

I-;

riilll

ri:'*l groups. The proton that is on a carbon that is alpha I t,'i carbonyls has a pKa value of roughly 11, so an ]1ii x-13 is strong enough to remove that proton. This

HrC HN

-X

CH" NH

a nucleophile that can react with alkyt halides, r!:si other electrophiles. In these two examples, sodium ,lrrlLlr: t,:e is chosen to prevent competition with the
rr""i-rL:.
iiillilrfl
ulriliiitLtlu

lE:s::ifi cation reaction.

H:C

CH:

Hrc

CH:

{"o

,\ hat is the structure of urea?

B.o

3.

Which of the following molecules will NOT undergo


decarboxylation when treated with heat?

",*Aot
{f_

*A *Y
I

D.o

"r*A*",
NH

^,,.j\.L",
c'oD'o\oH

B.

roUo"
T

NHz NHz

",.\o
69

o "\:r"

ri:

-:::nt O by The Berkeley Review@

GO ON TO THE NEXT PAGE

84. Why is sodium ethoxide chosen in the synthesis?


A

Passage XIV (Questions 88 - 94)


Carboxylic acids can be produced by one ofseveral ways. Carboxylic acids are useful in synthesis as an intermediate in numerous reaction pathways. Carboxylic acids are weak acids that when added to water will partially dissociate into ion.

B. C. D.

Sodium ethoxide is a strong base that will not result in transesterification of the ethyl ester. Sodium ethoxide is a weak base that will not result in transesterification of the ethyl ester.
Sodium ethoxide is stereo-specific. Sodium ethoxide is regio-specific.

their carboxylate conjugate base and hydronium

85. What signals would be observed in the proton NMR of


malonic ester?

Carboxylic acids play a distinct role in biological chemistry. because of their acid-base properties and their synthetic flexibility. Figure I shows six of rhe ways in which
carboxylic acid can be synthesized:

A. A 2H doublet, a 2H doubler, and a 3H triplet B. A lH singlet, a2Htriplet, and a 3H doublet C. A 2H singlet, a 2H rriplet, and a 3H quarrer D. A lH singlet a2Hquartet, and a 3H triplet
86. Which alkyl halide must be chosen to tbrm
following molecule when carrying out
malonic ester?
the

Grignard Reduction:
RMgBr + CO2

--*

RCO2MgBT

H+

--+

RCO2H

Oxidation of an Aldehyde:

*An.

cro3

HzSo+_

oAo"

a synthesis from

Hydrolysis of an Anhydride:

A
H:C A. B. C. D.
2-bromopropane. 2-bromobutane. 2-bromopentane. CH2CH3

.A".JLor+,*Ao,
Hydrolysis of an Ester:

*Ao*,*
Ozonolysis

Hzo

H+/L rr /d!

o
lt

R'o

*Aon*
an AIkene:

ot

of

lH

Two alkyl halides must be used; methyl bromide


and ethyl bromide.

"><_ Hydrolysis of
RCN +
Figure

t.q ,,1o,5-

_-\on*oA
*
NH+*

87.

What is the final organic product after malonic ester is rreared wirh: 1) NaOEt, 2) CH3CH2CH2I, and 3) urea?

H2o:i

H+/A

a Nitrile: O

A.o

HN

A
o

B.o

*Ao,

NH

HN

Six reactions used to synthesize a carboxylic

NH

The large number of starting reagents offers varietl, synthesis based on the availability of starting compou

The acid products can react further to form either

(through transesterification under acidic conditions) or ami (by way of acid chloride formarion using SOCI2). An a

CH:

CH2CH2CHj

is formed by treatment of either an acid halide


anhydride with an amine.
8

or

A HN
H:C

D.o
NH

A HN
H:C

8. To synthesize

synthetic pathways

NH

butanoic acid, which of the followi will NOT work?

A. B. C. D.

Oxidation of 1-butanol. Hydrolysis of ethylbutanoare.

Ozonolysis

of 4-octene, followed by

oxidari

cHs

cH2cH2cH3

rvorkup with hydrogen peroxide. Treatment of 1-bromobutane with NaCN, follou by strong aqueous acid treatment.

Copyright @ by The Berkeley Review@

GO ON TO THE NEXT PA

89

How does propanoic acid exist after it has been added to water given that it has a pKu value of 5.0?

4.

What sequence of reagents will convert an aldehyde into


an amide?

ways.
Liate r

in

.te

we8k into

A. Fully deprotonated. B. 50Vo deprotonated. C. 25Vo deprotonated. D. Less thanTTo deprotonated.

ion.

A. l. NH3 2. SOC12 B. 1. CrO3/pyridineiHCl 2. SOC12 3.NH3 C. 1. CrO3/H2SOa 2. NH3 3. SOC12 D. 1. CrO3iH2SOa 2. SOCI2 3. NH3

mistry,

nthetic

'hich

0.

Which of the following reactions does NOT result in


the formation of a compound that can turn blue litmus
paper red?

A. B. . D.
C

Treatment of 1-octanol with CrO3/H2SO4.

Treatment of 3,4-dimethyl-3-hexene with ozone followed by reductive workup with dimethyl


sulfide. Treatment of maleic anhydride with acidic water. Treatment of ethylbenzoate with acidic water.

41

What is the product for the following reaction?

H3cH2cv' cH3

1. PBr3(et2O)

2. Ms(etro)

I oH
A.

3.cQ(e)
4. H3o+/

oH AA'H
C.

B' ,

D.

rylic

ac

Which of the following reactions is NOT reversible?

variety
mpoun
her

or

An

. Hydrolysis of an ester. B. Grignard reagent plus carbon dioxide. C. Hydrolysis ofan acid anhydride. D . Hydrolysis of a lactone.

le or
u

l.

Which of the following is NOT associated with


carboxylic acid?

followi

A. B. C.
oxidati {, fol

An aqueous pH less than

7.0.

A broad IR peak between 2500 and 3000 cm-l. The loss of one peak in the 1HNMR when D2O
added to the NMR sample tube.

D. A pKu value between 9.0 and 10.5.


lrri'right
@

PAG

by The Berkeley Review@

71

GO ON TO THE NEXT PAGE

Questions 95 - 100 are NOT based on a descriptive passage

98. Which

type of compound can be reduced by NaBHa?

95. Which of the following

structures represents the intermediate for a transesterification reaction under basic conditions?

A. Phenol B. Tertiary alcohol C. Ketone D. Ether


99. Which of the following
describes the hybridization of

A.

RO-

B.

the carbon number three in 3-hexanol?

\,s *O/"ao*,,
C.

R'O-CI

il

OR"

A. B. c. D.
10

rp tp2
rp3 The carbon is not hvbridized.

D.

+C

o il
I

0.

The product of 2-butanol with chromium (VI) oxide in

sulfuric acid would have which of the following lHNuRpeaks?


Singlet (3H), doublet (2H) and triplet (3H). Singlet (3H), triplet (2H) and doublet (3H). Singlet (3H), quarret (2H) and triplet (3H). Triplet (3H), doublet (2H) and triplet (3H).

*/"\o
6.

A. B. C. D.

Esters are NOT used as a reactant in which reaction? A. Claisen condensation

B. C. D.

Friedel-Craftalkylarion
Grignard reaction Transesterification

7.

What is the major product of the reaction below?

"Oh, what a feeling;

oHA ->
A.

CHEM-IST-RY!l!',

c(" c("

B.

l.D D 11. A 16. C 21. C 26. C 31. A 36. D 41. D 46. C 51. A 56. D 61. B 66. C 71. D 16. B 81. B 86. B 91. D 96. B
6.

1.

2.D
D

12. B

11. B

22. D 27. C 32. B 31. A

42. C 47. C
52. A

57. B 62. D 61. C 72. A 17. C 82. B 87. B 92. B 97. A

3.A 8.A 13. D 18. C 23. B 28. A 33. A 38. D 43. C 48. B 53. A 58. B 63. A 68. B 13. D 78. B 83. C 88. D 93. D 98. C

4.8 9.A 14. D 19. C 24. C 29. A 34. B 39. C 44. B 49. B 54. A s9. D 64. A 69. C 74. A 19. D 84. A 89. D 94. D 99. C

10.C 15, c
20. A 25. B 30. B 35. D 40. A 45. C 50. D

s.D

s5. c 60. C 65. D 70. A 75. D 80. C 85. D 90. B 95. A


100.

Copyright @ by The Berkeley Review@

YOU ARE SO VERY FINISHE

.?

ion of

Choice D is correct. According to the answer choices, either statement I or II must be true and either statement III or IV must be true. The pKo values are given in the question, so the question centers around determining the relative acid strength of phenol and methanol. Phenol has a lower pKo, so it is a stronger acid than methanol. Vethanol is therefore a weaker acid than phenol. This makes statements II and IV the true statements, which makes choice D the best answer. Resonance is a stronger effect than the inductive effect, so statement II is more significant than statement IV. Pick D, and let correctness set you free.
Choice D is correct. For the base to be strong enough to deprotonate methanol, its conjugate acid must be a weaker acid (have a higher pKu) than methanol. Choices A and B can immediately be eliminated, because they are roth weak bases, and therefore are not strong enough to deprotonate methanol, a very weak acid. The conjugate acid of NaOH is water, which has a pKo of 1,5.7 (roughly equal to methanol), so it will deprotonate about half rf the methanol molecules. The conjugate acid of H3CCH2Li is H3CCH3, which is one of the weakest acids inor'\,n, so H3CCH2Li is definitely strong enough to deprotonate methanol. Pick D, and correct you will be. This :-'-restion can be reworded to read "which of the following bases is the strongest?", because the one base strong :rough to deprotonate methanol must be the strongest base of the choices.

'ride in lorving

Choice

A is correct. Because the electrophile has only one carbon, elimination is not possible, resulting in a :-.action where only the substitution product is observed. The best nucleophile is also the strongest base, because r e strongest base donates eiectrons most readily. Of the four answer choices, C and D can be thrown out, because --::\- are phenoxides (the conjugate base of a phenol) while choices A and B are the conjugate bases of alcohois. - :,enols are more acidic than standard alcohols, so phenoxides are less basic than standard alkoxides. Between -: rice A and B, the chlorine atoms on choice B (H3CCCI2ONa) are electron withdrawing and thus decrease the , - :r.pound's basicity and nucleophilicity. Pick A to make your day.
tlhoice B is correct. Electron withdrawing groups increase the acidity of a compound by stabilizing the negatir-e - ,'.:ge on its conjugate base. As the conjugate base becomes more stable, the acid becomes more acidic. The :::-:,gst acids of the phenols listed in the question (and it could be any type of compound, not just phenols, as ...: as all four choices are the same class of compound), is the one with the strongest electron withdrarving ::.-rP. The strongest electron withdrawing group is the nitro group, which withdraws by resonance, making .- ,:e B the most acidic. Pick choice B and a correct choice is what you'll see.
-F.oice D is correct. Because fluorine is more electronegative than chlorine, it is more electron withdrar,r'ilg :hlorine. This means that fluorine atoms on the backbone of the molecule increase the acidity even more so -,:. :he chiorine atoms. An increase in acidity results in a lowering of the pKu. The only value for pKu 1ou-er '," :: ' I.2 is choice D,1.0.4. You'll no doubt feel glee, if you just picked choice D.

:--,.:.

Dl
C C

-:.:ice D is correct. The

A
B B

reaction in choice A is an Sp2-reaction, because the electrophile is primary and .--::'"-ria is a good nucleophile. Sp2 reactions require an aprotic solvent, so ethanol is not a good choice. Chorce -' .ilnhated. The second reaction is a Grignard reaction, which cannot be carried out in a protic solvent. h-r a ,: solr'ent, the alkyl magnesium bromide species can deprotonate the protic hydrogen, thereby destror.ins .::glard reagent. Grignard reactions must be carried out in an aprotic solvent, so choice B cannot use ethano. :.lr-ent. Choice B is eliminated. In choice C, an aldehyde is being oxidized into a carboxylic acid. -\

D
C C

D D
C

.:'.i must be inert if it is to be useful. Because primary alcohols can be oxidized by ]ones reagent - : H2SOa), ethanol is reactive under the reaction conditions. This makes ethanol a bad choice for the ::.:, so choice C is eliminated. The last reaction involves the deprotonation of an alpha hydrogen on ::etic ester. Because the ester can undergo both deprotonation and attack at the carbonyl carbon, the base -. chosen matches the leaving group of the ester, in this case ethoxide. The ideal solvent is the conjugate -: :he alkoxide, so that it cannot generate another possible nucleophile for transesterification. Given that , -:e is the base being used in choice D, ethanol is the ideal solvent. Choice D is your kind of choice.

D B

.\
C

--:r:e D is correct. The E1 elimination reaction is a two-step process that involves the formation ot a . -: ::-:ion intermediate. The most stable carbocation is a tertiary carbocation. The only tertiary alcohols of
:-- -i .'. er

choices is choice D, which makes choice

D the best answer.

SHED::

, The Berkelcv Review@

/c

CARBONYLS

&

ALCOHOLS EXPLANATIONS

8.

Choice A is correct' The best nucleophile will undergo transesterification most readily. The nucleophile that most readily undergoes transesterification must have-the greatest partial negative charge and the least steric hindrance' Considering they are all five carbon alcohols'ana ,o,rgnty in basicity, their differences in nucleophilicity revolve around steric hindrance rather than partial"qrrJt negative charge. The least steric hindrance is associated with the primary alcohol. This eliminates choice C (a secondary alcohol) and choice D (a tertiary alcohol). Choice A iJa straight chain alcohol with less steric hindrance than choice B (a primary alcohol with a branched R group), so choice A is the best answer. Choice A is correct' Both primary and secondary alcohols can be oxidized to carbonyi compounds, therefore statement A is a false statement. Choice A is not true in regards to alcohols. Alcohols do ror* nydrogen bonds, and outside of alcohols, few other organic compounds forri hydrogen bonds (amines, amides, acids are some of the few)' Statement B is a valid statement. Piimary alcohols mak"e the best nucleophiles, because they exhibit the least steric hindrance in the transition state, therefore statement C is a valid statement. The infraied peak for an alcohol is broadened due to hydrogen bonding, and hydroxyl bonds show in th" t;001;*-i'L"g", irr*"t"* statement D is a valid statement. Pick A to start your correct answer filled day. Choice C is correct' A primary alcohol will oxidize into either an aldehyde or a carboxylic acid, depending on the conditions, but it cannot be oxidized into a ketone. This eliminates choices A and B. A tertiary alcohol does not undergo oxidation, which eliminates choice D. The oxophilic carbon (carbon bonded to the hyhroxyl oxygen atom) in a secondary alcohol has two bonds to carbons, so when it is oxidized into a carbonyl, it still has two bonds to carbon' A carbonyi compound where the carbonyl carbon has two bonds to other carbons is a ketone. Choice C is pretty unbeatable when it comes to correctness on this question. Choice A is correct' The alcohol group takes priority according to IUPAC nomenclature rules, so the compound L. automatically something-2-pentanol. This makes only choiJe A possible. It is not necessary to determine the exact name in light of the answer choices. Pick A to be correct. In the interest of learning, you should note thal the substituents are listed in alphabetical order according to nomenclature rules. Choice B is correcl Tosylating an alcohol,-in this case isopropanol, results rn the substitution of an alkoxy grou' for the chloride' The methyl group on the benzene ring should remain unaffected, so choices C and D are eliminated' Choice A is missing the oxygen atom from'isopropanol, so it is eliminated. The best answer choice B. This could be solved blsimply i,iUrtitntitrg an isopropyl group for the R in Figure 1.
i_<

fto
m0w t1M
'firffiUr
uwrolffii
,dqlliiinrrlrn

dmm

9.

Chr
lhdhil

ffiuW

ffmnmut4

10.

rffi

(nii

l[hum
lmmirlm

11.

&gm

'l'2'

13'

Choice D is correct. As stated in the passage, ketals cannot be formed under basic solutions and do not react in basic solutions' This means that the best answer is choice D, no reaction. Under acidic conditions, both acetone (choice A) and 1,2-ethylene diol (choice C) will form.

't4.

Choice D is correct. The reaction as drawn involves a ketal and water in the presence of acid catalyst. This water and acid will destroy the ketal group and consequently form a vicinal diol and ketone. The ketai carbon (carbon with two become the alcohols. The answer is drawn below:

',.vg"#,."*;;H,Jfi;.:H:1fi::'"i]ili"::T:::Hl3;:H?"*::"l3tj,.*""
Ether carbons become

the alcohol carbons.

f*fr:iH:
15.

t"6+-X.i
:

The best answer is choice D.

Choice C is correct. As shown in the sample reaction in Figure 1, a diol and a ketone react in the presence catalytic acid to yield a cyclic ketal. The diiference between -*ris question and the sample reaction in Figure 1
choice is answer C.

:;; T:ff ; l[ :?ffi:"#il :;iil:'+i:",1


&

Copyright @ by The Berkeley Review@

74

C{RBO\YLS

ALCOHOLS EXPLANATIONS

Choice C is correct. According to the reaction in Figure 1, diols and ketones react in the presence of catalytic acid :-' vield a ketal. The ketal is a protecting group for either the diol or the ketone, depending on the objective of -,:'e protection reaction. To protect a ketone, you can add a diol and catalytic acid. Therefore, to protect a diol, ', 'ru can add a ketone and catalytic acid. The best answer is choice C. If a vicinal diol reacts with an ester, it ""':ll undergo transesterification. The ester formed is reactive, so it cannot serve as a protecting group. This =-:rnrnates choices A and B. Choice D should be eliminated, because a ketone and alcohol will only go as far as :e hemiketal when reacting under basic conditions.

Choice B is correct. The carbon that bridges the two rings, which has two sigma bonds to oxygen, dictates the -:,ctionality of the molecule. Because there is a hydrogen attached to that carbon, along with two alkoxy i:rups (OR groups), the compound is an acetal. The best answer is choice B. The carbon is shown in the drawing
:

alo\r'.

(
:

;*,r"Tii ffi:',*H*r".'ff;.'#r

-hoice C is correct. Sodium methoxide is a strong base, methanol is an alcohol, and acetone is a ketone. As .-,:ed in the passage, a hemiketal is formed when mixing excess aicohol with a ketone in the presence of base. l'= mixture in the question results in the formation of a hemiketal. The best answer is choice C. Hemiacetals .--: hemiketals are formed under basic conditions, while acetals and ketals are formed under acidic conditions. l'=-ause no acid is present in the reaction mixture, choices A and B are eliminated. No aldehyde is present, so : = :roduct cannot be a hemiacetal, which eliminates choice D.

*roice C is correct. The formate leaving group and acetate leaving group should be fairly synonymous in that --:-'' are both conjugate bases of roughly comparable weak acids (there pKu values differ by about 1.0). ---:rrding to Table 1 in the passage, acetic acid has a G-value ol 1,.7, so formate should have a value of roughly ChoiceC, 1'.B,istheclosesttol.T,soitisthebestanswer. BecauseapKuof -T.0correiatestoaB-valueof - : :o have a G-value o{2.0, the acid would have to have a pKu value around 0.

:::-'n. In

-hoice A is correct. The leaving group in the reference reaction shown in Figure 2 of the passage is the ethoxide the reference reaction, the ethoxide anion is displaced by a methoxide anion. Ethoxide anion is also :::.ed in addition to the ester. Choose A for the result of correctitude.

r:r-,"'r1

l:loice C is correct. HBr falls between HI and HCI in terms of acidity, so bromide should have a B-value that of chlorine and iodine. Its G-value is found between 2.5 and 2.8, making 2.7, choice C, the best

:::rrvise we can't recognize whether a reaction has transpired. Sodium is a spectator ion, so labeling the ;-um is pointless. This eliminates choice A. Because the carbonyl remains intact in this reaction, 180 ':=-rng of the carbonyl oxygen would be pointless. This eliminates choice B. Because the alkyl group of the .::onyl moiety remains intact in this reaction,2H labeling of the alpha hydrogens would. Ue pointtess. This , :lnates choice C. The reaction is best monitored using an isotopically labeled nucleophile. This is ' JCD3/HOCD3, which makes choice D the best answer.
-:-oice B is correct. Taking the log of a number less than one results in a negative value. This means that if k*, then the log value of their ratio would be negative and thus the B-value would :: :.egative. A negative B-value occurs when the observed rate constant is less than the reference rate constant. = -- = observed rate constant is less than the reference rate constant when the observed rate is less than the rate of -- = :eference reaction. This means that choice B is the best answer.
' -=::ience were greater than

-ioice D is correct. This study requires that the ieaving group be distinguishable from the nucleophile,

:rl

O by The Berkeley Review@

75

CARBONYLS

&

ALCOHOLS EXPLANATIONS

24. Choice C is correct' The worst leaving group according to Table 1 is the alkoxide group, with a B-value of 0.2, an ester is less reactive than an anhydride and an u.id hulid". This eliminates choices B and D. An ami (NRz-) is more basic than an alkoxide, so according to the trend between basicity ana leuvirrg group strength, tlT alkoxidg rlis makes.the amide_the }|,:T:,::J':1"^l,]TTq^gf:P. answer. The term amide has multipt"least'reactive carbonyl compound the choices. Choice C is the best ;;";;;;;;ad; ,. nomenclature' It is both the negatively charged conjugate base of un u*il ""n"*t (deproionated form) and a carbr compound with an amine group at the alpha-position. "
25.

choice B.

Choice B is correct' The most reactive carbonyl compound has,the best leaving group attached. Using the riom B_value. A rarser a_"ur"" i? ?.ai"uu,u of greater reac: The largest B-value is found with iodide, thus 11e acetoyl iodide Is the the most ,uu"ti**oi;" .:.*;"il't" u question' This eliminates choices A and C. The anhydride is more reactive than the ester, so the best answer

ill?:l::,,1:,::.::l*I :Tt:,l"l"t*

26. Choice C is correct. To be sy:rthesized from 2-pentanone, the final product must have the new alkyl gr bonded to either the first or third carbon of the 2-pentanone. The answei choice compounds

oD.o ilii tH3cH2cH2cat-or*raru, Hrc-c)crtcurc+, HzC. oror12cHzcH3 *r.t, I "rzir, I From cArbon l Kr r;Xi Fromcdrbon5
C.
lt

A.O

B.O

are drawn below:

il

,ua-a

Fr

/ cH2c-r3 Fromcarbon3

fcH2cH2c

Choice C is formed from the alkylation of carbon 5, which CANNOT be carried out on 2-pentanone using enolate synthesis pathway shown- in the passage. The reaction can only occur on carbon 1 or carbon 3, wJ eliminates choices A, B, and D and confirms thal choice C is the best answer.
27.

Choice C is correct. The product, 3-hexanone, results from the deprotonation of carbon 1 (the less hindered thus kinetic carbon)' Product A is formed at lower temperatures, implying that product A is the kinetic pro< By association, Product A is 3-hexanone. At 0"C, thl percentug"-of 3lhexanone iuia 27.9"h. The f:::l'T","f^11::u"one. is the range, so correct choice must hive a percentage that lies between these values. only 35% lies within choice C is the best answer.

i;';;.;;;';r.ji"

"

28. choice

A is correct. The addition of a four-carbon alpha, beta-unsaturated ketone to butanone via a lv reaction results in an eighlcarbon product, so choices C and D (each with only seven carbons) are elimi The kinetic enolate of butanone tur,rlt" from the deprotonation of "urfor, the product must "ffi" H" lT^"T "ng]3te, answer choices are have the R group added carbon one from the reactant ketone (brrtar *9 remaining drawn beloir, wlih the butanone in bold script and the new R group 1" in standard script:

;;;. F;r;;-f# il;;;;

A.

B.

l
rin",iJcarbon

t
rin",iJcarbon

Only in choice A,6-oxo-2-octanone, did the alkyl group add to the kinetic (less hindered) carbon. Choice the best answer. In choice B, the alkyl group added to the thermodynamic carbon. to
Choice A is correct. Because LiN(CH2CH3)2 is a bulkier base than LiN(CH3)2, the kinetic product will be favored with LiN(CH2CH3)2 than LiN(CH3)2. This should be observed at all temperatures. Product A is kinetic product, so the best answer is choice A.

Copyright @ by The Berkeley Review@

76

CARBONYLS

&

ALCOHOLS EXPLANATI

30.

Choice B is correct. Product A is the kinetic product, so it must be formed by way of the kinetic intermediate. This can be confirmed by the data in Table 1, which shows that an increase in temperature results in an increase in Product B and a decrease in Product A. The kinetic product decreases with increasing temperature. The best answer is choice B. Choices C and D can both be eliminated, because they are the same answer. The term transition state intermediate is an oxymoron at best, because there is no such thing. Pick B and move on. Choice A is correct. At 40"C using the small base potassium hydride, the thermodynamic product would be the major product of the reaction. The more substituted alpha carbon is carbon 2 of the cyclic ketone. The ethyl group will add to this carbon, resulting in both the presence of an ethyl group and a methyl group on the C-2 carbon of the ring. The final product is 2-ethyl-2-methylcyclohexanone, choice A. Choice B is correct. To offer competition between kinetic and thermodynamic enolate, the molecule must have hydrogens on two alpha carbons that are unequally substituted. In choice A, the two alpha carbons are carbons 2 and 4. The two carbons are unequally substituted due to the presence of the methyl substituent on carbon 2. In choice B, the two alpha carbons are carbons 2 and 5. The problem is that carbon 2 has no hydrogen due to the presence of the two methyl substituents on carbon 2. There can exist no competition, because only one alpha carbon can be deprotonated on the molecule. In choice C, the two alpha carbons are carbons 1 and 3. The two carbons are unequally substituted due to the presence of the methyl substituent on carbon 3 (the fourth carbon in the butanone chain). In choice D, the two alpha carbons are carbons 2 and 5. The two carbons are unequally substituted due to the presence of the methyl substituent on carbon 2. The methyl group on carbon three plays no part in the reaction.

t1.

Choice A is correct. When an alcohol is oxidized into a carbonyl (ketone when the alcohol is a secondary aicohol), the oxidation state of carbon increases (an increase in oxidation state is associated with oxidation). Because the oxidation state decreases in choices C and D, they are both eliminated. The carbon in a secondary alcohol is sigma bonded to two carbons, one hydrogen, and an oxygen. The active carbon neither gains from nor loses electrons to carbons to which they are bonded. The active carbon gains an electron from the hydrogen to -,r'hich it is bonded and it loses an electron to the oxygen to which it is bonded. Overall, the active carbon of a secondary alcohol neither gains nor loses electrons in its bonds, thus the oxidation state of a secondary alcohol is zero. The correct answer must be choice A. Drawn below is a short cut technique for determining the oxidation :tate by considering each of the four bonds to carbon. H OH
I

OxidationHsC

HsC

Overall: 0 + 0+

CFI?

cH3
+2

1-1=

Overall:0+0+1+1=

f,hoice B is correct. Basic permanganate solution will oxidize a secondary alcohol into a ketone. The ketone -..-ned from the oxidation of 2-hexanol has the oxygen bonded to the same carbon in the both reactant and ::oduct, so the organic product is 2-hexanone. Because pentanal is an aldehyde, choice D is eliminated. The :eatment of 2-pentanol (also a secondary alcohol on a straight carbon chain) with basic permanganate solution :rrerates a yield of 81.7% (as listed in Table 1.) The yield when using 2-hexanol should be about the same, so !-.19i, is the best choice. The best answer is therefore choice B. The additional methyl group in 2-hexanol -:::-rpared to 2-pentanol should not cause much difficulty because it is located at the end of the chain, where =::rrc hindrance is minimal. Choice D is correct. Primary and secondary alcohols can be oxidized, while tertiary alcohols cannot be - r:dized. The compounds 2-methyl-3-hexanol and 2-ethylcyclopentanol are both secondary alcohols, so they ::n both be oxidized into ketones, eliminating choices A and B. Choice C (3,3-dimethyl-1-pentanol), is a ::rmary alcohol so it can be oxidized into either an aldehyde (in a non-aqueous environment) or a carboxylic ::rd (in an aqueous environment). The only tertiary alcohol in the answer choices is choice D,Z-methyl-2: - tanol.

-.-iht O by The Berkeley Review@

77

CARBONYLS

&

ALCOHOLS EXPLANATIONS

36.

that is important. Na2Cr2o7 (sodium dichromate) is an oxidizing agent, so the alcohol that"is chosen must be an alcohol that can be oxidized. Phenols cannot be oxidized, so choice A is eliminated. Both 1-methylcyclohexanol and 3-ethyl-3-heptanol are tertiary alcohols, therefore neither of those alcohols can be oxidized. This fact eliminates choices B and C. The correct compound is 2-methyl-1-octanol, a primary alcohol, making choice D the best answer.
J,/.

Choice D is correct' Because the solution turned green, there is evidence that some reaction took place. The color green is not important, it is the fact that a color change transpired

Choice A is correct' Choice A represents the conversion of a four carbon primary alcohol into a four carbon ketone' This is rzol possible, because the carbon bonded to oxygen has changed from ih" first carbon to the second. carbon' The oxygen does not migrate in oxidation reactions. Choice g is lhe oxidation of a primary alcohol (2methyl-1-pentanol) into a carboxylic acid, which is possible. Choice C is the oxidation of a primary alcohot (1pentanol) into an aldehyde,-which is possible. Choice D is the oxidation of a secondary atcohol (cyclohexanol) into a ketone, which is possible. Only choice A is not possible.
Choice D is correct' Chromium gets reduced when chromic oxide oxidizes a primary or secondary alcohol. V\4ren chromium is reduced, its oxidation state is reduced. Chromic oxide will o*idir" etiranol (a primary it will not react with 2-methyl-2-butanol, because it is a tertiary alcohol. Because of this fact, alcohol) but the oxidation state of chromium decreases with the addition of ethanol but the oxidation state of chromium remains constant upon the addition of 2-methyl-2-butanol. The best answer is choice D. Choice C is correct' To or reducing agent, a compound must be a reactant in the reaction. The ?." 3l gidizing two reactants are 2,4-pentadiol (C5H12o2) and Tio2. Because the diol gets oxidized into a diketone functionality, it must be the reducing agent. This is true, because red.ucing agents get oxidized. This means that Tio2 is the

38.

39.

oxidizing agent. Choose C, for correctivity glee.

40' Choice A is

correct' Compounds that turn litmus paper red are acidic. primary alcohols are oxidized into carboxylic acids.while secondary alcohols are oxidized to ketones. Tertiary alcohols do not oxidize. This makes choice A the correct choice.

41'

Choice D is correct' The formula C6H12o indicates that the compound has only one degree of unsaturation. A cyclic ketone would require two degrees of unsaturation (one for the ring and Jne for the carbonyl r-bond). By having only one unit of unsaturation, the cyclic ketone structure is not posJible.
Choose D.

42. Choice C is

correct' Compound A is positive with Jones test, so it can be oxidized. Compound A is also positive for the nucleophile test,- so it is likelyan alcohol, given the only heteroatom is oxygen. primary and secondary alcohols can be oxidized, so Compound A is eitheia primary oi secondary alcohoi."This makes choice A a valid correlation, and thereby eliminates it. The iodoforh test is used to dltermine whether the compound is a methyl k9!o1e' Compound B is the only unknowl that tested positive with the iodoform reagent, so Compound B is most likely a methyl ketone. Thii makes choice g a vaha correlation, and thereby eliminates it. By u process of elimination, only choices C and D are possible. Because Compound C did not react with Jones, reagen! it cannot be oxidized. An aldehyde can be oxidized into a carboxylic acid, so Compound C cannot be an aldehyde' This makes choice C an invalid correlation, so choice C is the best answer. A compound that is negative with all four tests in the passage, has one unit of unsaturation, and contains one oxygen atom can be either a ketone (but not a methyl [etone) or a cyclic ether. Neither a cyclic ether or ketone will be oxidized bv Jone's -reagent or Tollen's test. Neither is nucleophilic, so they wilt not undergo a substitution reaction ;h ; alkyl halide in the presence of silver cation. Neither is a methyl ketonJ, so they will yield a negative iodoform test' As t,"h, th"r" is a chance that Compound C is either a cyclic ether or a ketone, making choice D a valid statement. Choice D is eliminated.

43. Choice C is

correct' This question is a test of the information that you have memorized. Both primary alcohols and aldehydes can be oxidized into carboxylic acids, so choices A and B are eliminated. A hemiacetal will convert back to an aldehyde in the presence of water and either an acid or a base. Given that an aldehyde can be oxidized into a carboxylic acid Lnder either acidic or basic conditions, choice D is also eliminated. As a general rule, any carbon bonded to both an oxygen and a hydrogen, can be oxidized, with the exception of ether or ester. of the choices, only the ester cannot be oxidized. Choice C is the best answer.
7A

Copyright @ by The Berkeley Review@

CARBONYLS

&

ALCOHOLS EXPLANATIONS

)lor

1+.

um
ed. are C.

)on )nd

()(1-

Choice B is correct. An aldehyde can undergo oxidation to a carboxylic acid, reduction to a primary alcohol, or a substitution reaction to form a new n-bonded compound such as an imine. The most common test involves the formation of a carboxylic acid using an oxidizing agent such as CrO3 in H2SO4 or KMnO4 in KOH. These both involve the transition metal in test reagent changing oxidation state and thereby changing color. This makes choices A and C invalid, and both are eliminated. A testing reagent should always be the limiting reagent (in lower concentration than the compound for which it tests), so that the change in color is not masked by the excess reagent that did not undergo a color change. This eliminates choice D. The best answer is choice B. Although an aldehyde can be reduced into a primary alcohol and reducing agents are often rich in hydrogen atoms, the common reducing agents (LiAlHa and NaBH4) do not undergo any phase or color changes. This means that a compound rich in hydrogen atoms is not a good choice for the reagent used to test for the presence of an aldehyde functional group. Choice C is correct. To begin with, in organic chemistry, oxidizing agents contain oxygen and are best described as "rich in oxygen." A11 four of the compounds contain oxygen, so you must recall what each reagent does. KMnO4 is used to oxidize an alkene into a vicinal diol, oxidize a primary alcohol into a carboxylic acid, oxidize a secondary alcohol into a ketone, or to oxidize an aldehyde into a carboxylic acid. This eliminates choice A. H3CCO3H is a peroxy acid which can be used to oxidize a ketone into an ester (known as the Baeyer-Villiger reaction) or to oxidize an alkene into an epoxide. This eliminates choice B. HIOa is used to oxidatively cleave r-icinal diols into two carbonyl fragmenti. This eliminates choice D. This leaves only choice C to choose. Mg(OH)2 is used as a base, but it has no immediate use in organic chemistry as an oxidizing agent. Choice C is correct. LiAlHa reduces many things into an alcohol, such as acids, ketones, aldehydes, and oh yes... esters. Pick C for that fresh, minty, "I'm alive and picking right answers" sort of feeling. The Hz/Pd reagent is used to hydrogenate (reduce) alkenes. HCI/Zn is used in the reduction of a ketone into an alkane (Clemmensen reduction), and BH3 is used to add in an anti-Markovnikov fashion to an alkene.

rol)

len
but

ion
ant

lvo

ity,
the

il
nto

his

A
Br.

Choice C is correct. Wolff-Kishner reduction is carried out on both ketones and aldehydes. The question is reduced to: "Which of the following reagents can react with either an aldehyde or a ketone?" Lithium aluminum hydride reduces both ketones ands aldehydes into alcohols, so choice A is eliminated. A Grignard reagent, such as methyl magnesium bromide, reduces both aldehydes and ketones into alcohols of one more alkyl group. Choice B is eliminated. Clemmensen reduction is carried out on the same compounds that Wolff-Kishner :eduction is carried out on, so choice D is eliminated. Ozonolysis with oxidative workup can be used to make ketones and carboxylic acids, but it does not react with ketones (or aldehydes). This makes choice C the best
erlswer.

1\'e
a_rv

Jid

sa

IB ;a
nt,
an

Choice B is correct. Choice A can be eliminated immediately, because reduction corresponds with a decrease in rxidation state, not an increase. The central carbon of a ketone has two bonds to oxygen and two to other carbons, so it starts with an oxidation state of +2. This eliminates choice C. The carbonyl carbon is converted into an :lkyl carbon with two bonds to carbons and two bonds to hydrogens, so it finishes with an oxidation state of -2. lltis makes choice B the best answer. The conversion and oxidation determination is shown below. H
Loss of 4 e

bis t'e

trir-e

Ja
oLc

,-dl
:an _ia

Choice B is correct. Following Wolff-Kishner reduction, a carbonyl carbon is converted to an alkyl carbon. The :est of the molecule remains intact, so no new chiral center is generated. This eliminates choices A. The :arbonyl carbon starts with sp2-hybridtzation (three o"-bonds and one n-bond) and finishes with sp3:rr-bridization (four o-bonds), which makes choice B a true statement. The carbonyl carbon loses bonds to oxygen :nd gains bonds to hydrogen in the process, so it is reduced. However, nitrogen loses bonds to hydrogen and gains :onds to nitrogen, so it is oxidized. Each nitrogen goes from an oxidation state of -2 to 0 during the reaction, so :oth nitrogen atoms are oxidized. This eliminates choice C. Figure 1 shows that one water molecule, not three, '. formed overall, so choice D is invalid, and therefore eliminated.
::eht @ by The Berkeley Review@

79

CARBONYLS

&

ALCOHOLS EXPLANATIONS

50'

Choice D is correct. The hydrazine derivative is definitely larger than ammonia, NH3, but its greater size does not generate greater acidity due to steric hindrance. The protonls not being driven off by crowdiig, ,o choice A is eliminated' The hydrazine moiety does not have any cyclic array of n-Jto-r,-so it does not become aromatic upon the gain or loss of a proton. This eliminates choice B. Hydrazine may in fact have a stronger inductive effect (from the second nitrogen) than ammonia, so choice C cannot be eliminated. when the protoi is lost from nitrogen in the hydrazine derivative, the new lone pair is located on an atom adjacent to a n-bond. This allows for thefone pair to resonate to the carbon of the n-bond, offering it more;"btfi due to resonance. This makes choice D a true statement and the best answer.

"\fir'
51'
answer.
52. Choice

n2\'

Choice A is correct. Hydrazine and basic water reduce ketones and aldehydes into alkyl groups. They leave other functional groups intact, unless they are reactive with basic water. The hydro"yr fro"p in unreactive under basic conditions, so it will remain unaffected. This means that the product must contain a hydroxyl group and-an-ethyl group on thecyclohexane backbone. Choices C and D are eliminated, because they don't include the hydroxyl group. The alcohol functional group gets higher priority than an ethyl group u."ordi1g to IUpAC convention, so 3-ethylcyclohexanol is the correCt rupeCt"rame for ih" .o*pornd,kaking choice'A the best

answer.
53.

A is correct' Wolff-Kishner reduction requires basic conditions while Clemmensen reduction requires acidic conditions. Both involve an aqueo,rs enrri.on*ent, so neither can be employed for water sensitive compounds' This eliminates choice D. Wtjlff-Kishner reduction, by employing a'hydrazine molecule, takes advantage of a carbonyl compound's ability to react with a nucleophilic nitrogln."It shtuld not be employed for a molecule that is reactive *ith an amine at a site other than the carbonyt".u.uon.-'ir,i;;l;;#rt.toi." gNo acid is employed in a Wolff-Kishner reduction, so it is safe for acid sensitive molecules. Clemmensen reduction uses acidic conditions, so it will react with acid sensitive groups. This means that Wolff-Kis reduction is preferential to Clemmensen reduction when the molecul-e is icid sensitive. Choice A is the Choice A is correct. Water is the side product when hydrazine reacts with a ketone such as
2

methylcyclopentanone, so the product is most easily determined by removing the oxygen of the carbonyl carbcn and two hydrogens from the trittogu_tr in phenylhydrazine that hasiwo
and nitrogen. This makes choice A the beJt answer.

"i;g';;;;;;;.;;;*;;il.,
H

H..t.
N
I

-/Ph

-----+
H:C

N/N-

Ph

=A L-J
&
ALCOHOLS EXPLANATIO

54'

Choice A is correct. It is known that anhydrides and acid halides are highly reactive, so choices C and D can be eliminated. A thio ester C-S bond is weaker than an ester C-o bond, so the least reactive carbc compound is the ester (choice A). The chart of pKu values can be used to verify this theory. an alcohotlas higher pKa value than the thiol, so the alkoxy i"urrlng group is less stable between choices A and B, makir
choice A the best answer.

Copyright @ by The Berkeley Review@

80

CARBONYLS

loes

Ais
atic

tive
:om

)ws
rkes

Choice C is correct. The Grignard reagent (methyl magnesium bromide) adds on one carbon (a methyl group), so in order form 2-pentanol as the product, you need to react H3CMgBr with a 4 carbon carbonyl reactant and add the methyl group to the first carbon of the four carbon chain. Because there are six carbons in the products of choice A and choice B, the choices are narrowed down to either choice C or choice D based on the four-carbon electrophile requirement. The best way to get a secondary alcohol as the product is to react H3CMgBr with butanal, which will yield the product with the alcohol group on carbon number 2. Pick C for an altogether great experience and correct answer. Choice D will form 2-methyl-2-butanol.

Choice D is correct. The reaction of a Grignard reagent with a carbonyl yields an alcohol. Stereoisomers are formed when the alkyl groups are all different on the alcohol carbon. For this to be true, the Grignard nucleophile must add an alkyl substituent different from the alkyl groups present on the carbonyl. Choice A is eliminated, because the ketone has an ethyl and methyl group attached. The Grignard reagent adds an ethyl group. The alcohol that is formed is achiral (3-methyl-3-pentanol). Choice B is eliminated, because the ketone is symmetric (has two ethyl groups), so the product must be symmetric and thus achiral. An ester when reacted u'ith excess Grignard reagent yields an achirai tertiary alcohol (it adds the same R-group twice), thus choice C is eliminated. Choice D is best, because the final alcohol has an ethyl group, methyl group, and hydrogen attached. This makes the alcohol asymmetric and thus chiral. The product mixture is two enantiomers.
raVe

tive

,-{c
best

oup ude

Choice B is correct. Aldehydes are carbonyls with at least one hydrogen and often one carbon attached to the carbonyl carbon. The addition of a Grignard reagent places a second carbon on the electrophilic aldehyde site and thus reduces the aldehyde to a secondary alcohol. Pick B for the inner peace that a correct answer can bring.

,ires

live
rkes I for

H"C o
RMsBr ___-jt+

OH

H"O+

eB.
ISCN

LNCI

best

Choice B is correct. An ester has both a leaving group and a carbonyl with which to react. The first R-group to :ttach converts the ester into a ketone by displacing the alkoxy leaving group. The ketone that is formed as the ,:'.termediate product can react with a second equivalent of the Grignard reagent and thus add a second R-group. Tne final product is a tertiary alcohol with two identical R-groups. The conclusion is that a Grignard reagent :eacts twice (not just once) with an ester. Pick B to tell the world "I know Grignardl" Choice D is correct. The Grignard reagent is a strong base, so an aprotic solvent such as ether is chosen. This -:'.akes choice A a valid statement. Because the carbonyl compound is converted into an alcohol in a Grignard :=action, a bond to oxygen has been lost. This means that the carbon has been reduced, making choice B valid. A -:ignard reagent can be used to form a secondary or tertiary alcohol, as shown in the passage. To form a primary -'.1:ohol, a Grignard reagent can be added to either formaldehyde or the epoxide ethylene oxide. This makes -:.oice C valid. Becausgthe Grignard reagent is a strong base, it can react with the weakest of acids to form an ,rl-<ane. This is why the reaction must be carried under anhydrous, aprotic conditions. It is thus not true that the -::gnard reagent should be added under acidic conditions to obtain the highest yield. Pick D and move on with ::.other correct answer under your belt.

:-::ns an anhydride. This question is asking for the relative reactivity of an anhydride. Acid chlorides are :: -'re reactive than an anhydride, so choice A is eliminated. Amides are less reactive than an anhydride, so -:.oice B is eliminated. Esters are less reactive than an anhydride, so choice C is true. The difference in :.ectivity between anhydrides will not be distinguishable by pKu values that only differ by 0.1. There is a
::--ie, thus choice

Choice C is correct. Propanoic acid is the conjugate acid of propanoate, which when bonded to carbonyl group

:,-ierence in steric hindrance to consider. Choice D may or may not be true, but regardless, choice C is definitely D is assumed to be false within the context of this question.

rlhoice B is correct. The most acidic hydrogen on the molecule is the H bonded to the alpha carbon (marked on --: molecuie as carbon b). The best answer is choice B. The aldehyde proton is not as acidic as the alpha proton, ::ause the lone pair that would form on the carbonyl carbon is not stabilized by resonance, like the alpha

-::lon.

'

-.::.'r O by The Berkeley Review@

AI

CARBONYLS

&

ALCOHOLS EXPLANATIONS

62. Choice

D is correct' counting carbons starting from the carbonyl, the molecule is a 4-carbon chain with a .wh"," methyl group bonded to the numbei 2 carbon. The Zompound hu, an aldehyde functional group, so the correct IUPAC name is 2-methylbutanal, answer choice D.

63. Choice

A is correct' AI aldehyde with excess alcohol ft: presence of an acid will lose a water molecule, and go on to form an acetal' It is an equilibrium reaction, i" but the excess alcohol will push the reaction to the acetal (product side of the reaction)' A hemiacetal is formed when the reaction is carried out under basic conditions. Choice A is the best answer.

64'

Choice A is correct' The reaction is an aldol condensation. The acid workup causes elimination and thus ensures the formation of the o,G-unsaturated ketone from the G-hydroxyk"to.r"'it t"r-ediate product. choice D is eliminated, because it has seven carbons, which is not possible. Choice C is not an or,,G-unsaturated ketone, so it can be eliminated' Choice B, when broken apattby a retro-aldol reaction, would yield fragments of two and four carbons' not three and three carbons. Choice B is ellminated. The only ot8-unsaturatea ketone composed of two three carbon units is choice A. Choice D is corrprt' This is just one of those things that you need to learn. The conversion of a ketone into an enol or an enol into a ketone is referred to as tautomerization. Select D as your u.,r*"r. Tautomerization is involved in glycolysis in the conversion of glucose-6-phosphate into fruciose-6-phosphate, which is catalyzed by isomerase.

65'

66- Choice C is correct' This is a case of recognizing the weakest base.

only one of the answer choices is not a strong it must be the weakes$u,". iu,uo,", ate, Cos2-,is not a very ilH3l:::"i:j:,r":,n:T::l3lii^nl.*g:lr,oby resonance involving the double bond (C=o strong base, because the electroni u." itubii=ir"d rc-bond). It therefore the weakest base out of the choices listed, and thus is the one not strong enough to deprotonate the on the alpha carbon' Choice c is your choice for a brighter tomorrow, assuming of course that the brightness tomorrow
depends on your answer choice for this question.

67.

Choice C is correct' only a ketone and an aldehyde can undergo aldol condensation reactions. An acid wr than lose the less"acidic alpha;,;;;;; undergo the condensa reaction. Choice C, the carboxylic acid, cannot undergo aldol condensation.

f::ilf':li::"'f"_ij1":,':':ll-ol::flsroup

68.

most acidic proton on a B-ketoester is the alpha hydrogen that when lost results in Choices C and D are etiminated, because neither is an arpl ;?,T"1it:: Choice A is only tonjugated _ hydrogen' to. keto carbonyl, while choice B is conjugated to both the fhe carbonyl and the ester carbonyl' rire*most acidic proton is proton b, thus yo,, ,noria fi"k.hoi"e n.

Choice B is correct'

Jh" t:?:91t*.to the most n-tonds.

69.

choice c is correct. As stated in the passage, the lowest yield is found with the B-ketoester ani intermediate that has no alpha- hydrogen. For "this to occur, the starting ester must have only one aI hydrogen' of the choices, oniy choice C"has just one alpha hydrogen. you should pick C. The reaction is below.
Nothing protonates the alkoxide, thus the

H"C

'Fou,#
II

equilibrium favors the reverse reaction.

HrC,,^

o JL

H3C H

!
HeC

oEt+

'"flor,
H3C H

ooo

--I

H"C

H"C ' ,H
HsC

oEt
cFI3

-OEt

No alpha hydroien, therefore the alkoxide leaving group cannot deprotonate the final B_ketoester.

70. Choice A is correct' A Claisen reaction is not possible with esters that have no alpha hydrogen. of the cho only choice A has no alpha hydrogen. This makes choice A the best Ernswer. a, ulolt of interest, this mole is not known to exist as a stable compound at room temperature.
Copyright @ by The Berkeley Review@

a2

CARBONYLS

&

ALCOHOLS EXPLANATIO

ha
ect

-lt.

Choice D is correct. The product of an ester reacting with itself by way of a Claisen condensation reaction is recognizable from the carbon chain length on the carbonyl portion of the ester. In the answer choices, the two alkyl groups can be highlighted to recognize the alkyl groups that must be present on the two esters that react, considering that only the alkoxide leaving group is lost. The alkyl groups are highlighted for choice A are: H3CCH2COCH2CO2CH2CH3, therefore choice A is formed from the condensation of two different esters: H3CCH2CO2CH2CH3 and H3CCO2CH2CH1. This same analysis of the product for choices B, C, and D will

lead to the conclusion that choice D is the correct answer. In choice D, the starting ester
(C5H5)CH2CO2CH2CH3 which leads to (C5H5)C[12COCH(C6H5)CO2CH2CH3. Choice A:
rres

is

)is oit
our
:1\'O

NaoEt fl o -n- + -flH3cn2c oEr HsC oEt tloEi Hrcn c+ont


HsC

Choice B:

o
oEt

o
OEt

o
oEt

H3CF{2C o
-,4.+
OEt

/t-

NaOEt -----.--.-.-.> HOEt

HsC

rnol
r-ed

bv

Choice C:

ong
-er)'

Choice D:

H5C6H2C o

HsC'

ll -oEt NaoEt
HoEt

Hucurqcuont
o

It is eH
,s

of

H5C6H2C OEr -:"

A+

o
H5C6H2C

,(

oEt HoEf urc.urcfort QL


I

NaoEl

ll

o
ll

coHi
ruld tion
Choice
possesses two ester sites so an intramolecular Claisen reaction is Claisen reaction results in a cyclic B-ketoester. This eliminates choices B and D. -:.e chain contains six carbons, so choice C can be eliminated, because it requires seven carbons excluding the ;--<oxv group to form a six-carbon ring. The best answer is choice A. The reaction is shown below.

::.sible. An intramolecular

A is correct. The starting molecule

il
1na
Ipha keto

E,oqoEt
o

H#Eto

oEt

* rr"A6
o o

nion lpha
o\\-n

lll'loice D is correct. The two esters can either react with each another or they can react with themselves. This :==:lts in four structural isomers being formed. Of the structural isomers, two have one stereocenter, therefore -,',. of the four structural isomers have two possible stereoisomers (enantiomers). The total number of isomers is : =refore six. Choice D is correct. The isomers are drawn below:

H,cUoEt ,AAJLBA
H:C

H"cu"c$oe,
o
H3cH2c

A+
A

OEt

xuost ll HrC-. ,A. -OEr -i-trHOEt

/\ HH
B

H:c\Aoet - AB:
CHs

oo

H3cVoEt
cH3

urcnc$ont
cF{3

O\S

ri

i-::,

3 by The Berkeley Review@

a5

CARBONYLS & ALCOHOLS EXPLANATIONS

74'

llt':*l* choice TT:::1111: :lll;: the choices, B (methyl "tnu.ouiuj'il:"# ?y;'.rE;r"'J;-;;:J'8"(:ffi?"1?,T:lJj6ti:: hydrogens' This narrows the question down to eittrer ethanoate)' The key feature in the spectrum_is the singlet far down "ioi." A (metlryl propanoate) or choice c (ethyl field"carrsed by a methyl bonded to the oxygen' This peak imPlies that the compound must hav"e a methoxy group on thu carbonyl carbon. pick A for optimal satisfaction. The structures are dru-r, below:
o
o

Choice A is correct' The ratio of the integrals for each signal in the proton NMR of spectrum I is given as J:2:3;. eigJrt total hidrogens or some multiple of eight total hydrosens. oi

tY:*:

:j

o
-ocH2cH3

*i,.?".Aoc"H"
Methyl Propanoate a: Triplet (3H) b: Quartet (2H) c: Singlet (3H)

*,.AoJH,
Methyl Ethanoate
a: Singlet (3H) b: Singlet (3H)

HsC

"Abc

"-i--

OCFI2CFI3

Ethyl Ethanoate a: Singlet (3H) b: Quartet (2H) c: Triplet (3H) The bolded hydrogen will be found the furthest down field.

Ethyl Methanoate
a: Singlet (1H) b: Quartet (2H) c: Triplet (3H)

t
flmr

75. Choice D is correct' The ratio.of the integrals for each signal in the proton NMR for spectrum in Figure 2 is 2 :2 that,the comporlnd isolated fro"m solution has ten hydrogeis. Given that the reactan: l;"':,^Tt'_1---11t lajor eight hydrogens, it can be concluied that some reaction took place, which eliminates choice A. Choice lrag (ethyl acetate-) has eight hydrogens, so choice B is eliminated. This narrows the question down to either choic C (propyl ethanoate)

lno*l#':11^*,ln"j^o^'_".1: spectrulm II to the carbonyl. Because :li :.':*1, has no slnglet, choice C is eliminated. u ,,,,glet ror the CH3 adjace tt*?;;;;ril'" f,;!:' downfield implies that the compound must have ur,""tnyl group on the ester oxygen. pick D to know what ri feels like. The structures are driwn below: o o o o

or choice D (ethyl propanoate). Key features in spectrlm II include the quartet f "cnfi""
c.(nr.onrl ethanoate) would

ft,.h,.Ao.;,,
Methyl Propanoate a: Triplet (3H) b: Quartet (2H) c: Singlet (3H)

H:C

"Abc

'oCH2CHr

a HsC

d ocH2cH2CH3

bc

ab
H3CH2C

Ethyl Ethanoate
a: Singlet (3H) b: Quartet (2H) c: Triplet (3H)

(
cd
ocH2cH3

llu

Propyl Ethanoate a: Singlet (3H) b: Triplet (2H)


c: Sextet (2H)

d: Triplet (3H) The bolded hydrogen will be found the furthest down field.
76.

Ethyl Propanoate a: Triplet (3H) b: Quartet (2H) c: Quartet (2H) d: Triplet (3H)

ffi m

ffi

m,

{mr MN

m
dtu

Choice B is correct' An ester is more reactive than all carbonyl compounds that have a leaving group worse th the alkoxide leaving group. Leaving groups worse than the alkoxide leaving gro.rp would be more basic th the alkoxide anion. Because both halides and carboxylates ur" gooal;;G?-tr"ps (as well as weak ba acid halides and acid anhydrides are both more reactive than esters. This eliminates choices A, c, and D. only choice remaining is amides, choice B. The NR2- leaving group of the amide is a bad leaving therefore amides are not very good electrophiles. Choile B is the best selection.

ffiu

ffih

Choice C is correct' By using an amine nucleophile rather than an alcohol nucleophile, the product that formed is an amide oppot"J "t be more to an ester. The amine is a better nucleophile than the alcohol is in Reaction so the reaction should favorable than transesterification. The equilibrium constant for the reaction greater than one' Regardless, the question asks for the product of the reaciion, which is the amide formed wh the amine replaces the alcohol l_eavrng group. The amine nucleophile is'x-eihytamine therefore the b,r answer is N-ethylpropanamide (H3CCH2NucbcH2cu3), choice C. This can be deiermined by just adding ethyl amine in same way as the ethanol ii added to the ester in the sample reaction carried out at reflux.
Copyright @ by The Berkeley Review@

a4

CARBONYLS

&

ALCOHOLS EXPLANATIO]

:8.

Choice B is correct. The purpose of the acid in the reaction mechanism is to protonate the carbonyl to make the ester more electrophilic. tn a later step, the acid protonates the alkoxy leaving group to form an alcohol, which is a better leaving group than alkoxide. The role of the acid in the reaction is Le"st stated as serving to protonate the ester electrophile and in doing so make it more reactive. This makes choice B correct and desirible io you. Choice D is correct. If water were the nucleophile in lieu of the alcohol, the product would be a carboxylic acid. When carried out under basic conditions, this reaction is known as saponification. Hydrolysis of an ester is common in biological systems. The carboxylic acid formed in the reaction would have the OCH3 group replaced - l by an OH group. This carboxylic acid would be propanoic acid (CH3CH2CO2H), choice D. Choice C is correct- The deuterated solvent is used so that the solvent (which normally contains hydrogens) does not interfere with proton NMR spectrum of the compound. The deuterium atoms donot registertn proton NMR, so the solvent is invisible when it is deuterated. The best answer is therefore choice-C. Deuierium exhibits little to no affect on pH in organic solvents. This eliminates choice D. The solvent should have no effect on the magnetization of the NMR sample nor should the solvent prevent any reaction as solvents are inert. Choices A and B can both be eliminated.

Choice B is correct. The structure of urea must be determined by comparing the final barbiturate structure iproduct of the lower pathway) with the substituted malonic ester. Urea must contain two nitrogens bonded to a central carbonyl. The alkoxy groups of the ester are leaving groups when the urea substitutes. The structural deduction is drawn below. The drawing represents retrosynihetic analysis of the final product. pick choice B.

HN

Urea NH

o
Nrtr

HrrvA

cH3cFlo\AocF{2cFL
R

:roduct of an amine and ester is an amide.

Choice B is correct. Urea has a carbonyl carbon separating the two nitrogen atoms, so if the carbonyl oxygen of 'rea is replaced by two methyl substituents on the carbon, then the final product will be the same as tne ptoa""t :rom urea,_except that it will have two methyl groups at the formerly caibonyl carbon. It is not differeni at any 'iie other than the carbon between the two nitrogens (the former carbonyl carlon). The correct choice must have ie two methyl Sroups bonded to the central carbon in lieu of the carbonyl oxygen. The best answer is choice B. To answer this question requires knowing what urea looks like. If you don't iiow, then you must predict what otild happen if the diamine were added to the diester. The correct choice can be deduced if you know that the "'

Choice C is correct- Decarboxylation occurs with B-ketoacids. A11 four answer choices have carboxylic acid :-rnctionalities, but only choices A, B, and D have B-keto groups on the backbone. The ketone functionality of :iroice C is_ on the gamma carbon, therefore the molecule will not undergo decarboxylation. The best answer is
--:-,erefore choice C.

C.

OH
H.C

Choice A is correct.,The strong base used to deprotonate the alpha carbon can also act as a nucleophile by ::iacking the carbonyl carbon. Sodium ethoxide iJ chosen as the bur" ,o that it matches the substituent on the :arbonyi carbon of the ester. This means that if the ethoxide undergoes transesterification, it will generate the ;:rrLe ester (the leaving group of the ester ils the same moiety as the nucleophile). Sodium ethoxide is a strong =rough base to deprotonate the alpha hydrogen, but ii will not change the ester when it undergges -:ensesterification. The best answer is choice A. :ht o by The Berkeley Review@

g5

cARBoNyLS & ALcoHoLS EXPLANATIoNS

85'

Choice D is correct. Malonic ester has three types of protons, thus there are three signals in its proton spectrum. The structure of malonic ester is shown belowwith each of the hydrogens labeled:

rya

Clnr

o
lt

o
ll

ifi lr

frp\.ott\.,"\
/\ HH ec

ba
ocF{2cH3

a: 6 hydrogens adjacent to 2H thus a 3H triplet b: 4hydrogens adjacent to 3H thus a 2H quartet c:2 hydrogens adjacent to 0H thus a 1H singlet

SF

Ch,r
-_Ji"

:::a

:r.
:2:
The true the absolute

ratio of hydrogens is 6 : 4 : 2, but because NMR shows only the relative quantities and not
quantities, the NMR shows a ratio of 3
86.
7.

Based just on the ethyl groups, resulting in a 2H a quartet and a 3H triplet, the best answer is choice

D.

:il:

ffisr
.,*.,

Choice B is correct. There are four carbons added to the central carbon of malonic ester, so choices A and C can both be eliminated. The addition of methyl bromide followed by the addition of ethyl bromide to malonic ester bromobutane must be added to form the tertiary R substituent. "tioic" B is the best choice. Choice

:flerr

only adds three carbons total, not four. This eliminute,

i ;;lary

alkyl bromide like

2,

87. Choice B is correct- The product will be the exact product shown in the lower synthesis pathway in Figure only the generic R group is now a straight chain propyl group. This makes choice B the best answer.

1,

88'

Choice D is correct' To synthesize butanoic acid, one of several methods can be applied. Oxidation of a priman aicohol will work, so choice A is feasible. Hydrolysis of an ester will form an acid, so the four carbon carbonr-fragment of the ester will become a four carbon carboxylic acid. Choice B is a viable ,y.rtn"tl. put1r-urOzonolysis of an alkene works if the addition of ozone is followed by oxidative workup and the alkene mus: have vinylic hydrogens (hydrogens on the carbons of the n-bond). Choice C is thus valid. Treatment of an alkr bromide with the. cyanirls nucleophile followed by acidic workup forms a carboxylic acid. The p..;i;;i,h choice D is that the carbon chain increases by one carbon with the cyanide nucleophile so the product will te pentanoic acid, not butanoic acid. The best answer is choice D.

ffi
{ry#5iilf." Wu@ffior

89' Choice D is correct. Because the pKu for propanoic

acid is 5.0, it is a weak acid, implvine that it onlr partially dissociates in water. With a pKu value -of 5.0, the Ku value is 1.0 x 10-5. This mein's tn?t fo, r J if acid solution, the concentration of both H+ and conjugate baie are 10-2.5,which is less ttran " r0-i, or7oh. The dissociation is less than 1%. The Ku value is applied ui follo*r,

r"

tcT*S?llxl[CH3CH2CO2H]

[cHacHzCoz-]2
[CH3CH2CO2H]

= 1.0 x 10-s = 10 x 10-6

This imglies that the relative value of [CH3CH2Co2-] to [CH3CH2Co2H] is 3.1 x 10-J to 1. This value is less than one percent confirming choice D.

fiO x 10-3 to 1 which is

90'

passage/

treated with chromic oxide and sulfuric acid, which oxidizes the primary alcohol into i carboxylic acr Choice A results in a product that will convert blue litmus paper to red, ,o ihoi." A is valid. In choice B, L alkene is tetrasubstituted, therefore the carbonyls that are foimed must be ketones. They cannot be oxidized reduced) into a carboxylic acid. This means that the product in choice B witl not convert blue litmus paper red, so choice B is the best answer. In choice C, the anhydride is readily hydrolyzed into two equivalents acetic acid' This eliminates choice C. In choice D, an ester is hydrolyzed into an alcohol and a carloxylic This eliminates choice D. The best answer is choice B.

Choice B is correct. A carboxylic acid will turn blue litmus paper red when added to it. From the nature of t it should be implied that the.litmus paper is used to detect the acid. In choice A, a primary alcohol

91" Choice D is correct. The sequence of reagents used will add a carboxylic acid group to the alcohol carbon (cari Z) It required background information to know that PBr3 converts an alcohol into an alkyl bromide. If didn't know that before, then it's officially background knowledge now. This final produ.t i, .o,''por"J .' carbon with an ethyl group, methyl group, and a"carboxylic acid [rorp attached. This makes choice D the br ans\4/er.
",

Copyright @ by The Berkeley

Choice C should have been eliminated, because it contains six (rather than five) carbons.

Review@

a6

cARBoNyLS & ALCOHOLS EXPLANATIOI

choice B is correct. At general rule, hydrolysis reactions are reversible, because each step is an equilibrium I step' The Grignard reaction is not reversible however, therefore the best answer is choice B. Choices A and D should
have been eiiminated, because a lactone is an ester (a cyclic ester), thus they are the same answer.

correct. As a general rule, the pKu value of an acid is roughly 5.0. This makes choice D not true. Acids do.result in aqueous pH values less thin 7.0, because they are acidlc (choice A). The broad IR peak results from the hydrogen bonding and is found between 2500 cm-1 ur'ri gooo .* 1 i.hol;; ei t"r, than an alcohol stretch because the bond is weaker (and thus easier to stretch). Acids will exchange protons with a protic solvent, so the acidic hydrogen can readily be deuterated. This results in the disappea"rance of a peak in the proton NMR ichoice C).
Choice D is correct. listed in the passage, the addition of thionyl chloride (SoCl2) followed by an amine to -As a carboxylic acid will result in the formation of an amide. This eiiminates choices A and C. An aldehycle is oxidized into a carboxylic acid by the addition of H2So4 and Cro3 as shown in the pu*ug". n ; ;;;;;wer is :herefore choice D.

Choice D is

Choice A is correct. Under basic conditions, the alkoxy nucleophile group attacks the sp2-hybridized carbonyl :arbon of the ester (see below) to form un sp3-hybridized tetrthedr;t iniermediate rt,ith the negative charge .:rfted to the carbonyl oxygen. The best answer is choice A.

J$

.->

R'O

.ta

--+ /.. R
OR''

oR"

Intermediate

choice B is correct' The Claisen condensation reaction involves the deprotonation of an alpha hydrogen on a' :Slr, rsulting in a highly nucleophilic carbanion, which attacks another ester in solution. The ultimate :: duct of a Claisen condensation reaction is a B-ketoester. The Claisen condensation reaction definitely uses an ":er reactant, so choice A is eliminated. A Grignard reaction involves the addition of a nucleophilic alkyl ':- :qnesium bromide species to a carbonyl compound. An ester certainly qualifies as a carbonyl compound. The -:rqnard reagent can add twice to an estet, iesulting in the synthesis of a tertiary alcohol with at least two -ertical alkyl groups. The Grignard reaction can use an ester reactant, so choice C is eliminated. -::nsesterification, as the name implies, involves the changing of an ester by changing the alkoxy group. This -- lone by substituting one alkoxy group for another on"the"carbonyl carbon. A"t.ur-rr"rterification :':initely uses an ester reactant, so choiie D is eliminated. Friedel-Kraft alkylation is not a reaction reaction you are :=-'rired to know, but that is irrelevant in this question. The term alkylation impiies that an alkyl group is : =-rg added, and this does not lend itself to the uie of an ester. An ester is likely to add an :=:cts' This means that a Friedel-Kraft aikylation does not require an ester reactant, so acyl group"wnen it choice B is the best

-hoice A is correct. Treatment of a G-diacid with heat will drive off water (dehydrate the B-diacid) to form -- corresponding acid anhydride. Choice A is "-' :rsible, so acids can be formed from anhydrides an acid anhydride, so it is the best answer. The reaction is upon the addition of water.
a carbon with at least two bonds to oxygen n-bond from carbon to oxygen). Only choice C (a ketone) has two bonds to oxygen, so it must " = ::e best answer' Phenols, tertiary alcohols, and ethers all involve carbons that only have one Uo"r-ra to oxygen, . -:.ev are all eliminated.

-roice C is correct. In order to be reduced, the compound must have


:
- --1

o-bond and

: ::ed

-:roice C is correct. Carbon number three, the third carbon, of the alcohol is involved in no n-bonds and it is to four other atoms, so it must have sp3-hybridization. Pick C for optimal results.

3 by The Berkeley Review@

a7

CARBONYLS

&

ALCOHOLS EXPLANATIONS

100. Choice C is correct. Secondary alcohols, when oxidized, form ketones. The product following the oxidation of 2-butanol using a chromium VI species is 2-butanone. The product is drawn below along with its proton NMR
peaks. Pick choice C.

ll " u - C- b -- CFL CH{ " HzC'

a: 3 hydrogens next to a carbon c: 3 hydrogens next to a carbon

with no hydrogens: Singlet (3H) with 2 hydrogens: Triplet (3H)

b: 2 hydrogens next to a carbon with 3 hydrogens: Quartet (2H)

Copyright @ by The Berkeley Review@

aa

CARBONYLS

&

ALCOHOLS EXPLANATI

SeCtiOn VI
byTodd
D-Glucose

Monosaccharides

"' :"11i:'":J:[r,::[*".
iii. Common D-Sugars
Anomers

ii. Pentoses and Hexoses

Carbohydrates '' i"il:;""fferism ii.


Bennett nVO

iii. tlaworth Projections iv, Mutarotation v. Isomer ization

o" "zf HO{-H


u3f oH ,, sl -f "" cH2oH o rH HOH?C - \_ -o
J

Oligo' and polysaccharides


a) Linkages (Acetal Connectivity) b) Disaccharides c) Trisaccharides d) polysaccharides

i. Linkages ii. Cellulose and Starches


iii. Branching

1 i

"flffio"
t

bn L

chemical Keactions and rests


a) Nitric Acid Sugar Oxidation b) Osazone Test c) Benedict's Test d) Tollen's Test e) Periodate Oxidation f) Kiliani-Fisher Synthesis S) Ruff Degradation h) fMohl Degradation

B-D-glucopyranose

OH OH

JOH

{-O- (G-D-glucopyranosyl) -B-D- glucopyranoside

Biological Apptications
a) Blood Types b) Glycolysis

Speci altztng

in MCAT Preparation

Carbohydrates Section Goals


oB
Be able to recognize ommon monosaccharides. There are certain monosaccharides that recur in both organic chemistry and biochemistry that you should recognize and be able to draw. These include glucose, ribose, fruciose, mannose, andgalactbse. it may be easiest to recall how the other sugars relate to glucose. For instance, mannose is the C-2 epimer of glucose. If you know the structure of glucose, then you know the structure of mannose. Be able to inter-convert between Fischer and tlaworth Prqiections. Some passages and questions will assume that you can translate between structures. For instance, the Fischer projection may be given in the passage, but the question may center around the structure in a Haworth projection. In translating structures, there are three separate points to observe. The chirality of the anomeric carbon is dependent on the direction of the attack. The chirality of the penultiinate carbon (carbon five in aldbhexoses) is constant according to the D or L status,'and the backbone hydroxyl groups on the left in the Fischer projection are up in the Haworth projection. Be able to solve the chiralitv of an unknown suqar ftom reaction data.
data presented. This makes for an ideal passage, althougbr the MCAT'writers raiely use this information in a passage, just single questions. _You may recall evaluating whether a suiar would oxidize into an optically active or inactive diacid.
From organic chemistry class, you may recall solving the identity of a sugar by evaluating the reaction

:]I

lll

$.

av

&L

,0rl

ml
mn

fifid,l1

0Drun

@v

,ed

yi\[i
m,ni

M
1m
l&m

@?

Be able to recognize the chirality of cyclic and linear sugars. It is rather simple to solve for the chirality of selected. carbons in both the Fischer and Haworth proiections, aftbr you have done this once. The rules are the same, so determine a quick way to identify the R and'S configuration of each hydroxyl group of the sugar Be able to identify common disaccharides.
As with the monosaccharides, there are certain disaccharides that recur in organic chemistry and biochemistry. The disaccharides you must recognize include sucrose, lactose, and maltose. Know the two monosaccharides that comprise the disatcharide and the glycosidic Iinkage binding them.

W
{fllmfl

dfl

'v
@v

Be able to recognize the linkage associated with a given disaccharide. The linkage of a disaccharide is defined by the chirality of the anomeric carbon and the carbon of the sugar containing oxygen of the linkag"e. The disac-charide that presents the greatest difficulty is sucrbse, which is conipbsed of two anomeric carbons (one that is cr-1 and thebther that is G-2). Be able to distinguish epimers and anomers. You must be able to identify the difference in chirality between diastereomers, which in the sugar questions are commonly presented as either alpha or beta in regards to an anomer or epimels.

I{now the common reactions involving sugars and sugar derivatives.


This booklet contains a large number of sugar reactions, perhaps more than you need. Key reactions include the Kiliani-Fischer synthesis, Ruff degraclation, osazone formation, and nitric acid oxidation. The passage usually provides plenty of reactions, so don't memorize them, understand them.

Know the common polysaccharides and their biological significance.


You must recognize the difference between glycogen and cellulose. The structural difference simply involves the li[kage, but the difference in reattiv-ity is significant.

Organic Chemistry

Carbohydrates

Introduction

Carbohydrates are organic compounds that contain carbon, oxygen, and hydrogen in a 1 : 1 : 2 ratio. The term is derived from carbon that has been hydrated in a ratio of one carbon to one water, which explains the C : H : o ratio of 1 : 2 : 1. when we consider carbohydrates, we typicarly think of them as ,qugars. For this section, we will focus exclusively on sugars and their organic ;hemistry and biochemistry applications and exampres. while we tike a 'lecidedly organic chemistry perspective on this material, you should pay extra attention to any subjects that bridge organic chemistry and biochemistry. Some .rf this material will overlap with topics in the metabolic components and metabolic pathways sections of the biology books, but it will be piesented in a rnanner that aims at chemistry concepts and short cuts. "\-e ,--omenclature that describe sugars by their functionality and carbon count. In '"ddition to knowing generic nomenclature, you must be able to identify specific =ramples of aldohexoses, aldopentoses, and ketohexoses. Common sugais that should recognize in all traditional structural representations (Fischer, ' _ou :{arvorth, and chair) are glucose, ribose, fructose, galactose, and mannose. we

shall consider rules and definitions for sugars first. There is generic

"'ill present mnemonic devices to help with the recali of these structures. ::ereochemistry is a significant part of sugar nomenclature, so it is necessary to :.row the stereochemistry associated with the prefixes of D and L.
l lonosaccharides of five carbons or more are typically found in cyclic structures ::ther than straight chain structures. To form the cyclic structure, a hydroxyl :roup attacks a carbonyl carbon to form either a hemiacetal (if the sugar is in :. Jose) or a hemiketat (if the sugar is a ketose). Converting between linear and --rlic representations involves knowing how to draw thehydroxyl groups at =:ch carbon. we shall also consider the relationship of sugars. In the cyclic ftrm,

...ain and cyclic forms, epimers are possible. we shall emphasle


,
.

-'u must be able to determine what anomer is represented. In both the straight

:reochemistry terminology.

.' -'nsidering much of sugar chemistry involves multiple sugars, we will address :-, cosidic linkages and how to recognize the type of linkage. we will consider =---1nv examples from disaccharides, to trisaccharides, to oligosaccharides, and -:.a11y polysaccharides. Alpha and beta linkages impact the reactivity and '::rctural nature. To cleave iinkages, specific enzymes are required. oi most --,:oriety is the enzyme to cleave aipha linkages in glycogen, amylose, and ,-:--r-lopectin, which aliows us to break down starches. we lack the enzyme to ,.=:.r'e beta-linkages, so we are unable to breakdown cellulose.

: n'ill also consider chemicai reactions of sugars. There are three types of - :mical reactions we shall address. one type is used to identify the chiiility at '::cific stereocenters, such as treatment witn HNo3 and osalone formation.
"

::e aks

-:,other type increases the length of the carbon chain of the sugar. The last type down the sugar, one carbon at a time. The last feature we will consideiis -. biologicai reactivity. We wiil look at glycolysis from a practical perspective, ::rer than the detail seen in biochemistry. we will also ionsidertlood typ", :1 the impact of stereochemistry on the recognition of the glycoproteins that =:ermine the blood type.

- :r,right @ by The Berkeley

Review

9l

Exclusive MCAT Preparation

Organic Chemistry

Carbohydrates

Monosaccharides

Ivlonosaccharides
Straight Chain Monosaccharides Monosaccharides have the basic formula Cr.,H2r.,Orl, giving them one oxygen

attached to every carbon and one unit of unsaturation. The one

unsaturation is present as either a carbonyl group or a ring. To form the cyclic structure from the carbonyl structure, a hydroxyl group must attack the carbonyl to form a hemiacetal or hemiketal, depending on whether the carbonyl form is in aldehyde or a ketone. Monosaccharides have generic nomenclature for their structures that depends on their carbon chain length and the type carbonyl functional group. Aldehyde sugars are aldoses and ketone sugars ariketoses. The length of the sugar is also considered in the name. Four carbon sugars are tetroses, five carbon sugars are pentoses, and six carbon sugars are hexoses. Combining length and functional group yields sugar names such as:
Aldohexose: Aldo - hex - ose = aldehyde - 6 carbon - sugar. Ketotetrose: Keto - tetr- ose = ketone -4carbon-sugar. within each chain length, there are a set number of stereoisomers, each with a different common name. The number of stereoisomers can be derived from 2n

unii of

f.;

$-I
r*.d[

:{r

(where n = the number of chiral centers in the molecule). For example, an aldotetroses has two chiral centers (carbon 2 and carbon 3), so there are four possible aldotetroses. The Fischer projection for each of the four aldotetroses are shown in Figure 6-1.

J"" H-F
oH
cFI2OH

nIoH HOtH
I

HoJ-H H-t- oH
cFI2OH

"Y"

,etllodl{m
&r,lffrUwu

notn
no*H
cFI2O H

,MngtrumrE

Mffmmw
1mmillii{rnmmo

cFI2OH L-(+)-Threose Figure 6-1

D-(-)-Erythrose

D-(-)-Threose

L-(+)-Erythrose

*--#'LtlT

ry

"Y

The last chiral center (on the penultimate carbon) in the D-isomers of the sugars has R-stereochemistry, while the last chiral center in the L-isomers of the sugars has s-stereochemistry. There is no correlation between D and L with (+) and (-) optical rotation. As seen when comparing L-threose to D-threose, D- and Lsugars with the same name are enantiomers of one another. That is why one is designated as (+) and the other as (-), indicating they have opposite direction rotations of plane polarized light. sugars are routinely represented by Fischer projections, which are short hand for the top view (top perspective) of a compound in its fully eclipsed conformation.

@-,4!uilfli{

The Fisher projection is derived from the straight chain form which is not the most stable form, but is the easiest form to draw. Any substituent on the right or left side of the main backbone is projecting out at you. In bold wedge drawing, Fischer projections look like bow ties. When the fourth priority substituent is coming out at you, you may recall that the chiral center is the opposite of what you see. This means that in Fisher projections, if the fourth priority substituent is on the side of the chain, then you must take the opposite rotation of the arc that is viewed for the chiral center in two dimensions to ascertain its chirality. The translation into a Fisher projection from a three dimensional structure is shown in Figure 6-2.
*

-:r

right

by The Berkeley Review

92

The Berkeley Review

Organic Chemistry
Rotate the sugar to the all eclipsed form

Carbohydrates

Monosaccharides

A
OH

Viewing
Perspective

OH

cH2oH

HoIs
"**

HoJ-n

"Y:
.1_

Se"'
cH2oH Figure 6-2

'"J

H u

oHH H

FilI"'"
OH

cH2oH

H oHoHH

l",doses and Ketoses ---'oses are sugars with an ardehyde functional group. Ketoses are sugars with

ardotetroses and two =:rtetroses.. A hydroxyl group in parenthesis indicates that the chirality is *-:-:,o!vfl. The only ketotetrose known is ervthulose.

':lne functional group. Figure 6-3 shor.is two

CFLOH --:.-,lotetrose -

-*o -T(oH) -f- oH


I

t" H-l-

gFr2oH H

on

HO
cFI2OH L-Aldotetrose cFI2OH L-Ketotetrose

cH2oH
D-Ketotetrose

Figure 6-3 :',: toses and Hexoses r i - -is are five carbon sugars, whether it be a ketose or aldose. Hexoses are six -- r -r. sugars. Most sugars encountered in biology are pentoses and hexoses. I ;--:e 5-4 shows a pair of D-pentoses and a pair of 6_h"*oser.
r':

-''<o
(oH)

nlo
I

lroul -oH
:FI2OH

H-t- (oH) H-l- foHr HtroHi HIOH


I

CH"OH

u-F oH
cH2oH

t; H--l-

CH,)OH

roHl

H-tI

u-l-

,,-l-ro.,l
roul oH
cH2oH
D-Ketohexose

t'"

cH2oH
D-Aldohexose Figure 6-4

"-iopentose

D-Ketopentose

,: :-iht @ by The Berkeley Review

93

Exclusive MCAT preparation

Organic Chemistry
Example 6.1

Carbohydrates

Monosaccharides

How many aldohexoses are possil-.le?

D.

4,4 8.8 c. 16
32

Solution An aldohexose is a six carbon aldehr-de sugar. Carbon one is an aldehyde, and carbon six is a primary alcohol, so neither carbon 1 nor carbon 6 is chiral. Carbons 2, 3, 4, and 5 are all chiral, so there are 1.6 (24) possible combinations ol chiral centers (i.e., 2R, 3R, 4R, 5R etc.), n'hich makes choice C the best answer. There are sixteen aldohexoses of n'hich 8 are D-stereoisomers (carbon 5 has R
stereochemistry) and 8 are L-stereoisomers (carbon 5 has S stereochemistry). Example 6.2

How many 2-ketohexoses are possible?

4,4 8.8 c. 16
D.
A
32

j1'

ltu

Solution 2-ketohexose is a six carbon ketone sugar. Carbons 1 and 6 are primart' alcohols, and carbon 2 is a ketone, so neither carbon 1, carbon 2, nor carbon 6 is chiral. Carbons 3,4, and 5 are chiral, so there are 8 (23) possible combinations ot chiral centers (i.e., 35,45,55, etc.) which makes choice B the best answer. There are eight 2-ketohexoses of which 4 are D-stereoisomers and 4 are Lstereoisomers.

T]ll"""""""'U

ffiurarCIiil

[.-sur,ry

tuW
m[Wri0r.u*ch.

Example 6.3 How many D-aldopentoses are there?

Mlliltmume

4.2 8.4
D. A

t" r(

,n-

-,sd0[

sw

mm

c.8
16

ilD' [n6
$.n$tniliirnn

Solution D-aldopentose is a five carbon aldehyde sugar in which carbon 4 (the penultimate carbon) has R-stereochemistry. Carbon 1 is an aldehyde and carbor 5 is a primary alcohol, so neither carbon 1 nor carbon 5 is chiral. Carbons 2, 3 and 4 are chiral, so there are 8 (23) possible combinations of chiral centers (i.e2R, 33, 43; 2F., 3R, 45; 2R, 35, 4& 2& 3R, 4R; 25, 35, 43; 25, 3R, 45; 25, 35, 4R; ani 25, 3R,4R). There are a total of eight aldopentoses of which four are D-isomers and 4 are L-isomers, which makes choice B the best answer. If you are wise, picir
choice B.

lMhrnf*rtt',oq,q

'tlilnme

mumr,m

(0mumlms

i@Uiim
,4IMTW
,lilhs

nr,mmn

fdw

trd

aft

,u

Copyright

by The Berkeley Review

94

The Berkeley

Organic Chemistry

Carbohydrates

Monosaccharides

Figure 6-5 shows the eight aldopentoses along with their common names. Again, the D-isomer and L-isomer of Jparticurar sugar are enantiomers (mirror r-ig"rj of one another and haveopposiie optical rotations from one another (tne + uia in parenthesis indicates the direction of the rotation of plane polarized light.)

d
t.

o" no*n
tt--.|-

t:7o H-f-ou
cll2oH

t-Fo

u*ou
"o-l- " no*r,
cll2oH
L-(+)-Arabinose

.to-l-

"J"o' ,,-ltt
cFI2OH

tYo

Ho-+-

"o-f *, rtotH
cFI2OH L-(+)-Ribose

rf
r. R

L-(+)-Lyxose

L-(-)-Xylose

t-y'o .r-l- o.t


cFI2OH D-(-)-Ribose

H\/o

"T:" u-]- o"

,,o-l-.t n-f- o.t


cFt2oH D-(+)-Xylose
Figure 6-5

t-l-.,t

"J"o' .r-lH-F ou
cH2oH
D-(-)-Arabinose

to-f- t ro-lI

" H-l on
cFI2OH

D-(-)-Lyxose

In-

;is ;of
ere

*
-

e.above sugars represent the complete group of aldopentoses (there are eight -:al). As pointed out in Example 6.3, ther" are equal arnounts oi D-r.rgurc Jr-,d -sugars in each group of equivalent carbon sugars.

LErample 6.4
:---r 3

"

rich of the following terms best describes the relationship between D-(-)ose and L-(+)-Ribose?

{,

Anomers

ts. Epimers C. Enantiomers D. Diastereomers


Solution
(the
rbon

)?
(i.e.,

and
mers

pick

:enters vary, the two structures are mirror images, which makes them =nantiomers. This makes choice C correct. Choice A is eliminated, because to be lrlomers, the structures would have to be cyclic. If they were cyclic structures, --1e nomenclature would have included either nlphaorbeta at the start and the ::ame would have been ribofuranose, which implies that it is a five-membered ring :lgar. choice B is eliminated, because the sugars vary at more than one carboi irey vary at carbons 2,3, and 4). Diastereomers refer to stereoisomers rn which :-ct all of the chiral centers change (diastereomers are not mirror images). This =-:rninates choice D. Pick C for optimal results.

:ecause D-(-)-ribose and L-(+)-ribose vary at all of their chiral centers (compare re two structures in Figure 6-5), they are enantiomers. when all of the chiral

-opyright O by The Berkeley Review

95

Dxclusive MCAT Preparation

Organic Chemistry

Carbohydrates

Monosaccharides

c.,

Example 6.5 VVhich of the following terms best describes L-(+)-Lyxose and L-(-)-Xylose?

A. B. C. D.

Anomers
C-2 Epimers C-3 Epimers

--1

::

Enantiomers

:*

Solution
Because L-(+)-Lyxose and L-(-)-Xylose vary at only the second carbon (compare

the two structures in Figure 6-5), they are referred to as epimers. If all of the chiral centers vary, the two structures are mirror images, which makes them enantiomers. Not all of the chiral centers drffer, so choice D is incorrect. Choice A is eliminated, because to be anomers, the structures would have to be cyclic, which they are not, as implied by the names. choice C is eliminated, because the sugars vary at carbon 2 and not carbon 3. This narrows it down to choice B. The two structures vary at carbon 2, so they are C-2 epimers.

To complete our coverage of monosaccharides, let us consider aldohexoses. Figure 6-6 shows the eight D-aldohexoses. Do enjoy looking at them.

o
OH OH

t-oo
H--lI

Enat
,i'urF,Lu

o
HO H
HO

oH

H OH H
OH

HO+H

"Yo
H

J[

H OH

HoIH
I

HO+H

ulou
cH2oH

uoIH ulon
I

uo-l-

C-, D.
$ufouu
ffire#r

t.

cH2oH
D-(-)-Gulose

CH.OH
D-(-)-Idose

cH2oH
D-(+)-TaIose

i:nmrmre

lrugtm

D-(+)-Galactose

,qlrlirtrrrnnrrrin

n*

o
OH OH OH

"I"" u-f
H-lH-lD-(+)-Allose

oH oH oH

Ho+ H

"I
I

H-ro Ho4rr

furuun:

o
HO HO H H OH OH

lll_.u_4"Lll{.

ffinnom[

"+ H+

uJ- oH u-l- ou
I

uIoH
cH2oH
D-(+)-Altrose

c- [m" f,
mi*uttlriln

m[-

ffi- f,-

cH2oH

CH 20H
D-(+)-Glucose

cH2oH
D-(+)-Mannose

Figure 6-6
Example 6.6 D-mannose and L-gulose vary

lhqwd
)t;.r

trffi
u-r

hwdu

chiraLity at which carbon(s)?

A. 2 only B. 3and4only C. 5 only D. 2and5only


Copyright O by The Berkeley Review

llM

h- d[

96

The Berkeley Review

Organic Chemistry
Solution
Because D-sugars
aldohexoses must differ in chirality at carbon

Carbohydrates

Monosaccharides

differ from L-sugars at the penultimate carbon, the two

--gulose and D-mannose must have the same chirality at carbon 2, because Dr.rlose and L-gulose differ at all chiral centers. This makes choice C the best l:lswer. The two strucfures are shown below.
D-(+)-Mannose

5. This eliminates choices A and B. lhe question comes down to whether D-mannose and L-gulose vary at carbon 2 -'t not. According to Figure 6-6, D-mannose and D-gulose differ al carbon 2, so

Hao uo-f- H

L-(+)-Gulose

nolu
u-lI

oH

nlor-r
cH2oH
:r.ample 6.7

r+

;:T

H
H
OH

CH 20H

:,ch of the following sugars contains the fewest carbons?

t. : l'

D-Threose

)-Galactose
L-Fructose L-Idose

: : "ution

, : the same number

..::tose and idose are listed in Figure 6-6, so both are aldohexoses. They each of carbons, so choices B and D are eliminated. Fructose is 'nrrhexose, which means it too has six carbons, Iike the aldohexoses. This --:-rates choice C. Threose is an aldotetrose. Because aldotetroses contain only .-: :arbons, choice A is the best choice.

: ,.::'.ple 6.8 ' -- rse is all of the following EXCEpT the:

'I I "

--a epimer .--3 epimer ---i epimer :-i epimer

of D-altrose. of D-giucose. of D-galactose. of L-talose.

;tion rxlg to Figure 6-6, D-allose has all of its hydroxyl groups on the right in its .--=: projection. D-Altrose has all of its hydroxyl groups on the right except - -rroxyl 2, so it is in fact the C-2 epimer of D-allose. This eliminates choice - -'-lucose has all of its hydroxyl groups on the right - ' the c-3 epimer of D-allose. This eliminates choice except for hydroxyl 3, B. D-Talose has ail of r:oryl groups on the left except for hydroxyl 5, so L-talose must have all of ' ::orvl groups on the right except for hydroxyl 5. L-Talose and D-allose are :' :r,inrS, which eliminates choice D. Galactose has two hydroxyl groups on -:: and two on the right, so it differs from D-allose at two chiral centeri. As
-- rs
:

not the C-4 epimer of D-allose, making choice C the best answer.

trt

O by The Berkeley Review

97

Exclusive MCAT Preparation

Organic Chemistry

Carbohydrates

Monosaccharides

Common Monosaccharides Memorizing the names and structures for all sugars is no doubt a little pointless, but knowing the common monosaccharides is not a bad idea. to**on monosaccharides in biology include ribose, glucose, mannose, fructose and galactose. To help recall the common sugars, we shall present a mnemonic for a few (glucose, mannose, and ribose) and then know how the others relate to them. To help recall glucose, we have a not so nice way to remember the position of the hydroxyl groups. If you flip the page off with the fingernail oi yout middle finger pointing towards the page, your finger tips are positioned" just as the hydroxyl groups of glucose are positioned in the Fischer projection. This helps with enantiomers too. D-glucose is found using your right hand while L-glucose is found using your left hand. Given that your left and right hands are enantiomers (mirror images), D-glucose and L-glucose are also enantiomers. Mannose can be recognized using a similar finger trick. By making a gun figurine using your middle finger and index finger as the barrel, your finger tips again mimic the positions of the hydroxyl groups in the Fischer projection for mannose. You can say "man uses guns, so the gun hand position represents mannose." Ribose has all the hydroxyl groups on the right side of the Fischer projection, so "ribose is all right." This provides three solid mnemonic devices for recalling three common sugars. Other monosaccharides can be learned relative to those three. The straight chain Fischer projection of some of the more famous D-monosaccharides (famous implies that they are found in biological systems) and a suggested memory aid for each one are shown in Figure 6-7.

td

ffi ffi

m
ffi@

,m

Jfn

tmh

@,
'@

iimfir'Itl

'dtrf,r ,m

uG&

,m

WWM

H H H H

Y:"
*o"

n{o
no-l-H

-l- o"
cH2oH

H HO H H

D-Ribose "Ribose is all rightl"

OH H OH

uo-l-u

OH

HIOH
I

nA

ori

Ho+ Ho+

"Y o H+

ffi

mplm

cH?oH
HO *" H

H+ OH

OH H H

Io

{o.t
cH2oH

cH2oH

cH2oH

cH2oH
D-Galactose C-4 epimer of glucose

*o"

D-Mannose "p*x* glucosel" "Man uses his gun!"

D-Glucose

D-Fructose
ketose of glucose

Figure 6-7
The terms below the name of each sugar refer to a way in which each structure can be recalled. "F***'r of course stands for "flip off", right?

Stereoisomerism in Sugars The majority of sugar chemistry centers around the stereochemistry of the hydroxyl groups. As such, we shall focus on stereochemistry of sugars more than the chemistry of alcohols, carbonyls, hemiacetals, or acetals. In sugar chemistry, it is critical that you be able to determine how stereoisomers relate tc one another, whether they are enantiomers, epimers, anomers, or structuraisomers. The term epimers should not be mistaken with anomers, which refers ta two sugars in their ring forms that vary at the hemiacetal (in the case of aldoses or hemiketal (in the case of ketoses) carbon in the ring. It is easiest to say tha: anomers vary in chirality at the most oxidized carbon while epimers vary i:: chirality at any one of the less oxidized carbons. Anomers vary in chirality at the most oxidized carbon while epimers vary in chirality at any one of the less oxidized carbons.

fl
d

Copyright O by The Berkeley Review

9a

The Berkeley Kevier

Organic Chemistry

Carbohydrates

Monosaccharides

:1

I
a

I.
.e

.e

\S

;e :e
q

Epimers Epimers are sugar diastereomers that vary at one stereocenter (chiral center) in the carbon backbone. D-Threose and D-erythrose are C-2 epimers, because they vary at carbon 2. D-Glucose and D-Galactose are C-4 epimeis, because they vary at carbon 4. In the straight chain form of a monosaccharide, the carbonyl carbon is most oxidized, but because ithas sp2-hybridization, it has no chirality. The last carbon has no chirality, because the carbon is primary with two hydrogens. This means that two sugars cannot be epimers at the last carbon or the carbonyl ;arbon in the straight chain form. The straight chain form is not the most stable form, but it is an intermediate when one anomer is converted into the other lnomer. Anomers
The most stable form of a sugar is the cyclic form, which is formed when a :r'droxyl group attacks the carbonyl carbon yielding a hemiacetal. A new chiral :enter is formed when the hydroxyl attacks the carbonyl. Because the attack can :;cur from either top or bottom, there are two diastereomers formed. The two iiastereomers that result when the ring structure is formed are referred to as rromers and the new chiral carbon is the anomeric carbon. The two anomers are ::ferred to as the alpha (axial for D-pyranoses) and thebeta (equatorial for D:r'ranoses) anomers. The beta anomer is typically more stable than the alpha :romer. It is only in the cyclic form (furanose form for five-membered rings and ::,'ranose form for six-membered rings) that anomers are possible. Figure 6-g

LrI

fS

or
its

er

or
\-e
US

rsl

::.ows the anomers of glucose, o-D-glucopyranose and B-D-glucopyranose.

HOH"C

HOH2C.

Ho \\

o
OH

Ho$oH
B-D-glucopyranose All substituents have

HO HO

equatorial orientation

cr-D-glucopyranose All substituents have equatorial orientation, except the anomeric OH

OH

Figure 6-8

the
:lore
_rgar

:,r membered rings are referred to as pyrans while five membered rings are :=:erred to as furans. The name B-D-glucopyranose comes from the anomeric -'.'Jroxyl group being cis to carbon 6 (B), D because the last chiral center (carbon i has R-stereochemistry, gluco from the sugar glucose, and pyranose because it -' a six-membered ring sugar. All cyclic sugars are named in a similar fashion. 'l-rnosaccharides come as either five-membered or six-membered rings, so you :: -rpld be familiar with the term furanose as well as pyranose. Ribose in its -'-" :Lic form is a furanose as you are no doubt familiar with from biology.
In cyclic sugar structures, the term alpha technically means that the anomeric oxygen is trans with the last carbon and the term beta technically means that the anomeric oxygen is cis with the last
carbon.

te to

rrral
rs to
rses)

that

rvin
I
I

I
I

,'"*

99

Exclusive MCAT Preparation

Organic Chemistry
Mutarotation

Carbohydrates

Monosaccharides

straight chain intermediate. conversion from one anomer into the other is known as mutarotation. Figwe 6-9 shows the key structures in mutarotation.

The two anomers of a sugar exist in equilibrium in aiao and can interchange via

H
a

ftrd

o
6

fum

wd

OH

HOH,C

OH

HO

H
OH OH

Ho--4
HO
OH

OM

,im
ffim

3
(64%)

'oH

G-D-glucopyranose

-:-

Mn

o-D-glucopyranose
(36%)

cH2oH
Straight chain
(< 0.1%)

W trr
mrm i@Iu

sffiil

Figure 6-9
Example 6.9

Mutarotation involves which of the following?

A. B. C. D.

Sugardecomposition.
Sugar dehydration.
Sugar oxidation.

Stereoisomerization.

Solution
As shown in Figure 6-9, mutarotation involves the conversion between anomers. sugar decomposition occurs with degradation or digestion, which does not describe mutarotation, so choice A is eliminated. Sugar dehydration involves the loss of water by the sugar, which does not occur with mutarotation, so choice B is eliminated. Dehydration is accomplished chemically by a highly hygroscopic
substance such as concentrated sulfuric acid. Sugar oxidation would require-an oxidizing agent, which is not present in mutarotation. Choice C is eliminated. The process of mutarotation involves the conversion from one stereoisomer into

another, which is described by the tercn stereoisomerization Choice terrific answer for a question like this.
Example 6.10 aiao , D-glucose is MOST stable in its: alpha anomer of the pyranose form. beta anomer of the pyranose form.

D is one

In

A. B. C. D.

straight chain form. furanose form.

Solution
The pyranose (six-membered) ring form is more favorable than the straight chain and the furanose (five membered) ring form. This eliminates choices C and D. The beta anomer, with the anomeric hydroxyl group in equatorial orientation, is

more favorable than the alpha anomer. This eliminates choice A and makes
choice B the best answer.

Copyright O by The Berkeley Review

loo

The Berkeley Review

Les

Organic Chemistry

Carbohydrates

Monosaccharides

iaa

ris

)'
OH

Haworth Projections The Haworth projection is a simplified representation of cyclic sugars. They have no physical relevance, but are easy to draw (just like Fisher projections). They do not show the three dimensional spacing that chair conformations show, but they emphasize positions above and below the ring, which is the major focus rvhen considering sugars. Converting from the Fischer projection to the Haworth projection is easy once you get the hang of it. To go from the straight chain to the ring form, recall the phrase "downright uplefting," which applies to carbons 2, 3, and 4 in an aldohexose, but not the penultimate carbon (which has its oxygen in the ring) or the anomeric carbon. Any hydroxyl groups on the right side in the Fischer projection end up below the ring in its cyclic structure. Any hydroxyl groups on the left in the Fischer projection end up above the ring in its cyclic structure. Numbering the carbons makes it is easier to convert to the Haworth orojection, because it helps to keep track of carbons 2,3, and 4. The anomeric carbon is given the far right position in the Haworth projection. Figure 6-10 Haworth projection of D-galactopyranose. 'hows the generation of the
on

OH .'. H

right

down HO

OH
6t

C FI2OH

)mers.
es
,-es

OH
rL

not

the .ce B is
'scopic rire an mated.
.er into

left H ,..w HO
6'

"ffi"
HOH
Figure 6-10
6

OH
C I-{2O H

is

one

-:.e anomeric carbon is shown at the far right in furanose rings just as it is in :i'rarlose rings. Fructose and ribose are classic examples of naturally occurring :Jarlose rings. The carbonyl group of fructose is on carbon 2, so when oxygen of . ','droxyl 5 attacks the carbonyl, the ring is five atoms in size. Figure 6-11 shows :.e Haworth projections of cr-D-mannopyranose and B-D-fructofuranose.

ucHroH

rlht chain

HOH2C

OH

H
ln

HO
2'

cH2oH

and D.

HO

tation, is

HH
ct-D-mannopyranose

HOH
B-D-fructofuranose

d makes

Figure 6-11

'Review

-,l.right

by The Berkeley Review

lol

Exclusive MCAT Preparation

Organic Chemistry
Example 6.11

Carbohydrates

Monosaccharides

l :

The following structure is the Haworth projection of what sugar?

fl

]T

t:

A. B-D-glucopyranose B. G-D-mannopyranose
C. D.
a-D-glucofuranose u-D-mannofuranose

Solution
Choices C and D are eliminated immediately, because the ring in the question is six-membered, not five-membered. Translating from the Haworth projection to the Fischer projection might make it easier to determine the identity of the sugar.

Hydroxyl 2 is up, hydroxyl 3 is up, and hydroxyl 4 is down in the Haworth projection, so hydroxyl2 is left, hydroxyl3 is left, and hydroxyl4 is right in the straight chain form. This corresponds to the Fischer projection of mannose. The best answer is choice B. It might be easier to decide whether the compound is glucose (choice A) than it is to deduce the sugar's identity. B-D-glucose has the hydroxyls alternating between up and down as you move from carbon to carbon. The structure in this question doesn't show this, so it can't be glucose. The only choice left is mannose, the C-2 epimer of glucose. You may viant to verify that only the second carbon is different from B-D-glucose. The structures of the various sugars are drawn below:
6CH2OH

o
sOH HO
2

OH

OH

HOH
B-D-glucopyranose

HH
B-D-mannopyranose

H H
OH

H OH HO OH
H

ucrtrott

OH

cH2oH
D-glucose

cH2oH

HOH
o-D-glucofuranose

HH
cx-D-mannofuranose

D-mannose

The important skill to develop here is the ability to quickly convert from the ring structure to the straight chain structure, as these are the common ways in which you will see the sugars.

Copyright

by The Berkeley Review

r02

The Berkeley Revieu

Organic Chemistry
Isomerization

Carbohydrates

Monosaccharides

In glycolysir, tt" ".rrv_" io rructose _-nto )rocess similar to tautomerization converts dihydrJxya"itor^," pr.,orphate into slvceraldehyde 3-phosphate- Figur" o-rz ,rrr*s the step-by-step conversion of an aldose into a ketose in both u"ia una Ju ". Ketose-Aldose Isomerization (base and

Lr addition to mutarotation, monosaccharides can convert from an ardose into ketose via an enediol intermediat". ihil conversion is known as ketose_ardosea ::omerization

6?h?;p;;;;lJ;;"1,;:r,:::';i::,ru"il',r:i;T:"":*ll',r"i,:,;

uo-l- u H-l- oH +ry*


ur-l- ort
cH2oH

"Yt H*otr

acid catalyzed)

H
deprotonate H on C-2

HO

H H

OH H OH OH

Hoprotonate Y' 'o-on

H OH
deprotonate OH on C,2

HO

C-t

-!r._

Ho#u
I

u-fI

H+OH
oH
cH2OH

-=-

H {o-

oH
OH OH

H. H-

cH2oH

+
t,
H

',"::H,|f".
H.o\-g
HO^_
H

CF CH,zoH protonate c-1

H#oH uo-Fu H*or-r


H-fout cH2OH

deprotonate ||- o"

H-r_
protonate
-__l-

*{'no-l-n H#ou
HtOH
I

tl

Ho*H

oH F O*tt

I Hdeprotonate OH on C-2

HtOH HtOH
cH2OH

H( o-

cH2OH
Figure
6_12

HH-

f +

OH

o
H
OH

OH
i

CH, zoH

that in the reaction mechanism. The requirement are being either gained '::ration of either the enol (under acidic conditions) for the conversion is the where tn" .u.uo'yl oxygen '' :rotonated and the alpha hydrogen is deprotonated, or tne rormation of the ':"rate (under basic conditrons; #nere the arpha hydrogen is deprotonated. 1en isomerization is catalyzei a,y un it often proceeds through an "-,'yrne, '" :nine intermediate. As Figure ol-n shows, conversion tetween C-2 epimers ;--icose into mannose) arso pr"oceeds tnto"g^ an enedior intermediate.

e boided

-ost

hydrogen atoms are the active hydrogens

r\I I

u
++H*

H'oy. H

Isomerization of glucose into mannose (acid catalyzed)

t-t+oH * -'+H |
I

.-+-oH=-' -t- oH
cFI2OH

Ht oH Ho+H -H* _uo-Fn

Ho\-H l- on

H-o\uo-f,_f_

Ht ou

.r-:

cFI2oH
Figure 6-13

l-Glucose

H--oH H-J- oH cFI2oH

*H.__-

a,o#n

o\-H uo*H
_Ho

H-+-oHF
on
cFLoH

"o-l-u, .r-l o"

H-l

ou

Enediol intermediate

cF{2oH
D-Mannose

::,. right

by The Berkeley Review

l05

Exclusive MCAT preparation

Organic Chemistry

Carbohydrates

Monosaccharides

Example 6.12 In the conversion from glucose into mamose, one of the intermediates is:

A. B. C. D.

ribose. a six carbon enediol.


galactose.
a

five-carbon ketose.

Solution Under basic conditions, the conversion involves the deprotonation of an alpha hydrogen, which forms an enolate intermediate. protonation of carbon 2 regenerates the monosaccharide as one of two epimers, mannose or glucose. Under acidic conditions, the conversion involves the protonation of the cirbonyl oxygen and the deprotonation of an alpha hydrogen, which forms an ened.iol intermediate. Protonation of carbon 2 regenerates the monosaccharide in one of two stereoisomer forms, which leads to either mannose or glucose. The best answer is therefore an enediol, choice B. If enolate and enediol were both choices, then the reaction conditions would have been required. Because enolate is not a choice, it is safe to assume that the conditions are acidic. Figure 6-13 shows the conversion. No carbon is gained or lost during the conversion pto""ss,

_^rExample 6.L3 what is the name of the process that converts 3-phosphoglyceraldehyde into 1phospho-1,3-dihydroxyacetone?

A. B. C. D.

Aldehydosis
Enediolosis
Ketose-aldose isomerization Transphosphorylation

Solution
change in numeral is a result of a change in the functional group priority numbers, and the phosphate group in fact never changes its position. As suctr, transphosphorylation is an incorrect answer choice designed to trip up people who didn't draw the structures and assumed the numbering changea le"arrt" *tt group moved. Choice D is eliminated. An aldehyde is converted into a ketone, so enediolosis is not an apt description. This eliminates choice B. The overall conversion is from an aldose into a ketose, so the best term to describe the process is ketose-aldose isomerization. This eliminates choice A and makes
choice C the best answer. The numeral preceding the phosphate group changes from 3 to 1, which would lead one to believe that the phosphate group changed positions. However, the

Copyright

by The Berkeley Review

to4

The Berkeley

Organic Chemistry

Carbohydrates

Oligo- and Polysaccharides

Off$6s*dHfiffidu$;Hilfl,,,,ildffiab.6 des
This section addresses disaccharides, trisaccharides, oligosaccharides, and polysaccharides. Disaccharides result from combining two monosaccharides, trisaccharides result from combining three monosaccharides, oligosaccharides result from the combination of two to eight monosaccharides, and polysaccharides result from combining several monosaccharides.
Linkages (Acetal Connectivity) sugars combine when a hydroxyl group of one sugar attacks the hemiacetal carbon of a second sugar, displacing the hydroxyl group from the hemiacetal carbon to form an acetal. This forms a glycosidic linknge, resulting from a dehydration reaction of the two hydroxyls involved. The terms used to describe the orientation of the anomeric hydroxyl group in a cyclic monosaccharide (alpha ard beta) are also used to describe the orientation of the oxygen in the glycosidic -rnkage. When the anomeric oxygen is cis with respect to the last carbon (carbon rumber 6 in an aldohexapyranose), the linkage is said to be beta. Disaccharides
The names for disaccharides are derived from the two component sugars where --he left sugar (using hydroxyl 1 of an aldopyranose) is considered a substituent

-'n the right sugar (using either hydroxyl 4 or 6). The left sugar gets an "osyl" .uffix and the right sugar gets an "oside" suffix. For instance, sucrose consists of :lucopyranosyl and fructofuranoside. In sucrose, the linkage is 1,2, which is rare. -r most cases, the linkage is 7,4, as in the disaccharides maltose, cellobiose ,ormed from the hydrolysis of cellulose), and lactose. These common

:saccharides all contain at least one glucose residue in one of its anomeric forms. -nkages are named for the carbons involved as well as the actual structural .tereochemical orientation) of the anomeric carbon. A picture says so much :.ore than words (at last check the picture was valued at 1000 words), so for that :eason, Figure 6-74 is a picture of four common disaccharides.
HO
I
l

CH 20 H

_o

HOFI2C

o
cH20H
Sucrose

OH
linkage

HO

B-glycosidic

OH

ri
e

Lactose

.:

a-D-glucopyranosyl)-G-D-fructofuranoside) (o-1-B-2 -linked disaccharide)


HOFI"C

( -O-(C-D-galactopyranosyl)- c-D-glucopyranoside)
(G-1,

4-linked disaccharide)

t.-rG\--O.

HOFI,C

Ho\,a\,oS/or
linkage

HOH,C

uo

B-glycosidic
Cellobiose

oH Ho\-za-\-.oH
OH

]-\-'Ho\'^{\

01 a-g] ycos id i c

Iinkage

'or, I uourc.

HO

Maltose

OH

OH

-O-(G-D-glucopyranosyl)-B-D-glucopyranoside)
(B-1, 4-linked disaccharide)

(4-O-(o-D-glucopyranosyl)-a-D-glucopyranosicle) (o-1, 4-linked disaccharide)

Figure 6-14
:lil'

rpyright

by The Berkeley Review

l05

Exclusive MCAT Preparation

Organic Chemistry

Carbohydrates

Oligo- and Polysaccharides

Trisaccharides Trisaccharides are an oligosaccharide comprised of three monosaccharides. Trisaccharides contain two glycosidic linkages. one of the most common trisaccharides is raffinose, which entails a a-D-galactopyranose forming a linkage with its anomeric hydroxyl group with hydroxyL6 of iucrose. Figure 6--15 shor.is raffinose in the Haworth projection and three dimensionally more valid chair form.

OH

\
cH2oH

HOHCX
Raffinose
Trisaccharide of a-D-galactose, u-D-glucose, and B-D-fructose (u-7,4-and o-1-B-2 linked trisaccharide)

HO

oHl
HO

HO

cH2oH

Figure 6-15
Polysaccharides Polysaccharides are exactly what the name implies: multiple sugars. we shall treat polysaccharides as polymers made of four or more monosaccharides. Most are linked from the anomeric carbon of the sugar on the left (carbon 1) to the fourth carbon on the sugar on the right (carbon 4), a'J.,4 glycosidic linkage. Long

chains of sugars (starches like amylose and amylopectin) are common in biological systems. we store our excess sugar in long polymers such as glycogen (poly u-D-glucopyranose), which has 1,4-alpha linkages between ialacent glucopyranoses and 1,6-branching about one out of every ten glucopyranose residues in the polymer. The most abundant of all polysaccharides is-cellulose (poty G-D-glucopyranose) which is used for structural purposes in plants. we are unable to digest cellulose, because we lack the enzyme needed to break B-1,4glycosidic linkages. we produce alpha glucosidase (which cleaves only alphalinkages of poly glucopyranoses) but not beta glucosidase. Figure 6-1-6 shows
cellulose, amylose, and glycogen.

Copyright @by The Berkeley Review

l06

The Berkeley Review

Organic Chemistry
o

Carbohydrates

Oligo- and Polysaccharides

oH
(B-

HOH2L

"t'

Cellulose
1,4-linked polysaccharide)

l"

o
H OH

HOHHOHHOH
Amylose
(

"t

ct-

f{ o
HO

,4Jinked polysaccharide--no branching)

cH2oH

OH

r^v!./

HocH.

s;ph"
(

Glycogen

a- 1,4-l inked polysaccharide-- 1,6-branchin g)

Figure 6-16
There are no 1,6-linkages found in cellulose, only 1,4-linkages. This means that ;ellulose is a linear polymer with no branching. Strands of cellulose are held together by hydrogen bonding between the ring oxygen and the hydrogen on hvdroxyl 3, making the structure uniform on the microscopic level. Each glucose :esidue is approximately 5.1 angstroms from carbon 4 to carbon 1. The
1

llucopyranose monomers alternate orientation in cellulose to allow hydrogen :onding, as shown in Figure 6-16.
E-xample 6.14

t
3

I l 1

3ranching in amylopectin is attributed to what type of linkage?

I t
e

.\. c.

4,4
7,6

B. 4,1
D.
6,4

e
e

ttS

Solution 3ranching is attributed to 1,6-linkages, although in dextran the monomers are -rnked by 1,6-glycosidic linkages. This is memorization for the most part. \mylopectin and glycogen have branching, while cellulose and amylose do not har.e branching. The best choice is C.

Copyright

by The Berkeley Review

t07

Exclusive MCAT Preparation

Organic Chemistry

Carbohydrates

Chemical Reactions and Tests

ffiHffid

ruffiofi$:l 6ii#Egffi

"I

Nitric Acid Sugar Oxidation Nitric acid is rich in oxygen, making it an oxidizing agent. vvhen an aldohexose is treated with nitric acid, both terminal carbons are oxidized into carboxylic acid groups. If the product, an aldaric acid, is meso, then the compound is optically inactive. using retro-analysis, the chirality of the hydroxyl grolps in the original monosaccharide can be determined. Figure 6-12 shows the aldaric acid
formation reactions of four aldohexoses.

rfr

d o

il

Nitric acid sugar oxidation


OH OH

ifi

ru

ffi

H
HO

OH

HO
HO

HO

H HNo? __+ oH H2o, A


OH

H
OH OH
OH

H
OH OH

HNOl
H?o, a

HO

H H

H H

OH OH

-+

cH2oH
Glucose

cH2oH
Mannose

Glucaric acid (optically active)

o OH Mannaric acid (optically active)


O-rH HN03

H OH HO HO H
HNO3 HO

OH OH

H OH OH

-OH
OH

H H
OH

H H
OH

OH OH OH
OH

-_-_+

H2o, L

HO

OH H2o, a OH

iiifr

cH2oH
Galactose

-:>

ffiil

oH

cH2oH
Allose

@
&d Allaric acid
(optically inactive)

Galactaric acid

(optically inactive) Figure 6-17


Example 6.15 The addition of nitric acid to galactose results in: A. an optically active compound.
an optically inactive compound.

& tu
r@

B. C. D.

the loss of a carbon. one formaldehyde and five formic acids.

Solution
becomes optically inactive, because the diacid form is meso (it has a mirror plane through the bond between carbon 3 and carbon 4). This can be seen in Figure 617. Choice B is the best answer.

Nitric acid oxidizes an aldose into a diacid (aldaric acid) product. Galactose

Copyright

by The Berkeley Review

l08

The Berkeley Review

Organic Chemistry

Carbohydrates

Chemical Reactions and Tests

Osazone Test The osazone test has appeared frequently on the MCAT, despite its obscurity in the minds of most organic chemists. The repeated use of the reaction in passages is a testament to the test writer's goal to present topics that are not commonplace for the sake of asking questions that at initial glance look unfamiliar, The osazone test should be more aptly named t}.e epimer test. Vlhen treated with

three equivalents of phenyl hydrazine, an aldose or 2-ketose undergoes a substitution reaction to form an imine, followed by the oxidation of an alcohol into a carbonyl, and lastly a second substitution reaction forming a second imine $oup. Imine moieties form at carbons 1, and2, so any chirality at either carbon is lost. As a consequence, C-2 epimers yield the same osazone. Figure 6-18 shows that D-mannose, D-fructose, and D-glucose, all yield the same osazone.
Osazone (C'2 epimer) test

N-NFU

uo-#H

no-I- ru o" " -l--

H+OH

uo-J-

n-l-

oH

"-F cFl2oH

ort

H*ou
cI-I2OH
D-Glucose

=N-NIjZ uo-l-u HJ-on


H

-1-

oH

cFI2OH
D-Glucose

D-Marrnose

phenylosazone Figure 6-18

3e

osazone test works in conjunction with other information to zero in on the -dentity of an unknown sugar. For instance, if an unknown D-aldohexose yields -.ne same osazone as D-galactose, but does not have the same optical rotation as )galactose, then it must be either the C-2 epimer of D-galactose (D-talose) or the ietose of D-galactose (D-tagatose.)

tsenedict's Test (Fehling's solution) f,enedict's test oxidizes the sugar as copper dication gets reduced. In getting :educed, the copper solution goes from blue to red for a positive test. The aldose :ecomes an aldonic acid, which under basic conditions is deprotonated to form 'r aldonate. Figure 6-19 shows Benedict's test.

o\-u

Benedict's test (Fehling's solution)

Hf
Ho-+-

on
H
2Crt2* -+
5

H*ou u-F on
cFI2OH
Glucose

0H-

o\o n4on HoIH rilon n*ou


cFI2OH
Gluconate

Figure 6-19

J''rpyright

by The Berkeley Review

l09

Exclusive MCAT Preparation

Organic Chemistry

Carbohydrates

Chemical Reactions and Tests

Tollen's Test The Tollen's test involves the reduction of silver ion by an oxidizable sugar. As silver ion is reduced to silver metal, if the solution is spun within the coitainer, thel silver precipitates on the inside walls of the container, resulting in a so called silver miror. As silver is reduced, the sugar is oxidized. the ildehyde functional group oxidizes quickly, resulting in a rapid reaction with an aldose. Primary and secondary alcohols can also be oxidized, so eventually a ketose will react too. The Tollen's test is said to be positive if the silver mirror forms rapidly, indicating the presence of an aldose. Figure 6-20 shows the Tollen's test.
Tollens test

"Y" H+OH
H

o\
Yo-*unn. H+OH 2 Ag(NH?)r. HO*U 2OHH+OH
I

o-l- H
H-]-on H*ou
cFI2OH
Glucose

Ho-#t-l
2

"Yo_
|

NFr4*

----

Ag(Nu.)r. no--{- u
2

n*ori
cFI2OH Ammoniumgluconate

OH-

H-l- oH

"*o., cFI2OH
Figure 6-20 Ammonium mannonate

Example 6.16
The Tollen's test tests for:

C. D.

A. a reducing sugar. B. an oxidizing sugar.


an acidic sugar.
a ketohexose.

Solution

agent. This makes the Tollen's test a test for a reducing sugar (hemiacetal oi aldose). Choice A is the best answer.
Periodate Oxidation A strong oxidizing agent such as periodate , ro4-, oxidatively cleaves the carboncarbon bonds of a sugar. After complete oxidation, the terminal carbons increase by one bond to oxygen and internal carbons increase by two bonds to oxygen, forming several one carbon compounds as products. The complete oxidation of
erythrose, an aldotetrose, is shown in Figure 5-21.

The Tollen's test results in the reduction of silver, so the sugar must be a reducing

Periodate oxidation

H--{- OH
cFI2OH Erythrose

u-l- oH

IOa-

-----+

HCO2H

+ H2CO +

3IO3

Figure 6-21 Copyright O by The Berkeley Review

rro

The Berkeley Review

Organic Chemistry
Example 5.17

Carbohydrates

Chemical Reactions and Tests

The addition of excess periodic acid (HIOa) to fructose yields: A. one formaldehyde and five formic acids. B. one formaldehyde, four formic acids, and carbon dioxide. C. one formaldehyde and four formic acids.

D. two formaldehydes,
Solution

three formic acids, and carbon dioxide.

Fructose is a 2-ketohexose. with excess periodic acid, carbon-carbon bonds are oxidatively cleaved. The terminal carbons (carbons 1 and 6) are oxidized up one level from primary alcohols to formaldehydes (from one C-o bond to two C-o bonds). Because two formaldehyde molecules are formed, the best answer is choice D. The carbonyl carbon (carbon 2) is oxidized up two levels from a ketone io carbon dioxide (from two C-o bonds to four C-o bonds). The secondary alcohol carbons (carbons 3,4, and 5) are oxidized up two levels from alcohoh tt formic acids (from one C-o bond to three C-o bondi), resulting in the formation rf three formic acid molecules. The result is two formaldehyd.es, one carbon cioxide, and three formic acids, choice D. An reaction summary is shown below.

HO- CF{2OH
HO
I

\-^
H

HO=
OH

-o

o=c=o

no*u
OH OH

_ -rv

L_^

n*on
OH OH

-ru \_^ _ -FU \,_^


HO H HO H

HO H

H*on
CFLOH

OH

Ho- cF{2oH

!o

Kiliani-Fischer Synthesis Kiliani-Fischer synthesis increases the carbon chain of an aldose by one. The carbon is added as a cyano group to the carbonyl carbon, so the chain -:',creases by one carbon at the aldehyde end. After reduction to an imine and :idrolysis, the cyano group is converted into an atdehyde. The result is the :rrmation of two aldose epimers with an additional carbon. For instance, when .:'ing D-arabinose in Kiliani-Fischer synthesis, both D-mannose and D-glucose C-2 epimers) are formed. Glucose is72% of the product mixture, because of the ,:.rral influence of carbon 2 of the aldopentose (D-arabinose). Figure 6-22 shows --:..e Kiliani-Fischer synthesis of two D-aldohexoses from D-aralinose. An oH :roup in parenthesis represents the possibility of both chiral centers.

}e

:::::::::::::::erv

-opyright

by The Berkeley Review

lll

Exclusive MCAT Preparation

Organic Chemistry

Carbohydrates

Chemical Reactions and Tests

Kiliani-Fischer synthesis

oY"
KCN uo-tsr+ __>

N lil
C

u*ronr
HO+H

1..H2/PdBaSOn

*,-f- o.,
cH2oH
D-Arabinose

t-l.ot

H"O

z.urolt{

.,-f-

t-J-,rt
o.,
cH2oH

o\-u o\-H t-J-,rt to-l" toJ-rt to-l-*., t-l- ot ,,-l-,r0,

'-l- o' cH2oH


D-Glucose
(72%)

.,-f-

o.,

cH2oH
D-Mannose
(28%)

Figure 6-22
Example 6.18
The Kiliani-Fischer synthesis on a D-aldopentose produces: A. a fifty-fifty mixture of enantiomeric D-aldohexoses.

B. a major-minor mixture of diastereomeric D-aldohexoses. C. a major-minor mixture of diastereomeric D-ketohexoses. D. a fifty-fifty mixture of diastereomeric D-aldotetroses.
Solution Fischer-Kiliani synthesis adds a carbon to an aldose, so with a five-carbon reactant, a six-carbon product is formed. This eliminates choice D. The product is an aldose, because the carbonyl group must form at a terminal carton, so choice C is eliminated. The products are diastereomers (C-2 epimers), so choice A is eliminated. Because attack at the carbonyl carbon is lnfluenced by the asymmetry of the rest of the molecule, the mixture is not fifty-fifty. Choice B is

s*. ff,r
.h
m
,f,_.

t"*

Example 6.19
processes?

The addition of cyanide to a sugar is involved in which of the following

!
Siuu

A. Nitric acid oxidation B. Nguyen degradation C. Kiliani-Fischersynthesis D. Ruff degradation


Solution
As seen in the first step of the Kiliani-Fischer synthesis in Figure 6-22, the cyanide anion adds to the carbonyl carbon of an aldose to yield u cyutro extension of the originai aldose. The best answer is choice C. Because oi the influence of the other chiral centers, the cyano group does not add in afifty-fifty manner to the

Logr

'U

tl:

Tlne
TilrXilr

..4r

n
c.
F}

top and bottom of the carbonyl. This results in a diastereomeric mixture of products. Nitric acid oxidation involves nitric acid, so choice A should have been eliminated. Degradation of a sugar results in the loss of a carbon, not the

:y'"*
Copyright
@

gain, so the reagents should not contain carbon. This eliminates choices B and D, although choice B is not an actual sugar reaction, so you may have eliminated it

ftwiiu
r0mwLlt

*-lllb I

rflqp

by The Berkeley Review

ll2

The Berkeley Review

: ![ST
1r

Organic Chemistry

Carbohydrates

Chemical Reactions and Tests

Ruff Degradation The word degradation implies that something is breaking down. In Ruff degradation, an aldose loses its first carbon, resulting in an aldose of one less carbon. As we saw in the carbonyl section and in several biochemistry examples, iosing one carbon is readily accomplished by decarboxylation. In order to decarboxylate the terminal carbon, it must be oxidized first to a carboxylic acid and then to carbon dioxide. Bromine in water effectively converts the aldehyde into a carboxylic acid (or in the ring form, a hemiacetal into a lactone). The carboxylic acid is oxidized into carbon dioxide by peroxide in the presence of a ferric cation catalyst. CO2 leaves and decreases the chain length by one carbon. Figure 6-23 shows the Ruff degradation of D-glucose. It is rarely used with aldopentoses, because when the product is an aldotetrose, the yield is low, likely lo to the difficulty encountered with a smaller ring in the cyclic form.

Ruff degradation

o\-t

uoJ-n u-l- on

H-]-or.

uf

Br, ca(oH), --' -lI2-' Hzo Hzo

Ho
H H

+" ab -# ou
Fe"'

Y""

HO+H
u

HIor-i
-[oH
cH2oH

oH

cH2oH
D-Glucose

*""
cH2oH

D-Gluconate

D-Arabinose

Figure 6-23 Lxample 6.20 Ruff degradation results in: ,\. the gain of a carbon onto carbon

B. the loss of carbon 1 from the chain. C. the gain of a carbon onto the last carbon of the chain. D. the loss of the last carbon from the chain.

1 of the chain.

5olution )egradation implies breakdown, so Ruff degradation is associated with a loss of :arbon, not a gain. This eliminates choices A and C. As shown in Figure 6-23, it the first carbon that is lost, so choice B is the best answer. -o
Erample 6.21

lhe addition of bromine in water to a sugar is involved in which of the following


:rocesses?

{. Nitric acid oxidation B. Wohl degradation C. Kiliani-Fischer synthesis D. Ruff degradation


Solution -{s seen in Figure 6-23, the combination of bromine and water will oxidize a ronosaccharide into an aldonic acid (monoacid) in the first step of the Ruff Jegradation. The best answer is choice D.

Copyright

by The Berkeley Review

I t5

Exclusive MCAT Preparation

Organic Chemistry

Carbohydrates

Chemical Reactions and Tests

Wohl Degradation Wohl degradation is essentially a Kiliani-Fischer synthesis in reverse. An aldose is first converted into-its oxime using hydroxylamine to which acetic anhydride and acetate are added to acylate the hydroxyi groups and dehydrate the oxime into a cyano group. The cyano group is removed under basic conditions, which also hydrolyze the esters on the sugar. The result is an aldose of one less carbon. Figure 6-24 shows the Wohl degradation of D-ribose. Wohl degradation

"f

rIo*,
D-Ribose

H-F

OH

H--F .-.>
-

OH

a o ili
CH2OAc

o\'
cH2oH

H2NOH

o"

Hzo

H-J-

Hf

oH

oH

cH2oH

cH2oH

*# ut oac N"ofc,f :+oA"NuocH,-;f:: "t1ec


Figure 6-24

D-Erythrose

Copyright

by The Berkeley Review

tt4

The Berkeley Review

Organic Chemistry
Biid[##fi $ffi iiiiffi ffi

Carbohydrates

Biological Applications

plicariffi

Blood Types As a general rule, natural sugars have D-chirality. This is because our enzymes recognize D-sugars, and therefore select for D-sugars. One notable exception to the "D only" rule includes the presence of L-fucose, a reduced form of Lgalactose, in the glycoproteins of blood type markers. An oligosaccharide defined as a polysaccharide with anywhere from two to eight monosaccharides) Ls bound to the amino terminal of a protein on the surface of a red blood cell to function as the antigenic determinant. Figure 6-25 shows the terminal sugars of :he three common antigenic markers. You should be able to ascertain from the slructures why type O is the universal donor. Type O (Glycoprotein)

o:1

\1 CH.

6
prote ln

Types A and B (Glycoprotein)

HO CF{.OH
(for Type HO CFLOH

cF{2oH

=OH

o
HO

"1
HO
Figure 6-25

--- ucose is the sugar

with a CH3 group, rather than a CH2OH group, at the :::bon 6 position. It is bonded to the central galactose residue via an alpha-7,Z:*. :osidic linkage. Where type O differs from types A and B is that is does not -::.-e a fourth monosaccharide linked to hydroxyl 3 of the central galactose ::,i:Cue. Types A and B differ from one another at the carbon 2 position of the ' ':rth sugar. Type B has an ordinary galactose residue as the fourth sugar, and :--:s has a hydroxyl group on carbon 2, while type A has the carbon 2 hydroxyl ::up replaced by an N-acyl group.
,

,:r'right

by The Berkeley Review

l15

Exclusive MCAT Preparation

Organic Chemistry
Glycolysis Highlights

Carbohydrates

Biologicat Apptications

ll

Glycolysis is a ten-step metaboLic process that converts glucose into two pyruvate molecules. During the process, two net equivalents of adenosine triphosphate (ATP) are formed from adenosine diphosphate (ADp) and inorganic phospnate. Glycolysis is oxidative in nature (given that glucose is being bioken do#n;, so two equivalents of the oxidizing agent NAD+ are required". However, because glycolysis takes place in the cytoplasm, the NAD+ levels are kept low, so it gets used up quickly. Figure 6-26 shows the overall glycolysis reaction.

i(

r{

;[

fi

l0

2H2O

o-

tfi

fit

q 2 ATP

d
2

ADP
2 NAD+

4 ADP

HOH
Glucose

ATP

,m

CFI3

2 NADH

Pyruvate

,l@

rd

Figure 6-26 Glycolysis is presented in detail in the Biology II book. The goal in presenting

& of some common organic reactions. The MCAT is a thinking test, so we will focus on the big picture and the logic behind selected steps. Tire energy values for each step are derived from in aiao cond,rtrons, not standard .or-rditior-rr. Glycolysis can be segmented into three stages, the first of which converts glucose into two equivalents of glyceraldehyde-3-phosphate (steps I - v), the seJond of which__converts glycera.ldehyde-3-phosphate into a-phosphoglycerate (steps VI and vII), and the last of which converts 3-phosphoglyceraie ;ito pyrrr.rute ('steps Vru - X). Figure 6-27 shows Stage I, the first five steps, of glycolysis
Step I:
AG = -8.0 kcal/mole cFI2OPO32aspects of glycolysis in the organic chemistry book is provide biologicai example!

TN

ffi

th

rffi

fin
L@0

m!

Arn

Step II:
AG = -0.6 kcal/mole

f'11'3t
OHH

L@m

fu,ou

m@

Hexokinase

Phosphoglucose

ffi

isomerase_ .--

HOH
Glucose

ATP

ADP

OH

HOH
Glucose-6-phosphate

Fructose-6-phosphate Step III:


AG = -5.3 kcal/mole

o\-t
Ho-1_H
cFI2OPO32I

Phosphofructokinase

K:
OH

Glyceraldehyde 3-phosphutu
Triose phosphate isomerase

cttoPo32-

ft
t@

Ct-t.OI'O.--

Aldolur"

@
@-

""Y;
Copyright
@

Step V:
AG = o'6 kcal/mole

t., .|
cFI2OH

ILJ

IWT

CF{2oPo"2-

Step IV: AG = -0.3 kcal/mole

whr{

Glyceraldehyde-3-phosphate

Dihydroxyacetone phosphate

OHH
Fructose-1,6-bisphosphate

lrlndj
iil ?

Figwe 6-27
by The Berkeley Review

@"
The Berkeley Review
q-,l0[

l16

Organic Chemistry

Carbohydrates

Biological Applications

step I (First committed step in glycolysis): Conversion of glucose into glucoser-phosphate releases a great deal of energy, so it is the first committei step in llvcolysis. A pathway's first committed step is the first essentially irreversible :eaction in the pathway. This is true in many mechanisms. once a highly :avorable step transpires, the rest of the steps in the mechanism rapidly follow. -n the case of step I, in addition to the energetics not allowing for the back :eaction, once glucose is phosphorylated, it carries a negative charge, so it cannot tffuse through the plasma membrane. Although glucose is too ptlar to diffuse -:"rrough the membrane on its own, it crosses with the aid of proteins called :lucose transporters. Glucose transporters act like enzymes in th;t their activity -' reversible (if glucose levels get too high, then they can pump it out) and .:ecific (they recognize glucose, not glucose-6-phosphate). step I ii a regulatory ':ep, meaning that the activity of the enzyme is controlled (allosterically)by the -.r-els of ATP and other metabolites. Regulation always occurs at irreversible .:eps (rather than reversible steps), so as a helpful hint, you could look for steps ::upled with the conversion of ATP into ADp, because that generally makes the ::ep exergonic (AG < 0). Steps I and III are points of regulation.
5teps IV (Reverse aldol reaction); Following isomerization of an aldose into a -*:tose and the addition of another phosphate group in steps II and III, the : rlecule is poised to undergo a reoerse aldol rcaction. tn step IV, a six-carbon :.:lecule is cleaved into two three-carbon molecules by a reveise Aldol reaction :-at breaks the bond between carbon 3 and carbon 4 of fructose-1,6-diphosphate

phosphate (DHAP) and glyceraldehyde-3-phosphate ^lorm --3-P)' Fructose-1,6-bisphosphate is a B-hydroxyketone, which undergoes the ::.'erse aldol reaction. In step v, DHAP is converted into G-3-p by wiy of an -'''nerization reaction. By having so many steps, some of the intermediate :rLrducts in glycolysis can be used in other pathways. one example is the :,*'ersion of G-3-P into glycerol and then into fatty acid triglycerides.

dihydroxyacetone

of step v, no oxidation has occurred. In stage II, the carbon-based rpounds are oxidized, releasing energy. The energy is "stored" in a phosphate :',rd. It is at this point that NAD+ is required. Figure 6-28 shows stuge ri or i ,-;olysis, in which G-3-P is converted into 1,3-bisphopshoglycerate.
---'
::

o)

oPo32-

Phosphoglycerate

H-r
'-.
ceraldehyde-3-phosphate
Step

oH

-Z-T
ADP

kinase ATP

o\z H-f

ou

cH2oP032-

cH2oPo32-

VI: l,3-Bisphosphoglycerate Step VII: 3-phosphoglycerate ^ AG = -0.4 kcal/mole AC = 0.3 kcal/mole


Figure 6-28

i:ep VI (Oxidation of an aldehyde into a carboxylic acid derivative): Oxidative :r:sphorylation of G-3-P into 1,3-bisphosphoglycerate is the only oxidation step - :ir-colysis. It is the oniy step where the number of bonds from carbon to , - ien increases. oxidation releases energy, which is "stored" by adding a ----<phate group. oxidation occurs at carbon L as it gains a bond to o*yg"t-t r::n converted from an aldehyde into an acid carboxylate. The term pi r j-cates that there is an inorganic phosphate gained by the reactant. At pH = - - inorganic phosphate exists mostly in its dianion form. Dephosphorylati,on of :-bisphosphoglycerate into 3-phosphoglycerate in Step ViI, releases a " - - sphate, which serves to regenerate ATP that was invested in earlier steps.
, ,:,. right
@

by The Berkeley Review

tl7

Dxclusive MCAT Preparation

Organic Chemistry

Carbohydrates

Biological Applications

In the last stage of glycolysis, the compound loses a phosphate to generate more ATP and then undergoes isomerization and elimination to generate the final product, pyruvate. These three steps are shown in figwe 6-29.

"Y"

Step VIII:
AG = 0.2 kcal/mole

Step IX:

"\"

AG = -0.8 kcal/mole

H-roH
curoror2-

Phosphoglyceromutase

uf

oPor2cFI2OH

Enolase_
Y

o\o l" oPo3ctt


Phosphoenolpyruvate

ffi
@

ll-

ffi

3-Phosphoglycerate

2-Phosphoglycerate

Hzo

ffi

oADP +

H*

o
CHs

ATp

Step X: AG = -4.0 kcal/mole

Pyruvate Figure 5-29

Steps

IX (Enolate formation):

Conversion of 3-phosphoglycerate into 2-

phosphoglycerate positions the phosphate for enol formation in Step IX. Because these two structural isomers have the same bonds, this reaction results in a minimal change in energy. Conversion of 2-phosphoglycerate into phosphoenol

pyruvate involves dehydration. The double bond formed from elimination has

@il

phosphorylated hydroxyl group attached, making the compound an enor. Conversion of phosphoenolpyruvate into pyruvate completes glycolysis. You should not memorize glycolysis, as the amount of time invested to do so would not have a good return in terms of increasing your MCAT performance. However, you should have a basic idea of what happens overall. It is more important that you recognize the type of reaction than it is to know details of any particular step. A passage may present one or more steps of glycolysis and ask you questions from both an organic chemistry and biochemistry perspective. You should know that glucose is converted into pyruvate and that there is a net generation of 2 ATP and a release of energy. It is an oxidative process, so a molecule is broken down. You can appty the general principles discussed here to other pathways, so learn the concepts and insights.

m h m

uq

1) 2) 3)

Regulatory steps usually involve ATP.

Multipstep pathways produce intermediate products that can feed


other pathways.
species poor in FI, such as

Oxidation breaks down molecules and requires an oxidizing agent (a NAD+). Oxidation releases energy.

Copyright

by The Berkeley Review

lla

The Berkeley Review

Organic Chemistry
Oxidation and Reduction of Pyruvate

Carbohydrates

Biological Apptications

anaerobic breakdown. Pyruvate can undergo fermentation, where carbon 2 is reduced from a ketone into an alcohol, resulting in the formation of lactate, l'hich regenerates NAD+. we cannot reduce lactate any further (only yeast and bacteria can convert pyruvate into ethanol), so it is excreted as waste. pyruvate can also undergo oxidative decarboxylation. Converting a carboxylic acid group into co2 results in the gain of a bond to oxygen, so it is oxidation and m.tit therefore be coupled with reduction of NAD+ into NADH. To make these reactions easier to grasp, you should focus on the oxidative or reductive nature of 'Jre pathway. Figure 6-30 shows glycolysis, followed by three potential pathways Jrat pyruvate can take.

Once formed, pyruvate has a few potential fates. It can undergo either aerobic or

o
IRI

Lactate

H CHs

HO HO

o\o
Glycolysis

OH

-__+
Glucose

OH
Figure 5-30

F" CHs
Pyruvate

rRl> CO2 +

CH3CH2OH

[o]>

CO2 + Acetyl CoA

latabolism is the biological process of breaking down a sugar. Given that the :eneration of ATP is considered critical to energy storage, let us finish by briefly -:oking at an ATP molecule. It is made of a nucleoside (comprised of adenine :nd ribose) and a triphosphate group linked at hydroxyl 5. Adenosine :rphosphate is shown in Figure 6-31.
Nucleoside

Triphosphate

o
-O

rt I o-l oI

P-; O- P- O-P
I

il

- O-

o-

4
Adenine

Hydrolysis of ATP cleaves here and releases -7.3 kcal/mole

OH
Figure 6-31

HO

Ribose

is a monosaccharide (usually ribose) bonded at carbon 1 to one of bases in either DNA or RNA. Linkages between phosphate groups generate - 3 kcal/mole when they are hydrolyzed, meaning that when ATp gains H2O, it ,-.aves to form ADP, P1 (inorganic phosphate), and H+ with a LG = -7.3
,--e

-- nucleoside

:al./mole. This is a valuable tool for storing energy in small increments.

r19

Exclusive MCAT Preparation

Organic Chemistry
Monosaccharides

Carbohydrates

Section Summary

Key points for Carbohydrates (Section 6)

1.

b) c) d)

Sugars are named for carbon count and carbonyl type a) Tetrose = 4 C, Pentose = 5 C, and Hexose = 6 C

ii. Glucose: "F*** glucose!", and flip it off. iii. Mannose: "Man goes with gun.,' iv. Galactose: C-4 epimer of glucose v. Fructose: ketose of glucose 2. i. ii. iii.

Aldose = aldehyde sugar; Ketose = ketone sugar D-sugar has last chiral center = R; L-sugar has last chiral center s = common monosaccharides from biorogy shourd be memorized i. Ribose: "Ribose is all rightl"

Cyclic sugars are either furanoses or pyranoses a) Pyranoses are six-membered ring sugars
Beta anomer has last carbon cis to anomeric hydroxyl group Anomeric carbon is the most oxidized carbon (has two bonds to o) B-Glucopyranose has all substituents in the equatorial position Furanoses are five-membered rings i. Ribose and fructose are the two most common examples

b)

Oligosaccharides and Polysaccharides 1. oligosaccharides contain 2 - 8 monosaccharides, polysaccharides contain ? a) Glycosidic linkages bind them (water is released when linkage forms) i. Disaccharides include sucrose, lactose, and maltose ii. Polysaccharides include cellulose, amylose, and glycogen iii. Linkages are o or B, named for the component hydroxyl groups iv. Enzymes that cleave glycosidic linkages are specific

b)

i. ii. iii. 1.

Branching is typically done using hydroxyl 6 of a sugar in the polymer Cellulose (B-linked) and amylose (*-linked)have no bran&ing

Amylopectin (cx-linked) has branching about 1 out of 30 monomers Glycogen (u-linked) has branching about 1 out of 10 monomers

Chemical Reactions and Tests

a) Nitric acid (HNo3) oxidizes terminal Cs to acids: optical activity test b) Phenylhydrazine (OHNNH2)forms an osazone: C-2 epimer test c) Tollen's test (Ag+) oxidizes C-1 of aldoses: aldose test d) Periodate test (IOa-) oxidizes sugar by cleaving all C-C bonds e) Kiliani-Fischer synthesis increases an aldose by one C; forms epimers 0 Ruff and Wohl degradation decrease an aldose by one C
Biological Examples

Chemical reactions and tests determine chirality, add Cs, or remove Cs.

1. 2.

a) b) b)
Copyright O by The Berkeley Review

Blood types have glycoproteins with different stereochemistry Type o contains 3 sugars (Glc-Gal-Fuc) and Types A and B contain 4 L-Fucose is the third sugar, de'iating from the natural sugars are D rule Glycolysis breaks down glucose into two pyruvate molecules and energy a) Forms a net of 2 ATP and 2 pyruvates for further reaction
FIas one oxidation step; mostly isomerization and phosphate exchange

t20

The Berkeley Review

Organic Chemistry
Monosaccharides

Carbohydrates

Section Summary

Key Points for Carbohydrates (Section 6)

1.

b) c) d)

Sugars are named for carbon count and carbonyl type a) Tetrose = 4 C, Pentose = 5 C, and Hexose = 6 C

2.

Glucose: "F*** glucose!", and flip it off. Mannose: "Man goes with gun." Galactose: C-4 epimer of glucose Fructose: ketose of glucose Cyclic sugars are either furanoses or pyranoses a) Pyranoses are six-membered ring sugars i. Beta anomer has last carbon cis to anomeric hydroxyl group ii. Anomeric carbon is the most oxidized carbon (has two bonds to iii. B-Glucopyranose has all substituents in the equatoriar position b) Furanoses are five-membered rings i. Ribose and fructose are the two most common examples

i. ii. iii. iv. v.

Aldose = aldehyde sugari Ketose = ketone sugar D-sugar has last chiral center = R; L-sugar has last chirar center = Common monosaccharides from biology should be memorized
Ribose: "Ribose is all right!"

o)

Oligosaccharides and Polysaccharides

1.

b)

oligosaccharides contain 2 - 8 monosaccharides, polysaccharides contain ? a) Glycosidic linkages bind them (water is released when linkage forms) i. Disaccharides include sucrose, lactose, and maltose ii. Polysaccharides include cellulose, amylose, and glycogen iii. Linkages are o or B, named for the component hydroxyl groups iv. Enzymes that cleave glycosidic linkages are specific
Branching is fpically done using hydroxyl 6 of a sugar in the polymer i. Cellulose (B-linked) and amylose (u-tinked) have no branching ii. Amylopectin (u-linked) has branching about 1 out of 30 monomers iii. Glycogen (o-linked) has branching about 1 out of 10 monomers

Chemical Reactions and Tests

1.

a) Nitric acid (HNo3) oxidizes terminal Cs to acids: optical activity test b) Phenylhydrazine (OHNNH2) forms an osazone: C-2 epimer test c) Tollen's test (Agr) oxidizes C-1 of aldoses: aldose test d) Periodate test (IOa-) oxidizes sugar by cleaving all C-C bonds e) Kiliani-Fischer synthesis increases an aldose by one C; forms epimers 0 Ruff and Wohl degradation decrease an aldose by one C
Biological Examples

Chemical reactions and tests determine chirality, add Cs, or remove Cs.

1. 2.

Blood types have glycoproteins with different stereochemistry a) Type O contains 3 sugars (Glc-Gal-Fuc) and Types A and B contain 4 b) L-Fucose is the third sugar, deviating from the natural sugars are D rule Glycolysis breaks down glucose into two pyruvate molecules and energy a) Forms a net of 2 ATP and 2 pyruvates for further reaction b) Has one oxidation step; mostly isomerization and phosphate exchange

Copyright O by The Berkeley Review

t20

The Berkeley Review

Carbohydrates
Passages
l5
I OO

Passages

Questions

Suggested schedule: I: After reading this section and attending lecture: passages I, III, V, VI, & X Qrade passages immediately after completion and tog"your mistakes. II: Following Task I: passages II, IV, vil, & XI (za questions in J6 minutes) Time yourself accurately, grade your answers, and review mistakes. III: Review: passages VIII, IX, XII, XIII, & euestions 92 - I OO Focus on reviewing the concepts. Do not worry about timing.

,1fu

Speci altztng

in MCAT preparation

E.V.f.E.

I. II. III. IV. V. VI. VII. VIII. IX. X. XI. XII. XIII.

Qlucose and Qlucopyranose Fischer and Haworth Projections

(r -7)
(B

- t4)

Sugar Conventions
Monosaccharides versus Disaccharides Amylose, Amylopectin, and Cellulose

(r5 - 22)
(23 - 2e)
(5O - 56)

Unknown LAldopentose trlucidation Unknown D-Aldohexose Elucidation


Kiliani-Fischer Synthesis Osazone Derivative Test Clycolysis Reactions

(37 - 43) (44 - so)

(sr - 57)
(58 - 64) (65 - 7r) (72 - 78) (7e - 85) (86 - er) (e2 - lOO)

Blood'lYpes Biochemistry of Sugars Combustion of Sugars


Questions not Based on a Descriptive Passage

Carbohydrates Scoring Scale Raw Score


MCAT Score

85 - 100

65-84
47

64

15-15 10 l2 7 -9

53-46

l-32

4-6 l-3

oassage

(Questions 1 - 7)

2.

*-':

The most stable form of glucose is a hemiacetal ring --!-ture. The ring is formed when the hydroxyl group on ".::on 5 attacks the aldehyde group to form a hemiacetal.
h1'droxyl can attack the carbonyl carbon (sp2-center) from

D- Glucose and D-galactose are best described as which of the following?

, :.3r the top or bottom of the n-bond. As a result of this . ..x. there are two possible diastereomers that can form. *:: aldehyde carbon forms the new stereocenter. These two

A. Anomers B, C-2 epimers C. C-4 epimers D. Enantiomers


3.
D-Glucose and L-glucose are:

- ,:i: itatural sugars have D-configuration, it is perhaps easier :.:nk of the beta anomer as the one with the hydroxyl -: *r up (equatorial) in the ring form. Both anomers of D_ : -: -rvranose are drawn in Figure I below.

,.rble diastereomers are referred to as the alpha andbeta ' -:ers of the glucopyranose. The beta isomer (anomer) is ": red as having the anomeric hydroxyl (on carbon one) cis . . respect to carbon number six in the ring form. Because

A. B. C. D.
4.

anomers.

C-5 epimers.
diastereomers.

enantiomers.

D-Glucose has which of the following stereochemical


arrangements?

H
HO

I." Ir
-l- o"
*o"
cH2oH
5

A. 2R, 3R, 4R, 5R B. 2R, 35, 4R, 5R c. 2s, 3R, 4R, 5R D. 25,35,4R,5R

H H

How many stereoisomers are possible for a linear


aldohexose?

HOH,C OH OH
;

\
HOHoC

Ho \\ Ho-\--\

D.
6.

4.4 B. 8 c. 16
32

-D-glucopyranose

oHl oH a-D-glucopyrunor"
In the beta-anomer of D-glucopyranose, the first carbon
has the hydroxyl with what orientation?

Figure
ll

Anomeric forms of D-glucopyranose

Ire B-anomer is favored by a ratio of 64Vo to 36Vo over ,r r-:nomer. Hydroxyl groups on adjacent carbons in the : ,.r form hydrogen bonds from all orientations except . When the hydroxyl groups are equatorial they ':-,rltco less steric repulsion than when they have axial : - :Iion, but hydrogen bonding is slightly reduced. , --:: the B-anomer is more abundant, the reduced steric ' -,:: .ce of the equatorial orientation must be of greater r r."1nce than the slight loss in hydrogen bonding. .- _:;n bonding is possible from the gauche orientation " :::d with diequatorial hydroxyl groups.
',i-hich

A. Cis B. Planar C. Axial D. Equatorial


7.

The oxygen present in the 1,4-linkage in


disaccharide below is part of what functionality?

the

HOHIC

is the more stable anomer of D-glucose?

"n-1-\.-Q Ho\-1\-o
OH

\.

The alpha-anomer, because the hydroxyl group has equatorial orientation.

ts. The beta-anomer, because the hydroxyl group has equatorial orientation.
I

The alpha-anomer, because the hydroxyl group has

axial orientation.

D. The beta-anomer, because the hydroxyl group has


axial orientation.

A. Ketal B. Acetal C. Hemiketal D. Hemiacetal

;rt

by The Berkeley Review@

t23

GO ON TO THE NEXT PAGE

Passage

ll

(Questions 8 - 14)

The most stable form of most monosaccharides is the cyclic form, rather than the straight chain form. This is particularly true of the aldopentoses and aldohexoses. Although it is common to represent aldohexoses in their straight chain form, for most sugars, less than one percent exist naturally in the straight chain form. The straight chain form of a monosaccharide is represented by the Fischer projection, while the cyclic form is often represented by the Haworth projection. Cyclic structures come in five and sixmembered rings, referred to as furanose anrJ, pyranose respectively. The Fischer projection of D-mannose ancl the Haworth projection of B-D-mannopyranose are shown in Figure

polysaccharide consisting of only B-anomers wi 1,4-glycosidic linkages is commonly known as:

A. glycogen. B. cellulose. C. sucrose. D. lactose.


10.
A11

of the following are true of aldopentoses EXCEPT they are likely to form furanose rings. carbon 4 determines the designation as D or L. carbon 2 is the anomeric carbon in the cyclic forn they have four chiral centers in their cyclic form.

l.

A. B. C. D.
1

nvo
H

CHrOH

1.

no-lno-J-

What is the molecular weight for the trisaccharide from two glucoses and a mannose with 1,4-B-linkages

HI
I

s-l- oH
D-mannose

n os

cH2oH
Figure

HH
B-D-mannopyranose

. B. C. D.
A

414 486 504 540

grams/mole grams/mole grams/mole grams/mole

m'

12.

The glycosidic linkage in most polysaccharides is wh_

Fischer and Haworth projections of mannose

of the following?

Polysaccharides, such as cellulose, are formed from a dehydration reaction of cyclic monosaccharides. One water is lost fbr every glycosidic linkage that is formed. The glycosidic linkage between the two monosaccharides in a disaccharide involves the anomeric hydroxyl of one sugar (referred to as the glycosyl group) and a hydroxyl group of a second monosaccharide (referred to as the glycoside). For instance, 4-O-(B-D-galactopyranosyl)-a-D-glucopyranoside, lactose, results from a B-1,4-linkage between hydroxyl-4 of u-D-glucopyranose and carbon- l of B-D-galactopyranose. A glycosidic linkage is named for the two hyclroxyl groups that undergo dehydration to fonn the linkage.

A. B. c. D.
I

1,1

4,4
1,4

2,s

3.

The cyclic form of an aldopentose is a:

A. furanose ring with an acetal functional group. B. furanose ring with an ketal functional group. C. pyranose ring with an acetal functional group. D. pyranose ring with an ketal functional group.
14.
The following structure has what glycosidic bond?

I.

Which ol the fbllowing Haworth projections represents


the C-3 epimer of R-D-mannopyranose?

A.

cH,oH

B.

An O-glycosidic bond between the carbor


methanol and the anomeric oxygen of cr-mannc,, methanol and the anomeric oxygen of B-mannor*.

B. An O-glycosidic bond between the carbon

C. An O-giycosidic bond between the oxyge:


methanol and the anomeric carbon of c[-manno*

D. An O-glycosidic bond between the oxyger


methanol and the anomeric carbon of B-mannose

Copr ri-eht O by The Berkeley Review@

124

GO ON TO THE NEXT PA

aassage

lll

(Questions 15 - 22)

15. All of the following


are true EXCEPT:

statements about a D-aldohexose

Sugars are organic compounds made exclusively of -;ir:on, hydrogen, and oxygen. In a monosaccharide, the ratio i :arbon-to-hydrogen-to-oxygen is I :2 : l. As the name ;-:ohtdrate implies, for every carbon atom, there is a water l:-;cule in the formula. By convention, sugars are namecl irr: rhe number of carbons and given the suffix "ose', to rT,:-:ate that they are a sugar. The highest priority functional ;:"::o of the sugar is indicated by the prefix. For instance, an , :.,hexose is a six carbon sugar with an aldehyde functional r'::1. In an aldohexose, the first carbon has an aldehyde

. B. C. D.
A

It contains five secondary alcohol groups.


Its straight chain form has four chiral centers. Its most oxidized carbon is carbon l. Its fifth carbon has R-chirality.

16. Which of the following sugars


ketohexose?

can be classified as

'.1:r;rional group and the remaining five carbons have ,::r,\yl groups. The stereocenters have variety, but the ' r.r.-:ctivity is the same. Figure I shows the eight possible
"
-

*,'iohexoses.

o
OH

"Yo
H--+OH

,, ao
HoJ-g

A. B. C. D.

D-Mannose D-Fructose D-Glucose


D-Galactose

H H
OH
CHz OH I-

no-lI

n{os HIon
cH2oH

nolH
I

17.

What is the C-5 epimer of L-glucose?

n-l- oH u-l- on
cH2oH
D-(+)-Mannose

A. D-Idose B. L-Mannose C. D-Allose D. D-Glucose

- rGalactose

D-(+)-Glucose

o
OH OH

"Y"
HO-+-

"Yo
HO--+H

18.

The C-2 epimer of D-Gulose is also the:

nlos
on
I

HIoH
n-lI

. B. C. D.
A

C-3 epimer of D-Glucose


C-3 epimer of D-Idose C-4 epimer of D-Allose

C-4 epimer of D-Altrose

H
OH

HoJ-n

n-l-

H-l-

oH

on
1

cH2oH

9.

CH2OH D-O-Idose

cH2oH
D-(+)-Altrose

The Tollen's test involves an oxidizable sugar reducing

silver cation to silver metal, which can then


sugars yields no silver precipitate?

be

-*

- i-Gulose

identified by its precipitate. Which of the following

Hyo
HO-{H

Hyo
H+OH

Ho-l-

so-lI

n n

H-l.-on
cH2oH
D-(+)-Talose

u-f

n-l-

n-]-os
oH oH
2

A. B. C. D.

Mannose
Sucrose

Glucose
Galactose

0.

cH2oH
D-(+)-Allose

Sugars naturally exist in cyclic conformations in their most stable form. The ring is formed when an OH group attacks the carbonyl carbon. The most stable form of fructose would be which of the following?

Figure

Eight D-aldohexoses

:ructose is also a six-carbon sugar, but it has a ketone ,rr:: rnality on carbon 2, rather than an aldehyde functional rLll"r rr: It is referred to as a ketohexose.

A. B. C. D.

Hemiacetal Acetal

Hemiketal
Ketal

,.

litl,i:iEht @ by The Berkeley Review@

125

GO ON TO THE NEXT PAGE

I1.

Which of the following aldohexoses is NOT an epimer of D-talose?

Passage lV (Questions 23 - 29)


Monosaccharides typically have a formula of CnH2nOn.

A. B. C. D.

D-Galactose. D-Idose. D-Mannose. D-Glucose.

where n is an integer. Depending on the number of carbon-s in the monosaccharide, it may have a preference for a cyclic structure (a hemiacetal or hemiketal) over the straight chain form. To cyclize, the hydroxyl group on the penultimate carbon attacks the carbonyl carbon, which turns the carbon$ carbon into the anomeric carbon. Generally, aldopentoses and ketohexoses form furanose rings, while aldohexoses forrm pyranose rings, as shown in Figure l.

22.

Treatment

of which of the following

sugars with
CH2OH

NaBH4 will yield an optically inactive product?

A. B. C. D.

Glucose Mannose Ribose Fructose

n* ront n*on
cH2oH
D-aldohexose

H-ts (oH) n*toHr

HH
D-aldohexapyranose

H-#

n-F oH

n-l-

roHl

roul

cH2oH
D-aldopentose

HH
D-aldopentafuranose

CH,OH

n*ronr n-l- roHr n-F on


cH2oH
D-ketohexose

t-o

(oH)

cH2oH

,dt

D.

HH
D-ketohexafuranose

rS.

t"
@1u

Figure 1 Generic cyclic monosaccharides \\tren t*'o cyclic monosaccahides combine, they lose rvater molecule as a side product. The resulting disacchari is descnbed b1, the common oxygen between the rings.
glvcosidic linkage is described by its respective position each ring. In almost all disaccharides, one sugar uses anomeric hvdroxyl group to form the glycosidic linkage. disaccharide is typically drawn with the sugar using anomeric oxl,gen in the linkage on the left and the susar on the right. When drawn in this standard method. sugar on the left is given an "osyl" suffix while the sugar the ri_eht is "ide" suffix. -eiven an

Copyright @ by The Berkeley Review@

126

GO ON TO THE NEXT PA

13.

The C-2 epimer of galactose is the same as which of the

following sugars?

27

A trisaccharide

made from three unique aldohexoses has:

. B. C. D.
A

The C-2 epimer of glucose. The C-2 epimer of mannose. The C-4 epimer of mannose. The C-4 epimer of glucose.

A. two acetal linkages. B. three acetal linkages. C. two hemiacetal linkages. D. three hemiacetal linkages.

l{.

The linkage in sucrose is best described as:


2

8.

Exhaustive methylation of a disaccharide converts all of

the hydroxyl groups into methoxy groups. Upon Al1 of the OCH3 groups return to being OH
groups.

hydrolysis under acidic conditions, what is observed for

HOH
A. ul-a2 B. al-82
C. t31-a2 D. B2-a2

"ffd",""
OH H

the fully methylated disaccharide?

A. B.
C. D.

Only the glycoside loses OCH3 groups, nor rhe glycosyl group.
The disaccharide racemizes at every chiral center.

The two carbons involved in the linkage will have hydroxyl groups and no methoxy groups.

The glycosidic linkage of maltose is what type of


functional group? 29 HO
HO

In the following disaccharide, rhe glycoside is:

A. B. C. D.

H
Hemiacetal

OH

Hemiketal
Acetal

Ketal

A. D-galactose. B. L-galactose. C. D-mannose. D. L-mannose.

The following monosaccharide relates in what manner


:o B-D-glucopyranose?

{. B. C. D.

It is an anomer of B-D-glucopyranose.
It is an conformer of B-D-glucopyranose. It is an enantiomer of B-D-glucopyranose. It is an epimer of B-D-glucopyranose.

rnn',ight @ by The Berkeley Review@

t21

GO ON TO THE NEXT PAGE

Passage V (Questions 30 - 36)


Human beings possess the enzyme for cleaving the alpha-1,4-glycosidic linkage of starches. We can break down polysaccharides such as amylose, amylopectin, and glycogen. However, we are unable to digest cellulose, because it contains B-D-glucopyranose residues connected by B-1-4glycosidic linkages. Foods rich in cellulose pass through us undigested. Both amylose and amylopectin are made from cD-glucopyranose, drawn in Figure 1.

31. Why is amylase


A. B. C. D.

released

in both saliva and by

pancreas into the small intestine?

Salivary amylase is destroyed by the gastric fluids.


Pancreatic amylase is weaker than salivary amy

Salivary amylase is weaker than pancreatic amy


Pancreatic amylase cleaves amylose with B-lin while salivary amylase cleaves o-linkages.

2.

Which sugar below CANNOT be absorbed into blood?

HO
HO

OH

A. Fructose B. Galactose C. Glucose D. Lactose


3

Figure 1 cr-D-Glucopyranose
polysaccharide made of a-D-glucopyranose with exclusively cr-1,4-glycosidic linkages. Amylopectin is like amylose, except that about one out of every thirty of the glucopyranose monomers has an a-1,6-glycosidic linkage to a branched strand of o-glucopyranoses. Glycogen also has
a

Amylose is

3.

The difference between amylose and amylopectin is

. B. C.

amylose has more branching.

amylose has B-1,6-branching, while amylo


has a-1,6-branching.

branching, but significantly more than amylopectin. All three polysaccharides are ultimately broken down into smaller fragments, such as the disaccharide maltose. We absorb only monosaccharides into our bloodstream, so starches must be enzymatically cleaved in the mouth and small intestine.
Even disaccharides are broken down into monosaccharides to be absorbed from the intestine into the blood.
3 34

amylose has no branching, while amylopectin


a- 1,6-branching.

D. .

amylopectin has no branching, while amylose


a- 1 ,6-branching.

What is NOT found in glycogen?

0.

Which of the following disaccharides is maltose?

A.

HOH.TC

"o-1-\B.

o\\z-ot OH HO-\-za-\OH

HoHoc

. Monomers of a-D-glucopyranose. B. o-1,3-linkages C. Acetal functional groups D. o-1,6-linkages


A
3

OH

5.

Ho+\zo.

HOHrC

Which of the labeled hydroxyl groups are involved linkages in amylopectin?

uo\ t \ IHOH,C oH
HOH,C

ot-\-o.

Ho\-.a-\OH

ou

C.

no's\zo.

no\

\\ o*or OH HO\/-t\
OH
3

HoH2c

A. B. C. D. 6.

I I

and and

II only

III only

I, II, and III only I, III, and IV only

D.

"""M\ b"lXL=o.
Ho\,.a-\
oHl
OH 128

HOH.C

Amylose is eventually broken down into:

A. cellulose. B. lactose. C. pyruvate. D. sucrose.


GO ON TO THE NEXT

Copyright @ by The Berkeley Review@

lassage Vl (Questions 37 - 43)

38.

Compounds C and D are BEST described as which of


the following?

-':,:

Compound A, an unknown L-aldopentose, is treated with acid to yield Compound B, a diacid referred [o as an ." -.;ric acid, as shown in Reaction I below. Compound B '.'",,s no specific rotation ([up] of 0'). This is to say that

. :iacid does not rotate plane polarized light when an


-3truS

A. Anorrers B. C-2 epimers C. C-3 epimers D. E,nantiomers

solution of the diacid is analyzed using

polarimeter.

Compound

Reaction I

HNOr

ComPound B

( apl ol0")

9.

Which of the following sugars is NOT possible for the


identity of Compound A?

'r

Compound A, when treated with a sequence of reagents .s a slight modification of the Kiliani-Fischer synthesis , --:h increases the length of an aldopentose by one carbon), ' :. trvo diastereomers that vary in their chirality at only .: :rrbon. The two epimers are referred to as Compound C - : :ompound D. Carbon I of Compound A is labeled with

i.

A. Ribose B. Xylose C. Glucose D. Lyxose

-: ,lompound D. This implies that the l3C label is not on ., .-Jehyde carbon in Compounds C and D. Reaction 2 is

The I3CNMR peak that starts at 202 ppm in :round A but is fbund at 78 ppm in both Compound C

0.

Compound
ITIASSES?

B has which of

the

following molecular

'

' n below.
1. HCN

ItrmPound A

2. H2lPd(BaSO 3. I{jO+ / A

a)

Compound C + Compound D

. B. C. D.
A

132 grams/mole 148 grams/mole 164 grams/mole 180 grams/mole

Reaction

41.

The melting point of Compound B:

,ltrmpound C when treated with nitric acid yields a diacid r :-.I (Compound E) that shows a specific rotation ([op]) -:'. Compound D when treated with nitric acid yields a .*.: product (Compound F) that shows a specific rotation ol 0'. This is to say that compound E is optically , :. while compound F is optically inactive. The :-,, .1nS ore shown below as Reaction 3 and Reaction 4: i-ompound

. . C. D.
A
B

is greater than the melting point of Compound A. is equal to the melting point of Compound A. is less than the melting point of Compound A. is less than the boiling point of Compound A.

HNO:
Reaction

ComPound E ap.l of -26")

42.

Aldohexoses

in their most stable form would

NOT

show which of the following IR peaks?

-ompound

HNo:
Reaction

ComPound F ([ upl oi 0".r

A. B. C. D.

1140 cm-1 1715 cm-1

2980 cm-l 3480 cm-l

r,. --3 the


r

sing the optical rotation data presented, it is possible to identity of the unknown L-aldopentose (Compound ihe chirality of each hydroxyl group can be determined -: r time from the optical activity data.

--

3.

The second carbon (C-2) for Compound A:

- -{orv many
:ldopentose?

stereoisomers

are possible for

an

. B. C. D.
A

must have R stereochemistry.


must have S stereochemistry. can have either R or S stereochemistry.

is achiral.

\. 2 B. 4

D.

c.

16

"

:reht @ by The Berkeley Review@

129

GO ON TO THE NEXT PAGE

Passage

Vll

(Questions 44 - 50)

45.

The stereochemistry of carbon 3 in the Compound X

Compound X, an unknown D-aldohexose, undergoes a X is first treated with nitric acid and the purified product (Compound y) is placed into a polarimeter to analyze its optical activity.
sequence of chemical reactions. Compound

Compound

A. B. C. D.

definitely R. definitely S.
variable (either R or S). achiral (no chiral center).

shows an optical rotation,

water, and finally hydrogen peroxide. The product (Compound Z) has a molecular mass of 150 grams/mole. Compound Z is then treated with nitric acid to yield a new product (Compound W), which when placed into a
polarimeter shows no optical activity.

Compound X is then treated with the following sequence of reagents: first bromine in water, second ferric carbonate in

[sO =

26t].

46. Compound X, when treated with hydrogen

cy

followed by hydrogen gas over palladium metal


barium sulfate and then treated with acidic water yield which of the following?

Compound

X is then treated with one equivalent of

periodic acid, which breaks the bond between carbons 3 and4, oxidizing carbons 3 and 4 up one level to aldehydes. This six-carbon hemiacetal fragment (Compound e) is then hydrolyzed out of the hemiacetal form and converted into two smaller fragments, both three carbons long (Compound R and Compound S). From the optical rotation data, the researcher concludes that carbons 3 and 4 have opposite chirality. The reactions are summarized in Figure I below:

A. B. C. D.
47. A

A D-aldopentose.
An L-aldopentose.

A D-aldoheptose.
An L-aldoheptose.

reverse aldol reaction involving Compound X an aldotetrose (formed by breaking the C-2:C-3

o\" lHt(oH)

^o, L Br2(aq)

Compound Y

The aldotetrose when treated with sodium borohy yields an optically active tetra-ol. This confirms whi of the following conclusions about the chiral centers Compound X?

optically active
Compound Z

H_F fOgr Ht (OH) Hf oH


cH2oH
Compound X

2. Fq(CO3)3raq) 3. Hzoz(aq)

(MW = 150 gimol)

. B. C. D.
A
4

Compound X Compound X Compound X Compound X

has chirality of 3R and 4R.


has chirality

of 35 and 4R.

has chirality of 4R and 5R. has chirality of 45 and 5R.

l*o,
V

8.

HIOa

Compound W optically inactive Compound Q

How many stereoisomers are possible chain form of a D-aldohexose?

for the

ooo
HOH
1

A. B.

4
8

"5*" '\'""'."i:i":.
Compound R Compound

D.

c.

16

32

HOH 5

&

ComPound S

49. Which of the following reagents, when

Figure

Reaction summary of Compound X experiment

added aldohexose, would yield the same conclusion chirality as adding HNO3?

The reaction involving the Br2 liquid in water is known as Ruff degradation, which removes carbon I and oxidizes carbon 2 into an aldehyde, while leaving the sugar chain intact from carbon 3. The stereochemistry of the backbone

A. B. C. D.
5

NaBHa
H2SOa
Br2(aq)

KMnO4(aq)

does not change during this reaction. The structure of


Compound X can be deduced from this information.

0.

44. Treatment of Compound X with sodium borohydride


(NaBHa) would yield which of the following?

Treatment of an aldohexapyranose with one equ of HIO4 breaks the sugar between:

A. B. C. D.

An An An An

optically optically optically optically

active diacid.

inactive diacid. active hexa-ol. inactive hexa-ol.

A. B. C. D.

carbons 1 and2. carbons 5 and 6. carbons 2 and3. carbons 3 and 4.

Copyright @ by The Berkeley Review@

130

GO ON TO THE NEXT P

Passage

Vlll

(Questions 51 - 57)

2.

Which of the following pair of sugars are C-2 epimers?

-'.,-tion. The cyano group is then reduced into an imine ' .r;h is subsequently hydrolyzed into an aldehyde. An

-,:ns the Kiliani-Fischer synthesis. Kiliani-Fischer , --:hesis employs a cyano nucleophile in the first step. The :, ino group attacks the carbonyl carbon in an addition
.nple using ribose is shown in Figure
1.

The length of a sugar can be increased by one carbon

A. B. C. D.
53

D-Ribose and D-Altrose D-Glucose and D-Talose D-Mannose and D-Allose D-Idose and D-Gulose

In which of the steps in Kiiiani-Fischer synthesis is the


sugar reduced?

o
HO

oH r. HCN/KcN

H
OH OH OH
T

OHffi
CH2OH - - +)-Ribose

H H H

oH 3. Hro+/

uI oH u-f oH
cH2oH
D-(+)-A11ose

HIoH

H;4,o H{on

A. B. C. D.

Steps
Steps Steps Steps

I only II only I and II only I, II, and III only.


an

CH2OH D-(+)-Altrose

54. Which of the following is


Kiliani-Fischer synthesis
?

intermediate

in the

Figure

Kiliani-Fischer synthesis starting with ribose

\
tr -

,: different D-aldopentoses
,ir .,. n in Figure
2.

researcher carried out the Kiliani-Fischer synthesis on to determine the effects of steric

A. B. C. D.

Aldehyde

Amine
Ester

Imine

-.ance on stereoselectivity. The four D-aldopentoses are

yo 'yo
cH2oH
CHrOH
D-Arabinose

Hyo

55. The product mixture following Kilianisynthesis is best described as:

Fischer

HtoH To" HtOH HotH Ho-lH HtOH HtOH TOH


- -Ribose

cH2oH

. B. C. D.
A
5

two structural isomers in equal concentration. two structural isomers in unequal concentration.
two diastereomers in unequal concentration. two enantiomers in equal concentration.

CH2OH

D-Xylose

D-Lyxose

Figure

Four D-aldopentoses

6.

Which structure represents B-D-talopyranose?

A.

B.

Table 1 lists the product distribution for each synthesis.


rldopentose D-Ribose

Major Product
81% D-Altrose
'72Vo

Major Product
197o

D-Allose
D-Gulose D-Talose

l-Arabinose

D-Glucose

28% D-Mannose
297o 24Vo

)-Xylose
)-Lyxose

llVoD-Idose
J6Vo D-Galactose

Table
.ldopentose should be used?

To synthesize the following D-aldohexose,

what

O H -\. B.
C. D.
D-Arabinose D-Lyxose D-Ribose D-Xylose
@

OHOHH

OH

cH:oH

H H OHH

57

Kiliani-Fischer synthesis converts

an:

A. B. C. D.
131

aldohexose into an aldopentose. aldohexose into a ketohexose. aldopentose into an aldofuranose. aldopentose into an aldohexose.

:lsht

by The Berkeley Review@

GO ON TO THE NEXT PAGE

Passage lX (Questions 58 - 64)


An unknown sugar is extracted from the fruit collected from the Racaniqui Tree native to Willoughby, Montana. The sugar is isolated by coiumn chromatography in 99.9Vo purity as shown by lgNUR. The moleculai*us was fbund to be approximately 342 +2 grams per mole. To have that
molecular mass, the unknown sugar must be a twelve-carbon disaccharide. The sugar shows a specific rotation of +126.2. .

9. Which of the following pairs of


represent Compound

structures

o''

and Compound II?

H--+

(o
OH

uo-l-r+

The disaccharide linkage is cleaved rather easily with

H-f
u'

H-J-

oH

HoIn

ao-|.u

"x""
cH2oH

ou

H-f on

mild acid, and two six-carbon monosaccharides, Compound I


When treated with three equivalents of phenylhydrazine, both monosaccharides form osazones. Compound I yields an osazone with a specific rotation of +42.6" while Compound II yields an osazone with a specific rotation of +31 .2' . Figure I shows the reaction of glucose with three equivalents of phenylhydrazine. and Compound

cH2oH

II, are isolated.

HO-+-

'' (o
H

uI
t

n-l-

os
oH

o
H
HO
OH

H./, N-NHO
HO-t3

H-f

nolH uoIH
H-f

HO..F

N-Mro
H

oH cH2oH

on

cH2oH

H
OH OH

OHN-NH2
HOAc

H H

n{

on
oH

u-f

""y" H{ uo-l- u ""y"


HO-+H
HC

cH2oH Figure

cH2oH
+ NHj + C6H5NH2

H-f

H-l- ou

ou H-f oH
CH2OH

HoIn
o

on

Osazone formation starting from glucose

CH2OH

The specific rotation of the osazone of D-glucose is +54.6'. Both Compound I and Compound II are naturally
occurring sugars, so they are assumed to be D-sugars. When both sugars are treated with nitric acid, each one is oxidized into an aldaric acid (a diacid with carboxylic acid groups at both carbon one and carbon six). Both of the aldaric acids formed are optically active. This implies that neither aldaric acid is meso and that both sugars are aldohexoses, rather than

"

"Y* u-]-on
Ho-l--n

HO H
HO

H
OH

n-f

H
OH

on

ketohexoses.

2-ketohexose can also react with three

cH2oH

CH"OH

equivalents ofphenylhydrazine to form an osazone derivative. The information from the derivatives naffows the identity of each aldohexose to one of four choices. To identify the

60. Which of

the following aldopentoses optically active aldaric acid?

will

yield

il

original sugar, the physical properties such as melting point


and specific rotation can be compared to the values of the four

o'n
H+-

B.H

possible D-aldohexoses. To determine the linkage of the disaccharide, the hydroxyl groups can be glycosidically

(o
OH

H
HO

labeled. After hydrolyzing the labeled disaccharide into its


two monosaccharides, the sugar with hydroxyl four unlabeled is the glycoside (sugar on the right in standard notation). This allows for the disaccharide to be identified precisely.

r-f-

H-l-

oH

ot "

T", #n
*o"
cH2oH

I ( l

CH2OH

58. How many stereocenters


disaccharide?

are present

in the original

"'tY o
HO-+ H

,m ,il

4.1 B. 8

D.
Copyright

c.

10
11

'+ Hot

OH H

""Y x+

H
OH OH

r( D

CH zoH by The Berkeley Review@


132

CH zoH

GO ON TO THE NEXT PA

61

i or Compound II?

Which of the following sugars CANNOT be Compound

Passage

(Questions 65 - 71)

I. Mannose (the C-2 epimer of glucose) II. Talose (the C-4 epimer of mannose) m. Galactose (the C-4 epimer of glucose) A. I only B. II only C. I and II only D. I and III only
Which of the following choices shows a sugar paired with the incorrect reason for eliminating it as a possible structure for either Compound I or Compound II?

The early part of glycolysis involves the conversion of D-glucose into glyceraldehyde-3-phosphate. Glycolysis is carried out in ten enzymatically catalyzed steps. The first four steps of glycolysis, forming glyceraldehyde-3-phosphate,
are shown below:

Ho-l-H

uJ-

on

n-l-

nJ-oH
on
CH.,OH

"Yo H-F HoIn Hton


ATP ADP

OH

n-f-

oH

A.
H H H
H

CH2OPOj2

oB.
OH OH OH OH
HO HO

o
H H
OH OH

STEP

I:

Glucose to Glucose-6-phosphate

AG"=-4.0kcal/mole

H H

Ho-t-H

rL HtoH
jT-n

cH2oH

cH2oH Will yield


an optically

Fo no-f-n tsomerase a
n-Tun -

cH2oH

Will yield the same


osazone as glucose

inactive aldaric acid

HtoH

C.
HO

o
H
OH

D.
CHrOH

CH2OPO32STEP II: Glucose-6-phosphate


CHrOH

-lou
CH2OPO32-

to Fructose-6-phosphate

H
HO

H
OH

no-l- s u-]-on
H-]I

fo

AG"=+0.4kcallmole

OH

notn
H-lSTEP

Ho

CH"OPO"2-

uIoH
oH

CHrOH Will yield an optically


inactive aldaric acid

cH2oH

Will yield the same


osazone as glucose

cH2oPo32-

-ry
Phospofructokinase

uo*n H*os

fo

ATP

ADP

uf

os

CH2OPOj2-

III:

Fructose-6-phosphate to 1,6-Bisfructophosphate

AG'=-3.4kcallmole
Which of the following choices contains two sugars
that

will form the same

osazone?

CHTOPOT2-

A. B. C. D.

D-ribose and D-xylose (C-3 epimers). D-idose and D-altrose (C-4 epimers).
D-talose and L-talose.

D-mannose (C-2 epimer

of D-glucose) and D-

fructose (ketose of D-glucose).

l-' Aldola.^ cHroPOr2HO--H i -- -J tt"-En+u H+os CH2OH Hf I

L:^

-ntoH
l-

'vo

OH

CH2OpO32-

CH2OPO32n"4.

\\rlrat happens to the three phenylhydrazine molecules?

STEP

IV:

1,6-Bisfructophosphate to Dihydroxyacetone-

{. . C. D.
B

phosphate and Glyceraldehyde-3-phosphate

One adds to the sugar and two are oxidized. Two add to the sugar and one is oxidized. One adds to the sugar and two are reduced. Two add to the sugar and one is reduced.

AG'=*5.7kcallmole
Dihydroxyacetonephosphate undergoes isomerization to

form glyceraldehyde,3-phosphate. Steps I-IV correspond to common reactions in organic chemistry, so the mechanism for each step follows standard organic chemistry processes.

l :ri'right

by The Berkeley Review@

133

GO ON TO THE NEXT PAGE

a ketone in Step II goes through a process known as tautomerization. Step III is similar to Step I. The breaking of the carbon-carbon bond in Step IV is an example of the
retroaldol reaction. The formation of a B-hydroxyketone from a carbonyl is a common reaction in synthetic chemistry.

The addition of the phosphate group in Step I is similar to esterification of a carboxylic acid with an alcohol. The role of the enzyme is to align the molecules so that the hydroxyl group on carbon 6 reacts to gain the phosphate, rather than another hydroxyl group. The conversion of an aldehyde into

0.

Which of the following sugars is fructose-6-phosphate.'

Pd

gH2oPg32' CH"OH

cH2oPo12

ptrr
pil-I
mhr

Ifi[B
lfmrr
hdim,

D.

65. G-3-P,

glyceraldehyde-3-phosphare, and DHAp,

Ben mfu

dihydroacetonephopshate are best described as;

dM

A. B. C. D.
66

Sm
ffiW
ruflmr

configurational isomers.
diastereomers.

epimers

structural isomers.

If D-galactose (the C-4 epimer of glucose) undergoes the same four reactions shown in the passage, how will the products differ from the products of glucose?
A. B. C.
G-3-P will have a different chiral center. DHAP will have a different chiral center. Both G-3-P and DHAP will have different chiral
centers.

71

Glycolysis converts glucose into:

A. . C. D.
B

two pyruvates and generates a net two ATps. two pyruvates and generates a net four ATps.
three pyruvates and generates a net two ATps. three pyruvates and generates a net four ATps.

D.

The products

will be identical to the products

formed by the glucose reaction.

67

. If carbon 5 of a single D-glucose


products?

is enriched with 13C, where will the enriched carbon (label) be found in the
Carbon Carbon Carbon Carbon
1 of DHAP.

. B. C. D.
A
6

3 of DHAP. 2 of G-3-P. 3 of G-3-P.

8.

What is the overall free energy change from Step Step IV of glycolysis?

to

A . + 2.1 kcals per mole. B. + 1.3 kcals per mole. C. - 1.3 kcals per mole. D . - 2.1 kcals per mole.

9.

How many chiral centers are present in the two final


producrs (c-3-P and DHAp)?

A. B.

D.

c.2
3

Copyright @ by The Berkeley Review@

t34

GO ON TO THE NEXT

P.

Passage

Xl

(Questions 72 - 78)

Blood types in human beings are determined by the ,,,:r saccharide derivative attached to the amino terminal of a : :tein in the wall of the red blood cell. The success of a : . od transfusion depends on the compatibility of antibodies --.. recognize the markers on red blood cells as equivalent or ,,:isn. If the antibodies do not recognize the foreign red

In all three blood types, the galactose forms a bridge via hydroxyl two to a fucose residue. In blood type A, the
galactose has a N-acetylglucosamine linked to hydroxyl three while in blood type B, the galactose has a second galactose residue linked to hydroxyl three. These polysaccharides are referred to as the antigenic determinants of the red blood cells. These, along with the Rh factor, determine the feasibility of a transfusion.
7

'rd cells as invaders, then the transfusion is possible. :,rd type O, the universal donor, contains a trisaccharide, ' -,,1e types A and B contain tetrasaccharides. Figure 1 shows .-. oolysaccharide associated with Type O blood, Figure 2 ' '.,, s the polysaccharide associated with Type A blood, and : :-re 3 shows the polysaccharide associated with Type B

2.

What linkage is present between N-acetylglucosamine


and galactose in blood type O?

A. u-1,2-linkage B. B-1,2-linkage C. u-1,4-linkage D. B-1,4-linkage

cH2oH

\ -o*o -\-/"Ho\$

73

H*+ o4Jo, 'ry


luo
HO

o{o I CH:f,o,.in
associated with Type O blood
7

NHI

How many acetal and ketal functionalities are present in the type O blood antigenic determinant?

A. 0 B. I
D.

c.2
3

I .ure 1 The polysaccharide


HO

H,cyzo|.$jtq
_.1 CH2OH 7f -

ct{]oH

4. If the CH3 group of fucose were replaced by CH2OH,


how would the molecule differ from glucose?

o - T/4 ,['PS1]-"e\
H.rC u
CH-r

T,! -o).o "Y-"i


NHI

. . C. D.
A
B

It It It It

would would would would

be the C-2 epimer of D-glucose. be the C-2 epimer of L-glucose. be the C-4 epimer of D-glucose. be the C-4 epimer of L-glucose.

\ protein
7

\Ho
HO

5.

Which of the fbllowing molecules is B-D-2-acetamino2-deoxyglucose?

zure

The polysaccharide associated with Type A blood

A.

CH"OH

B.

CH"OH
t

"f ."ro"
MoNHI

CH2oH

Hod/or
Q:Q

OH

_ I CH2OH .4- ,___t

T.,{ -n).o,
H.rC

"pPvs-"d\
L/f
\"o

/oH
CH:
D

Q:Q

NH

"Y-"-q \ protein
cHr

cHr

'

cH2oH

'

cH2oH
OH

HO
ri

HO-\//1l "n-^\-o.
o-c
6l /oH
NHz

Ho$/or Ho-\//$/
Q:Q

llure

The polysaccharide associated with Type B blood

In the three blood types, the first sugar attached to the ':' :-n is N-acetylglucosamine. Attached to the first sugar " . i.4-glycosidic linkage is galactose.
.

/ \

o
NHe

nght @ by The Berkeley Review@

135

GO ON TO THE NEXT PAGE

6.

The formation of L-fucose from an L-aldohexose would require what type of reaction?

Passage Xll (Questions 79 - 85)


Glycolysis is an example of a biological process
generates energy that is ultimately stored in the form adenosinetriphosphate, ATP. ATp is able to store because its phosphodiester bonds are very weak and result the release of energy when they under go hydrolysis.

. B. C. D.
A

Reduction specific for carbon 6. Oxidation specific for carbon 6. Reduction specific fbr carbon 4. Oxidation specific for carbon 4.

hydrolysis of ATP into ADp releases 7.3 kcal/mole energy. Table 1 shows the energy of hydrolysis for se phophorylated compounds that are common in bi
77

In the type B antigenic determinant, what has been added to hydroxyl three of the central galactose?

Compound Phosphoenolpyruvate

AG'' (kcal/mole)
-14.8 -10.3

A. a-galactofuranose B. B-galactofuranose C. o-galactopyranose D. B-galactopyranose

Acetyl phosphate
Pyrophosphate Adenosine triphosphate (ATp) Glucose 1-phosphate Glucose 6-phosphate

-1.9
-

t.J

-5.0 -3.2

Glycerol 3-phosphate
7

aa

8. How many chiral centers


antigenic determinant?

are present in the type A

Table
for the reaction at a pH of 7.

D.

A. 12 B. 16 c. 20
24

The AG'' term represents the standard free energy c The conversion from one structure into another st

in Table I involves a phosphate group transfer.

favorability of the transfer reaction can be ascertained the energies of hydrolysis for the two reactions. The phosphate group transfer agent is phosphoenolpyruvate. loss of the phosphate group by phosphoenolpyruvate subsequent conversion from pyruvate enol into pyruv shown in Figure 1.

};,",,+;*T
Figure

Conversion of phosphoenolpyruvate into pyruvate enol and pyruvate

The substantially large energy of hydration a with phosphoenol pyruvate is attributed in part to
lavorable conversion from an enol into a ketone. The

is the more stable of the two tautomers, so the equili


constant is greater than ketone.
79

for the conversion of an enol

What is the approximate change in free energy for conversion of pyruvate enol into pyruvate?

. B. C. D.
A
Copyright @ by The Berkeley Review@
136

+16.4 kcal/mole +6.7 kcal/mole

-6.1kcallmole
-16.4 kcal/mole

GO ON TO THE NEXT

I,l

Which of the following compounds can couple with the conversion of ADP into ATP in an overall favorable
fashion?

84. Hydrolysis of which of the following

compounds

releases approximately 5 kcal/moles of free energy?

A. B. C. D.

Acetyl phosphate
Glucose l-phosphate Glucose 6-phosphate

CH2OPO32

QPo:2-

Glycerol 3-phosphate

! n. The high energy associated with the hydrolysis of


compound with a phosphate group can be attributed to:

CHoOH

D.

cH2oPoj2-

A. B.

the strong phosphodiester bond, where the strength can be attributed to its aromaticity.
the strong phosphodiester bond, where the strength can be attributed to its strong resonance. can be attributed to electrostatic repulsion.

C. the weak phosphodiester bond, where the weakness D. the weak phosphodiester bond, where the weakness
can be attributed to the inductive effect.

85.

How is the structure for fructose-1,6-bisphosphate best


described?

CH2OPOj2-

CH2OPO32-

\\rhich enzyme catalyzes the conversion ofphosphoenol pvruvate, PEP, into pyruvate?

o\o
I

o\o
AG = - 15 kcal/mole
I

,foeo:2 F ATP H il H
{. PEP dehydrogenase B. PEP reductase C. Pyruvate kinase D. PEP aldolase

ADP

fo cHr

OH

A. B. C. D.

It It It It

has a furanose ring with alpha orientation. has a furanose ring with beta orientation. has a pyranose ring with alpha orientation. has a pyranose ring with beta orientation.

'Ihen glyceraldehyde-3-phosphate, G-3-P, converts to


-.-3-bisphosphoglycerate, what is NOT true?
OPO32-

CH2OPO32-

l-

o"

----->

OH

CH2OPOj?-

. . C. D.
A
B

Carbon I is oxidized.

NAD+ is required.
Carbon 2 undergoes no redox chemistry.

Biotin is required.

,i' ::_iht O by The Berkeley Review@

137

GO ON TO THB NEXT PAGE

Passage

Xlll

(Questions 86 - 91)

7.

How much CO2(g) is produced from the metabolism


10.0 grams of D-glucose,
up in the reaction?

c*ff

Sugars, when metabolized by animals, generate a substantial amount of energy. Many naturally occurring
monosaccharides and disaccharides can be extracted from the

if

all of the glucose is

used

fruits of plants. Among the most common naturally

A.

10 x --l

180

x 44 grams COZG)

occurring sugars are fructose, sucrose, and glucose. Figure I shows six common biological monosaccharides.

'vo

B. 10x I xLx44gramsCO2(g) i80 I C. l0 x 18q x 6 grams COz(g)


44

"oI"o" H-lH*os
CH2OH
D-Glucose

"-l-.,"

noJ-H no-l-n

"r""
D-Galactose

"** cH2oH

noJ-H Ho-l-n n-l--,r" u*oH


CH2OH D-Mannose

D.
88.

l0 x

180

L* -Lsrams CO?re) 644

The BEST explanation for the differences in heats

combustion
following? A. B.

for the aldohexoses is

which

of tk

The bonds are different in the different sugars. The arrangement ofthe hydroxyl groups is di

H-l- o" no-l-n

'vo

between sugars, causing differences in hydro


bonding.

"Yo
H-{OH

CHrOH

tn

C D

n-f-

o"

n-lH-r
I

oH oH

Ho-.|-H H-l- o"

. .

Only some of the sugars have resonance stabilin,.


Only some of the sugars are aromatic.

il

CH2OH D-Xylose

cH2oH
D-Ribose

u*oH
CH2OH D-Fructose

89. If the heat capacity of water is 4.18 J/g K, how much will the temperature of 1000 g of water insulating bomb calorimeter rise if it absorbs all of the
generated by burning 15.0 grams of ribose?

Figure

1 Six common

A.

4.91 'C
41.4 "C 49.7 'C 201 "C

I I

monosaccharides

The more heat released per gram of a sugar, the more effective the sugar is as a fuel. Most thermodynamic data, such as the heat of combustion (AH) and free energy changes (AG) under standard conditions, are listed in terms of kcals/mole. Table I shows the thermodynamic values associated with the sugars in Figure 1.
Sugar Glucose Mannose Fructose
Galactose

B. c.
D.

It

,tr

90. The amount of CO2(g) produced by combustion w be greatest from 1.0 gram of which of the followi
sugars?

AHcombustion

(kcal/mole)
-2538 -2512 -2482 -2410

AGcombustion (kcaUmole) -2827


-2801 -2711

A. Glucose B. Sucrose C. Ribose D . A11 monosaccharides


COZ(g) per gram.

I ( I

produce the same amount

mr

JI

-2759
-2341

Xylose
Ribose

-2102
-2016

91.

-2315
1

The amount of energy produced by combustion be greatest using 1.0 gram of which of the followi
sugars?

I (
D

Table
8

6.

What is the stereo-configuration for D-xylose?

A. 2R, 3R, 4R B. 2R, 35, 4R


C. D.
25, 3R, 4R 25, 35. 4R

A. Glucose B. Xylose C. Ribose D. All monosaccharides


energy pef gram.

W] produce the same amount

{rum

il" f-

C. D138

Copl right @ by The Berkeley Review@

GO ON TO THE NEXT PA

Questions 92 - 100 are NOT based on a descriptive passage

8.

The Fehling's test is positive when copper goes from an oxidation state of +2 to +1. What change in the sugar corresponds to a change is in color for the solution in

Fehling's test?

I2.

Decomposition of glucose with excess HIO4 would yield which of following products?

A. B. C. D.
1.3

4 formic acids and 2 formaldehydes.


5 fbrmic acids and carbon dioxide. 4 formic acids, carbon dioxide and 1 formaldehyde. 5 formic acids and 1 formaldehyde.
9

. An aldose becomes a carboxylate B. An aldose becomes a hexa alcohol C. An aldose becomes an acetal D. A ketosebecomes acarboxvlate
A

9.

What is the molecular formula of a trisaccharide formed from two aldohexoses and one aldopentose?

What is the mass percent of carbon in an aldopentose?

A. B. C. D.

33Vo

C by mass

31% C by mass
40Vo C by mass 44Vo C by mass

A. C17H36O15 B. C'7H34O15 C. C13H32O16 D. C13H36O1g


10

0.

r-$. After treatment with nitric acid, a sugar is converted into a diacid (aldaric acid). Which of the following sugars would yield an optically active diacid upon
rreatment with nitric acid?

An aldopentose has how many units of unsaturation?

A. B.

0
1

c.2
D.
3

A. B. C. D.

D-Mannose D-Galactose

D-Ribose

D-Allose

In order for a D-aldopentose


,,r'hat stereochemistry?

to become optically inactive upon treatment with nitric acid, it must have

{. 2R, 35 B. 25, 3R C. 25, 35 D . All of the choices


inactive diacid

"Just studv

above

will lead to an optically

itl"

lrr

fubose differs from deoxyribose in all of the following ',IAYS EXCEPT:

.{. B. C. D.
Lrr

Ribose has more stereoisomers. Ribose has one more chiral carbon. Ribose is the C-2 epimer of deoxyribose. Ribose is more oxidized than deoxyribose.

,.:nes

,\-hen two monosaccharides form a disaccharide, what NOT occur during the reaction? The loss of a water molecule The formation of an acetal from a hemiacetal The formation of a glycosidic linkage The oxidation of the anomeric carbon

r. . C. D.
B
p,,

1.B D 11. c 16. B 21. D 26. D 31. A 36. C 41. A 46. C 51. D 56. C 61. D 66. D 71. A 76. A 81. C 86. B 91. A 96. C
6.
139

2.C
1.
12. 11. A 22. C 21. A 32. D 31. C 42. B 47. D 52. D s7. D 62. C 61. C 72. D 71. C 82. C 87. B 92. D 97. D
B C

D 8.D 13. A 18. D 23. C 28. D 33. C 38. B 43. B 48. B 53. C 58. C 63. D 68. C 13. D 78. C 83. D 88. B 93. C 98. A
3.

4.8 9.8 14. C 19. B 24. B 29. C 34. B 39. C 44. C 49. A 54. D 59. B 64. D 69. B 14. D 19. C 84. A 89. C 94. A 99. A

5. C
10.C
15. A 20. C 25. C 30. D 35. D 40. D 45. A 50. D 55. C 60. D 65. D 70. A 15. B 80. A 85. A 90. B 95. A
100. B

-1-'1

O by The Berkeley Review@

NO MORE CARBOHYDRATES!

Choice B is correct. According to the rules of steric hindrance, the equatorial orientation is more stable than th. axial orientation for the anomeric hydroxyl group on carbon 1. As stated in the passage, the G-anomer of Dglucopyranose has its anomeric hydroxyl gtot p irthe equatorial position. Because ihere is 64"/obetaanomer an; only 36"h alpha anomer formed ,tpon .yciiration, the beia anomer must be the more stable of the two anomers From the drawing in Figure 1 and the information presented in the passage, this question should have been eas,, You really should choose B.
2.

]I
ffi.

This eliminates choice D. Glucose and Galactor" ut" C-4 epimers of one another. Epimers are suga: diastereomers that differ in chirality at only one of the carbons ln backbone of the straight ui' form of ih. sugar' To get this correct requires knowing some sugar facts. Welcome to MCAT preparation and "t have a nice da,,
after you choose

Choice C is correct. Anomers are diastereomers of the same sugar that vary in chirality at the anomeric carbor. b9 the same sugar, the sugar must have the same name. ThiJ eliminates choice A. Enantiomers vary at all c: Jo the chiral centers. In the case of enantiomeric sugars, they are named the same, except for the D or L prefi1,.

C.

Choice D is correct. By definition, the D and L forms of the same sugar are mirror images of one another, whir tlll lhe two compounds are enantiomers of onJanother. The n aia L forms of a sugar have opposite values (differing only in sign) for their optical rotations of plane-polarized 'tir light. This'mean, that one-rotates plane-polarized light in a clockwise fishion while its enantiomer rotates plane-polarizec light in the counterclockwise directi,on. In this question, as would be the case when comparing the D and L isomers of any other sugar, D-glucose and L-glucose are enantiomers of one another. Choose D for best results.
means (by definition)
4.

Choice B is correct. Hopefully the structure of glucose is permanent in your memory (using the ,,right-hanc meJho$' for D-glucose), glucose in its straignt cniin Fischer projection is 2 right (R), 3 iefi (S),"a right in) tlsnl (n) It is important that you remembel tnat hydroxyls on tire teft are s #a nyaroxyls on the iighi ana : are R i:the Fischer projection of standard aldohexoses. This makes choice B the best unr-"r.
Choice C is correct. An aldohexose has chiral centers present at carbons 2, z, 4, and 5 for a total of four chirain the molecule. The maximum number of stereoisomers is found using the formula 2n where n is the number of chiral centers present in the molecule. In this question, plugging in fJur for the value of n yields 2n =
centers

5.

24, which equals 16, choice C.


6.

Choice D is correct. Looking at the structures in Figure 1, it can be seen that the anomeric carbon (carbon 1) ha= the hydroxyl group in the equatorial position in the G-anomer. Choose D for optimal satisfaction. Cis and tran are poor answers, because to be cis or trans, it must be relative to somethrng. In actuality, the B-orientation is defined as cis with respect to carbon six in an aldohexapyranose, but thai t,aJnot implied ir, tni, question. Choice B is correct. The 1,4-glycosidic linkage between any two monosaccharide units involves the hemiaceta, OH of carbon-1 of one sugar with a hydroxyl oJ carbon 4 of the other sugar. These combine to form an acetal when carbon 1 is anomeric. Pick B for correctitude sensations. Had the anomeric carbon been carbon-2 (as in the case o: a cyclic ketose), the glycosidic linkage would be2,4 and the functionalitr.n'ould be a ketal.

Hot\c-

,,o\)i\.*:i\A-\--q
b"l\ H\
140
Anomeric C L. bo:ri.i :.': OR, OR', C, ANd H

HOFIC ' \

Acetals are formed from an aldehyde qroup, U I so the monosaccharide is an aldose

HCFTC

\l "l'
lmI
:
* - i':'

._

Copl'right O by The Berkeley Review@

C\RBOHYDRATES EXPLANATIONS:

Choice D is correct. When the straight chain form of a sugar is converted into its Haworth projection, the hydroxyl groups originally on the left side of the sugar are found above the ring and hydroxyl groups originally on the right side of the sugar are found below the ring (recall the mnemonic "downright uplefting;;. Mat''oose has the hydroxyls on carbons 2 and 3 on the left. The C-epimer of D-mannose therefore has a itraight chain with only the hydroxyl on carbon 2 on the left. In the Haworth projection of the C-3 epimer of B,D, mannopyranose, the hydroxyl groups on carbons l and2 are above the ring and the hydroxyl groups on carbons 3 and 4 are below the ring. This makes choice D the best answer. Choices A and B should have been eliminated early on, because they have s-orientation at carbon 1. As a point of interest, choice C is B-D-glucopyranose.
Choice B is correct. This question is best answered from straight memorization, although a good guess can be derived from the passage. Lactose is a disaccharide made from glucose and galactose, not a polysaccharide, so choice D is eliminated. Sucrose is a disaccharide made from glucose and fructose and not a polysaccharide, so choice C is eliminated. Glycogen has alpha linkages between saccharide monomers, while cellulose (the polysaccharide we can't digest) has beta linkages between the saccharide monomers. We lack the enzyme to cleave the B-1,4-glycosidic linkage. Choice B is the best answer. Choice C is correct. When you consider the most common aldopyranose, ribose, it is reasonable to assume that a furanose ring is the most stable cyclic form for aldopentoses in general. Choice A can be eliminated. The penultimate carbon determines the designation of D or L for a sugar. In a five carbon sugar, carbon 4 is the penultimate catbon, so choice B can be eliminated. In the cyclic form, furanose ring, all of the carbons except carbon 5 have chiral centers, so it is true that a cyclic aldopentose has four chiral centers. Choice D is eliminated. In an aldopentose, carbon t has two bonds to oxygen, so in its cyclic form, carbon 1 still has two bonds to oxygen, making it the anomeric carbon. Carbon 2 is not the anomeric carbon, so choice C is correct. Choice C is correct. For any trisaccharide, the total weight is the weight of the three monosaccharides minus the weight of the two waters lost in forming the linkages. When three 6-carbon monosaccharides are combined, there are two bridging linkages. Calculating this value gives 3(180) - 2(18) = 540 - 36 = 504 grams/mole. Choose C to show off those math skills of yours. The fact that the linkages were 1,4-Iinkages is i moot point rvhen considering the mass of the compound. Either way you look at it, the polysaccharide loses one water per linkage, regardless of the exact linkage. Choice C is correct. This question tests straight memorization, so remember it correctly, then choose C. Should vou not recall the fact that polysaccharides form linkages from carbon one to four (when dealing with pyranoses), then the passage also gives you a subtle hint, by giving only one example of a linkage, and the example is a 1,4-linkage. Choice A is correct. An aldopentose is most likely to form a furanose ring when the hydroxyl on the penultimate carbon (carbon 4) attacks the aldehyde carbon. This eliminates choices C and D. Because the original sugar is an aldehyde, the structure is a hemiacetal and not a hemiketal. The best answer is choice A.

sugar. The nucleophile is methanol, which attacks using the lone pairs on its oxygen atom. This eliminates choices A and B. This question now focuses on whether it is the c[-anomer or the G-anomer. The methoxy group is irans to carbon 6, so it is the u-anomer, making choice C the best answer.

Choice C is correct. The glycosidic bond forms when a nucleophile displaces the anomeric hydroxyl group of the

Choice A is correct. An aldohexose has one aldehyde functional group, one primary alcohol functional group, and four secondary alcohol functional groups. This makes choice A the best irlr-"t. An aldohexose has chiral centets at carbons 2,3, 4, and 5, so choice B is eliminated. The aldehyde carbon has the most bonds to oxygen of any carbon, so it is most oxidized. This eliminates choice C. For a D-sugar, the penultimate carbon has Rchirality, so for a D-aldohexose, carbon 5 has R-chirality. Choice D is eliminated.

A ketohexose is a six-carbon sugar with a ketone functionality (likely on carbon 2) in its If it were in the cyclic form, the term becomes pyranose or furanose depending on the ring size. Since fructose is the only ketose listed in the choices given, it is best for you to choose B.
hnear structure.

Choice B is correct.

:rright

by The Berkeley Review@

t4t

CARBOHYDRATES EXPLANATIONS!

17.

Choice A is correct' W1:" the hydroxyl group on carbon 5 of an aldohexose is inverted, the sugar becomes the opposite type of sugar. This means that the C-5 epimer of L-glucose must be a D-sugar, eliminating choice B. ln order.to form D-glucose from L-glucose, all of ihe chiral cinters must differ, because the two structures are enantiomers. Only one chiral center differs, so choice D is eliminated. The C-5 epimer of L-glucose has hydroxyl SrouPS on the right on carbons 3 and 5, and hydroxyl groups 2 and,4 on the left, which a"ccording tr Figure 1 is D-idose. The best answer is choice A.
Choice D is correct. D-Gulose has hydroxyl groups on the right for all carbons except carbon 4. The C-2 epimer of D-gulose has hydroxyl groups on left for carbons 2 and 4. The C-3 epimer of giucose has all of the hyarox.,groups on the right, so choice A is eliminated. The C-3 epimer of D-idose has thJhydroxyl group on the left for carbons 2,3, and 4, so choice B is eliminated. The C-4 epimer of D-allose has the nyato*yt on the left on carbon 4 only, so choice C is eliminated. The C-4 epimer of D-altrose has hydroxyt groups Erorp on the left for carborx 2 and 4, so choice D is the best answer.

18.

1.9.

Choice B is correct. Ketals can not be oxidized by Tollen's reagent, because they are stable under basic conditiom and they cannot be converted into aldehydes (which can be-oxidized). The key feature in a ketal preventinq oxidation is the absence of a C-H bond on the ketal carbon (which is the feature that prevents a ketone fror: being oxidized) ' The C-H bond is necessary for oxidation to occur. oxidation from the organic chemislrr perspective involves the loss of hydrogen as well as the gain of oxygen. For this question, chooJe B for greatest satisfaction. Choice C is correct' Fructose is a ketose, so its cyclic structure cannot be an acetal or hemiacetal. This eliminates choices A and B' Ketones, when added to alcohols in basic medium, go on to form hemiketals. The hemiketai h'i1l be in equilibrium with ketone. The question does not state whether the medium is basic or acidic, so ).oin must answer this from experience and memory. Monosaccharides for hemiketals (or hemiacetals if the structure starts as an aldose). This means C is the best choice. Epimers differ by one chiral center in their backbones. Talose differs from galactose m carbon 2, differc from idose at carbon 3, and differs from mannose at carbon 4. This means talose is air epimer crl galactose, idose, and mannose, eliminating choices A, B, and C. It differs from glucose at carbons 2 and 4., meaning it is not an epimer of glucose. This makes choice D the best answer.

20.

27.

Choice D is correct.

tYo

H o--f- H H H

o-l- n u-l- ou
I

o{u

uo-l-

Y:" HolH
H

Hvo H o-l- r
H

I
H

uAoH

u-lI

oH

HAon
I

oIu

o**,

;Y"
oH
cFI2OH

'10

"Y"" uoJ- H
u-f
u--l- on
oH
cFI2OH D-(+)-Glucose

ru

u-l

H*oH

L@

UD

cFI2OH

cFI2OH D-(+)-Galactose

cFI2OH

D-(+)-Talose

C
ffiu

D-(-)-Idose

D-(+)-Mannose

roe

)',

Choice C is correct. Treatment of an aldehyde with a reducing agent such as NaBH4 reduces the carbonyl gror: (C=O) into a primary alcohol. Following that same reactivit|, treatment of an aldohexose (or ketohexose) rr-: convert the sugar into a hexa-ol (a six carbon chain containing six hydroxyls, one on each carbon). Treatment an aldopentose will convert it into a penta-ol. The results fiom an- analytical reasoning perspective are simil to treatment of the sugar with nitric acid, because the product now has ihe possibility oifehg meso (indicat bv a lack of optical activity). Fructose is eliminated, because the new hydroxyl would result in rL/uru^yr vvuutu rcbulr ut the IOrIIlaIlon i LIte formation

ffih

G
.d[,

cfr

a new chiral center, thus there would be the formation of two diastereomers. At least one of the t,,,,has to be optically- active Glucose and mannose are eliminated, because they have no mir:; 1i1-:*i"?*"rs carbons three and four. plane between This makes ribose the best choice. Ribose will generate a penta-ol with mirror YrqrLL'rLurb through Lcuuurt three. rlcK cnolce L for tne tlngly sensation of correctivity. Gal plane slicing rrtruu6rt carbon Lrlree. Pick choice C lor the tingly o1 Galactc h'ould also yield an optically inactive product although it is not given ai un ur,r*", selection.

W
i@

mry

i'@

'i@

cm!

M
Copvright @ by The Berkeley Review@

t42

CARBOHYDRATES EXPLANATIO]

23.

Choice C is correct. Galactose is the C-4 epimer of glucose. The C-2 epimer of galactose varies from glucose at both carbon 2 and carbon 4, so choices A and D are eliminated. The C-2 epimer of mannose is glucose, so choice B is also eliminated. The only remaining choice, and the correct answer, is choice C. Talose (which is the C-2 epimer of galactose as shown in Figure 1) is the C-4 epimer of mannose. Pick choice C.

n*o

n_?o
(- A nL-r-

n\J-ra

,,Ion tt
lL-+_lL-._

"-l-o'
n

r-

H-l- oH H-T- oH
CFLOH
D-Glucose

ePrmers

Hot u H--l- oH
cFI2OH D-Galactose

ePlmers

":I: <-S1+- Jr"" HO+H ,rola,


Ho--Fu
eplmers

.,-J- o"

u*oH
cFI2OH D-Talose

"+o"
cFI2OH
D-Mannose

Choice B is correct. The glycosidic linkage involves carbon 1 of the glucose on the left and carbon 2 of the fructose on the right. This eliminates choice D. The glucopyranose ring has c,-orientation, so choice C is eliminated. In the fructose structure, the anomeric oxygen (involved in the linkage) is cis with carbon 6, so it has B-orientation. This makes choice B the best answer.

Choice C is correct. Glycosidic linkages involve an anomeric carbon with two OR groups, so the functionality cannot contain the prefix "hemi". This eliminates choices A and B. Because maltose is formed from glucose, an aldose, the functional group is an acetal. This makes choice C the best answer. Aldoses go on to form

hemiacetals as monosaccharides and acetals as polysaccharides. Ketoses go on to form hemiketals as monosaccharides and ketals as polysaccharides. Choice D is correct. In G-D-glucopyranose, all of the substituents on the pyranose ring have equatorial orientation. This is a piece of information you should have committed to memory. The structure in question differs from B-D-glucopyranose at carbon 3, where the hydroxyl group has axial orientation. This makes the structure a C-3 epimer of G-D-glucopyranose, making choice D the best answer. An anomer would vary in chirality at the anomeric carbon. This eliminates choice A. A conformer is the identical molecule rotated or contorted. If only one substituent changes from equatorial to axial while keeping the ring in the same orientation, then the structures are not conformers. This eliminates choice B. Enantiomers differ at every chiral center, so all of the centers would need to be axial for it to be an enantiomer. This eliminates choice C.

l"

Choice A is correct.

A trisaccharide has two glycosidic linkages. This eliminates choices B and D. When the risaccharide is made from three unique aldohexoses, then the linkages will be acetal linkages, rather than ketal linkages. The linkages involve carbons with two OR groups, so they are acetal linkages, rather than r.emiacetal groups. This makes choice A the best answer.
Choice D is correct. Hydrolysis under acidic conditions hydrolyzes acetals and hemiacetals, but does not affect ethers. When a disaccharide is exhaustively methylated, all of the hydroxyl groups become methoxy groups. -{11 of the methoxy groups are ethers except for the anomeric carbon of the glycoside (sugar on the right), which :oes from a hemiacetal to an acetal. This means that the anomeric carbon of the glycoside and the anomeric

III

:arbon of the glycosyl group making the linkage are both acetals. When hydrolyzed, they will generate :l-droxyl groups. All of the other oxygen atoms will be part of methoxy groups. Not all of the OCH3 groups
:elurn to being hydroxyl groups, so choice A is eliminated. The glycoside loses the methoxy group on its anomeric :arbon, but not any others. The glycosyl group does in fact not lose any methoxy groups. Choice B is not a good rlswer, but it cannot be eliminated. The disaccharide only racemizes at the anomeric carbons, so choice C is .,rminated. Following hydrolysis, the two carbons involved in the glycosidic linkage gain hydroxyl groups ,.,-hen water is added. The best answer is choice D.

r,'-ght

by The Berkeley Review@

143

CARBOHYDRATES EXPLANATIONS!

29' Choice C is correct. The glycoside

is typically the sugar on the right, recognized because it does not use it-. anomeric carbon for-the linkage. G-D-Glucopyranose his hydroxvl lroups tliat alternate up-to-down-to-up-tcdown in the Haworth project. The sugar ot-t ih" right follows the saie puit"rrl as B-D-glucopyranose, except fo.. carbon 2, which has its hydroxyl group up insteaJof down. This means that the glycoside is the c-2 epimer o: G-D-glucopyranose, making choicJ C, D-rr-rir-r.ror", the best answer.

30'

Choice D is correct. Maltose is a disaccharide made from two rx-D-glucopyranose structures. The linkage must b. alpha, so choices A and C are eliminated. Choice B is eliminatea,"uecause the hydroxyl group on the glycosid(sugar on the right) has beta orientation. The best answer is choice D.

31'

Choice A is correct. Amylase breaks down the cr-giycosidic linkage. However, it is not stable under highir. acidic conditions, where it is reaclily hydrolyzed. Gistric. fluids uti nignty acidic, so amylase is destroyed r the stomach' It must be released uguitl try thl pancreas. This makes cho"ice'A the best answer. Both pancreatrand salivary amylase cleave u-glycosidic linkages, so choice D is absoluteiy wrong. The two forms of amylas; are identical enzymes, so they are equivalent in strength. This eliminates choices ts and C.
monosaccharides, so choices A, B, and C are eliminated. Lactose is a disaccharide of"galactose and giucose, so r:

32' Choice D is correct.

The blood can only absorb monosaccharides. Fructose, galactose, and glucose ar-

cannot be absorbed into the

which draws water in and results in diirrhea.

disaccharase lacta-se, therefore you cannot break lactose into galactose and glucose. Since we cannot absorb thdisaccharide, the iactose does not get absorbed and instead g6es to feed our"bacteria (which metabolize it to ga: and toxic metabolites). This results in pain and an increase in the number of molecules in the gut and lumer.

blood. Choice D is the best answer. In lactose intolerance, we don,t have

th..

33'

Choice C is correct. It is stated in the passage that amylose has no branching, so choices A, B, and D ar. eliminated' It is also stated in the puttug" thit amylopectin has o,-1,6-branchiig at about one out of ever.. thirty glucose residues. This confirms thatihoice C is the best answer.
Glycogen is a starch made from exclusively a-D-glucopyranose with a significant amount c: 1,6-branching' This eliminates choice A. The standard glycosidicii"r.ug; in the polyrucJharide is s-1,4, s: glycogen contains both s-1,4-linkages and s-1,6-linkages. This eliminates Jhoice D and makes choice B the bes: answer' Because glucose is an aldehyde,_all of the linkages involve anomeric carbons that are part of an acet.functional group. This eliminates choice C.

34' Choice B is correct.

35'

Choice D is correct. Amylopectin is a polysaccharide of a-D-glucopyranose that is held together by a-1,=glycosidic linkages with an occasional 1,6-glycosidic linkage ro. "b.ur-r.i,irrg. This means that thi oxygen ator.: on carbons 7,4, and 6 are involved in glycosidi. lit-rkug"s. Hydroxyl I is oi carbon 1, so the correct choice mu:contain I' This does not help, because every answer selection contains I. Hydroxyl II is on carbon 2, so the corre." choice must not contain II. This eliminates choices A and C. Hydroxyi rfu is on carbon 4, so the correct choic; must contain III. This does not help, because the remaining answer seiections (choices B and D) both contains IIj Hydroxyl IV is on carbon 6, so the correct choice must contain IV. only choice D contains IV, so it is correct. Choice C is correct. Cellulose is another polymer of giucose, similar to amylose, but with 13-1,4-glycosidr: hnkages' This etiminates choice A' Lactose il a disacchaiide made from glucose and galactose, so lactose cannc: be formed from the break down of amylose. Choice B is eliminated. Sucr6se is a disaccharide made from glucos. and fructose, so sucrose cannot be formed from the break down of amylose. Choice D is eliminated. Amylose broken down into glucose, which is absorbed by the blood and transported. Some of the glucose undergoe, glycolysis (that which isn't stored), resulting in the formation of pyr.r.rit". Choice C is the best answer.

36'

37" Choice C is correct. An aidopentose

has three stereocenters, located at carbons 2,3, and.4. The maximum numbe: of stereoisomers possible is found by 2n where n is the number of stereocenters. The reason the word ,,maximurr, is chosen is that if one of the stereoisomers happens to be a meso compound, then the total number of possibl. stereoisomers will_decrease by one. This won;t be a problem with the sugars, because sugars are not mesc o---* -' Because there are three stereocenters, there will be eighi possible stereoisomJrs. Choose C.

Copyright @ by The Berkelcy Review@

I44

CARBOHYDRATES EXPLANATIONS:

Choice B is correct. Because the cyano nucleophile can attack the carbonyl carbon from either side (not necessarily equally from either side however), carbon one in the reactant monosaccharide (which becomes carbon 2 in the product) generates a mixture of two chiral centers on carbon two of the product. No other chiral centers will change, so only the chiral center at carbon two differs between the two product molecules. This makes the two compounds diastereomers (non-superimposable and not mirror images). More specific than diastereomers when dealing with sugars is the term epimer, given to sugar diastereomers that vary at only one carbon in the backbone. This makes choice B the best answer.
Choice C is correct. Compound A is said to be an L-aldopentose. You should know from your information base that glucose is a six carbon sugar (aldohexose). This means that the unknown sugar cannot be glucose. The correct answer is choice C. Ribose you should know is an aldopentose, so choice A definitely should have been eliminated. Without knowing exactly what xylose and lyxose are, they cannot be eliminated' Xylose is the C-3 epimer of ribose and lyxose is the C-2 epimer of xylose.
Choice D is correct. Compound A is an aldopentose with a molecular mass of 150 grams per mole. When oxidized at both terminal carbons, the mass increases. This eliminates choices A and B. The formula for the aldaric acid formed is C5H3O7 so the molecular mass is equal to 60 + 8 + 772 = 180 grams per mole. Choice D is therefore the correct answer. You could also have solved this knowing that when it is oxidized, it gains two oxygen atoms and loses two hydrogen atoms, resulting in a net gain in mass of 30 grams/mole. Choice A is correct. Compound B has increased hydrogen bonding compared to Compound A, because carboxylic acids form stronger hydrogen bonds than alcohols. The increase in hydrogen bonding will manifest itself as an rncrease in melting point. The best answer is therefore choice A. Choices C and D should exclude one another, because if choice C were true, then choice D would have to be true.

Choice B is correct. Aldohexoses in their most stable form are actually aldohexapyranoses, the six-membered ring form of a monosaccharide. The ring forms when the hydroxyl group on carbon 5 attacks the carbonyl carbon. This means that there is no carbonyl absorbance observed in the IR spectrum of the most stable form of the sugar. The carbonyl peak in the IR is just above 1700 cm-l, so no peak is obserr.ed there. Choice B is the best answer. Choice B is correct. Because Compound B is optically inactive, it must be a meso compound. In order for the compound to be meso, there must be a mirror plane through the molecule. The molecule contains five carbons so the mirror plane must slice through carbon three reflecting carbon 2 onto carbon 4. Compound B is drawn with carbon 2 having accurate stereochemistry as determined by the optical inactivity of the diacid.

HO
On the left because H it is an L-suear \

On the left because of the mirror plane

'\u= \

(oH) (oH)
H
4

HO

-+

HNG

"o-crLou
5

HO HO Compound B optically inactive diacid

Compound A Unknown L-aldopentose


:ir:

if the hydroxyl group on carbon 2 is on the left in the Fischer projection of compound B, then it must be on the left
Ln

the fiscner projection of compound A (the original sugar). Hydroxyl groups on the left in the Fischer
S

projection are assigned

stereochemistry. Pick B.

I
:
.r{

Choice C is correct. Treatment of Compound X, an aldohexose, with NaBH4 will reduce the aldehyde iunctionality of carbon 1 to a primary alcohol. Because carbon 6 is also a primary alcohol, the two groups are tclentical. Generating matching groups at the terminal ends of the sugar is similar to what occurs when a sugar is oxidized with nitric acids (in which case both terminal carbons become carboxylic acid groups). Because the diacid of Compound X is optically active (it has an optical rotation associated with it), the reduced form (a hexa-alcohol) is also optically active. The best answer is choice C.

right @ by The Berkeley

Review@

145

CARBOHYDRATES EXPLANATIONS!

45.

Choice A is correct. Because Compound W (the oxidized aldaric acid derivative of Compound z, the aldopentose formed from the Ruff degradation oi Compound X) shows no optical activity, it must be a meso diacid' This means that the second and fourth carbons oico*pourrd w have opposite stereochemistry. This means that the hydroxyl groups in question are on the same side of the backbone in the Fischer -two projection' Carbons 2 and 4 in Compou"a w are originally carbons 3 and 5 from Compound X (the original aldohexose)' Because the original aldohexose is a D-iugar, the fifth carbon has R chirality. This makes the third carbon R, because the hydroxyl group is on the sarie side as hydroxyl five in the Fischer projection. Ali sugars drawn on the right in a Fischer projeition have R stereochemisiry. lict a.
Choice C is correct' Treatment of an aldohexose with the reagents for the Kiliani-Fischer synthesis generates a sugar with one more carbon in the backbone (an aldoheptosei. Because the chain increases by one carbon, an aldopentose is not possible, so choices A and B are eliminated. f.he addition of C=N- occurs at carbon 1 of the original aldohexose. The chirality of the penultimate carbon remains the same, because the penultimate carbon is still the same carbon. This makes choice C the best answer.

46.

N
llt
C

n-lH-l-

Jr:"",
roni
cFI2OH

H H H

roHl CN- H
H

"+oH'
17

-F(oH) *o.t
I

trottr
cFI2OH

-]-

-l-

lour ,ott,
H2

.t-J- ro"l n-f-roru


t*o.,
I

t F*t n-l- roHl

rll-roHr
H*

Jr:"",
oH

s[

Pd (BasQ)

H--F(oul --+ Hzo


cFI2OH

-=+

u-F

H--|.- roHl u--.1-roHl


cFl2OH

The penultimate carbon (next to the last carbon) has D chirality in the final sugar (aldoheptose).

' choice D is correct. A reverse aldol reaction

cleaves between the alpha and beta carbons of a beta hydror.r carbonyl compound' This means that Compound X, an aldohexose, is cleaved into a two carbon fragment (fron carbons 7 and2) and a four carbon fragmeni(from carbons 3 through 6). The four carbon fragment as stated in the question is an aldotetrose.

the aldotetrose is reduced with_sodium borohydride, the four carbons (3 through 6 from the origina; aldohexose) have hydroxyl groups. Because the tetra-ol of the aldotetrose is optically active, its two chiral centers have like stereochemistry (If they were R and S, then the compound would t" -"ro and opticallr inactive)' This means lft the two hydroxyl groups are on opposite sides of the sugar backbone in the Fischm projection' Carbon 3 of the aldotetrose (carbon 5 fiom the original aldohexose) must be R given that the origiruil aldohexose is a D-sugar. This means that carbon 4 from the"original D-aldohexose has S chirality (which pu:s it on the opposite side from the fifth hydroxyl in the Fischer projection). pick D for best results. " The draw',g below summarizes the reactions.

After

1[

3ctt"oH (oH)
OH
Reverse aldol H __-+

3ct-troH

rI

;(OH) +
6crtrou
5

NaBFd

(oH )
OH

OH

u,-[j,o' ut-L o" .1,


cFI2OH

Ho-f--u
=

H-loH .15
n

cF{2oH

aldotetrose

tetra-ol

optically active tetra-ol

48'

Choice B is correct. The number of possible stereoisomers is 2n where n is the number of chiral centers. The ,",a.1: of n in the case of a straight chain uldoh"tor" is four. 24 16, so there are sixteen possible stereoisomers for r = aldohexose' of these 16 possible stereoisomers for an aldohexose straight chain, ur" D-sugars and e are L-sugars. This make B the best answer. Pick B, and do what is best... L terms of"ight answering that is.
146,

Copyright O by The Berkeley Review@

CARBOHYDRATES EXPLANATIO]

49.

aldehyde to a primary alcohol making both terminal carbons identical without affectinglthe other carbons. This is analogous to the results when using nitric acid. Nitric acid is cheaper than sodium btrohydride, but as a general rule, the MCAT has stayed away from questions about cost effeciiveness of chemical reictions. pick A and show off your correct answer selection skills. Choice B (H2SO4) dehydrates a sugar to carbon, while choice C (Br2(aq)) will oxidize only carbon 1 into a carboxylic acid, not both terminal carbons. Choice D (KMnOa(aq)) will oxidize everything on the sugar, so choice D is a bad answer. It is bad to pick bad answers; don't be badl
i0. Choice D is correct. The most stable form of an aldohexose is the pyranose ring structure. This ties up the OH of carbon 5' Because periodic acid (HIOa) cleaves 1,2-diols, the sites are limited io neighboring diols aiCl and C2, C2 and C3, and C3 and C4. The least sterically hindered is the C3 and C4 pair of alcohols. Pick D. This information could be extracted directly from the passage, which is common on the MCAT.

(with HNO3 they are both carboxylic acid groups). The addition of NaBHf to ihe sugar will reduce the

Choice A is correct. The addition of HNO3 results in a compound with identical groups on the terminal carbons

D-aldohexose. To determine the D-aldopentose needed to synthesize the D-aldohexose in the question, using retrosynthetic analysis, carbon 1 must be removed and carbon 2 must be turned into a aldehyde group.

The addition occurs at carbon 1 of the D-aldopentose, turning carbon L into carbon 2 in the D-aldohexose products. This means that the chirality of carbons 2,3, and 4 in the D-aldopentose match that of carbons 3, 4, and 5 in the

Choice D is correct. Using the Kiliani-Fischer method, a D-aldopentose is used to synthesize a D-aldohexose.

H*-OH

uo_{_u

uIoH
ou
cF{2oH

u-l-

uo*H H*ou
cFI2OH

H-l-

oH

D-Xylose

Upon matching structures with the four D-aldopentoses shown in Figure 2, the best answer is D-xylose, choice D. Choice D is correct. To be C-2 epimers, the sugars must be diastereomers that vary in their chirality at carbon 2. Ribose is an aldopentose and altrose is an aldohexose, so they are not even isomers, let alone epimers. Choice A is eliminated. The only way to answer this question, without actually knowing what the sugars are, is to employ the data in Table 1. The products of Kiliani-Fischer synthesis arc C-2 epimers. Glucose and talose are rot C-2 epimers and neither are mannose and allose. This eliminates choices B and C. Idose and gulose are :ormed as products when xylose undergoes Kiliani-Fischer synthesis, so idose and gulose are C-i epimers,

naking choice D the best answer.

Choice C is correct. Reduction in organic chemistry is most simply viewed as either the gain of bonds to
:.r-drogen or the loss of bonds to oxygen. In step 1, the carbonyl carbon loses one bond to oxygen (in forming a new :ond to carbon), so carbon 1 is reduced in Step I. This eliminates choice B. in Step II, reduction occurs, because the :\-ano group gains a hydrogen on carbon and a hydrogen on nitrogen. The symbol [H] refers to reduction, so Step II ,s definitely reduction. This eliminates choice A. Step III involves the hydrolysis of an imine into an :idehyde. This is where the general definition needs to be a bit more specific. Both oxygen and nitrogen are rore electronegative than carbon, so when carbon goes from being bonded to nitrogen to being bonded to oxygen, it ::as not been oxidized or reduced. This means that Step III is not oxidation or reduction, so choice C is the best r:'swer. This is a case where oversimplification can hurt. Technically speaking then, oxidation occurs when the : --rmber of bonds to more electronegative atoms increases and / or the number of bonds to less electronegative atoms r:creases. So, for those of you who considered Step III to be oxidation, you still got lucky in thai you didn't .-*ure it to be reduction and choice C was still your choice. But for edification, know the more correct definition : oxidation and reduction for organic chemistry.

tlhoice D is correct. In the Kiliani-Fischer synthesis as shown in Figure 1, and aldehyde is first converted into a -..Jroxy nitrile compound. The nitrile is reduced into an amine. The amine is hydrolyzed into the aldehyde. ': the choices, only choice D, an imine, is an intermediate in the synthesis. Choice D is the best answer.

,:t

O by The Berkeley Review@

147

CARBOHYDRATES EXPLANATIONS!

55.

Choice C is correct. The two products formed from Kiliani-Fischer synthesis, according Table 1, are C-2epimers of one another. Epimers are diastereomers, which eliminates choices A, B, and D. The two diastereomers are formed in unequal concentration, because the presence of a chiral center in the reactant makes one side of the aldehyde easier to attack than the other. Choice C is the best answer. Choice C is correct- Talose differs in chirality from glucose at carbons 2 and 4. In the Haworth projection of G-Dglucopyranose, the hydroxyl groups are up (above the pyranose ring) on carbons 1 and 3, and dtwn in carbons l and 4. This means that talose has the hydroxyl groupJ on carbons i and. 4, up, so all of the hydroxyl groups ar up in l3-D-talopyranose. For talopyranose, the hydroxyl groups on carbons 2,'3, and.4 must bL up, io Jnoice c i: the best answer. Choice D is correct- Kiliani-Fischer synthesis adds one carbon to the sugar structure. Choice A is eliminated because a carbon is lost, not gained. Choice B is isomerization, where thi number of carbons does not change This eliminates choice B. In choice C, the nurnber of carbons in the product is not listed, so the choice can neithe: be confirmed nor refuted. In choice D, the number of carbons has increased by one. This makes choice D the bes: answer.

56.

57.

58.

Choice C is correct. The unknown disaccharide in the passage is comprised of two cyclic monosaccharides formej from aldohexoses (aldopyranose structures). A cyclic aldohexose molecule has five stereocenters (one at evert carbon except the terminal carbon, carbon 6). Given that each monosaccharide has five chiral carbons, ; disaccharide must have ten stereocenters total. pick choice C.

59.

sugars wiil lead to an osazone that matches either glucose or mannose, neither sugar leads to an optical-; inactive aldaric acid, and the two sugars are not epimers of one another.
60.

Choice B is correct. Choice A is eliminated, because the structure on the left is glucose. Neither sugar can b,; glucose, because the specific rotations of the osazones of both sugars do not mat"ch the specific rotation of tl-: osazone of glucose. Choice C is eliminated, because the structure on the left is rrlu.nor", the C-2 epimer u: glucose. Neither sugar can be mannose, because the specific rotations of the osazones of both sugars do not matc: the specific rotation of the osazone of glucose (which would also be the osazone of mannoie). Choice D --. eliminated, because the two structures ate C-2 epimers of one another. Because the two sugars form osazon* with different specific rotations, the two sugars cinnot be C-2 epimers of one another. In choice B, neither of tlo:

Choice D is correct. When an aldopentose is oxidized with nitric acid to form an aldaric acid, the two carbons ;: observe for optical activity are carbons 2 and 4. if the hydroxyl groups on carbons 2 and.4 are on the same side id the chain in the Fischer projection, then the compound is meio ind inus optically inactive. It is only in choice D that the two hydroxyls on carbons 2 and 4are on opposite sides. Choice D is the best answer. Choice D is correct. The osazone of Compound I has specific rotation +42.6", the osazone of Compound il hnr specific rotation +37.2' , and glucose yields an osazone with a specific rotation of +54.6' . The osazone of manncse will also show an optical rotation of +54.6', because the C-2 epimer of glucose. This means that fe ^urlnor"l, unknown sugars cannot be either glucose or mannose (selection I). It is possibi" for thJ unknown sugars to be eil",s talose or galactose according to the osazone test. Because the aldaric aiids of both Compound I anl Compoun; I

61.

means that the unknown sugars cannot be galactose (selection

are optically active, they are not meso. Galactose leads to an optically inactive (meso) aldaric acid. I::s III). Compounds I and II cannot be mannose :u' galactose, making choice D the best answer. For those of you who chose B, because you forgot it u,a. n "CANNOT" question, try to limit your scream to a few seconds... "AAAAHHHHHHHHFiHHHHHHHHII:

62.

Choice C is correct" Choice A wilt yield an optically inactive aldaric acid when treated with nitric ac.i. because it will have a mirror plane slicing the carbon-carbon bond between carbons three and four. Neifisr Compound I nor Compound II yields a meso aldaric acid, so choice A cannot be Compound I or CompounC Choice B is mannose, the C-2 epimer of glucose. Choice D is fructose, the ketose isomer of g1.r.or". Both choices and D are eliminated, because they will both yield the same osazone as glucose and Comp6unds I and Ii genei; a different osazone than glucose. Choice C will not lead to an opticalty inactive aldaric acid, because it will :r be meso after oxidation. The best option for you is choice C.

Copyright @ by The Berkeley Review@

t4a

CARBOHYDRATES EXPLANATIO]

53.

Choice D is correct. Identical osazones will form from sugars with matching chiral centers from carbon three down. This means that C-3 and C-4 epimers will form different osazones, so choices A and B are eliminated. Choice C is a pair of enantiomers, which would generated enantiomeric osazones, so choice C eliminated.
Fructose is the ketose formed when mannose isomerizes from an aldose into a ketose. Mannose and fructose are identical from carbon 3 on, so they form identical osazones. The best choice is thus answer D.

";r: uo-]n-]H*oH

rI oH

Hq 3uq N-N HH

N-NFIZ

no-J-

F N-NFIZ =
H

n-F

ulon

H@ 3eq N-N HH

t; HoJ-H
u*oH

CFLOH

ou

utoH
cFI2OH D-fructose

cFI2OH
D-marrnose

cF{2oH Osazone product

a'4' Choice D is correct. Because the final osazone product after treatment of the sugar with three equivalents of phenyl hydrazine contains two phenyl hydrazine moieties, it is safe to assume that two of the three equivalents are substituting onto the sugar. The third phenyl hydrazine gains two hydrogens to form ammonia (NHs) and aniline (C6H5NH2). The gain of hydrogen is reduction, thus one of the three phenylhydrazine
molecules is reduced. The best answer is thus choice D.

Choice D is correct. Glyceraldehyde-3-phosphate and dihydroxyacetonephosphate are linear sugar structures with the same formula, so they are isomers of some sort. Dihydroxyacetonephosphate has no chiral center,

while glyceraldehyde-3-phosphate has one chiral center, so the two structures cannot be configuration isomers (isomers that have the same bonds, but different spatial arrangement). Epimers and diastereomers are configurational isomers, so if the two structures are not configurational isomers, they cannot be diastereomers or epimers' This eliminates choices A, B, and C. An aldose and ketose of the same carbon length are structural isomers. In this example, they each have one phosphate group, so choice D is the best answer.

no ts O chiral centers I
k5!os9 with

FH2oPo32- C3gso6p2- '


(same formula
-.

CFI26H

-different bonds)

{o but g--L Ou I
CFI2opo32-

al&se with one chiral center

Dihydroxyacetonephosphate

Glyceraldehyde-3-phosphate

Choice D is correct. Carbon 4 of glucose^becomes the aldehyde carbon in glyceraldehyde-3-phosphate. It loses its original chirality when it takes on sp2-hybtidization (associated with the aldehyde carbon), thus it has no chirality in the end. Galactose (the C-4 epimer of glucose) should therefore yield the exact same products as glucose if the enzymes were able to recognize galactose. The remaining chiral centers are identical between Dglucose and D-galactose, so the chirality of the products is the same. The best answer is choice D.

Choice C is correct. The lower three carbons (carbons 4,5, and 6) of D-glucose form the glyceraldehyde-3rhosphate molecule, so carbon 5 finishes as the middle carbon (C-2) of glyceraldehyde-3-phosphate. Choice C rs the best answer. Because of isomerization, the label is also found at C-2 of DHAP, but that is not a choice.
CFI"OPO.2H
il
!1

HO H
I

OH H Step

I g9
I

OH cFi2oH

OH H OH
cI-I2OPO32-

cFI2OH
+

Fo

l'J

fii

H-*C-OH

H.*C-OH

H- *C. OH I cltoPo32-

H-*C-OH
I

cFI2OPOq2-

H-*C-OH
I

cFI2OPO32-

'1":-ght O by The Berkeley Review@

t49

CARBOHYDRATES EXPLANATIONS!

58.

Choice C is correct. This is a question of Hess's Law as applied in thermodynamics. The overall free energy change is the sum of the individual free energy changes foi Step I through Step IV. The math to determine the sum of the four steps is carried out as follows: -4.0 + (0.4) + (-3.4) + (5.27 = -7.4 * 2.1 _1.3, which is choice C.
=

69.

Choice B is correct. The two terminal carbons in dihydroxyacetone phosphate (DHAP) have two equivalent hydrogens and the middle carbon has sp2-hybridization, so none of the are chiral. The first carbon in "uibor,, glyceraldehyde-3-phosphate (G-3-P) has sp2-hybridization and the third carbon has two equivalent hydrogens. Only the second carbon of G-3-P has four different substituents attached to it. The total number oi chiral centers between the two compounds is therefore just one. The best answer is choice B.

70. Choice A is correct. Fructose is a ketohexose with a ketone on carbon 2, so carbon 2 is the anomeric carbon in the furanose ring structure. The phosphate group is located on the last hydroxyl group (carbon 6) of fructose. Fructose, s'-D-fructofuranose, fructose-6-phosphate, and s-D-fructofuranose-6-phoiphate are shown below.

CI-I"OH

CFI"OH CFLOH

,[o

Ho-l-H

H-r- oH
I

,r-l6

uo-lOHH
c-DFructofuranse

,[o
H

cF{2oPo32-

9nzoH

o.t

u-l6

H-J-or-r
oH
CFI2OPO32-

CF{2OH

OHH

Fructose

Fructose-6-phosphate s-D-Fructofuranse-6-phospha:

Choice C should be eliminated, because carbon 3 has no hydroxyl


because the phosphate is on the wrong carbon.

group. Choices B and D are eliminated

7'l', Choice A is correct. This question tests your knowledge of glycolysis.

Glucose is a six-carbon sugar while pyruvate is a three-carbon structute, so two pyruvates form from one glucose molecule. This eliminates choices C and D. Early in glycolysis, two ATPs are invested, but two ATPs are formed per pyruvate, so the net result rs that two ATPs are formed in glycotysis. This makes choice A the best answer.

72.

Choice D is correct. The linkage can be determined directly by looking at the two sugars of interest out of the three sugars in the type-O blood antigen. N-acetylglucosamine is the ]irst sugar link"ed to the protein (the \acetyl should give this away), thus the galactose must be the second sugar fiom the protein ln the antigeru: determinant trisaccharide. The linkage is from carbon 1 of the galactose t6 carbon 4 of irl-acetylgiucosamiie bo way of the B-anomer of galactose. This means that the linkage is a B-1,4- glycosidic linkage, so cloice D is bes; The structure is drawn below.

HO6
4

HO

%.frSA, ovo
OH
HO

tt \

I
rux

protein

,Iffi

CFI,

&

73.

Choice D is correct-.

of aldoses, therefore there are three acetal functionalities (one for each sugar) rather than ke:
150 CARBOHYDRATES EXPLANATIO

All three sugars of the type

fr
O blood antigenic determinant are formed from the cycliza.;
0!

best answer is therefore choice D.


Copyright @ by The Berkeley Review@

functionalities. All three sugars are linked to something, so they are all acetals rather than hemiacetals. Tl

{u

ruffi

74' Choice D is correct. Fucose, differs from a normal sugar in that carbon 6 has been reduced from a primary alcohol i1!o a methyl group' If the methyl group is repLced with a primary alcohol group, then it becomes an
aldohexapyranose. This question centers on the relationship of the aldohexapyranose derivative of fucose and glucose' B-D-Glucopyranose has all of its substituents in equatorial position. Ho-"rr"r, because carbon 5 of fucose in blood type o has S-chirality, it is an L-sugar rather than a D-iugar. B-L-Glucopyranose also has ali of its substituents in equatorial orientation. The fuiose derivative with thJ hydroxyl group on carbon 6 has all of its substituents in equatorial orientation except the hydroxyl groups or, .urton, i q. The group on carbon 1 is "l^ra irrelevant in this question. The best ut t*e} is choice p, ine c-+^epimer of glucose. This means that when carbon 6 of galactose is reduced from a primary alcohol into an alkyl group, ficose is formed. L-Fucopyranose, Lgalactopyranose (L-fucopyranose with a hydroxyl group added to-carbon 6), and. L-glucopyraro.u *L all drawn below.

OH
la OH

OH
10

OH HOFIC HO HO HO
1cr

OH
HO
o-L-Galactopyranose
(a-L-Fucopyranose

OH

HO

w/

a-L-Glucopyranose

6-OH)

The correct answer is choice D.

The second structure_ (L-fucopyranose with a hydroxyl group added to carbon 6) is similar to that of Lglucopyranose, with the difference coming at the ihiral ."trl"t o? carbon 4. It is thus concluded that the second structure (the sugar formed when the CH3 group of fucose is replaced by CH2OH) is the C-4 epimer of L-glucose.

Choice B is correct. The 2-deoxyglucose portion of the name hints that the second carbon has no oxygen bonded to it, so choices C and D can be eliminated. Only choices A and B are deoxygenated at carbon z of thi ring. Choice A is an alpha sugar while choice B is a beta sugar. Choice B is therefore the best answer. iyranose

Choice A is correct. A normal L-aldohexose has a hydroxyl group present on carbon 6, while fucose lacks a hydroxyl grouP on carbon 6. This means that fucose is more tuirr.ua tirun u normal aldohexose, so fucose results from the reduction (loss of oxygen) of carbon 6. The best answer is choice A.

best answer is choice C.

Choice C is correct. The sugar is a six-membered ring (a pyranose), rather than a five-membered ring (a furanose), so choices A and B are eliminated. AII of the choic"r-ur" cyclic galactoses, so identifying the sugar is not necessary. As drawn in Figure 3, the galactose that has been added to hydroxyl group of .u.bo1l of the third sugar/ is upside down compared to the standard chair projection. The hydroxyl l-inkage is still from the axial orientation (trans to carbon 6), so it is the alpha anomer bf galactose. The sugar'is an cr-"galactopyranose, so the

Choice C is correct. The type A antigenic determinant has four sugar rings, each of which has chiral centers at carbons 1 through 5. This means that there are five chiral centeti p", sugar ring, and four rings total. Five centers times four sugars results in a total of twenty chiral centers. The .otr"c1 unr*", is choice C.

-i

Choice C is correct. According to Table 1, conversion of phosphoenolpyruvate into pyruvate releases 14.8 kcal/mole. Most oJ the other phosphate compounds in Table 1 release between 5 and 1b'kcals per mole when hydrolyzed' It can be assumed that hydrolysiJ of phosphoenolpyruvate into pyruvate enol releases between 5 to 10 kcal/mole. This means that the conversion from pyruvate'enol into pyruvate releases between 5 and 10 kcalslmole, given that the overall conversion from phosphoenolpyrurrut" lr-rto pyruvate releases about 15 kcals,/mole. The release of energy makes AG a negatirr" rrulru, so-c-hoices A and B are eliminated. The best answer is choice C, given that it is the only choice that fits within the range.
Choice A is correct. The conversion of ADP into ATP involves the gain of a phosphate, which according to Table 1 requires 7.3 kcals/mole. Only acetyl phosphate releases *or* thur"r 7.3 kcals/mole when it is hydrJyzed,, so choice A is the best answer. Choices B, C, and D all release less than 7.3 kcals/mole. 151

"'

, .:r'right O by The Berkeley Review@

CARBOHYDRATES EXPLANATIONS!

81'

choice C is correct. \Atrhen bonds are broken, energy must be added to the system. when bonds are formed, energlis released from the system. An energy releasinf reaction such as hydrolysis of a phosphodiester thus resu]F from the breaking of weaker bonds and formatioi of stronger bonds. This means tnat tfre phosphodiester bonc must be weak, eliminating choices A and B. The weakness can be attributed to repulsion from the negatir-t charges located on the oxygen atoms in the phosphate groups. Repulsion causes the bond to elongate, where longer bonds are weaker bonds. Choice C is the best answir. Hydrogens are not gained or lost w^hen phosphoenol pyruvate, pEp, loses its phosphaie group to ADP, so the enzyme is not PEP dehydrogenase. Choice A ii eliminaiea. nnospnoenol pyruvate, pEp, = not oxidized or reduced in the conversion, so the enzyme is not PEP reductase. Choice B is eliminatei Phosphoenol pyruvate, PEP, does not make or break any cirbon-carbon bonds in the conversion, so the enzyme rs not PEP aldolase. Choice D is eliminated. A phosphate group is transferred, so the enzyme is a kinase. The bes: answer is pyruvate kinase, making choice C the best answer.

82' Choice C is correct.

83'

Choice D is correct. When glyceraldehyde-3-phosphate, G-3-P, converts to 1,3-bisphosphoglycerate, th. number of bonds to oxygen on carbon 1 increases. This means that carbon t has been oxidLed, elim"inating choice A' The oxidizing agent used to oxidize the aldehyde into a carboxylic acid is NAD+ (which is poor in H, , property of an oxidizing agent), so choice B can also be eliminated. Carbon 2 starts and finishes with the exa.-: same bonds, so it is neither oxidized nor reduced. This eliminates choice C. By process of elimination, choice D is the best answer. The passage_ provided no information to know this fact, so you either had to know fror* . memory that biotin is not required or you had to use your chemistry logic to eliminate incorrect answer choices.
Choice A is correct. According to Table 1, 5 kcals/mole is released when glucose-1-phosphate is hydrolyze; This means that you need to determine which structure represents glucose-"1-phosphite. bhoi.", B and D ar eliminated, because the phosphate group is on carbon o. tn B-o-glu"copyranoi", tr," hydroxyl groups alternarn from up-to-down-to-up-to-down when vilwing from carbon 1 throrfih This is only oUr"irr"a ir-, choice A "urUo.,4. so choice A is the best answer Choice A is correct. The ring is a five-membered ring, so choices C and D are eliminated. The anomeric carbon $ carbon 2, on the right half of the sugar as drawn. rhe hydroxyl group on carbon 2 is down, while carbon 6 is u: This means that the two groups are trans to one another, -utirig thl sugar an alpha sugar. rhe besiln;;;;; choice A.

84'

85'

lu

(
85'
Choice B is correct. A shortcut for evaluating the chirality of sugars in the Fischer projection is to realize tha: all hydroxyl groups on the right side of the baikbone are located on an R stereocenter while all hydroxyl grouF$ on the left side of the backbone are located on an S stereocenter for standard sugars drawn according to conventicn with the carbonyl carbon at or near the top. The procedure for obtaining"the stereochemistr/ of one of 1'c stereocenters is drawn below: Method
1

r@

0m

ffi

M m

nYo
H>i< OH Ho-].-H I g>I< 6g i
cFI2OH D-Xylose
Draw the Fisher projection

Method 2

nvo
I

m
ffip

OH
H

.t iruti"li".

Solving the

HO H
S

first I -\ yi.ii' Hti< oH

ssffs

m
,SU,

offi

OH
cFI2OH on the right.
R for hydroxyls

for hydroxyls

n-l- on
D,XytJse

on the left"

D-Xylose
For sugar is in the standard Fisher projection, hydroxyls on the right have R chirality and hvdroxyls on the left have S chiraiity

i*o'

mM

rms

with Counterclockwise normally indicate 3-dimensional accuracy. The Hs an s stereocenter, but because tf,e are in front in Fisher projections. hydrogen is coming out of the plana
the stereocenter is reversed to R.

The compound has chirality of 2R, 3s, 4R, so choice B is the best answer.

mmfll

Copyright @ by The Berkeley Review@

152

CARBOHYDRATES EXPLANATIONS!

i-.

multiplying by the molecular mass of carbon dioxide (a ). The long-hand math goes as follows:

Choice B is correct. From the balanced equation, there are six CO2(g) formed for every one glucose that reacts. To solve this question, you must convert the glucose from grams to moles, convert moles of glucose to moles of CO2(g), and then convert moles of CO2(g) into grams of CO2(g). This process requires dividing by the molecular mass of glucose (180), multiplying by the ratio from the balanced equation for carbon dioxide and glucose (6:1), and then

10.00gC6H12o6" lmole * -

6COz ,44grams=10x 1 x6x44q.Co) 180 grams 1 C6H12O6 1 mole 180 1 "

The set up is what is requested in the question, so the best answer is choice B.

:i.

Choice B is correct. The best explanation for the difference in heats of combustion between the aldohexoses (glucose, galactose, and mannose) is structural in nature. They all have the exact same bonds (thus they are stereoisomers) so according to the simplistic application of Hess's Law, they should yield the same amount of heat. Because the heats are so different, there must be a difference in stability. The more stable the molecule, the less heat that will be given off. Because all of the aldohexoses have the same bonds, choice A is eiiminated. Sugars are neither aromatic, nor do they have any stabilizing resonance forms. This eliminates choices C and D. The only choice left is answer B, stating that the sugars have different orientation is space and thus different hydrogen bonding. The sugars also exhibit differences in their steric interactions. The best answer of the choices is choice B.
Choice C is correct. The given in this question is 15 grams ribose which, when divided by the molecular weight of ribose (150 g/mole), gives 0.10 nroles of ribose. The AH.o-bustion for ribose is lisied in Table 7 as -2076 kJlmole. This means that the 0.10 moles of ribose generates 207.6kl (207,600 J) of heat when it is burned (roughly 50), making choice C the best choice. The mathematical layout is drawn for you below:

'

(oxidized). Plugging into the equation E = mCAT yields which is approximut"ty ?90'900 ::::ry^K, 1000 x 4.18 1000 x 4

15gC5H1so5x lmole x2076kL=207.6kJ&E=mCAT...AT= E 150

207'600
1000 x

grams 1 mole

mC

4.18 4.I8

=207.6 =200 =50"C


4

see a calculation this lengthy on your MCAT, but you should still understand the set up and the theory behind the math.

It is doubtful that you will

--

Choice B is correct. The most COz(g) results from the combustion of the compound with the largest mass percent of carbon. This is because for every carbon within a sugar, one CO2(g) molecule is formed. Therefore, thl more carbon by mass in the compound there is, the more COZ(S) that is formed upon complete oxidation. The mass percent of carbon in any monosaccharide is 40%. The mass percent of carbon in sucrose (C9H22O1) is greater than the mass percent of monosaccharide, because the difference in the ratio of C:H:O between sucrose and the monosaccharides is due to a loss in H and O from the water. This increases the relative abundance of carbon in sucrose compared to a monosaccharide giving sucrose a greater mass percent of carbon than a monosaccharide.
The mass percent of carbon in sucrose mass percent of carbon in sucrose gram of sucrose than from the combustion of one gram of any monosaccharide. Choose B and be huppy.

1'44 = 144 = 72 and. 72 > 72 = 0.40 ... 72 > +o%. Becatrse the 1.M+22+776 342 177 171 180 777 is greater tltan 40"/o, more CO2(g) will be produced from the combustion of one it'

",

A is correct. For this, you must use the AH in terms of k] per gram, where the kJ/g value is found by dividing the kJ/moles by the molecular mass of the sugar. Monosaccharides have masses that are multiples of 30 g/mole, so the numerators should be simple to work with. The following calculations show the energy per gram values for each of the sugars given in the answer choices. The correct answer is the sugar with the highest numerical value.
Choice Glucose.2538 =423

180

30

Xylose '2102 -420'4 -

150

Ribose .2076

30

- 415.2 150 30

Ribose gets eliminated immediately, because it is equal in mass to xylose but produces less energy. The possible answers are narrowed down to either glucose or xylose. Instead of solving the math, it is easier to reduce the two l'alues to something over a common denominator (in this case 30). Glucose results in the greatest numerator, making choice A correct.

:vright

by The Berkeley Review@

153

CARBOHYDRATES EXPLANATIONS!

92.

Choice D is correct. Addition of an excess of periodic acid to a monosaccharide results in the oxidative cleavag" of ail of the carbon-carbon bonds in the sugar. Aldehydes are oxidized into formic acid , primary alcohols ar; oxidized to formaldehyde and the secondary alcohols associated with internal carbons are oxidized by losir: both carbon-carbon bonds to form formic acid. The product distribution for the decomposition of glucose is dran: below:

-----+
H

OH+ H_+
o

HO
H H

------+ D-glucose
CFI2OH
93.

-J(

In the end, the full oxidation of glucose yields 5 formic acids and 1 formaldehyde. Choose D for correctness.
Choice C is correct. The molecular formula for xylose is C5H16O5. The molecular weight of xylose (or any oth": aldopentose or ketopentose) is 60 + 10 + 80 = 150 grams/mole. The mass of the five Iarbons'is 60 grams. Tlmeans that the mass percent of carbon in xylose is 60/150 x 100% = 40"/o. As a point of trivial interJst, the ma.. percent of carbon in any monosaccharide is always 4O'h. Choose C and feel happlly correct.

94. Choice A is correct. In order to be optically inactive, the compound must be either meso or achiral. In the case .: sugar derivatives, the diacid (aldaric acid) formed after oxidation of the terminal carbons must be meso to :,. optically inactive. D-mannose has stereochemistry of 25,35,4R,5R, which does not yield a meso aldaric ac-: when oxidized, so the aldaric acid derivative of mannose is optically active. D-galactose has stereochemis5 of 2R,35,45,5R, which yields a meso aldaric acid which is opticallyinactive. D-"ribosehas stereochemistrr':r2R, 3& 4R and is five carbons long (which you should know from memory), so the third carbon would no longer :*

a chiral center in the aldaric acid form. The aldaric acid of D-ribose ir *ero and therefore optically inactii': D-allose has stereochemistry of 2R, 3R, 4R, 5R, which would form a meso aldaric acid *ni.n is opticai-r inactive. The aldaric acid of D-allose is meso and therefore optically inactive. The only sugar to ftrm ;: optically active aldaric acid derivative of the given choices is D-mannose. The best answer is choice A.

HO H
HO

HO

o
OH

o
OH OH

--+t
H

oFt-OH

OH OH HO Aladirc acid derivative


of D-Mannose
(not meso

HO H

HO

.'. active)

Aladircacid derivative of D-Galactose (meso .'. inactive)

HO Aladirc acid derivative of D-Ribose (meso .'. inactive)

:!_

HO

..:
:_

Aladirc acid derivative of D-Allose (meso .'. inactive)

Copvright @ by The Berkeley Review@

154

CARBOHYDRATES EXPLANATIO\SI

95. Choice A is correct. When treated with nitric

acid, the two terminal carbons (carbon 1 and carbon 5) of an aldopentose get oxidized into carboxylic acid functionalities. This results in the formation of a 1,S-dicarboxylic acid (aldaric acid). After the oxidation reaction, only carbons 2 and 4 can be chiral centers, because the third carbon now has two identical substituents attached to it (making carbon 3 achiral.) In a D-aidopentose, carbon 4 has R stereochemistry. This would mean that carbon 2 must aiso have R stereociremistry in the original sugar, although that changes in the aldaric acid, because the priorities are different. Onty choice A has iiris feature. The stereochemistry of carbon 3 is irrelevant.

no

{o Ha-(oH)
I

HO

HO

Ho
OH

/'o
I

(oH)

ronl

sQt-

--+I
H HO Aldaric acid

(oH|
OH

--+{
H

(GH}
OH

H-l-I

l->\l

H--Fos u-T-iour
r-rT
I

oH

vrf]'

cFI2OH
D-Aldopentose

HO

cr-LoH
2&3unkrown

(opticallyactive)

(opticallyinactive)

derivative hractivg so it must be meso of D-Aldopentose OH-4 on right ... OH-2 on right
OH-3 is unknown
b

/ c C-3 is achiral

96'

Choice C is correct. Ribose differs from deoxyribose in that carbon 2 has no hydroxyl group in deoxyribose. This means that carbon 2 does not have four unique substituents, so it is not a chiral fhis matls choice B a "Lnt"t. valid statement, which eliminates it. By having one more chiral center, ribose has more possible stereoisomers, making choice A a valid statement, and thus it is eliminated. Ribose, by having a hydroxyl group rather than a hydrogen on carbon 2, is more oxidized than deoxyribose. This makes cho]ice D a valid statement, which eliminates it. Ribose and deoxyribose do not have the same molecular formula, so they cannot be stereoisomers, let alone epimers. This makes choice C a false statement and therefore the best answer.

i-'

Choice D is correct. When two cyclic monosaccharides form a disaccharide, a glycosidic linkage is formed. Linkage formation occurs when a hydroxyl on one sugar attacks the anomeric carboi bn the other stigar. A water molecule is released as the leaving group, as the hemiacetal with the reactive anomeric carbon gels converted into an acetal' This makes choices A, B, and C all true. The anomeric carbon starts and finishes *ith t*o bonds to oxygen, one bond to carbon, and one bond to hydrogen. This means it is neither oxidized or reduced, so choice D is not observed, making it the best answer.

Ho+c-q
H\

HOFLC

OH+

HO

HO

HO HO

o
HO

Hemilcetal because
anomeric C is bonded to: OR, OH, C, and H anomeric C is bonded to: OR, OR', C, and H

Choice A is correct. When copper goes from an oxidation state of +2 to +L, it has been reduced. The color of the solution changes when copper changes oxidation state, so a color change corresponds to an oxidation-reduction reaction involving copPer ion and the sugar. Because copper is reduced, the sugar must be oxidized. If an aldose forms a carboxylate (deprotonated form of the carboxylic acid), it has been Jxidized. This makes choice A a correct answer. If a sugar forms a compound with only hydroxyl functional groups and no carbonyl group, it has been reduced. This eliminates choice B. If an aldose becomes an acetal, the rurmber of bonds to"o*yg"n and hydrogen has not changed, so it has neither been oxidized or reduced. This eliminates choice C. Ketonei do not oxidize into carboxylic acids, so choice D is eliminated.

,:..ight

by The Berkeley Review@

155

CARBOHYDRATES EXPLANATIONS!

99. Choice

molecule is

eliminated. Choice B represents the sum of the formulae of all three monosaccharides, but with the loss of two water molecules, the formula is C17H3go15, making choice A the best answer.

A is correct. A trisaccharide has two glycosidic linkages. For every glycosidic linkage, one water lost. An aldohexose has a formula of C5H12O6 while a aldopentor6 irut a formula"C5H1gO5. A trisaccharide from two aldohexoses and one aldopentose has seventeen carbons, so choices C-and D are

100. Choice

B is correct. An aldopentose has one n-bond and no rings. It can form a cyclic structure, but that has n bonds and one ring. Either way, the compound has only one unit of unsaturation. The best answer is choice B.

Copyright @ by The Berkeley Review@

156

CARBOHYDRATES EXPLANATI

Section Vtr
Nitrogen Compounds
tty Todd Bennett

Nitrogen Compounds (Non-Biological) al Amlnes


Basicity and Acidity Nuelcophilicity iii. FormaLion of Amines iv. Reactions of Amines v. Hofmann E,limination b) Imine Chemistry i. Formation of Imines ii. Imine-Enamine lsomerization c) Amides Formation of Amides iii. Reactions of Amides d) Amino Acid Synthesis i. Hell-Volhard-Zetinskii Synthesis ii. Strecker Synthesis iii. Reductive Amination of g-Keto Acids

i. ii.

i. ii.

Stru ct u re

ir,

+H."-+H+

CHr
I

T,;
pK a(R) =

?*n
o.u/
Hisiidine (Basic Amino Acid)

Nitrogen Compounds (Biologicat) a) Amino Acids


Strcutures and Classification Isoelectric points iii. trasy Calculation of pI b) Proteins i. Structure Features ii. Structural Levels c) Biological protein processes d) Biochemistry Lab Techniques i. Gel Electrophoresis ii. Affinity Chromatography iii. Sequencing (?rimary Structure) iv. Cutting/Fragmentation v. Edman's Reagent

^*

\:J

i. ii.

Nitro gen- Containing Compound s

Section Goals

oB aB
@3

Know the amino acids that affect the tertiary structure of proteins.
You must know the structures for cysteine and proline. You should know that cysteine forms disulfide bridges that result in cross-linking"within protbins. Proline, because of its cyclic structure, will cause structural abnormalities likes bends, kinks, and turns. You should have air idea of what structural features are most affected.

Be able to recognize and classify amino acids according to their side chain. Amino acids are classified in many ways, including hydrophobic, hydrophilic, acidic, basic, polar, and aromatic. Each of the classificatiohs gives vou iriforniation about tfie reactivitv of the amino s eives you information the reactivity acid side chain. With proteins, the side chain ii most important, because the amino and carhoxvl c6ain is imoortant. becarrse carboxyl terminals are involved'in the peptide tinkage. Be able to determine the isoelectric point for amino acids and proteins. The isoelectric pH is the pH at which the compound carries no net charge. For all amino acids except those with a basic side chain, it can be determined by averaging the values for pKrl and pKu2. Fbr proteins and the basic amino acids (histidine, lysine, and arginine), you must dverage thb two pK6 values that involve the zwitterion (neutral molecule). This is most easily found by first determining the charge of the protein (or amind acid) when it is iully protonated.

@?

Ilave an understanding of lab technigues such as gel electroDhoresis.


The basics of gel electrophoresis and isoelectric focusing involve placing the protein or ami.no acid between the charged plates of a capacitor, and allowing the compound to migiate through a viscous gel that offers reslstance. The natuie of the charge on the compound is deteriiined by the migration features.

"?

Know the structural definitions and structural features of proteins. It is important to know what is meant by primary, secondary, tertiary, and quaternary structure,
although some definitions overlap. You shbuld,recognize G-pleated sheeis and hbw they are arranged (parallel or anti-parallel). Have -a basic idea of s-turns and u-helices.

t?

Be able to determine the sequence of a peptide from chemical information. Sequencing a protein involves treating the protein with denaturing reagents, and then sequentially determining the component amino acids or peptides that fragment from fhe protein when it-is treated with a sequencing enzyme (digestive enzyme). You don't necessarily need to memorize the reagents, but you must be able to sequence a protein when provided with the reagent (or enzyme) and its function. You must know Edman's reagent.

lftt:

Know the basicity of amines and the effects of alkyl substituents.


Perhaps the most common weak base in organic chemistry is the amine. The pKu of a standard protonated alkyl amine_is in.the range of 9 to 11. This makes the amine a weak base, and the ammonium cation a weak acid.
,i,.1-li

:.t: li:
OTU

@?

Know that amine compounds can exist in either the free base or acid salt form.
There are several common amine compounds used for medicinal purposes as antibiotics, antipyretics, and analgesics. The MCAT likes to present organic molecules that 6ave a definite biological apflication.

t::

ilrr:
f,e{f

'v

Know how amines react to form amides. There are several instances in biochemistrv where an amine will react with a carbonvl comnound to form an amide bond. The most obvious'case is the formation of proteins from u.ni1'to acids. For
in aitro reactions, you must be aware of the solvent, because of the acid/base properties of the amines.

'ifilr,lr

nri

Iili:r1",:

ifinl.iLLm

Organic Chemistry

Nitrogen Compounds

Introduction

Compounds containing nitrogen make up a surprising large percentage of the known organic molecules. Many oxygen-containing such as alcohols "o*polrttai and carbonyls have equivalent compounds where the oxygen is replaced by a nitrogen. For instance, alcohols and primary amines are similar in that sense. Because nitrogen has two hydrogen atoms instead of one, Iike an alcohol, its reactivity is not identical to an alcohol, but it is nonetheless similar. Ketones and reactivity is close to one another. The significant difference in an imine is that its tautomer, an enamine, has different reactivity than an enol, because nitrogen and oxygen are different in electronegativity. we shall draw as many analogies to oxygen-containing compounds as we can.
We wiII start by considering amines of all types. Unlike the drastic difference in reactivity between an ether and an alcohol, amines do not change their reactivity significantly as they become more substituted. Steric hindrance and electron density on the nitrogen are impacted by substitution, but not to the degree that iertiary amines are significantly more or less reactive than primary amLes. of significance when considering amines is their basicity and their nucleophilicity. This makes up the bulk of reactions carried out by amines. Amine basicity and nucleophilicity is not as clear cut and easy to undeistand as one might hope. f.he :rnpact of steric hindrance, hydrogen bonding in solution, and eleitroniis is not as predictable as it is with most other compounds.

aldehydes share a strong structural similarity to imines, and their relative

-{fter considering amines and imines, we will address some of the less common rilrogen-containing compounds such as hydrazines, oximes, and azides. There ale not a great deal of examples of their reactivity, and it should be expected that :: they appear on the MCAT, then the test writers will provide a subsLntial pool :: information with them. we shall consider amides last, as they will serve as a :ansition from non-biological examples to biological examples. Amide bonds, :eferred to as peptide bonds when shared between two amino acids, make up the ::r'rndation of the primary structure of a protein. Amides are formed and :",-drolyzed readily under biological conditions, so they are an ideal building :,ock for much of biological chemistry. we will address in aitro synthesis o1 ;rino acids as our segue from non-biological examples into biologicil examples : i nritrogen-containing compounds.
"''e shall spend a significant amount of time covering biological examples of "-:rogen-containing compounds. The majority of the text will be devoted to the ;:rno acids and proteins, but we shall also address briefly other compounds ;' '.h as the common bases in DNA and RNA as well as some common clisses of :::epounds found in neurobiology. we will focus on the structure and function

:'::-'teins and amino acids, The MCAT has placed an emphasis on iaboratory rr,:eriments in their previous exams, so we will address the logic and theory

:r ;he compounds, leaving the biological aspects to the biology books. of -reatest interest will be isoelectric points and an easy way to determine them. '*<tlv, we will address biochemistry laboratory techniques that pertain to
l"-::nd electrophoresis, _affinity chromatography, and sequenciirg. Althougir trt:e are many other techniques in biochemistry, these three are chosen becau"se rr ::reir frequency on the MCAT and their applicability to proteins and amino u-:-. A11 other biochemistry laboratory technlques covered by o.tt course can be 'r,:rd in the biology books.
;c",right O by The Berkeley Review

159

Exclusive MCAT Preparation

Organic Chemistry

Nitrogen Compounds

Non-Biological

Non-Biological Nitrogen Compounds


Amines Amines are compounds with a central nitrogen that has three sigma bonds to either carbon or hydrogen. The degree of substitution of the amine is described as primary, secondary, or tertiary, where a nitrogen with al1 three sigma bonds to carbon is known as a tertiary amine. To complete the octet about the nitrogen, there is also a lone pair of electrons on the nitrogen. The lone pair is said to be in arrsp3-hybrid orbital. If the nitrogen has a double bond (both a pi and a sigma bond) to carbon, the compound is known as an imine. If a hydrogen on the amine nitrogen is replaced by a hydroxyl group, the compound is known as a hydroxyamine. Nitrogen when bonded to nitrogen by a sigma bond is ahydrazine. The structures of some nitrogen-containing compounds are shown in Figure 7-1.

r-

*{"
H

"

o-*v"" H
1"

-*v" " R'


Amine

o-*$"o" R'
3'Amine

Ammonia

Amine :'-

2"

,r-ilv"H o"
Hydroxylamine

N;'r xH,

"''

-N\cn ,
Imine

Ho/*oao.
Oxime

Hydrazine Figure
7-1

Example 7.1 \A4rich of the following compounds is an amine?

A.

H"C "\

/
L-1\

'QC
NHz

D.

/
H:C

N
I

O
H

NHz

A"

Solution Compound A has a double bond between carbon and nitrogen making it an imine. Imines can be reduced to form amines, but they are not amines themselves. Choice A is thus eliminated. Choice D has the nitrogen conjugated to a carbonyl group. This functionality is known as an amide. Choice D is thu-. eliminated. Choice C is a hydrazine, because of the nitrogen-nitrogen single bond. Choice C is thus eliminated. The only choice left is choice B. The compound is a primary amine, because only one carbon is bonded to nitrogen.
Amines are named in a similar fashion as ethers. The alkyl groups attached to nitrogen are named first, followed by the word amine. They can also be named as amino alkanes. Because of the muitiple name option, the likelihood of amine nomenclature questions is not too high. We'lI do one example just to cover all
bases.

im im

,[@

rfu

i@

ft
ffi
r@

tm

Copyright O by The Berkeley Review

r60

The Berkeley Revier-

Organic Chemistry

Nitrogen Compounds

Non-Biological

Example 7.2 \Alhat is the ever-so-wonderful IUPAC name for the following compound?

N
CHs

A. N-methylphenylamine B. N-benzylmethanamine C. Methyl aniline D. Methyl amino benzene


Solution

ft".?Tp:yl9

h?r

thus named N-methylphenylamine accordlng to the wild and adventurous IUPAC naming gang. The best answer is choice A.

an spJ-hybridized nitrogen, making the compound an amine. The ctmpound is

benzene ring (phenyt group) and a methyl group bonded to

-{mines are both weak bases and good nucleophiles. They are found in many neurobiology compounds, because many amines have stimulating effects on thl brain. some of the more biologically famous amines are shown in Figure z-2. HO

o-R,
(-)-Morphine Nicotine

'',\F"6"
Cocaine

ocH3

H"N

G.':l-'"'
Amphetamine

HO

t*FigureT-2

at,

-."Qr"-"'
ocH3
Mescaline

Epinephrine (adrenaline)

.ome of which you may recognize. Because of their medicinal and industrial ;ses, synthetic pathways to each are of some interest. Natural product synthesis '' a significant part of organic chemistry. what is meant by -tne term "natural :roduct" is a compound that is synthesized in a plant or animal. These -ompounds are targets of a great deal of synthetic laboi. In some cases however, -t is far cheaper to extract the amines from plants (i.e., nicotine) using both acid:ase extraction techniques, as well as organic solvent extraction. Foiinstance, it *' through extraction that caffeine is removed from the coffee bean. Extraction ::n be far easier than synthesis. For this reason, it is more useful to consider the :atural synthetic pathway rather than an in uitro synthesis. -opyright O by The Berkeley Review

The above compounds are found in various medical and household products,

l6l

Exclusive MCAT Preparation

Organic Chemistry

Nitrogen Compounds

Non-Biological

The lone pair of electrons can be donated to a proton making an amine compound a Brsnsted-Lowry base. The basicity of amines is well-documented. The protonated form of the amine is often isolated from aqueous solutions as the ammonium salt. For instance, ethyl amine (CH3CH2NH2) can also be found as the ammonium chloride salt (CH3CH2NH3+C1-) after the free base is treated with hydrochloric acid. The free-base form is defined as the deprotonated form of the amine. It is with the basicity of amines that we shall start their study.
Amine Basicity A common use for amines is as weak bases. Amines have pK6 values in the 4 to 5 range, which means that the K6 associated with the protonation of an amine is between 10-4 and 105. The varying basicity of amines ian be attributed to factors such as the inductive effect, steric hindrance, hydrogen bonding, and solvation. The seemingly chaotic order of relative amine basicity demonstrates the presence of both the inductive effect and steric factors. Steric factors would favor a smaller base, because of its ability to bind a proton with little to no hindrance. The inductive effect predicts that the more substituted amine is the most basic. The relative basicity of the methyl amines is: (H3C)2NH (pK, = J.27) > H3CNH2 (pK6 = 3.35) > (H3C)3N (pKu = 4.27) > NH3 (pK6 = 4.74).

Because the nitrogen of the amine has a non-bonding pair of electrons that it is free to donate, nitrogen atoms can serve as a nucleophilic center for the molecule.

ts

A lower pK6 value corresponds to a stronger base, so according to the pK6 data, a secondary amine is the most basic of all amines in water (with the same alkyl group on all of the amines). In organic chemistry, pK data is applied to determine which way an acid-base equilibrium will lie (as we considered in section I, pages 37-40 of Organic Chemistry book I). Reaction 7.1 shows a favorable proton transfer reaction involving amines of different substitution.
H3CNH2

5r

:t

:
-J -a'

+ NHn*

H3CNH3+

+ NH"
:"

Reaction 7.1
Because the methyl group is electron donating, methyl amine is more basic than ammonia. This means that the ammonium cation is more acidic than the methvi ammonium cation. The stronger base and the stronger acid lie on the reactant side, therefore the product side of the reaction is more stable than the reactant side of the reaction. This means that the reaction will proceed in the forward direction as written when starting from standard conditions. At equilibrium, the products are more abundant than the reactants, so the equilibrium constant (Kgq is greater than 1. The equilibrium constant for a proton-transfer reaction can be determined using EquationT.T (listed also as Equation 1.13 in section I).

:
E"r,

(q

Keq = lgPKa(product acid) - PKa(reactant

acid)

0.1,1 H.

If you don't recall the common acid-base equations, you should consult page 4C of Organic Chemistry book I. In organic chemistry, we are concerned with the predominant species in solution, so it is a good idea that you fully understani the Henderson-Hasselbalch equation for buffers, shown in a modified form as
Equation
7.2.

D.
5,r

lt

,--_

pH

= pKu (protonated species)

species] 1o, [Pgprotonated .-

i -*

(7.2t

[Protonated species]

Acid-base chemistry has been thoroughly covered in section chemistry, so we shall assume you have reviewed it thoroughly.

I and in genera-

:*'

Copyright

by The Berkeley Review

16,2

The Berkeley Reviex

Organic Chemistry

Nitrogen Compounds

Non-Biological

Example 7.3 Civen that the pK6 of NH3 is 4.74, what is the approximate pK6 of Et3N?

A. -0.32 B. 3.25 c. 5.31


D.
9.26

Solution

Alkyl groups are electron donating, so triethyl amine (Et3N) is more basic than ammonia. As a consequence, the pK6 for triethyl amine must be less than 4.24. This eliminates choices C and D. A negative pK6 corresponds to a strong base
and amines are weak bases, so choice A is too

low.

Choice B is the best answer.

Example 7.4

Diethyl amine is more basic than ethyl amine in an aqueous solution. If the pK6 for ethyl amine is 3.29, then the pKu for diethyl u**o.irrrn chloride is:

A. B. D.

c.

3.05. 3.89. 10.11. 10.95.

Solution
The question is asking for the pKu of a protonated amine, which lies between 9 and 11. This eliminates choices A and B. Because diethyl amine is more basic than ethyl amine, the pK6 for diethyl amine is lower than ihe pK6 for ethyl amine

(which is 3.29). The sum of pKu for diethyr ammonium catLn and pK6 for diethyl amine is 14. For the math to work out, the pKu for diethyl u*.norri.r* cation must be greater than 10.71. This makes choice D the best of a1l possible answers in this the best of all possible acid-base questions involving amines.
Example 7.5 \Alhat is the relative base strength of the following three compounds?

A. Compound II > Compound I > Compound IiI B. Compound I > Compound II > Compound III C. Compound III > Compound I > Compound II D. Compound III > Compound II > Compound I
Solution

G*"' ""iG*"' ','o$*',


Compound

"

Compound

II

Compound

III

are amines. The question comes down to the electron donating and withdrawing nature of the other functional group. Because carbonyls are withdrawing by resonance, Compound II is less basic than compound I. Because alkoxy groups are donating-by resonance, Compound III is more basic than Compound I. The relative order for basicity is thus Compound III > Compound I > Compound II, making choice C correct.

All three compounds

Copyright

by The Berkeley Review

165

Exclusive MCAT Preparation

Organic Chemistry
Example 7.5
\A/hat is the

Nitrogen Compounds

Non-Biological

pH of 0.1 M aniline, which has a pK6 of 4.2?

A. 2.6
8.8 77.4 D. 13.0 B.

c.

Solution Aniline is a base, so the pH is greater than 7.0. This eliminates choice A. Aniline is not a strong base, so a 0.10 M solution must have a pH less than 13. This eliminates choice D. For a pure weak acid or pure weak base, it is best to use the simplified equation learned in the general chemistry book. For a weak base, lt'e use: pOH =1/zpKA-r/21c9 [Base]. Plugging the values into the equation r,r'e get: pOH =1/2(4.2) -1/z(1) =2.L +0.5 =2.6. If the pOH is 2.6, then the pH is 11.4, making choice C the best answer.

Nucleophilicity
Weak bases, such as amines, make good nucleophiles and the nucleophilicity ot amines parallels their basicity for the most part. Deviations from this trend are due to steric hindrance. The majority of amine reactions in organic chemistninvolve the amine behaving as a nucleophile by attacking an electrophile ani either replacing a leaving group or breaking a n-bond. Amines do nucleophiJrc

substitution reactions without competition from elimination reactions. One problem with amines is that they are increasingly nucleophilic as more alkvgroups are added. This makes the synthesis of primary amines rather difficult because once formed, they are more reactive than ammonia. Typical amine

reactivity questions involve comparing the relative basicities

and

nucleophilicities of a series of amines, looking at the effects of substituents both through resonance and the inductive effect on reactivity, and amine structure
questions.

ExampleT.T What is the major organic product when ammonia, NH3, is treated excess eth';iodide, H3CCH2I, following workup?

A. B. C. D.

H2C=CH2

H3CCH2NH2 (H3CCH2)2NH (H3CCH2)3N

Solution
Because secondary amines are beiter nucleophiles than primary amines which i; turn are better nucleophiles than ammonia, the reaction proceeds until the alloi halide is exhausted. With excess alkyl iodide, the reaction is free to add as mantr ethyl groups to the amine as possible. The best choice shows three alkyl groups

would not form without a strong base, so it should have been eliminate,J

added to the amine. This makes the correct answer choice answer

D. Choice,\

immediately. It is possible that (H3CCH2)aN+, a quaternary amine, would forrr* but it is not listed as an ans\{'er choice.

Copyright O by The Berkeley Review

t64

The Berkeley Reviet

Organic Chemistry

Nitrogen Compounds

Non-Biological

B. benzoic acid. C. an alkyl halide. D. an ether.

Example 7.8 An amine can react with all of the following EXCEPT: A. an anhydride.

Solution An amine can react as a nucleophile with a highly reactive electrophile such as an anhydride. This eliminates choice A. An amine can react as a base, so when added to a carboxylic acid such as benzoic acid, it carries out a proton transfer reaction. This eliminates choice B. An amine can react as a nucleophile with an alkyl halide in a nucleophilic substitution reaction. This eliminates choice C. Ether is usually a solvent for many of these reactions, because an amine does not react with an ether. The best answer is choice D. The three reactions for choices A, B, and C are shown below:

-,,nt."jtto
o
HsCa

*t, -----+
*

oio*, ,r*lo
+ NH4+

Ao"
+

* NHs

*ru.r.Jl-..o

A RX
o

NH,

---->

Ri

+
NHz

HX

Example 7.9 lVhat is the major organic product for the following reaction?

+ NH3 {" NH
B.

--->
o
C.

D.

1}"
Solution

H,NUoH H,NUN',

+,"

-{mmonia is a good nucleophile that can break open the strained four-membered :it g by attacking the carbonyl carbon. The final product is not going to have a :our-membered ring, so choices A and D are eliminated. Amines react with esters to yield amides, so choice C is eliminated and the best answer is choice B.

Copyright Oby The Berkeley Review

165

Exclusive MCAT Preparation

Organic Chemistry
Example 7.10

Nitrogen Compounds

Non-Biological

\Alhat is the hybridization of nitrogen in a tertiary amine?

c. spr
D.

A. B.

so
sp'^

d2sP3

choice C the best answer. Example 7.11

Solution In a tertiary amine, nitrogen makes three bonds and has a lone pair of electrons. This means that there are four electron pairs surrounding ttitrog"r,, so the hybridization must involve four atomic orbitals. The hybridi"itionof-nitrogen in any amine, whether it is primary, secondary, or tertiary, is sp3. This makes

How can

it

Compound KMF8889?

be explained that Compound AWLg3g6 is more basic than


CH.
CH"

Compound AWL8386

o
I

Compound KMF8889

t'

A. Aromaticity
B. C. D.

reduces the availability of the electron pair Compound AWL8386. The inductive effect is greater in Compound KMFgggg. Steric hindrance is greater in Compound AWLg3g6. The nitrogen in Compound KMF8889 lacks hybridization.

for donation in

Solution
The lone pair on nitrogen in compound KMF8889 is tied up in the aromatic ring of the pyrrole. This makes it less available for sharing wlth a protic hydrogeri and thus makes the compound less basic. This makes choice A tire best ur,r-ur.

Copyright O by The Berkeley Review

r66

The Berkeley Review

Organic Chemistry

Nitrogen Compounds

Non-Biological

Formation of Amines Primary amines may be synthesized in a variety of ways. The synthesis of primary amines is not as simple as it may initially appear. You cannot just add ammonia to an alkyl halide, because the product (an alkyl amine) is a better nucleophile than ammonia and thus it can react a second and third time with another alkyl halide. This results in the formation of multi-substituted amine products (2' and 3' amines) with random distribution in the product mixture. Figure 7-3 shows six synthetic methods for forming primary amines. Some of the reactions can also be used to form secondary and tertiary amines.

Gabriel phthalimide synthesis (for 1' amines only)

oo
o

o
KOH -------->

e(*H

Hzo

-.+ l(--->

RX

c9
OH

o(".
o

+ HrN*R

OH

Azide reduction (for 1" amines only)

NaN" R-X ----l->

-N-

+ H"/Metal \\==N '' >


Hz

R- NH2

N2(s)

Nitrile reduction (for 1'amines only)

R-X NaC=N- R- C=N o

/ Metal-

R- CH2- NH2

Hofmann rearrangement (for 1" amines only)


il
X2

.z C:.

KOH(aq)

RNH2+CO2+KX+HX

NHz

Imine reduction (for 1" or 2' amines)

t-*-t
+
1. NaBHr(thO
I

o
il

N
NH" +
A

-/H
2. NHaCI(aq)

il

RR'

,zc:.
o
il

RR'

.zC:,
HH

.z 9:. RIR'
H

Amide reduction (for 1', 2', or 3' amines)


1.

,zC:. ./H RN
I

,zC:. ./H 2. NHaCI(aq) n N


--j+Figure 7-3
I

LiAlHa

\r

Copyright

by The Berkeley Review

167

Exclusive MCAT Preparation

Organic Chemistry

Nitrogen Compounds

Non-Biological

Imine reduction can form either a primary or secondaryu*ii.", depending on the imine, and amide reduction can form either a primary, secorrdiry, oriertiary amine. Figure 7-4 shows the formation of secondaty and tertiary amines.

Reduction of an azide or nitrile, the Gabriel phthalimide synthesis, and Hofmann reaarangement all produce a primary amine as their majoi product after workup.

Imine reduction

t-*-o
I

o
ll

RNH2*
R''

R'

.',

C:.

^*
o
il
R

_,,,t_-,, R' R" tNtrffi o,-[-o


z""Ti""
Amide reduction

N/R ll

r NaBHd(rho

R/c\N/
I

HH \ t" i. LiAtH. -/_\./_. 2. NHaCI(aq) R' N


I

H
2" amine

o
il

i]
R
I

H
-/"

1. 2.

LiAlH,
NHaCI(aq) R'

\ I
z-\

R/t-N/
R'

N
I

R'

3" amine -

\zvhich of the following compounds

CANNoT be reduced to form an amine?

A. B. C. D.

An amide An imine A nitro compound An alkyl hydrazine

Solution According to Figure 7-3, when an amide is treated with lithium aluminum hydride, it is reduced into an amine. when an imine (R2G=NH) is treated with sodium borohydride, it is reduced into an amine. This eliminates choices A and B immediately. Figure 7-3 shows neither a nitro group nor hydrazine being reduced into an amine, so memorization won't finish lhis question. A nitro compound contains one nitrogen, while hydrazine contains two nitrogen atorrr<Considering an amine only contains one nitrogen, it is more likely tf,at a nitro group is reduced into an amine (primary amine to be specific) than irydrazine. A nitro group can be reduced to form an amine by adding Clemmensen reagents (HCl(aq) and a metal catalyst), which eliminates choice C. An alkyl hydrizne (RHN-NH2) cannot be reduced to form an amine, because reduciion-will not cleave the nitrogen-nitrogen bond. This makes choice D the best answer. Thk also emphasizes a point about your review. Despite our best efforts to track the Copyright
@

tu
M
MM

Wh

nd
ffim

tu 0m tu
@

dh

ffia

tu

ffiP

by The Berkeley Review

r6a

The Berkeley Review

{rq

Organic Chemistry

Nitrogen Compounds

Non-Biological

MCAT and incorporate information they release about the exam, there will be some topics we miss. If you plan on memorizing all the information in our materials, you will be fine for the most part. But there will some subjects that they introduce that may surprise you. To prepare for this exam, you must emphasize general concepts and logic more than memorization.
Reactions of Amines The lone pair of electrons on nitrogen is readily shared, making an amine a good

nucleophile. The crux of amine reactivity involves the lone pair on nitrogen attacking an electrophilic carbon. As a general rule, either a ieaving gro.rp i, displaced or water is formed as a side product. This should help to predicfthe products of amine reactions. We will consider the addition of amines to alkyl halides, diazonium formation and the Hofmann elimination first and imine formation and hydrazine reactions later. Figure 7-5 shows a few amine reactions. Addition of an amine to an alkyl halide H3N: + R-X H3N+RX-, H2N+R2X-, HN+R3X-, --+ This reaction results in multiple additions, because the more
substituted an amine, the better it is as a nucleophile.
Reactions of diazonium salts

R-NH2 ff

NaNO"

R-N*=N: Cl- + NaCl + 2H2o

N2 is about as good a leaving group as there is, so diazonium salts are excellent electrophiles.

.\r-NH2 HC-

NaNOr

Ar-N+f

N: Cl-

CuX
(X = Cl, Br, C =N:)

Ar-X + N2(S)

Hofmann Elimination

*-tHr - ar{

NH2 3 cH3l-

o-atr-ari.*,cll3)r A '
\Ag2o /

\ p11g: CH2 + (HrC)rN + AgI


Least substituted alkene

Figure 7-5

in the first reaction, addition to an alkyl halide, there is a leaving group. F{ence, Jre nitrogen displaces the leaving group by way of a nucleophilic substitution :eaction to form an amine. As noted in Figure 7-5, there is the good chance that nultiple alkyl groups will add to the amine. In the second reaction, formation of : diazonium salt, there is no leaving group. Hence, we need to look for the water 5at forms. one nitrogen has the hydrogen atoms and the other has the oxygen :toms, so upon losing the hydrogens and oxygens, nitrogen atoms in need of :onds remain, so this reaction must form nitrogen-nitrogen bonds. In the last :eaction, Hofmann elimination, there is a leaving group, so the nitrogen displaces :1"1e leaving group to form an amine. With excess methyl iodide, it exhaustively rethylates to form a quaternary amine. The quaternary amine carries a positive ::1arge, making it an excellent leaving group. The long alkyl chain attached to -ie nitrogen can undergo E2 elimination, kicking out the leaving group and :rrming a least substituted alkene. We shall address the Hofmann elimination in rore detail, as it has appeared on previous versions of the MCAT. -opyright O by The Berkeley Review

169

Exclusive MCAT Preparation

Organic Chemistry

Nitrogen Compounds

Non-Biological

Hofmann Elimination

B-hydrogen resulting in elimination. The E2 reaction does not have rearrangement associated with it, therefore the product is the least substituted alkene. This is one of-few ways to synthesize a terminal alkene. Figure 74 shows the orientation of a generic molecule in a Hofmann eiimination reiction.

Ag2o, when hydrated by water, yields two hydroxide anions (oH-) urid t-o silver cations (Ag+), so it is a strong base. The silver cation can bind the iodide anjon and precipitate out of solution while the hydroxide anion deprotonates the

quatemary amine is anti to a B-hydrogen (hydrogen on the carbon adjacent to the carbon bonded to the quaternary amine), it canbe etminated with strong base.

mechanism. The Hofmann elimination reaction involves forming a cationii quaternary amine (nitrogen with four alkyl groups attached). when the

The mechanism for the Hofmann elimination reaction is a straightforward

E2

(TlcH,), r
The beta hydrogen and the quatemary amine are anti to one another for E2reaction.

N(crus
H lrr..

o{--4;" -+
H \-/
-OH Figure 7-5

prCpO, H-O\

/t

a...rrlH

It is important that the compound have B-hydrogens relative to the quaternary amine, otherwise there can be no elimination product. Be particularl! u*ur" of this when dealing with cyclic amines. Figure 7-7 shows Ho?mann elimination of
a cyclic amine.

*trt,

f*) \-,J

2cH3I

\-J
(H3C)3N

f*) ++ r,,.'*) tt'L ,r..-*\ a ?-) z-)


+ AgI +
Figure 7-7

/.tt,

ftu

tr.\

/.",

4..'\

Copyright

by The Berkeley Review

170

The Berkeley Revier

Organic Chemistry
'"-hen

Nitrogen Compounds

Non-Biological

I:rines

amines are added to a carbonyl, a nucleophilic attack at the carbonyl ,::bon takes place. If there is no leaving group on the carbonyl, then the reaction - irns an imine and water and is driven by its equilibrium. If water or imine is :=:noved (usually by distilling away the product with the lower boiling point), -:-tn the reaction is pushed forward. This is one of several reactions that can be ---:ognized by the water side product more easily than the organic product. The : - rnation of an imine, oxime, and hydrazone are shown in Figure 7-8, to present *--t ease of predicting the product after identifying the water that is formed.

1'Amine
R'
I

1"

Amine
R'
I

-NHH o

H Aldehyde

A
OH
I

--r A
RH
Imine

N,R

.z N:. HH o

A
OH
I

-+ HCI cat.
R

A
Imine

N.R

Ketone

:l'droxyamine
OH

Hydroxyamine

-N-. HH o

H Aldehyde
l

A
"l

--------->
A

A
Oxime

N/

HH
H

N:. ,u o
________->

A
Ketone

cat. HCI

RR

A
Oxime

N -oH

Hydrazine

Hydrazine

H-N
HzN-..

H,N -l N

-NHH o

RH

^*A A
RH

HH

N:. ,u
o

H2N\

Hydrazone

A
Ketone

ffioA'.
Hydrazone
R

Aldehyde
Figure 7-8

These reactions proceed considerably better with an aldehyde than a ketone. This is attributed to the greater electrophilicity of an aldehyde than a ketone. when a primary amine is used, the imine that is formed is referred to as a shiff bsse. Hydrazones are used in the identification of carbonyl compounds, particularly ketones and aldehydes. Other carbonyl compounds have derivatives as well, but the majority of derivatives are formed by substitution reactions with nitrogen compounds. The basic idea here is that by saturating an aldehyde or ketone with a compound that contains at least two hydrogen atoms on the nitrogen (ammonia, primary amines, hydroxyamine, or hydrazine), a C=O is converted into C=N. With excess water, the reverse reaction is observed.

Copyright @by The Berkeley Review

t7t

Exclusive MCAT Preparation

Organic Chemistry

Nitrogen Compounds

Non-Itiological

Example 7.L3 \zVhat is the major organic product of the reaction shown below?

Ph

oAo'.
A.o
-Jr.RNH
I

,N-N.

A*

B'

NH

c'
R

*Ao
Ph

NPh

D.

R'

N/

NHPh

R'

Solution Find the water and all will be fine. The oxygen comes from the ketone, so the two hydrogens must come from the nitrogen-containing species. Only one of the two nitrogen atoms has two hydrogen atoms, so that is the nitrogen that replaces the oxygen in the ketone. The result after removing the water and connecting the atoms is choice D. Connect-the-dot skills you developed in kindergarten may prove more useful on the MCAT than the volume of information you memorized during college. Probably not, but it felt good to say. Imine'Enamine Tautomerization ]ust as ketones and aldehydes undergo tautomerization in the presence of mild acid or mild base to form an enol, imines also undergo tautomerization to form an enamine. Figure 7-9 shows the equilibrium between an imine and an
enamine.

..N/, R"

o4ro:o /\
HH
Imine

ll

.. .,R" Nr

:111n"
T)r-

H\
+H'

., R"

&R<+ -\
H
Figure 7-9

H
Enamine

An imine can form only if the nitrogen compound has two hydrogen atoms on the nitrogen. When a secondary amine reacts with an aldehyde or ketone, the product L ut uttu-ine, because an imine cannot form. The formation of an
enamine from a secondary amine and ketone is shown in Figure 7-10'

R-n
H
I
I

-l-r'o
IIN+ HH
Aldehyde
Copyright
by The Berkeley Review

tl

-R"
Eliminationr

R-iu'R"

R'

NT_-> /tt\

R''

2" Amine

"'txo Ho/\

HHH Carbinolamine
Figure 7-10

"\*
Enamine

172

The Berkeley Keview

Organic Chemistry

Nitrogen Compounds

Non-Biological

Amides Amides are carbonyl compounds with a nitrogen bonded to the carbonyl carbon. They are formed when an amine or ammonia is added to a carbonyl compound with a leaving group. Figure 7-11 shows the three types of amides.

o
il

o
il

o
il R

R-c- *.t
I

R-c- NI

R,/c- N,/
I

H 1" Amide

R"
3" Amide

2'Amide
Figure 7-11

ruPAC nomenclature rules call for naming the alkyl groups on the nitrogen with a prefix of "N" first, followed by the carbonyl chain. The degree of substitution impacts the physical properties. Acetamide, H3CCONH2,has a melting point of 82"C and a boiling point of 227'C. N-Methylacetamide, H3CCONHCH3, has a melting point of 28"C and a boiling point of 204'C. N,N-Dimethylacetamide, H3CCON(CH3)2, has a melting point of -20"C and a boiling point of 165'C. Amides form H-bonds between oxygen's lone pair and the hydrogen on nitrogen, which explains the drop in boiling and melting points as substitution increases.
Formation of Amides Amides are typically formed from the addition of an amine to an acid anhydride, acid halide, or an ester, as covered in section V of Organic Chemistry book IL A primary amide can be formed by partial hydrolysis of a nitrile (R-C=N) and can also be formed from an oxime by way of the Beckmann rearrangement. The Beckmann reamangement results in the formation of an amide from an oxime through the addition of either a strong acid or PCI3O/PCI5. An oxime is formed by the addition of hydroxyamine to either an aldehyde or a ketone, as shown in Figure 7-8. Beckmann rearrangement is shown tnFigureT-12.

;ph,o
,(H --1l,

^:frJ'-I' /t oAo

.'l:,

rr

t?-*'

Beckmann Rearrangement

:o:

R/ oA*.o
-l

il

atii n\ ->J-, R Copyright

H !r;o'. R/ .-(.0*.T-- ::)':*'o/


n_.g.'.n
FigureT-12

"-'dYi rr

..d.. ttt/gX H
'o,

by The Berkeley Review

173

Exclusive MCAT Preparation

Organic Chemistry

Nitrogen Compounds

Non-Biological

Reactions of Amides we have seen that amides can be reduced into amines using lithium aluminum hydride. In addition to reduction, an amide can be hydrotyied into a carboxylic acid and amine. Primary amides can undergo Hofmann r"urrur,g"-ent to form a primary amine of one less carbon and carton dioxide. The rlaction does not work with substituted amides; it works only with primary amides. Figure z-13 shows the step-by-step mechanism for Hofmann rearrangement.

Hofmann Rearrangement

o-c:o
aa

/ + R-N: \

N
I

Co:

.>

il .i, \t
. lr>H

H-61-*n$;
iol

. C_N:

/H

to

.\-

{t
ll
-

t-''i-"
\

[i^Isocyanate '

piG: Yc:N.
o/J

H-Oi/
Figure 7-13

O q:*;/ "S-" ':.- |


R

H-9: I

'. ztT ^r
R

In the Curtius Rearrangement an acid chloride is treated with sodium azide NaN3 to form an isocyanate. The conversion goes through an acyl azide that when heated decomposes to_yield an isocyanate ana nitrogeln gas (a great leaving group). Just as seen in the Hofmann rearrangement, an iiocyanate ian ,rndergJ decarboxylation in basic conditions to geneiate a primary imine. Figure 7-14 shows the key intermediates of the Curtius rearrangement.

Copyright O by The Berkeley Review

174

The Berkeley Revier

Organic Chemistry
o

Nitrogen Compounds

Non-Biological

Curtius Rearrangement

oA., lt
llN"J

o:c:o

+ o-*,
uo'll tl
O:C:N\
h

-o x:it:N

li

oX.' \\

oA*:it-N

ll

--------

oA*-ir:N

?/

FigureT-'1,4

Amino Acid Synthesis An amino acid, as the name implies, is a molecule with an amine group and a carboxylic acid group. In an aqueous environment, the carboxyl terminal is deprotonated and the amino terminal is protonated, so it has at least two charged sites despite having no net charge. However, in organic solvents they exist as an uncharged molecule. There are three synthetic pathways for making amino acids: Strecker synthesis, Hell-Volhard-Zelinskii synthesis, and reductive amination of cr-keto acids. The three syntheses are shown in Figure 7-15.

Hell-Volhard-Zelinskii synthesis

OJON'
Strecker synthesis

W
N.H

----i>

NH"

"t'o,-,
NHz

il

A -oL-4,.+
HRH
Reductive amination of g-keto acids

H"N

.XL
H

CO"H

RH

*t'CO2H

N,H
CO2H

H"N
NaBH, NH,CI .* "-. Et2o Hzo

.X

CO2H

Figure 7-15

Copyright

by The Berkeley Review

Exclusive MCAT Preparation

Organic Chemistry

Nitrogen Compounds

Biological

ffifinu$itHl'

gen,,Comp6#Hft

Amino Acids Natural amino acids (s-2-amino carboxylic acids) are a critical part of biology as the building blocks of proteins. There are a total of twenty standard amino icids that humans code for. Although there are over 500 amino acids in nature, humans code for just twenty. In mRNA, amino acids are coded fotby a sequence of 3 bases (codons) using adenine (A), cytosine (C), guanine (G), and u.acil 1u;. For instance, histidine is coded as cAC or CAU. with 64 (43) combinations of codon triplets, there are multiple codes for some amino acids. Figure 7-16 shows the generic structure of an S-amino acids in both aqueous environment and lipid environment.

H.N+-

' \c'l.-\o /2,


HR
Figure

o il
c

t'*-.-t-on

lt

I ,,, HR
Uncharged form of amino acid

Zwitterion form of amino acid


(aqueous environment)
7-"16

(lipid environment)

bonding, cysteine, and proline, protein sequencing, and some enzymatic chemistry. We shall start with terminology and classification. Two terms that will be used repeatedly to describe the amino acids are hydrophilic and hydrophobic (based on their side chains). within the hydrophilic group, some
amino acids are acidic, some are basic, and some the others are polar. Structures and Classification There is a chiral center at the cr-carbon in all amino acids except glycine. Naturally occurring amino acids have s-stereochemistry at carbon 2, although in biochemistry we refer to that as an L-amino acid. The exception to the L corresponding to S rule is cysteine, where L-cysteine is the natural form, but it has R-stereochemistry at the o,-carbon. For the MCAT, you should recognize the twenty amino acids for which we code, along with general pKu data for the amino terminal, the carboxyl terminal, and the side chain of protic amino acids. Amino acids are categorized according to behavior, and although it may vary from textbook to textbook, it is generally based on water solubility and degree of protic activity. we shall classify amino acids into one of five categories: hydrophobic, semi-hydrophobic, hydrophilic, acidic, and basic. There are amino acids that seemingly should fit into muitiple categories, like aspartic acid, which is hydrophilic and acidic, but we sha1l have each amino acid appear only once.

Because amino acids are compounds which contain an amino group on the second carbon and a carboxylic acid, they have acid-base properties that you are expected to know. Amino acids are essentially just polyprotic acids where the all of the protons are weakly acidic. The amino terminal when protonated forms a cation with a pKu of roughly 9.7 + 9.9. The carboxylic acid terminal has a pKu of roughly 2.2 + 0.4. In long chains, they form proteins, which you are also expected to know. This section covers functional features of the amino acids, the acid-base properties of the amino acids, protein structure, the structural effects of hydrogen

Copyright

by The Berkeley Review

t76

The Berkeley Review

Organic Chemistry

Nitrogen Compounds

Biological

Hydrophobic amino acids Hydrophobic amino acids are "water-fearing", which means that they are not water soluble and they prefer to be situated in an organic (lipid) environment rather than an aqueous environment. In aqueous environments, hydrophobic amino acids are found in the core of a folded protein. Figure 7-17 shows the hydrophobic amino acids glycine, alanine, valine, leucine, and isoleucine as they exist in an aqueous environment at pH = 7.

Hydrophobic Amino Acids


pKaz

srN\

=el8
| '2. HH

o
ll
PKot = Z.JJ
PKaz =9'87

-zc\ copKaz =

HrN\

I H
o
PKor=2:t
CH2CH(CFI?)2

.zc\ oCHs
pKaz

il

PKaz =

9j2

o
lt

HsN\
H

''.- rKu -t?q

Glycine (Gly, G)

e.74 ll srN\ .zc\ coH

Alanine
(Ala, A)

=e.74 ll HrN\ .zc\ co-

cH(cFi")2 Valine (Val, V)

Ii

-zc\ coPKat =2'29

| |

I H

r2

pK,' =z.zz

CH(CFI3)CH2CH3

Leucine (Leu, L)

lsoleucine (Ile, I)

FigureT-17
S emi-hy drophobic amin o acids These amino acids are typically considered to be hydrophobic in most text books, but we segregate them from other hydrophobic amino acids, because they don't

pack as well as the alkyl side chains. These include the aromatic side chains (except for tyrosine) and the amino acids that are more water soluble than expected for an aliphatic side chain. Figure 7-18 shows the semi-hydrophobic amino acids proline, phenylalanine, tryptophan, and methionine as they exist in an aqueous environment at pH = 7. Semi-Hydrophobic Amino Acids

o
PKaz

=9'44

PKaz = 9'24

H'N\./t\o
H

il

o
PKaz =10'64

II

pKaz

=e.27

o
ll
oPKat = 2.17

:'*\./t\o\ \,J i\ rr

HsN\./t\

rz. PK*=z'ss
CH^

PKar=1'e5

n Proline

(Pro, P)

H cH2cH2scH3 Methionine (Met, M)

I2

o
Phenylalanine
(Phe, F)

l'

Figure 7-18

Copyright

by The Berkeley Review

t77

Exclusive MCAT Preparation

Organic Chemistry

Nitrogen Compounds

Biologicat

Hy drophilic amino acids

environment. In water, hydrophilic amino acids are found on the exterior of a folded protein. These are sometimes referred to as protic side chains. Figure 7-19 shows the hydrophilic amino acids serine, threonine, cysteine, tyrosine, asparagine, and glutamine as they exist in an aqueous environment at pH = 7. Hydrophilic Amino Acids
PKaz = 8'80

to be situated in an aqueous environment rather than an organic

Hydrophilic amino acids are "water-1oving", so they are water soluble and prefer
11lpia}

nuN\

pKaz =

I H

-zc\ cotz9Hz
I

PK,- =z'oz

e.13 ll H:N'\ -zc:. cor'-- PK* =2'tz I


H
CH"

pKaz =

e.12 ll HrN\ -zc\ coH


CH.

I r2 PKot=z'zt

o4Nu,
Asparagine (Asn, N)

tI

9Hz

o4Nu,
Glutamine
(Gln, Q)

pKaz =

e.46 ll H'N\./t\oH

o
pKaz =

srN\

e.1o
H

o
ll
vK,r- =z'ov

PKas = 10

r2 PK,.=zlo
CH2OH

-zc\ c-o '7

33 ll HsN\ -zc\ cH

OH Tyrosine (Tyr, Y)

Q,

=1007

CH(CFr3)OH

r---

o-

PK*=taa

CH2SH
PKaz = 8'36

Serine (Ser, S)

Threonine (Thr, T)

Cysteine (Cys, C)

Figure 7-19 Example 7.14 Which of the following amino acids is LEAST hydrophilic?

A. Serine B. Tyrosine
C. D.
Cysteine Leucine

Solution

Hydrophilic is defined as "water-loving," which implies that the compound should be water-soluble. Serine has an alcohol side group, so it is water soluble due to the hydrogen bonding of the side chain. Tyrosine has an alcohol side group as well, so it too is water soluble due to the hydrogen bonding of the side chain. This eliminates both choices A and B. Cysteine has a thiol (sH) side group, so it is water soluble due to the polarity of its side chain. Leucine has an
alkyl side chain, so it is hydrophobic. The best answer is choice D.

Copyright

by The Berkeley Review

The Berkeley Review

Organic Chemistry
Example 7.15

Nitrogen Compounds

Biological

Which of the following amino acids contains A. Valine B. Tyrosine

a benzene ring?

C. Histidine D. Isoleucine

Solution Phenylalanine, tyrosine, and tryptophan are the only amino acids with side chains that contain a benzene ring. of those three amino acids, only tyrosine is offered as a choice, so choice B is the best answer. Histidine has an aromatic ring, but it is not a benzene ring.
Example 7.16 \\4'rich of the following amino acids is classified as hydrophilic?

A. Proline B. Threonine C. Valine D. Phenylalanine


Solution
Proline, valine, and phenylalanine are hydrophobic, because they contain alkyl side groups. This eliminates choices A, C, and D. The only choice reft is threonine, choice B, which has an alcohol group as at the end of its side chain. The hydroxyl group forms hydrogen bonds, making threonine hydrophilic.
Choice B is the best answer.

.lcidic amino acids {cidic amino acids have side chains that lose a proton from their neutral state. They have three acidic sites, the N-terminal, the C-terminal, and the side chain. ln the case of Glu and Asp, the side chain is a CooH. Cysteine and tyrosine can also lose a proton from their neutral side chain, but it happens at a high enough pH that we generally think of them as polar more than as acidic. Figure 7-20 shows the acidic amino acids aspartic acid and glutamic acid as they exist in an aqueous environment atpH =7.
pKae =

10.01

O
ll

Acidic Amino Acids


PKas =

"r*\.-.-o_ '7 | vx", =t'oo


H 9Hz AK*=E'sa o- oI

e.e5 ll HrN\ ,zc:. co| H


'tzPKot

=zlz

CH" CH"

l'

Aspartic acid (Asp, D)

AK^z=a'zz o- oGlutamic acid


(Glu, E)

t-

Figure 7-20 Copyright O by The Berkeley Review

179

Exclusive MCAT Preparation

Organic Chemistry

Nitrogen Compounds

Biological

might wish to recall the mnemonic "his lies are basic," which means that histidine, lysine, and arginine are basic. For histidine, the lower nitrogen in the imidizole ring is the basic site. Figure 7-21 shows the basic amino acids lysine, arginine, and histidine as they exist in an aqueous environmen t at pH = 7
.

Basic amino acids Basic amino acids have side chains that gain a proton from their neutral state. The three examples, lysine, arginine, and histidine, have nitrogen-containing side chains. Like acidic amino acids, basic amino acids have three pKu values. you

Basic Amino Ac ids

o
pKaT = 9.20

o
PKaz = 8'99

H'N\c/t\o|'t7 PKot =zte H (CH,)o

il

HeN\./.\o_
H

ll

pKaa =

e.15
H

o
ll
o-

HrN\./.\

17 PKot=t'ez
(CHr).

pKaa = 10.80

t+pg,

'r- vKu = 1.81


9Hz
I

t-" NH
PKaz =

Lysine
(Lys, K)

0"";,';:21-,u*,,
Arginine (Arg, R) FigureT-21

6.0s\ t
Histidine (His, H)

N:./ ^N-H

Amino acids can be categorized in many ways, not just by their side chain, as we have done here. For instance, let us consider essential and non-essential amino acids. There are eight to nine essential amino acids, depending on a person's age. Essential amino acids are the amino acids we cannot synthesize in our body, io therefore we must take them in through our diet.
Example 7.17
\A/hich of the following amino acids has a charged side chain at pH = 7?

A. B. C. D.

Valine
Phenylalanine

Arginine
Cysteine

Solution At a pH of 7, the side chain for valine is CH3, the side chain for phenylalanine is CH2C6H5, the side chain for arginine is CH2CH2CH2NHC(NH 2)2+, and the side chain for cysteine is CH2SH. The only charged side group is found on arginine,
which makes choice C the best ansl\'er.

Copyright

by The Berkeley Review

rao

The Berkeley Review

Organic Chemistry
Example 7.18

Nitrogen Compounds

Biological

which of the following amino acids carries a net positive charge at pH = 8? A. Threonine B. Arginine C. Phenylalanine D. Histidine Solution
a +1 charge and the carboxyl terminal is deprotonated and carries a -1 charge. The compound exists as a zwitterion if the side chain is neutral. So, to have a net positive charge at pH = 8, the side chain must carry a positive charge. The only amino acid of ihe choices to have a positively charged side chain at pH = 8 is arginine, which has a side chain pKu of 13.2. This means that arginine has an overall positive charge

At a pH of 8, the amino terminal is protonated and carries

up to the pH where the amino terminal is mostly deprotonated. This makes arginine, choice B, the best answer. At pH = 8, the structures are as follows:
Threonine

Arginine
Protonated PKoz > 8

Histidine

Protonated O pKuz > 8 ll

Phenylalanine

Protonated

HsN\c-C-oHCH /\ HO CH:

H"N*

l'r,

\ cr
H

t-ar-

ll

HsN\c-C-o-

pKu:>S ll

Protonated
pK"rr8

HsN\c.C-o-

ll

,""""i:t

l',- ,?""i:, 1", i:i':, I -"


(CH")"

1", ,t-""i:t

Protonated

NH

PKo:>B
overall charge = 0

I
+1
overall charge = 0 overall charge = 0

HzNy'L: *rr',
overall charge =

Example 7.19
\A/hich of the following amino acids is involved in cross linking

within a protein?

A. Cysteine B. Threonine C. Proline D. Tryptophan


Solution
Two cysteine residues form a disulfide bridge (-CH2-s-s-CH2-) within a protein or between two separate primary structures (protein strands). The formation of the crosslink involves oxidation of the thiol groups, one of those facts you should know. The best answer is choice A. There are some fundamental structural features of proteins that result from properties of their component amino acids. Cross-linking of cysteine residues is one example. Another is strucfural turns, forced alterations in the structure of a protein, caused by proline. Proline is cyclic and thus unable to rotate freely about its sigma bonds. It is locked into a cyclic conformation, which locks the protein backbone into a tum.

Copyright

by The Berkeley Review

r8r

Dxclusive MCAT Preparation

Organic Chemistry
Example 7.20

Nitrogen Compounds

Biological

\Arhich of the following amino acids is

Mosr

soluble in water?

A. Alanine B. Serine C. Isoleucine D. Methionine


Solution
The compound that is most soluble in water of the choices is the hydrophilic compound. Because the side chain of serine contains a hydroxyl grorrp, it.* form hydrogen bonds. This makes serine hydrophilic, whiie alanine, isoleucine and methionine are hydrophobic. Choice B is the best answer.
Example 7.21

C. Tryptophan D. Methionine
Solution the twenty amino acids for which we code, only threonine and isoleucine have chiral centers in the side chain. The side chain of threonine is naturally R The diastereomer of naturally occurring threonine (that differs from threonine in the chirality of the side chain from R to S) is referred to as allo-threonine. The

A. Proline B. Threonine

which of the following amino acids has a chiral center in the side chain?

of

best answer is choice B.

Isoelectric Points
T, an amino acid exists as an ionic species. The amino terminal is protonated and the carboxyl terminal is deprotonated. The net result is zero overall charge (excluding the side chain in some amino acids), and the structure is referred to as a zutitteiion. The zwitterion is neutral overall but contains oppositely charged sites. The zwitterion form of an amino acid, because it is a salt, has a high melting point. The pH at which the zwitterion exists in highest concentration is referred to as the isoelectric pH. The isoelectric pH is obtained by averaging the two pK^ values that involve the neutral moleculeThat is to say that you average the pKu that takes the molecule from -1 to neutral with the pK6 that takes the molecule from neutral to +1. The molecule is a perfect zwitterion exactly midway between the two pKu values which include the neutral species. The isoelectric point coincides wiih an equivalence point on a titration curve, the first equivalence point of all amino acids except the ones wiilr basic side chains, where it coincides with the second equivallnce point. To calculate the isoelectric pH you must decide if the side chain is acidic, basic, m neutral. For amino acids with neutral or acidic side chains, the isoelectric point b fo"ld by averaging pKal with pKu2. For amino acids with basic side chains, the isoelectric point is found by averaging pKa2 with pKu3. Figure 7-22 shows z pictorial overview of the structure of an amino acid ai it chang:es with increasing pH. The R-group represents any neutral side chain.

In aqueous solution at a pH of

Copyright

by The Berkeley Review

ta2

The Berkeley Revier

Organic Chemistry

Nitrogen Compounds

Biological

-*/oHi'=-#o
HR
Overall churg",

i+L*-/"
HR
0
Overall "hurg",

HR
*1Overall .hu.g"'

-1-

. PLr + pL=-- PKuz


FigureT-22
Figure 7-23 shows a pictorial overview of the structure of an acidic amino acid as changes with increasing pH. The side group is specific for aspartic acid, but the charges are the same with any acidic side chain such as glutamic acid, cysteine, or tyrosine. Acidic side chains lose a proton from their neutral state to form an anion. Acidic side chains contain a hydrogen on either oxygen or sulfur.

it

n,*/oH
H
CFL

Pqr--

og,"/os!e**-/"

#on
Overall charge'

l'

""x
OH

";.
0
Overall chu.ge, -1-

H
Overall

gFr,
I

*1-

h"
"hurg",'2

Overall .hurg",

. PLr pr=-- + PKaz


FigureT-23 Figure 7-24 shows the titration curve associated with full deprotonation of an acidic amino acid, starting from its fully protonated state and proceeding to its fulIy deprotonated state,like it is shown tnFigureT-23.

Isoelectric Point

1,/z
Figure 7-24 Copyright
@

Equivdentsbase added

by The Berkeley Review

Exclusive MCAT Preparation

Organic Chemistry
Figure 7-25 shows
a

Nitrogen Compounds

Biological

pictorial overview of the structure of a basic amino acid as it pH. The side group is specific for lysine, but the charges are the same with any basic side chain such as histidine and argininu. "By definition, basic side chains gain a proton to form a cation at low "pH ,rut.r"s. Basic side chains contain a lone pair of electrons on nitrogen when neutral.
changes with increasing

H (CH,),

t-' NHs

l-' *NH,

(CHr)o

H tfuna
*NH,
Overall chu.ge,

Overali charge,

*2

Overall charge'

*L

Overall charge: -1-

ot=ryfu
Figure 7-25 Figure 7-26 shows the titration curve associated acidic amino acid, like shown in Figure Z-25.

with fully deprotonation of an

Isoelectric Point

pH=pK

pH=pK
HzA*

11

/z

ort - /2

Equivalents base added

Figure 7-26

For calculating the isoelectric point of an amino acid, it is first necessary to determine the nature of the side chain. The isoelectric point is the averagl of pK"1 and pKn2 for all amino acids except lysine, histidine, and arginin". f-h" isoelectric point of lysine, histidine, and arginine is an average of pKu2 and pK6. No matter what isoelectric point you are determining, you should remembei thit it's an average of two consecutive pKu values.

Copyright

by The Berkeley Review

ta4

The Berkeley Review

Organic Chemistry

Nitrogen Compounds

Biological

Easy Method to Get the Isoelectric point of a protein First assume that the,protein exists at pH = 1, where all sites on the protein are protonated. Carboxyl groups are uncharged when protonated, while imino sites carr.y a positive charge when protonated. only the amino terminal and the side chains of lysine, arginine, and histidine carry a charge (a positive one) at low pH, so the net charge of the protein is the rn- oi the lysires, irginines, and histidines in the protein plus one (for the N-terminal). consider the p"rotein tnFigareT-27.

o_@@o H"N-Met-Thr-Ala-

L;s-As "#Llhil"'i'"V"i'-o"
CHs

Gly

scH3 cHoH CHa NHro,AoH H -His\n, oA on


'4\ (NH HN-./

Glu- S"r4
cH2oH

Figure 7-27
The protein in Figure 7-27 canbe abbreviated as H6A3*, because when it is fully protonated, it carries a +3 charge, and there are six sites from which a proton may be lost (the N-terminal, the C-terminal, and the side chains of Lys,Asp, His, and Glu). Now consider the number of times the protein must be deprotonated lo reach zero charge. charge starts at +3, so the fully protonated protein _The nust be deprotonated three times to become neutral. This results in pK^3 eading to the zwitterion. Figure 7-28 shows an abbreviated titratior of u6d3i.'"

r{uA3*

+I

HsAr,

+L

*rno

PKae ,:

PKa+

H3A

tro-

*L

,ro'-

SE* or-

Figure 7-28

1-" p-I is an average of the two pKu values surrounding the neutral species (the :Ku that leads to the zwitterionind the pK2 that leads"from the zwitierion). In lis c1s9, the pI is found by averaging pKa3 with pKu4. The shortcut derived :rom this example is that the-total .hutg" at pu t to the subscript = "o-esponds - i the first pKu averaged to determine pt. This is because the pKu leading to the reutral species corresponds numericaily to the charge wherrfully protJnated. -onsider a protein with a +7 charge at pH = 1. It requires seven dlprotonations :,1 get to the neutral species, so pKuT is the pKu value that leads to the zwitterion.
Figure 7-29 surnrnarizes this example and any generic charge.

+/
E...-

PKar

+q

::

PKat

-*6
+(q-1)

-)

+t

PK^z +^:_
-tlp

PK

-rPrpI=

PKaz+PKaa
2

PKaq+t

PKuq+PKuq*t
2

+1

Figure 7-29

that numerical subscript and averaging it with the next so, if a protein has two lyr;i"t,-or,u arginine and ::ee histidines, then the pI is_an averige of pKuT and pkug. The sample protein :.s a lysine and histidine, so the pI is an aveiage of pfu3 a.,a pfua.

rK" value sequentially up.

rs by T: pI thefoundwithsumming the number of basic amino acids plus one, and --aking pKz

*-rpyright

by The Berkeley Review

ra5

Exclusive MCAT Preparation

Organic Chemistry

Nitrogen Compounds

Biological

The final step is to determine which pKu values are which. For the original protein shown, pKul is for the carboxyl terminal (this is always true). The rrom pKu inrormation' Figure 7-30

nffi:T"";1ii""-Tlffi;[ *r::lined
Lyr-A:pClv

pKa-=e7

@@oo HrN- N4et-Thr-Ala-

rr

I I # cHoH CHr # + scH3 trlnr


!'I 13

I .TJ

_A^ _ H " ro.'so-l -oH pKo =


#6
PKa = 3'9 #2 pKan=

I-

His- Clu- s"r4

ftr d'' ,!. Nrr r _9H Y,.2 - 4.3 .//


\ ,r"
#3

'oH cH2oH pKn=2..1


#1

6.1HNJ

Figure 7-30 The assignment of pKu values is sequential with increasing numerical value, so the pI for this protein is found by averaging pKa3 (4.3) and pKn4 (6.1), which results in a pI of 5.2. This means that at pH = 5.2 it exists as a zwitterion, at pH <
5.2

it exists

as a cation, and at

pH > 5.2 it exists as an anion.

^r -PKa3+PKa4 -4.3+6.7 - <'t -70.4 -"''

''-

Example7.22
\Alhich of the following statements is false about amino acids?

A. B. C. D.

I I

The carboxyl terminal always has the lowest pKu. A carboxylic acid side chain always deprotonates before the amino terminal. The sidechain group of lysine is less acidic when protonated than the amino terminal of lysine. The D-isomer of glycine is the predominant isomer in human beings.

fi

M.

,n

Solution
The carboxyl terminal has the lowest pKu value for all twenty of the amino acids for which we code. This can be verified by looking at the amino acids in Figures 7-17 through7-2L. This eliminates choice A. Because the carboxyl terminal has a lower pKu value than the amino terminal, the carboxyl terminal is more acidic than the amino terminal. By being more acidic, the carboxyl terminal loses a proton more readily than the amino terminal. This means that choice B is valii and thus is eliminated. Choice C requires that you look at the amino acids ir Figure 7-27. The pKu of the amino terminal of lysine is 9.2. The pKu of the side chain of lysine is 10.8. The lower pKu is associated with the amino terminal which implies that the amino terminal is more acidic than the side chain. This makes choice C valid, and thus it is eliminated. The only choice left is answer D. There is no D-isomer of glycine, because the side group for glycine is a hydrogertherefore glycine does not contain a chiral center. Without a chiral center, there can be no D or L configuration associated with the isomer. This confirms that the statement is false, making choice D the best answer. For the other amino acid-. the naturally occurring form is the L-isomer. This can be recalled by thinking a,i the phrase "L is for life." It can also be remembered by thinking amino acids are natural. Either way you choose to recall the meaning of the L label, amino aciis are naturally L. If you are bothered by the word "always" in choice A, sometimes an always is okay.

Copyright

by The Berkeley Review

l86

The Berkeley Reviel

Organic Chemistry
Example 7.23 tr\4rat is the isoelectric

Nitrogen Compounds

Biological

point for tyrosine?

A. B.

c.

2.2 5.6 6.2

D. 9.1,
Solution
Tvrosine is not a basic amino acid, so its isoelectric point is found by averaging

oKul and pKaz. The isoelectric point is solved according to the following
nathematics:

pI

PKal +PKa2 =2.2+9J.

222

- 11.3 = 5.65

This makes choice B the best answer. As a general rule, acidic amino acids have lI values less than 5.5 and basic amino acids have pI values greater than 2.5. All rther amino acids have pI values within the 5.5 to 7.5 range.

Example7.24

"\hat

is the isoelectric point for lysine?

.\.

5.7
6.5

c.

B.

D.

9.2 10.0

5olution
Lysine is a basic amino acid, so its isoelectric point is found by averagin g pKaZ and pKu3. The isoelectric point is solved according to the following mathemitics:

pI =PKa2+PKa3 =9.2+10.8 =20 = I0

222

This makes choice D the best answer. As a general rule, basic amino acids have :i values greater than7.5, so choices A and B are eliminated before any math :reeds to be done. Example 7.25 i\hat is the isoelectric point for the dipeptide alanine-arginine?

A. 7.5 B. 9.0

c.

11.1

D.

11.5

Solution
-{,lanine is a hydrophobic amino acid and arginine is a basic amino acid, so the :-.oelectric point for the dipeptide is found by averaging pKa2 and pKu3. As
-.',-ritten, the

peptide linkage is made from the C-terminal of alanine and the N:erminal of arginine, so there is no pKulgOOH; for alanine and no pKuglgr*; for arginine. For the dipeptide, pKu2 is the N-terminal of alanine and pKu3 is the ;ide chain of arginine. The isoelectric point is solved as follows:
p1 = PKa2

'zz

PKa3 = 9.9 + 13.2

23.1.

= 11.55
Exclusive MCAT Preparation

-opyright O by The Berkeley Review

ta7

Organic Chemistry

Nitrogen Compounds

Biological

This makes choice D the best answer. The difficult task in these questions is not

determining which pKu terms belong in the equation, but rather assigning a numerical value to each pKu term. If you've grown weary of continually turning back to the amino acids in Figures 7-77 tLrrough 7-21, then start making some approximations. Knowing that the N-terminal is pKu2 and its value is around 9 should tell you that the pI is greater than 9. This eliminates choices A and B, which helps to some extent. From here, you have to know the values. The MCAT test writers will provide pKu values if the question is this specific.
Example 7.26 VVhat is the isoelectric point for the dipeptide histidine-leucine?

A. 5.5 B. 6.7 c. 7.6


D.
9.2

Solution Histidine is a basic amino acid and leucine is a hydrophobic amino acid, so the isoelectric point for the dipeptide is found by averaging pKa2 and pKu3. As written, the C-terminal of histidine and the N-terminal of leucine make up the peptide linkage, so pKu3 is the N-terminal of histidine and pK62 is the side charn of histidine. The presence of the leucine does not alter the pI from what it woulc have been with histidine alone. The isoelectric point is solved according to the following mathematics:

pI= pK62 +pKu3 =6.1+9.2 -15.3

22

=7.65

This makes choice C the best answer. You should have been able to solve thi-.

particular question without consulting any other data. The side chain o: histidine has a pKu close to physiological pH, so you know that the pI is ar. average of roughly 7 and9, which means that the pI is about B. Only choice C i' close to that number. Another method for determining the best answer is tc consider that histidine is a basic amino acid. We know that the pI is greater thar 7.5,but it must be less than the pKu of the amino terminal, given that pKu: corresponds to the amino terminal. Only choice C fits in the 7.5 to9.2rcnge.
Example7.27 At what pH is the tripeptide serine-cysteine-isoleucine perfectly neutral?

A. 5.3 B. 6.1,

c.

p'

D.

8.4 9.4

Solution
The pH at which it is perfectly neutral is the isoelectric

point. Of the three amiri; acids, none are basic, so the isoelectric point for the tripeptide is found b', averaging pKul and pKu2. The tripeptide has three active protons, the Nterminal of serine, the side chain of cysteine, and the C-terminal of isoleucine The C-terminal is always pKal and in this case pKu2 is the side chain of cysteine The isoelectric point is found using the following mathematics:
61 =PKal

+PKa2

'222

=2.3+8.4 =70.7 =

5.35

Copyright

by The Berkeley Review

1a8

The Berkeley Kevier

Organic Chemistry

Nitrogen Compounds

Biologicat

This makes choice A the best answer. Choices C and D should have been eliminated, because there are no basic amino acids in the tripeptide. Cysteine is actually an acidic amino acid, but because most cysteine side chains are involved in cross linking, we rarery consider their acid-base propertie, i'prot"ir,r.
Example 7.28

l4lhich of the following tripeptides has the LOWEST pI value? A. Asp-Lys-Asp B. Glu-Lys-Glu C. Lys-Asp-Lys D. Lys-Glu-Lys

Solution
The lowest pI value corresponds to the most acidic tripeptide. Choices A and B have two acidic amino acids and one basic amino acid, while choices C and D have two basic amino acids and one acidic amino acid. This eriminates choices C and D' Choices A and B each have one lysine,.so both pls are found by averaging pKu2 and pKu3. In each tripeptide, ther! are five active protons (the N_terminal, the C-terminal, and th::: the siae ciains;. Both pKu2 and pKu3 correspond to the side chains of the acidic amino acids. tn choice-l, *," aliaic amino acid is aspartic acid while in choice B the acidic amino acid is gt.rtumi" acid. Aspartic acid has a lower side chain pKu than grutamic acid, so tti" t.ipuftiae in choice A has a lower pI than the tripeptide ln ctioice B. Choice A is the best answer.

Example7.29

-\t pH

= 5.0,

D. -1.
Solution
are protonatJd or deprotonated. wh"en the pH ;i;", than the pK,, the site is protonatga' jhe pH is greater than the pKu, the site is - ^{rren deprotonated. At pH = 5.0, histidine has a dJprotonated carboxyl terminal (pKu - 1.8), a protonated
To determine the charge of an amino acid at a given pH, first determine whether 'he sites

c.

B.

{.

histidine exists with a net charge of;

+2. +1.
0.

side chain (pKa = 6.1), anja terminal (pKu = 9..2). The carboxyl terminal .itriur a negative .hurg", while ttre side chain and amino terminal are both carry a positive The"overall charge is therefore a positive one. The best answer "frurg". isihoice B. Hiitidine at pH = 5.0 is drawn below: PKa=9'2 pH . pK
protonated

pr;;;";rJ;;ino

HsIr(

o- pH>pK

PKa = 1'8

H
PKu = 6.1

deprotonated

PH < pKo

protonated

9Hz

overall charge = +1 +1 -1 =

+1

-opyright

by The Berkeley Review

ra9

Exclusive MCAT preparation

Organic Chemistry
Example 7.30

Nitrogen Compounds

Biological

At pH = 9.5, tyrosine exists with

a net charge of:

A. +1 8.0
D.

c. -1

-2

Solution
deprotonated amino terminal (pKa = 9.1), and a protonated side chain (pKu = 10.1). The carboxyl terminal canies a negative charge, while the amino terminal is uncharged. The side chain is uncharged when protonated, so the overaltr charge is a negative one. The best answer is choice C. Proteins
Proteins are biological polymers built from amino acids. They are held together

At pH = 9.5, tyrosine has a deprotonated carboxyl terminal (pKa = 2.2), a

by covalent bonds in peptide linkages. A peptide linkage is formed when the amino terminal of one amino acid attacks the carbonyl carbon of the carboxr-l terminal of another amino acid, resulting in the loss of water. Because amino acids have specific stereochemistry, proteins are highly chiral (on average, therhave one chiral center for every amino acid in the protein). This large degree of chirality explains why enzymes are so selective.
Protein Structural Features and Levels The structure of a protein can be broken down into levels. The most elementan' level is the primary structure, which is defined as the sequence (connectivity) o: amino acids within the protein. To break down the primary structure, you mus: cleave the peptide bonds. Figure 7-30 shows a tetrapeptide (four- amino acii protein). Tetrapeptide (4 amino acid protein)

*/'
N
I

H
Peptide linkage (amide bond)

R3

Figure 7'30

In this case, the primary structure is the order in which the four amino acids a:e arranged from the amino terminal to the carboxyl terminal. The prima;r. structure of a protein or enzyme is determined by sequencing the protein one amino acid at a time. This will be discussed later. The interactions between ttw amino acids within the protein are responsible for the secondary structure" which is defined as the folding of the amino acids into their natural configuraticm within the protein. The chemical reasons for special features in the seconda--r' structure are most often attributed to hydrogen bonding, and sometimes to ctcei:-" linking through disulfide bridges and kinks and turns caused by the presence rff proline. Some of the structural features of interest in proteins are the s-helix and Copyright O by The Berkeley Review
19()

The Berkeley Revier

Organic Chemistry

Nitrogen Compounds

Biological

G-pleated sheets. In the o-helix, two residues near one another in the protein are held together by-hydrogen bonds. There are 3.6 amino acids per turn and each turn is 5.4 A in length. They coil like a phone cord. In B-pieated sheets, the amino acids strands are not coiled. They ire held together by hydrogen bonds from the_hydrogen on nitrogen to the carbonyl oxyge"n. Figure 7-31 shows a Bpleated sheet with antiparallel strands

-t _t
*r,/
il
o,,,.

R7 4

roH

-rvA1
"\

lll

Rzz

Rzo

\
il 9'. 9,
38

Is\ll
Rzs

rf

o
H
I

(^l t
5_

-N.
q
H

A-.4 T+i/r& 2-- I Its .fi t;


o
N.

Y il
o P
il ll

Rgz ,H

R+r

R,"H

H
I

N
I

N
I

o
Figure 7-31

{ disulfide bridge is formed when two cysteine resid.ues lose the H from the :hiol group to form a sulfur-sulfur bond. Disulfide bridging is most often
jnked protein is considered more oxidized than the protein without

:onsidered to be part of the tertiary structure. Because loss of hydrogen results rn an increase in the oxidation state of sulfur, this process is oxidatiae, so a cross-

-igure 7-32 shows the reductive cleavage of a disulfide linkage using the remical reagent B-mercaptoethanol.

crossTo break a cystine cross-linkage, one can add. a reducing agent such as :^khg. :imercaptoethanol, HSCH2CH2oH. As a side note, cross-litrtitr[ iJresponsible :or an increase in rigidity for the polymer (polypeptide in this case) due to the -oss of both flexibility and entropy. In terms of thermodynamics, the process of =oss-linking is driven by enthalpy and not entropy (recall that AG = lu - us).

-opyright O by The Berkeley Review

l9l

Exclusive MCAT Preparation

Organic Chemistry
,

Nitrogen Compounds
Vasopressin (antidiuretic hormone)

Biological

Cys-Tyr-Phe-

GIn-Asn- Cys-pro-Arg-

Gly

SH
I

Cys

-i

lr,,rA-tt i
s
I

NH-

-oH
Gly

-Tyr -Phe-

Gln-Asn-

CYt

SH Cys-Tyr

l"oA-tt
SH

-Pro-Arg

NH-

tt

Gln-Asn- Cys-pro-A.g
+

Gly

NH-

Horyt-rA.-ot
Figure 7-32

when any of the molecular interactions that hold together the secondary o: tertiary structure of a protein are broken, the protein is said to be denqtured 1nt longer in its natural form). The overall folding can be changed by the environment in which the protein exists. ln an aqueous environment, a protei:: will fold in such a way as to expose the hydrophilic side groups to the iolven: (water) while minimizing the exposure of the hydrophobic (alkyl and phenr. side groups to the solvent (water). in a hydrophobic envirotr-eni, a protein n-i; fold in such a way as to expose the hydrophobic side groups to the solvent (lipic while minimizing the exposure of the hydrophilic (hydrogen bonding) itde groups to the solvent (tipid). This overall folding of the protein is referred to a-. the tertiary structure. The tertiary structure is globular for many proteins.
Example 7.31

:-

t c

rl

ffi
Taking a protein from a lipid environment and placing environment would most affect which of the following?

it into an aqueous

&
ffi
G

,mg

A. Primary structure B. Secondary structure


C. D.
Tertiary structure Quaternarystructure

ffifr

'lbu
Solution A protein in a lipid environment has its hydrophobic side chains exposed to the solvent and its hydrophilic side chains in the inner core. A protein in an aqueons environment has its hydrophilic side chains exposed to the solvent and is hydrophobic side chains in the inner core. The conversion from lipi; environment to aqueous environment breaks apart the internal hydroge: bonding, which denatures the protein. This affects the secondary structure tr changing the structural features. The tertiary and quaternary structures are also affected, but only as a consequence of changes in the secondary structure. This question calls for the most specific change. The best answer is choice B.
Copyright
@

ru

lMfl um

rfls

.ft
&

&d

,m
fr,$n
,li@M

by The Berkeley Review

192

The Berkeley Revieu

Organic Chemistry
Example 7.32

Nitrogen Compounds

Biological

Turns are caused by which of the following amino acids?

A. B. C. D.

Cysteine

Histidine Proline Glycine

Solution proline is cyclic, it cannot freely rotate about its sigma bonds, forcing the protein chain to turn. Choice C is the best answer. The following pictorial representation shows the impact of the cyclic structure of proline on the turn.
Because

*y'
N

\*:
o
protein turn

proline residue

turn--d,

.A

"Y

NH

,*.,rJL

*^If
o

tr= H R

:'

Bent peptide containing proline

Blown up view of protein tum

v
E

Example 7.33
\Alhich of the following results in a denatured protein?

A. B. C. D.

Cleaving disulfide bonds Spiitting base pairs


Breaking alpha bonds

Forming of beta bonds

Solution
Breaking disulfide bonds requires the reductive cleavage of the S-S bond. This definitely changes the tertiary structure, thus denaturing the protein. Base pairing is involved in DNA and RNA, not proteins, so choice B is eliminated. Alpha bonds and beta bonds are throw away answers, so choices C and D are

eliminated. Choice A is the best answer. Unique Biological Process A common rule in the chemistry of amino acids is that they exist exclusively as Lstereoisomers in biological systems. An exception to this "all amino acids are L" rule is found in the bacterial cell wall. The cell wall of a bacteria is held together by a net-like structure that involves a cross-link between D-alanine and glycine. As a point of biological interest, B-Iactams such as penicillin act in a medicinal fashion by breaking this linkage and destroying the cell walls of bacteria. Again, these are not facts to be processed and stored, but they are interesting biological anecdotes that have been presented on previous MCATs. Discussing them here is simply to provide a little background, so if they show up again, there will be a small air of familiarity to them. Figure 7-33 shows the formation of the cross-link in the bacterial cell wall.

Copyright O by The Berkeley Review

t93

Exclusive MCAT Preparation

Organic Chemistry
O

Nitrogen Compounds

Biological

pentaglycinebridge TerminalD-Ala-D_Alaunit

HOHH til lt R-C-CHz-NH, + R'-N-C-C-N-C-COr- + t' ll Terminal Residue of the H CHe CHs

il

I I *l R-C- CH"-N-C-C-N-R' + H.NIC-CO?--r^' | ' ll | '


Gly-D-Ala Figure 7-33

il

HH

o CHs crosslink

CHs

i)

D-Ala

Gel Electrophoresis L9t us consider gel electrophoresis, a biochemistry lab technique based on the idea that charged particles migrate through eleciric fields. Particles migrate according to their charge (which impacts the strength of the electrical forceland their size-(which impacts their resistance to flow ur th"y travel through a gel). A polyacrylamide gel is chosen to offer resistance ro ihut the particles do not migrate too rapidly and thus not have the time to separate to a distinguishable amount. Figure 7-34 shows the schematic of an electrophoresis apparatus.

tu

ffi

&

+--_@

m-

Jl"

t
{c.

IL

oFigure 7-34

shows the relationship of the particle's mass and acceleration to ?ts charge, q, tllg strength of the electrii field, E, and the resistive force due to the drag urlt rrloro through the gel.

In gel electrophoresis, cations migrate to the cathode and anions migrate to the anode. This allows for separation of cations from anions. To get separation c,f like charges (cations from cations for instance), the particles "must migrate at different speeds. The speed depends on the acceleration, which dependJ on tlw electric force (F = qE) and the resistive force due to drag (Fa.ug). iquation i-J

,im ery

q onn m

ffi hr

ma=gE-Fdrag

(7Jl

The charge of an amino acid, polypeptide, or protein fragment depends on tle pH of the environment and the isoelectric point, pI, of the species. when pH k greater than pI, the environment is basic, resulting in a negatively chirged species. when pH is less than pI, the environment is acidic, resulting iir m positively charged species. The bigger the difference between the pH or *, environment and pi of the species, the greater the magnitude of its charge. Au

values. The other factor that influences the migration rate is the size of ti
species. Larger species experience greater resistance as they travel through medium. As a result, there are two types of electrophoresis tlchniques emp-.-lor to separate protein fragments. Copyright O by The Berkeley Review
1

the magnitude of charge increases, the electric force inireases, and thereiore acceleration increases, This means that the species can be separated by their I

194

The Berkeley

Organic Chemistry

Nitrogen Compounds

Biological

The first type of electrophoresis employs a pH gradient in the gel and the charged species migrate through the gel until they ieach a pH equalto their pI. At that point they have no net charge, so there is no electric force. This techniqle is referred to as isoelectric focusing. The second type of electrophoresis involves adding sodium dodecyl sulfate, sDS, to the piotein mixture. The protein fragments incorporate sDS into their secondary structure, resulting in every fragment taking on a negative charge. Larger protein fragments can incorporate more sDS than smaller ones, resulting in a greater magnitude of cirarge. However, larger species are more massive. As a result, the incorporation of sbs creates an environment where all of the protein fragments have the same massto-charge ratio. Given that they are experiencing the same electric field, any separation is due to differences in size, where smail species migrate faster than larger species, because they experience less resistance due to drag.
Example 7.34

What is true of the protein fragment in a mixture that migrates fastest in SDS PAGE and migrates to the highest pH value in a gel with a pH gradient?

A. It is the heaviest fragment in the mixture B' c.

aspartic acid. It is the heaviest fragment in the mixture and is most likely to contain lysine. It is the lightest fragment in the mixture and is most likely to contain aspartic
acid.

and is most likely to contain

D. It is the lightest fragment in the mixture


Solution

and is most likely to contain lysine

In SDS PAGE, the speed at which a species migrates depends on the resistance it experiences moving through the gel. Larger fragments experience greater resistance, so a species that migrates rapidly through the gel must be small. This means that the fastest fragment must be the lightest fragment, so choices A and B are eliminated. Because the species migrated to a high pH value, it must have a large pI value. The presence of lysine, arginine, and histidine increases the pI, so the species must have at least one of those three amino acids. Aspartic acid Iowers the pI value, so choice c is eliminated and choice D is the best answer. .{ff inity Chromatography Another technique that can separate proteins according to their charge is affinity chromatography. we will look specifically at ion-exchange chromatography. Ion-exchange chromatography separates by the affinity of charged protein- for oppositely charged sites on the polymer in the column (stationary phase). The stationary phase is made of an insoluble polymer, often polystyrene or cellulose, to which functional groups capable of carrying a charge have been attached. Typical anionic groups include sulfate on sulfonated polystyrene and carboxylate on carboxymethyl-cellulose. Typical cationic groups include diethylaminoethoxy on diethyiaminoethoxy-cellulose. Positively charged columns bind anionic species tightly and negatively charged columns bind cationic species tightly. A protein carries a positive charge when the solution pH is less than the isoelectric point, pI, of the protein. v\4ren a mixture of proteins is added to the column and ailowed to migrate down the polymer, selected proteins can be separated from the mixture by binding the column. Proteins carrying the same iharge as the column elute. To release a bound protein from an ion-exchange column, the polymer can be washed with a solution of varying pH or a solution of gradually rncreasing salt concentration. Both techniques for releasing the protein could denature it irreversibly, so the preferred technique is not always obvious. Copyright
@

by The Berkeley Review

t95

Exclusive MCAT Preparation

Organic Chemistry

Nitrogen Compounds

Biological

Cutting and Sequencing (Primary Structure Determination) Determining the primary sequence of a protein is critical in biochemistry. In the sequencing of proteins, peptide bonds must be broken and component amino acids identified. To sequence a protein, several reagents are involved. Sometimes it is best to break a large protein into smaller, more manageable fragments. Each fragment can be sequenced and then the fragments &n be reassembled to determine the overall sequence of the original protein. This means thai the original protein must be mirked in such u *iy that the first and last fragments are easy to identify. we will start by determining which amino
sequence can be accomplished by adding 2,4-dinitrofluorobenzene, 2,4-DNFB llanger's reagent), to the protein, followed by comprete hydrolysis using 6 M HCl. sanger's reagent binds the amino terminal of a protein, thereby labeling the

acid is first in the protein. The identification of the first amino acid in the

first amino acid. Complete acid hydrolysis of the piotein then yields ail o] the component amino acids, of which only one is marked. Figure 7-35 shows both
free 2,4-DNFB and a complexed amino acid residue

2,4-dinitrofluorobenzene

0
TO

HNq

Nq

;x:KI
Figure 7-35

"b,f

-)

Nq

Labeled amino acid residue

activity.

Sanger's reagent serves to mark the first amino acid in a protein and to ultimatelridentify it, but that's just the beginning when req.tetrcitrg a large protein wiiir several amino acids. To break the protein into smaller fragments,-biochemists mimic what nature does. we employ enzymes that break a large protein into smaller fragments by cleaving proteins at specific sites (bondsy. fne enzyme chymotrypsin, which is active in the small intestine, cleaves an amino acid cirain at the peptide linkage on the carboxyl side of phenylalanine, tyrosine, or tryptophan, As a point of biological interest, the pancreas actuaily releases chymotrypsinogen, an inactive species that gets activated once in the small intestine. chymotrypsin would cleave the twelve amino acid protein, Arg-CysGly-Ala-Phe-Thr-Met-Ala-Tyr-Cys-His-Leu, twice, Ieaving three smalier fragments. The three pieces are: Arg-Cys-Gly-Ala-phe, Thr-Met-Ala-Tyr, and Cys-His-Leu. Reassembling the three fragments can be done if the first frigmen: is known from the 2,4-DNFB result. Table 7-r shows various enzymei and chemical reagents used to break peptide iinkages, their specific activity, and for the digestive enzymes, their corresponding zymogen and sites of release ani

Copyright

by The Berkeley Review

196

The Berkeley Reviex

1{L

Organic Chemistry
Enzyme or Agent

Nitrogen Compounds

Biological

Cleavage activity C-side of Phe, Trp, and Tyr C-side of Arg C-side of Met C-side of Trp

Site (if applicable)


Released as chymotrypsinogen from pancreas into small intestine

Chymotrypsin
(digestive enzyme) Clostripain
(enzyme) Cyanogen bromide (chemical reagent)

O-Iodosobenzoate
(chemical reagent)

Hydroxylamine
(chemical reagent) 2-Nitro-S-cyanobenzoate (chemical reagent) Pepsin

Asp-Gly

bonds

N-side of Cys
C-side of Asp, Clu, and Leu C-side of Asp and Glu (Glu only under specific conditions)
Released as pepsinogen from chief cells into stomach

(digestive enzyme)
Staphylococcal protease (digestive enzyme)

Thermolysin
(enzyme)

N-side of Leu, Val, and Ile


C-side of Lys and Arg
Released as trypsinogen from pancreas into small intestine

Trypsin
(digestive enzyme)

TableT-l
sequencing is often done in multiple steps. some methods involve random hydrolysis followed by sequencing of the fragments and analysis of the overlap of information between fragments. There is also a systematic approach where ihe protein is broken down at specific linkages using different reagents and the overlap between fragments is again analyzed. Consider the following example. A twenty-seven amino acid protein is treated with various reagents in four separate experiments. The results of each experiment are listed below. 1. Treatment of the protein with 2,4-dtnitrofluorobenzene and then 6M HCI yields a histidine residue bonded to 2,4-dtnitrobenzene. 2. Treatment of the protein with chymotrypsin yields the following
fragments:

a) N-His-Val-Ser-Gly-Ala-Ile-Phe-C b) N-Leu-His-Gly-Thr-Tyr-C c) N-Val-Asp-Arg-Cys-AIa-Leu-C d) N-Val-His-Lys-Glu-Trp-C e) N-Cys-Ile-Met-Phe-C 3. 4.


Treatment of the protein with trypsin yields 3 fragments that are
19

amino acids, 5 amino acids, and 3 amino acids in length. Treatment of the protein with thermolysin yields a free histidine residue

a) N-Ile-Phe-C b) N-Leu-His-Gly-Thr-Tyr-Cys-C c) N-Ile-Met-Phe-C d) N-Va1-His-Lys-Glu-Trp-C e) N-Val-Asp-Arg-Cys-Ala-C 0 N-Val-Ser-Gly-Ala-C


Copyright
@

and a free leucine residue along with the following six fragments:

by The Berkeley Review

197

Exclusive MCAT Preparation

Organic Chemistry

Nitrogen Compounds

Biological

first amino acid in the protein, because of the reaction with sanger's reagent.

Frorn the first experiment, the conclusion you should draw is that histidine is the

the five fragment that would not have been fragmented by chymotrypsin. Combining the conclusion of the first experiment with fragments in the second experiment, we know that amino acids 1-7 are His, val, ser, Gly, Ala,Ile, and phe sequentially. We also know that amino acids 22-27 are Val, Asp, Arg, Cys, Ala, and Leu sequentially. The free leucine from Experiment 4 further supports that leucine is the last amino acid in the sequence. From this point, we will label each fragment from Experiment 2 A through E and use information from the other experiments to evaluate the overlap of information and determine the position of each amino acid. The five fragments from Experiment 2 are arbitrarily assigned the letters A through E as follows:
Fragment A: N-His-Val-Ser-Gly-A1a-Ile-phe-C Fragment B: N-Leu-His-Gly-Thr-Tyr-C Fragment C: N-Va1-Asp-Arg-Cys-Ala-Leu-C Fragment D: N-Va1-His-Lys-Glu-Trp-C Fragment E: N-Cys-Ile-Met-phe-C
Because we know the first and last amino acids, we know that fragment A is first, and fragment C is last. This leaves six possible combinations for the fragments produced in Experiment 2. The possibilities are: A-B-D-E-C, A-B-E-D-C, A-D-BE-C, A-D-E-B-C, A-E-B-D-C, and A-E-D-B-C. From Experiment 3, we know that the 3-AA fragment must be Cys-Ala-Leu, the last three amino acids. Trypsin cleaves the C-side of Lys and Arg, so the Lys-Glu bond is broken. This leads us to conclude that fragment D must be the fourth fragment, because the five amino acid fragment from Experiment 3 must end with arginine. This narrows the choices to two: A-B-E-D-C and A-E-B-D-c. From Experiment 4, it canbe inferred that Phe from fragment A must be bonded at its carboxyl site to either Leu, val, or Ile. This means that fragment E cannot be second in the overall sequence, which narrows the answer down to A-B-E-D-C. This makes the original protein: N-His-Val-Ser-Gly-Ala-Ile-Phe-Leu-His-Gly-Thr-Tyr-Cys-Ile-Met-phe-val-HisLys-Glu-Trp-va1-Asp-Arg-Cys-Ala-Leu. Be sure you work through the logic of

From the second experiment, we can conclude that leucine is the last amino acid in the protein, because leucine is the only carboxyl terminal amino acid in one of

sequencing.

Copyright

by The Berkeley Review

l98

The Berkeley Review

Organic Chemistry

Nitrogen Compounds

Biological

fifty amino acids maximum, depending on the amino acids), biochemists employ Edman's reagent, phenylisothiocyanate (H5C6N=C=s). This reagent r"q,t"t"rtiuily removes each amino acid one at a time starting from the amino terminal. This leads to the amino acid sequencing equipment used in most biology labs. Edman's reagent and its reactivity is shown in Figure 7-36.
I\-L-b

Edman's Reagent To sequence the amino acids in a small polypeptide (twenty to

Phenylisothiocyanate
S

il
Peptide

/c-

wH

G"

NH-peptide H

1t

ii-(* "

\*
NH-peptide

HzN

orr,"^ 1l T rransrertt_ry\
NH-peptide

H
Figure 7-36 Example 7.35 Hydrolysis of a peptide results in the cleavage of: A. the carbonyl group. B. acarbon-carbonbond. C. the carboxyi terminal.
the amide bonds.

D"

Solution Polypeptides are held together by peptide linkages, which are amide bonds between amino acids. To break apart a polypeptide, the amide linkage must be severed. This makes choice D the best answer.

Copyright

by The Berkeley Review

199

Exclusive MCAT Preparation

Organic Chemistry

Nitrogen Compounds

Biological

Example 7.36 2,A-Dinitrofluorobenzene reacts with the:

A. nitrogen of the amino terminal. B. carbon of the carboxyl terminal. C. nitrogen of the amide bonds. D. carbon of the side chains.
Solution 2,4-Dinitrofluorobenzene, known as sanger's reagent, is an electrophile that reacts with the nitrogen of the amino terminal of a protein by iay of a nucleophilic aromatic substitution reaction. The nitroget of th" amino terminal attacks the benzene ring nucleophilically to displace th"e fluorine on the aromatic ring in a two-step reaction. The best answer is choice A.
Example7.37
The tripeptide Lys-Arg-Ser would NOT:

A. have an overall positive charge atpH = 7. B. be water-soluble. C. from any disulfide bonds. D. have an isoelectric point greater than Asp-Glu_Thr.
Solution
The tripeptide has an amino terminal on lysine (pKa 9.2), acarboxyl terminal = on serine (pKa = 2.2), an ammonium side chain ori tyrin" (pKa 10.g), and a = guanidinium side chain on arginine (pKa =13.2). At pil 7,'ih. amino terminal = is protonated and thus positively chaiged, the carboxyl terminal is deprotonated a.d thus negatively charged, and thelwo basic side chains are protonated and thus each are positively charged. The overall charge is therefore positive two_ This means that "The tripeptide Lys-Arg-ser would, not have an overall positive charge at pH = 7" is an invalid statemenl, so choice A is eliminated. Because all sites on the tripeptide are charged, the compound is ionic and therefore :{1h." highly water-soluble. This means that "The tripeptide Lys-Arg-ser would not],rwater-soluble" is an invalid statement, so choice B is eliminatel. Because neithen lysine, arginine, nor serine contain thior groups (only cysteine does) there can be no disulfide cross-linking associated with this tripeptiie. This means that "The tripeptide Lys-Arg-ser would not have any disuiride bonds formed" is a valirr statement, so choice C is the best answer. The isoelectric point for the tripeptidc Lys-Arg-ser is an average of pKu3 and pKu4 (the two sidl chain pKu vaiues fc lysine and arginine), while theisoelectrii point for the tripeptide Asp-GIu-Thrb an-average of pKul and pKu2 (the carboxyl terminal pru value and the side chain pKu value for aspartic acid). The average of pKul u"a pru2 for Asp-Glu-Thr ise lower number than the average of pKul anapiua ro. LyJ--a.g-sei. This ,, that "The tripeptide Lys-Arg-ser would nothive inisoeiectric"point greater Asp-Glu-Thr" is an invalid statement, so choice D is eliminaied. Basic acids (lysine, arginine,, and hjstidine) always increase the isoelectric point acidic amino acids (aspartic acid and glutamic acid) always decrease isoelectric point. This question is difficult,lecause the language is crrmbe.st Making use of denotations next to each answer choice tik-e "ialse, because it

positive," may prove highly beneficial. you need to not only recall the subir 11!ter when working with these books, but you must also hone your test-ta skills.

Copyright O by The Berkeley Review

200

The Berkeley

Organic Chemistry
Non-biological Nitrogen-Containing Compounds

Nitrogen Compounds
7)

Section Summary

Key Points for Nitrogen Compounds (Section

1.

Amines (Contains a central nitrogen with either alkyl groups or hydrogens


attached) a) Basicity (Amines are weak bases)

i. ii. iii. b) c)

The pK6 for amines is between 3 and 5, the pKu of protonated


amines is between 9 and
11

Typically, 2' > 3'= 1", because of steric hindrance and H-bonding Common reagent in buffers with pH of 10 t 1

Nucleophilicity i. Good nucleophiles for 51112 reactions ii. Can undergo multiple additions when treated with alkyl halides
Amines can be synthesized in many ways

i. ii. iii. d)

Gabriel synthesis using alkylation of a phthalimide followed by hydrolysis Reduction of azides, nitriles, imines, and amides Hofmann rearrangement of an amide

Amine reactivity is based on nucleophilic substitution

i. Amines readily displace a leaving group from alkyl halides ii. Amines react with nitrite to form diazonium salts iii. Quaternary amines can undergo Hofmann elimination to yield
terminal alkenes where three of the alkyl groups are methyl groups
2.

1l

a)

Imines have nitrogen double bonded to a central carbon (R2C=NR') They react in a similar fashion to aldehydes and ketones

i. ii. iii.
.)-

They are in equilibrium with enamines The carbon is electrophilic and can be attacked by nucleophiles Oximes and hydrazones form in a similar way as imines

Amides have nitrogen bonded to a carbonyl carbon (RC=ONR') a) Amides are neither acidic nor basic i. Amides are formed from substitution reactions where an amine attacks an ester, anhydride, or acid halide ii. Amides are also formed from an oxime via Beckmann rearrangement iii. Amide bonds are referred to as peptide bonds when formed between amino acids

4.

Amino acids can be synthesized by several methods in aitro a) The alpha carbon has four things attached, an H, CO2H, NH2, and Rgroup, so synthesis routes center around adding these groups to carbon i. Hell-Volhard-Zelinskii synthesis activates the alpha-carbon of a carboxylic acid and then adds an amine ii. Strecker synthesis adds a cyano group to an imine and then undergoes hydrolysis iii. An alpha keto acid can be converted into an alpha imino acid, which is reduced into an amino acid iv. Gabriel synthesis, which is used to synthesize a primary amine can make an amino acid, where the phthalimide is added to an alkyl halide with a carboxylic acid group rather than an alkyl halide

Copyright O by The Berkeley Review

201

Exclusive MCAT Preparation

Organic Chemistry

Nitrogen Compounds

Section Summary

Biological Nitrogen-Containing Compounds 1. Amino acids have an amine group on the alpha carbon of a carboxylic acid a) They are classified according to their R-group i. Hydrophobic amino acids have alkyl side chains ii. semi-hydrophobic amino acids have functional groups that are only slightly water soluble iii. Hydrophilic amino acids have highly water soluble side chains, usually alcohols and amines

b)

Side chains can be acidic, basic, or neutral

c)

Acidic amino acids contain oxygen, lose a proton from their neutral state, and have isoelectric points less than 5.5 ii. Basic amino acids contain nitrogen, gain a proton from their neutral state, and have isoelectric points greater than 2.5 iii. Neutral amino acids do not gain or lose a proton from their neutral state and they have isoelectric points between 5.5 and 7.5 Isoelectric points, the pH where the charge is 0, are found by averaging the two pKus leading to and from the zwiiterion i. pI is an average of pKul and pKu2 for all amino acids except Lys, Arg, His, because "his lys are basic.', For Lys, Arg, and His, pI is an

i.

ii. iii. iv. 2. a) b)

and His plus one for the N-terminal, which gives the p*u t".i-, leading to the zwitterion when the pH of the environment exceeds pI, the species is an anion \A/hen pI exceeds the pH of the environment, the species is an cation

average of pKu2 and pKu3. The pI of a protein is found easily by adding the number of Lys, Arg,

Proteins are polymers of amino acids Primary structure is the amino acid sequence and peptide linkages secondary structure involves the interactions of nearby amino acids i. An alpha-helix is a coil in the backbone with 3.6 amino acids per tum ii. Beta-pleated sheets involve H-bonding of amides of nearby amino acid moieties

c)

Tertiary strucfure involves side chain interactions of amino acids that are far apart from one another i. Beta-pleated sheets involve hydrogen bonding of amides of nearbv amino acid moieties

d) Quaternary structure involves interactions between two


primary strands

different

3.

Protein lab techniques center around separation and sequencing a) Gel Electrophoresis

i. ii. iii. b) i. ii. iii.

Anions migrate to anode and cations migrate to cathod.e in an E field Buffered gel gets separation by pI: migrates until pl = pH of gel SDS-PAGE separates by size: bigger fragments migrate slower

sequencing involves analyztng the primary structure of a protein First treated with urea to break H-bonds and G-mercaptoethanol Edman's reagent, Ph-N=C=s, cuts one AA at a time from N-terminal manageable fragments which can be sequenced and reassembled

Enzymes can be used to break large proteins into smaller, more

Copyright

by The Berkeley Review

202

The Berkeley Revierr

I. Basicity of Amines II. Amine Basicity and Nucleophilicity III. Effects of Alkyl Qroups on Amine Nucleophilicity IV. Imine and Enamine Chemistry V. Amino Acid Synthesis in vitro VI. Amino Acid Trivia VII. Acidity and Basicity of Amino Acids VIII. Amino Acids and polypeptides IX. Physical Properties of Amino Acids X. Protein tlormones XI. Enzymatic Cleavage and Sequencing XII. Peptide Sequencing XIII. Amino Acid Stereospecificity in Synthesis
Questions not
Based on a Descriptive passage

(L

-7)

(8 - 14)

(ls

- 20)

(2r - 27)
(28 - 35) (36 - 43) (44 -

5t)

(52 - 58) (5e - 66) (67 - 75) (74 - Bt) (82 - 88)
(Be - e6)

ffi

rm

rm

mfr

(e7 -

1OO)

Amines and Amino Acids Scoring Scale Kaw Score 84 - 100


MCAT Score

t3- 15
IO-12 7 -9

66-85
47 65

34-46

l-33

4-6 L-3

Passage

(Questions 1 - 7)

3.

The trend for the relative basicity of amines in water is: secondary (2") > primary (1") = tertiary (3') > arnmonia. The

What can be determined about the equilibrium constant (K) for the following acidlbase reaction?

H3CNHj+ + NH3

order is explained by competing influences in water. The electron donating effect of the alkyl groups supports a relationship of 3' >2" > 1' > ammonia. This prediction is supported by the gas phase basicity of the amines and amine acid/base reactions conducted in an aprotic solvent. Reduced hydrogen bonding caused by the steric hindrance of alkyl groups supports a relationship of ammonia > l' > Z" > 3..
Differences in hydrogen bonding capacity are demonstrated by

A. K > 1 in both water and gas. B. K> I in water; K< I in gas. C. K< 1 in water; K> I in gas. D. K < 1 in both water and gas.
4.
Which of the following amines is the MOST basic?

the increasing sharpness of the N-H absorbance seen in


infrared spectroscopy. The sharpest peak is observed with the secondary amine and gets progressively more broad with

primary amines and ammonia. The two opposing trends compete to give the observed trend in water. Table 1 shows
the pKu values for some alkyl ammonium chloride salts.

A. B. C. D.
5.

NH3 H3CNH2 (H3C)2NH (H3C)3N

Compound
NHaCI (H3C)3NHCl

PKa
9.26 9.79
10.65
10.71

Charged compounds are found to be more soluble in

water than neutral compounds. At which of the following pH values would a tertiary amine be MOST
soluble in water?

H3CNH3CI H3CH2CNH3CI
(H3C)2NH2Cl (H3CH2C)3NHC| (H3CH2C)2NH2C1

A. B.

r0.73
10.75 10.95
1

D.
6.

c.9

5
11

Table

The pK6 for the conjugate base of any given acid can be tbund by using the equation pKb = 14 - pKu. This can be rseful for determining whether a compound is charged or

The pKa for carboxylic acids is between 2 and 5. In a pH = 7 buffered aqueous solution, lysine exists in which of the following forms?

A'aB.o
HzN

uncharged at a given pH value in an aqueous solution. Figure 1 shows the structure of a diamine at varied pH.

HzN
OH

\./^ / E, -N\'t

HH
o NH'
s,

H
I

H
C.

(CH.),
I

-N\,-

NH'

pH = 8.8

E.-NVNH,
pH = 12.8

NHz

"
D.
H3N1

(:"'lo
*NH:

pH = 10.8

Figure 1 Diamine at varying pH values


1

. In the gas phase, which of the following


cations is the MOST acidic?

ammonium

"'*.#o
H
(CHr)a

o-

A. B. C. D.
2

NH4+ H3CH2CNH3+ (H3CH2C)2NH2+ (H3CH2C)3NH+

l-'

(CH"), *NHr
I

NHz

7-

Why is there no mention in the passage of the broadness of the IR absorbance of an N-H bond of a
tertiary amine?

What is the approximare pK6 of CF3CH2NH2?

A. pKu > 10.71 B. 10.71 > pK6 2 7


C.1>pK6)3.29

D.

A. B. C. D.
205

Steric hindrance hinders N-H bond stretching. There is no N-H bond on a tertiary amine. Resonance shifts the absorbance to 3000 cm-I.

Symmetric molecules exhibit no IR absorbances.

3.29 > pKu

Copyright O by The Berkeley Review@

GO ON TO THE NEXT PAGE.

Passage

ll

(Questions B - 14)

10.

Amines are classified as weak bases, compounds that partially hydrolyze water to give OH- and its conjugate acid. Amines are one of the most common types of organic bases. The relative basicity of various amines can be predicted by comparing the substituents bonded to the nitrogen of the amine. Electron donating groups, when bonded to nitrogen, increase the basicity of the amine. This is to say that the basicity is directly dependent on the groups attached to the

To separate an amine from an amide, it would be best to use which of the following for extraction?

. Ether and water at pH = 3 B . Ether and water at pH = 7 C . Ether and water at pH = 11 D. Amides cannot be separated from amines.
A
1

1.

amine. Amines obey standard rules for aqueous phase acidbase chemistry. Reaction 1 is a proton transfer reaction involving a primary ammonium cation and ammonia. The
equilibrium for Reaction 1 as written lies to the left. H3CNH3+ + NH3

Which of the following statements is true about rhe relative basicity of amines compared to amides?

A. B.
C

Reaction

*H3CNH2 I

+ NH4+

Amines are more basic than amides, because amide: have their lone pair tied up in resonance. Amines are less basic than amides, because amides have their lone pair tied up in resonance. Amines are more basic than amides, because amides
are more sterically hindered.

The equilibrium constant for Reaction I is less than 1, because the methyl amine (on the product side) is a stronger base than ammonia (on the reactant side). The equilibrium
thus favors the protonation of methyl amine. By knowing the direction that an equilibrium lies, it is possible to determine the relative basicity of two compounds both

D.

Amines are less basic than amides, because amides


are more stericallv hindered.

io!

flh

12. The highest

boiling point is associated with which

mm

c,f

the following amine compounds?

m
,md

qualitatively and quantitatively. The pKu value for an acid (or the pK6 value for a base) can be derived from the equilibrium constant (K.O) for the reaction and the pKu of the other acid. Table 1 shows the equilibrium constants for a series of acidbase reactions with ammonium, NH4*. All reactions were carried out at 35'C.
Reactant Base

A. B. C. D.

NH3 H3CNH2 H3CCH2CH2NH2 (H3C)3N

rffi

K"O for reaction with NH4+ 1.00 25.7

13. The pKa of trimethyl ammonium

chloride-

NH:
H3CNH2 H3CH2CNH2 (H3C)2NH (H3CH2C)2NH (H3C)3N (H3CH2C)3N

29.s
30.9
50.1

(H3C)3NHCI, is 9.8. Benzene is electron withdrau'ins by resonance when the substituent has a lone pair crf electrons, so the pK6 of aniline, H5C6NH2, CANNOT be which of the following?

A. B.

3.6 4.8

3.55

D.

c.

9.2
10.4

32.4

Table I
8.
Which of the following compounds is the MOST basic?

14. Which of

the following changes to a primary arnirc will NOT result in a compound that yields a lower f,*q
Replace fluorine. Replace nitrogen Replace
a hydrogen on the carbon backbone

A. B. C. D.
9.

when undergoing proton transfer with ammonium?

H3CNH2

A. B.
C

H3CCH2NH2
(H3C)2NH

uiffi

(H3CCHz)zNH

. .

the hydrogens on the carbon bonded with a double bond to oxygen. one of the hydrogens on nitrogen wir. second alkyl group.

Which of the following reactions has a K"O of 0.030?

Replace the
hydrogen.

alkyl group on nitrogen wi*

A.

(I{3C)3NH+ +

NFI3 . ^

GIIC):N + NH4*
(HjC)3NH+ + N}I3
(I{3C)2NH + NFI4*

TC. GI jC)2NH2+ + NHj -iD. (I{3C)2NH+N}Ia+ . ^


(I{3C)3N + NFI4*

B.

(H3C)2NH2++N}Ij

Copyright O by The Berkeley Review@

206

GO ON TO THE NEXT PA

Passage

lll

(Questions 15 - 20)

6.

The IR absorbance for an N-H bond is observed at


approximately:

The nucleophilicity of an amine closely parallels its


basicity, where as the base strength of an amine increases, its nucleophilicity also increases. This holds true as a general trend with most deviations attributed to steric factors. Table I shows the pK5 and log K.O for an Syg2-reaction with ethyl chloride, H3CCH2CI, for a series of amines.

A. 3200 cm-1. B. 2700 cm-|. C. 1600 cm-l. D. 1300 cm-1.


17.
Which of the following statements BEST explains why
methyl amine is a stronger nucleophile than ammonia?

Amine NH:
H3CNH2 (H3C)2NH (H3C)3N

pKn
4.7
3.4 3.2

log K.
1.8

2.6
3.1

A. B.
C

The methyl group is electron withdrawing by the


inductive effect, making nitrogen electron poor.

4.2

0.6

The methyl group is electron donating by the


inductive effect, making nitrogen electron rich.

Table
are capable

The methyl group is electron withdrawing by


resonance, making nitrogen electron poor.

One problem with amines as nucleophiles is that they of undergoing multiple additions. It is difficult to isolate primary amines. To do so, an excess of amine is used in the reaction. When there is excess electrophile, the reaction proceeds readily to secondary and tertiary amines. To synthesize a primary amine, methods other than the reaction of ammonia with an alkyl halide are employed. Figure 1 shows alternative routes for synthesizing a primary amine.

D. The methyl group is electron donating


resonance, making nitrogen electron rich.

by

18.

Methyl amine, when added to (R)-2,chloroburane, would give which of the following major organic products?

Method 1: Gabriel Phthalimide Synthesis

r.

o 19.

A. B. C. D.

1-butene

2-butene

(R)-2-butyl methyl amine


(S)-2-butyl methyl amine

Which of the following labeled nitrogen atoms is the


MOST nucleophilic?

NHr#RNH2
3. RBr 4. NaOF(aq)/A
Method 2:
RC=N
RCH=NH

.N. $-H
b
a

Reduction of Nitrogen Compounds


Hr(e)/cat.
RCH2NH2

NHz

HzN

H2(g)/Ni
C2H5OH RCH2NH2

o
lt

LiAlH4

A. B. C. D.
2

Nitrogen Nitrogen Nitrogen Nitrogen

b
c

RC- NH2
RCH2N3

- -.-.-+ HoNNHT
Pd/cH3oH
RCH2NH2

ether

RCH2NH2

0. Methyl amine can react with all of the following


molecules EXCEPT:

Figure
1

A.

Synthetic routes to form a primary amine

B.

5.

Reduction of all of the following types of compounds will yield a primary amine, EXCEPT;

A. B. C. D.

",.A.,
C.

",.4o.",
D.

a nitrile.
a

primary imine.

H3C-

Cl

asecondaryamide. an oxime

nrc/o

at.

Copyright @ by The Berkeley Review@

207

GO ON TO THE NEXT PAGE.

Passage

tV (euestions 21 - 27)

21. Which of the following

primary amine, and a secondary amine respectively.

neutral imine. Secondary amines instead form compounds known as enamine.r, rather than form a cationic imine. Figure I shows the reactions of a ketone with ammonia. a
H

The reaction proceeds with both primary and secondary amines (in addition to ammonia), but the secondary amine cannot form a

constant close to one, so the reaction is driven by either the addition of excess reactant or the removal of a product.

hour for most amines. The reaction has an equilibrium

When ketones are exposed to amines (such as ammonia or alkyl amines), they can be converted into an imine and water. In the presence of a small amount of acid, conversion of roughly 99Vo of the ketone into amine occurs in about an

canied out via the enamine synthesis route? A . 3-pentanone into 2-methyl_3_pentanone B. Butanal into 2-ethylhexanal C. 3-hexanone into 3-heptanone D. Acetone into 2-octanone

conversions CANNOT

b'e

22.

What is the product for the following reaction?

"\)
A.
HjCH2CN

H2NCH2Cn3

4.

B.
HjCH2CHN

oA*,
o

+ NH3

=L
R'

N/

+ HoO
R"

C.

N-R
+ RNH2

HN

*A*,

=5

.A*',
*-R-o

t'o
23.
What is the product for the following reaction whe:
carried out under acidic conditions?

*,4*

+R2NH

.A*,,*n'o

Figure

Ketone to imine conversions

","{o
B.

Hzo

The ketone can be regenerated by adding water to the imine, as shown by the reverse reaction of the equilibrium. The imine cation formed from the addition of a secondary amine to a ketone can undergo the following synthetically
useful reaction (Figure 2).

*-R-*

*-*-*
oHI

.AcH2R,
*-*-*
--_l>

oA.r*'
*-R- *
R''-X

*Ao"*
R-.@N R

*A.n*,
I

24

What is the product of the fbllowing reaction?

R''
H+raor

11 nAcH*,
I

=+

*A.n*
I

R" Figure

R"

Ketone synthesis via an enamine intermediate


208

. A ketone and a primary amine. B. A ketone and a secondary amine. C . An aldehyde an<l a primary amine. D . An aldehyde and a secondary amine.
A

+"{

d>

Copyright @ by The Berkeley Review@

GO ON TO THE NEXT PAGE.

25. To convert 3-pentanone into 4-methyl-3-heptanone,


which of the following sequences ofreagents should be
used?

Passage

(Questions 28 - 35)

A. 1. R3NH+ 2. H3CCH2I 3. H3O+(aq) B. 1. R2NH/H+ 2.H3CCH2I 3. H3O+(aq) C . l. R3NH+ 2. H3CCH2CH2I 3. H3O+(aq) D. 1. R2NHIH+ 2. H3CCH2CH2I 3. H3O+(aq)

There are many naturally occurring amino acids (formally known as 2-aminocarboxylic acids) that can be isolated from

biological systems. Amino acids vary only in the alkyl


group attached to carbon

2.

Our body can synthesize some

amino acids, but others must be consumed in our diet. Amino acids can be synthesized in vitro using one of three

26. Treatment

of an imine with a reducing agent is similar to treatment of a ketone with a reducing agent. What is the product formed when the imine produced from the reaction of a ketone and primary amine is reduced using
A primary amide A primary amine
Asecondaryamine A tertiary amine

common methods. The first method involves the substitution of the amino group for a bromine functional group on the carbon 2 of an alpha bromo carboxylic acid. This method is known as the Hell-Volhard-Zelinskii synthesis, and it is shown in Figure 1 below.

***,ll

oo

LiAlHa in ether?

-E?- *lAon
t. ll
Br

Br,/PBr,

A. B. C. D.

.+*4*
oo
Br

6NH:*

+ NHaBr

Figure
2

Hell-Volhard-Zelinskii synthesis of an amino acid

7.

What are the products formed when N-ethyl-3-pentimine


is treated with acidic water?

A. B. C. D.

Ethanol and 3-pentanone.

Ethyl amine and 3-pentanone.


Ammonia, ethanol, and 3-pentanone.

Diethyl ether, ammonia, and 3-pentanone.

The second method for synthesizing amino acids in vitro involves the addition of a 2-bromodiester to a dicarboxylic imide (Gabriel's phthalimide). The product of this reaction is then heated in the presence of strong acid in water, which results in hydrolysis of both ester sites to form a diacid. The dicarboxylic imide is completely hydrolyzed under these

conditions, to yield an alkyl ammonium cation. After


enough time at an elevated temperature, the diacid undergoes

decarboxylation to form a monoacid, which in this case is an amino acid. This method is known as the Gabriel synthesis and it is drawn in Figure 2 below.

ot*B'ovou, *os]"::
HrN*{

\n"
o

Fo"

HgN*{

\o"
2

-*

H*(aq) A

HTN+J

\.,"

Fo" o
Figure
Copyright @ by The Berkeley Review@
209

Gabriel synthesis of an amino acid

GO ON TO THE NEXT PAGE.

a;ri

of a nitrile anion (C=N-) to the imine under acidic conditions. The product is then treated with strong aqueous acid at high [emperature to hydrolyze the cyano group into a carboxylic

The last method involves the addition of ammonia to an aidehvde to form an imine. This is followed by the addition

31. What is the side product formed during the Gabriel


synthesis of valine?

A.

B.

m
Mh

functional group. This synthesis method is known

as

rh= Srrecker synthesis and is drawn in Figure 3 below.

e(-'
o c

lm
fim ]{n[

m
CN

1. NH3

*A'
Figure

2. NaCN/HCN 3. ll3O+(aq) /A

3 Strecker synthesis

of an amino acid
D_

:8. In infrared spectroscopy,

rm
OH OH

rror
mffi

TMfi

CN

the O-H absorbances for

serine appear at which of the fbllowing values?

. B. C. D.
A

3200 cm- I and 3400 cm-l 2700 cm-l and 3400 cm-l 1700 cm-l and 3400 cm-l 1700 cm-i and 2i00 cm-l

32. How
A

many units of unsaturation are there in an amino acid with an aliphatic side chain?

nfrr

tu

dilr

. 0 units of unsaturalion B. 1 unit of unsaturation C . 3 units of unsaturation D . 4 units of unsaturation


33. How many signals do you expect in the 1HNUR
spectrum for L-cysteine, with a side chain of CH2SHobtained in a solution with deuterochloroform solvenr?

19. \\-hich of the following sequence of reagents would


1,

ield phenylalanine when added to 2-phenylethanal?

o H

2-phenylethanal phenylalanine Phenylalanine \ . 1. CrO3/H+(aq) 2. NH3 B. 1. NaCN 2. H+(aq)/A C. 1. NH3 2. NaCN 3. H+(aq)/A D. 1. 2-aminoerhanal 2. H+(aq)lA,
The following 1HNUR was run in CDCI3 on an amino acid synthesizedby the Gabriel method. Which of the

NHc

4.2 B. 3 D.
5

c. 4
34. The pKu for the side chain of histidine in aqueous medium is 6.1. What is true of its pKu in a
hydrophobic environment?

A. It is the same, because log functions cannot be


altered.

tollowing amino acids could it be?

B. It would be increased, because C.

the proton leaves

more readily to make the species negativell


charged.

more readily to make the species neutral. It would be decreased, because the proton leaves

D. It would

be decreased, because the proton lear-e* more readily to make the species neutral.

35. Which of the following amino acids, starting from


titrated by a strong base?

im

fully protonated state, has three equivalence points whea

7654321

-{. Glycine (R = H) B, Alanine (R = CH3) C. Serine (R = CHzOH) D. Phenylalanine (R = CH2C6H5)


C-;',
1-q51 @

A. Glycine B. Alanine C. Leucine D. Tyrosine

ft! lltrL

G.
GO ON TO THE NEXT PA

by The Berkeley Review@

Passage

Vl

(Questions 36 - 43)

7.

In adult humans, there are nine essential amino acids. The term essential is applied to amino acids that humans :nust obtain through diet, because they are not synthesized in :he cytosol. The essential amino acids include the aromatic rmino acids phenylalanine, tryptophan, and histidine, and
.ome hydrophobic amino acids like valine, leucine, and .soleucine. The other three essential amino acids are rethionine, threonine, and arginine. Of the essential amino
:cids, only histidine, threonine and arginine are hydrophilic. The water-solubility of histidine increases at a low enough

Removing a polypeptide from a lipid environment and placing it in an aqueous environment would:

A.

alter its primary structure by cleaving covalent


bonds.

B. alter its primary structure by cleaving hydrogen


bonds.

C. alter its secondary structure by cleaving covalent


bonds.

D. alter its secondary structure by cleaving hydrogen


bonds.

:H to protonate the side chain and form a cation. In neutral water, the carboxylic acid functional group is rble to deprotonate and become anionic, while the amino :unctional group gets protonated and becomes cationic. The rmino and carboxyl terminals of all amino acids are charged .r an aqueous environment at or near a pH of 7, causing most .mino acids to exist as zwitterions in neutral aqueous lolution. A zwitterion is the form of the amino acid in
.ites on it. The exceptions to the zwitterion at pH = 7 rule rre the highly acidic and highly basic side chains.
:,hich the overall charge is zero, but the molecule has charged

8.

Before determining the amino acid sequence of a large protein, an enzyme is often added, so that the protein is:

A. B.
C

denatured and turned into its straight chain form. cleaved at selected amide bonds and broken down into smaller fragments.

. .

in the reverse order and then transcripted. sliced into a cross-section and mounted on a slide for a better view of the component amino acids.
coupled with an identical protein

All naturally occurring


,-hirality in

amino acids have the

same

9.

Threonine, shown below, has what chirality?

The natural form for humans is the L.orm, which is a result of an S chiral center at carbon number :"r'o of the amino acid, with the exception of cysteine. Carbon number two is often referred to as the alpha carbon. Frgure 1 shows the generic structure for a typical L-amino rcid where the R refers to the side chain of the amino acid:

vivo.

HOH

H"N

H:N\ amino terminal


protonaled

HR
Figure

/z

-ZU\--_ 'O-

',' ll .^ff:1",!'iffi1"",

A. 25, 35 B. 2R, 35 C. 25, 3R D. 2R, 3R


4

L-Amino acid

0. At a pH of 7.4, the structure of phenylalanine


described as:

is BEST

Generic Amino Acid in Water

Six of the essential amino acids have a hydrophobic side :hain. A hydrophobic side chain reduces the water-solubility rf the amino acid. The amino acid still exists as a zwitterion rn water, as long as the pH is within two units of the amino acid's isoelectric point. The isoelectric point, pI, can be rcund by averaging the two pKa values for the amino acid that include the zwitterion in their reaction. For instance, the pI of glycine is found by averaging pKul and pKu2, because

A. B. C. D.
41.

uncharged.

an anion. a cation.
a zwitterion.

pKul involves zwitterion formation


zwitterion consumption.
3

and pKu2 involves

What pH would result if you were to mix 10 mL of fully protonated alanine with l0 mL of NaOH(aq) solution, where the concentration of the base solution is
1.5 times greater than the concentration of the alanine solution?

6.

There is at least one essential amino acid with each of


the following properties EXCEPT:

L. B.

2.35 6.11

A. B. C. D.

an aromatic ring.
a

hydrophilic side chain.

D.

c. 7.00
9.87

an R-steroecenter at the cr-carbon. a cationic side chain at pH = 6.

Copyright @ by The Berkeley Review@

211

GO ON TO THE NEXT PAGE.

2.

What is the isoelectric point for isoleucine?

Passage

Vll

(Questions 44 - 51)

PKal =

2.3

pKa2 = 9.7

A. 2.4 B. 6.0 c.7.0 D. 9.1


3.

An amino acid is given its name based on its having both an amine group and a carboxylic acid group. The
generic structure for a natural (chirally correct) amino acid drawn in Figure I below.
i*s

HzN\a/t\o,
4

o ll

Which of the following amino acids would generate the following titration curve when titrated with 0.10 M
NaOH(aq) starting from its fully protonated state?

HR
Figure

lu
Generic amino acid

The R represents various groups, each one corresponding

to a specific amino acid. This is to say that all amino acids are the same with the exception of the R group. The generic amino acid in Figure 1 is shown as it exists in a hydrophobir

environment. This is not how

it

exists

in an aqueous
aqueou_r

environment, however. In water, the amino terminal (amine group) is protonated and the carboxyl terminal (carboxylii

acid group) is deprotonated. Figure 2 shows the


form of a generic amino acid.

t,*-"-"-o1.0 A. Phenylalanine B. Glycine C. Lysine D. Aspartic acid 2.0


3.0 added

lt

HR
Figure

lu
anC

Equivalents of 0.10 M NaOH(aq)

Amino acid in aqueous environment

The pK6 values for the protonated amino terminal

the carboxylic acid group are approximately 9.5 and respectively. At physiological pH of '1.4, the pKu of

2.5 the

amino terminal is greater than the pH, so it is protonated br, the solution according to the Henderson-Hasselbalch equation_ At physiological pH, the pKu of the carboxyl terminal is iess than the pH, so it is deprotonated by the solution. Equatiom 1 is the Henderson-Hasselbalch equation.
PH = PKa +

log [A-]
IHA]

Equation 1 Henderson-Hasselbalch Equation


Using the Henderson-Hasselbalch equation, it is possible to calculate the ratio of the deprotonated form to protonateJ form for each site on an amino acid at a specific pH.

44.

Which of the following changes to n-propanol woulii NOT result in a lower pKu than n-propanol?

A. B. C. D.
Copyright @ by The Berkeley Review@
212

Oxidizing carbon
hydrogen.

n-bond to oxygen instead

by four electrons, resulting in a

of two o-bonds

tct

Replacing the oxygen with a sulfur atom. Replacing the propyl group with a phenyl ring.

Replacing the hydroxyl group with an amine


group.

GO ON TO THE NEXT PAGL

What charge does the following dipeptide cany at a pH

9.

of 5.0?

o
PKa=3'e

How many times must the fully protonated form of glutamic acid be deprotonated to form monosodium glutamate? (For glutamic acid R = CHZCHZCOZH)

W.-\
N
PKa = 2'0

A. 0 times B. I time C. 2 times D. 3 times

H (cHt4H
+ftHt PKu = lo'8
I

50. At

A. -1 B. 0
C. D.
+1

what pH value will the ratio of the +2 to of the following molecule be 10J

+l

forms

+2

V\*"=u,
I

\\.

When the pH of solution is greater than the pKu by l, what is the ratio of the deprotonated form to the
protonated form?

PKa= 9.2 + HgN. N


I

A. 1:10 B. 1:2 C.2:1 D. 10:


1

OH

PKo

1.3

(CHn),

*l

'r

NH3 PKo =

10.8

How do the three labeled protons on the following


molecule rank in terms of relative acidity?

HiN\

c+

A. 3.0 8.5.r c. 1.1


D.
8.2

c/ ll.H

c\
o*1
b

ll

CF2OH

51. Given the pKu values for each of the following


compounds, which of the compounds is the most basic?

A. Proton a > Proton c > Proton b B. Proton b > Proton a > Proton c C. Proton b > Proton c > Proton a D. Proton c > Proton a > Proton b
What can be said about the pKu of a hydrogen on sulfur, given that cysteine exists predominantly in the following form at pH = 7.4? Note that the hydrogen on sulfur is the second proton to be lost when cysteine is
deprotonated.

PKa

= 10'0

PKa = 8'4

PKa = 5'0

u'ry*"'
c.
o
D.

.ll HrN\a/t\n-

l\ H
A. B. C. D.
Irpyright
@

CH2SH

It is less than 2.5. It is greater than 2.5 but less than 7.0. It is greater than 7.0 but less than 9.5. It is greater than 9.5.
by The Berkeley Review@

oA

ry"

GO ON TO THE NEXT PAGE.

Passage

Vlit (euestions 52 - 58)

53. Hydroxyproline, shown below, is found in


beings, but it is not coded for in DNA.

humar:

naturally occurring polymers known as proteins. The human body codes for twenty amino acids. These twenty make up the majority of the amino acids found in enzymes, protein hormones, and other polypeptides. polypeptides are formed when amino acids link their respective amino and carboxyl terminals. Table 1 lists the twenty amino acids that have
base codons associated

Amino acids are the biological building blocks of

(,!",
What feature is NOT associated with hydroxyproline? A. Hydroxyproline has more than one chiral center. B. Hydroxyproline induces deviations in the seconda,-, structure of a protein. C . Hydroxyproline is hydrophilic. D . Hydroxyproline has a side chain pKu around 4.0.

\-**n

N-ot

with them:

Amino acid
Alanine

Code
A1a

R-Group CH:
(CH2)3NHC(NHz)z+

PKas
2.3/9.7 2.2/9.5/13.0 2.1/3.9t9.8 2.0/8.9

Arginine
Aspartic acid Asparagine Cysteine

Arg Asp Asn

CH2CO2
CH2CONH2 CH2SH

cyt
Glv
Glu Gln His Ile
Leu

r.8/8.4/10.6
2.3/9.6

Glycine Glutamic Acid Glutamine Histidine


Isoleucine Leucine

H
CH2CH2CO2CH2CH2CONH2 CH2C3H3N2 CH(CH3)CH2CH3 CH2CH(CH3)2

54. Which of the following peptide fragments has thr


HIGHEST isoelecrric pH?

2.U4.3t9.8
2.2/9.1

t.8t6.1t9.2
2.4/9.7 2.4/9.6 2.2/9.2/10.8 2.3/9.3

A. Met-Lys-Arg B. Gly-Phe-Ala C. Tyr-Asp-Glu D. Leu-Val-Ile


55. Which of these amino acids contains exactly one chi:ruI
center?

,t

Lysine Methionine
Phenylalanine

Lys
Met
Phe

(CH2)aNH:+
CH2CH2SCH3 CH2C6H5

I il

t.8/9.2
2.0t9."7

Proline
Serine

Pro
Ser

-CH2CH2CH2CH2OH

2.2t9.2

Threonine

Thr Trp

CH(CH3)oH
CH2CsH6N CH2C6HaOH CH(CH3)2

2.6/r0.3
2.4/9.4 2.1/9.1t10.1 2.3/9.6

Tryptophan
Tyrosine

A. Glycine B. Histidine C. Isoleucine D. Threonine


56. Which of
these dipeptides requires the lowest

&

rp

m
soluim

Tyr
Val

Valine

Table I
Each amino acid can be distinguished from the others by the R-group attached to the alpha carbon, known as its side chain. The R-groups in Table I are shown as they exist at pH = 7.4. Figure 1 shows the structure for a generic amino acid with a generic R-group.

pH to exist 100% as a zwitterion?

A. Leu-His B. Lys-Asn C. Asp-Cys D. Tyr-Arg


At pH = 7.0, which of the following amino acids u migrate to the cathode in a buffered electrophoresis gai"

.ll
"t*-a/"
Figure

o o

l\. HR

A. Alanine B. Aspartic C. Leucine D. Lysine


a

acid

Generic amino acid with natural stereochemistry

52. Which of the following polypeptides will carry

A. B. C. D.

positive one charge when placed in gastric acid (an aqueous solution buffered at pH = 1.5)?
Lys-Leu-Ile Arg-Cys-His Asp-Glu-Asn Glu-His-Phe

58. An isoelectric point less than 5.0 indicates


amino acid has what type of side chain?

thar

. An acidic side chain. B. A basic side chain. C. A hydrophilic side chain. D. A hydrophobic side chain.
A

Copyright @ by The Berkeley Review@

2t4

GO ON TO THE NEXT P.{

Passage

lX

(Questions 59 - 66)

61.

The GREATEST isoelectric pH is found with:

There are twenty amino acids for which human DNA ;odes. These twenty amino acids form proteins within the iuman body. The physical properties of the twenty amino
:cids are listed in Table

l,

shown below:

A. alanine. B. glutamic acid. C. histidine. D. phenylalanine.


62.
Which of the following structures represents the most common form of cysteine in a pH = 7.0 solution?

\mino Decomp Water Sol.


.\cid
G1

Temp 233"C 298"C

@ 25'C

- -rq

(8/too mI-) 25.2


16.5

t0lil

PKal
z.-'\a

PKa2 PKa3
9.78

\la
\/ai
Leu Ile

+8.5 +13.9
-10.8
+1 1.3 -8.2

2.35

9.81 9.12 9.74 9.74


9.21
10.64

A.
HzN

B.
HzN

315'C 293'C

8.9

2.29
2.33

2.4
4.1

284'C
280"C

2.32
2.11
1.95

CH2SH

cH2s

\Iet
Pro ?he T
Ser

3.4

220'C
283"C 289"C

t62.0
3.1

-85.0
-35.1
-3 1.5

C.

2.58 2.43

9.24 9.44 9.46 9.10


8.36
10.33 10.07

t.l
5.0 verv sol.
0.05
3.5
'\- l

228'C
225"C 342"C 234"C 185"C 210"C

-6.8 -28.3

2.19
2.09
1.88 2.21

H
63.

CH2SH

CHoS

Thr - \'s

+6.5 -10.6
-5.4

\I
-\ Sn

9.12
8.80

Arginine has a lower pKui than alanine, because the


carboxyl terminal of alanine is:

2.02
2.11

lln
j.sp

+6.1

9.t3
3.88
10.01

A.
9.9s
10.80
13.21
9. 15

0.6
0.9

+25.0 +31.4 +14.6 +12.5


-39.1
1

r.99
2.13

llu
_\S
lao

241'C

4.32 9.20
8.99 6.05

B.
C

225'C 244'C
281"C

very sol.
15.0

2.16

t.82
1.81

:l is

4.2

D.

Table

more acidic than the carboxyl terminal of arginine, due to the inductive effect of the anionic side chain. less acidic than the carboxyl terminal of arginine, due to the inductive effect of the cationic side chain. more acidic than the carboxyl terminal of arginine, due to the inductive effect of the amino terminal. less acidic than the carboxyl terminal of arginine, due to the inductive effect of the amino terminal.

'.

Amino acids are distinguished by the side chain attached :he alpha carbon. Within a linear protein, there is one ,. ::no terminal and one carboxyl terminal, so the side chains r,-iount fbr the behavior of each amino acid within a peptide.

64.

Tyrosine best separates from cysteine at

pH equal to:

5:i, tyhi"h of the following side chains would MOST LIKELY be found in the hydrophilic pocket of a
protein?

A. B. C. D.

the pKa (Cys), when using ether/water extraction. the pKa (Tyr), when using ether/water extraction. 7.0, when using ether/water extraction.

either

plg6

or

pltyr, when using ether/water

extraction.

A. -CH2CH2SCH3 B. C. -CH2C6HaOH D. -CH2CH2CONH2


-CH2SH

65. At pH = 4.0, what is true for histidine?


A

. B. C. D.

Most Over Most Over

exists with the carboxyl terminal protonated.


9070 exists as a zwitterion.

exists with the amino terminal deprotonated. l}Vo of the side chains are protonated.

r i,

How can the fact that lysine is more water soluble than
leucine best be explained?

66. Which of

B. C. D.
-

The side chain of lysine can form hydrogen bonds, while the side chain of leucine cannot. The side chain ofleucine can form hydrogen bonds, while the side chain of lysine cannot. The side chain oflysine is non-polar. The side chain ofleucine is polar.
215

the following amino acids would bind to column filled with DEAE-cellulose at a pH of 6.20? Alanine Glutamic acid Lysine Tyrosine

A. B. C. D.

:i"right O by The Berkeley Review@

GO ON TO THE NEXT PAGE.

Passage

(Questions 67 - 73)

68. In PTH, amino acids #1 through #34 make up the


through #43 can best be described as:

of the three protein hormones involved in mineral homeostasis. PTH is initially synthesized as a 115-amino
acid inactive polypeptide preprohormone. preprohornones

Parathyroid hormone (pTH) and calcitonin (CT) are two

active portion. Amino acids #35 through #94 protecx the structure from proteolysis. Amino acids #3-s

are later activated by removal of specific parts of their primary structure. The primary structure of parathyroid
hormone is shown below:

A. hydrophilic. B. hydrophobic. C. polar and anionic. D . polar and cationic.

9. In the endoplasmic

reticulum, prepro-pTH loses tu.c, methionine residues and a 23-amino acid peptide to form pro-PTH. Pro-PTH is transferred into the Golgi region where it is converted into PTH. What occurs in the Golgi region?

. Gain of an octapeptide . Gain of a hexapeptide C. Loss ofan octapeptide D. Loss ofa hexapeptide
A
B

70.

and

Chymotrypsin cleaves at the C-terminus of phe, TrgTyr. What fragments form when human pTH x

treated with chymotrypsin?

A
B

. .

Three fragments total, of length 22 amino acids, i ! amino acids, and 5l amino acids. Three fragments total, of length 27 amino acids. tr . amino acids, and46 amino acids.

C. Four fragments
D

total, of length 27 amino acids.

amino acids, 16 amino acids, and 32 amino acids. Three fragments total, of length 23 amino acids. j amino acids, and 50 amino acids.

l
i.

Figure 1 Parathyroid Hormone

its carboxyl terminal in the form of an amide. Both calcitonin and PTH are found in several animals, but the primary sequence varies from species to species. The
reactivity is sometimes altered by the substitution of different amino acids in the polypeptide.
67

Calcitonin is a 32-amino acid peptide hormone that has

71.

The fact that Metg and Metl g of human pTH can Lr replaced with nor-leucine (R = -CH2CH 2CH2Cllz with no observed change in reactivity implies that:
t

In human PTH, amino acid #46 is alanine. In bovine PTH, amino acid #46 is glycine. What is true when
comparing human PTH with bovine pTH?

A. sulfur has no effect on reactivity. B. nitrogen has no effect on reactivity. C. sulfur has a significant effect on reactivity. D. nitrogen has a significant effect on reactivity.

B. C. D.

The secondary structure is more disrupted in bovine PTH, because of its bulkier side chain. The secondary structure is more disrupted in human PTH, because of its bulkier side chain. The tertiary structure is more disrupted in bovine PTH, because of its bulkier side chain. The tertiary structure is more disrupted in human PTH, because of its bulkier side chain.

2.

Analogs of human-PTH should:

be structurally diverse between amino acids


through #34 compared to human-pTH.

#l!
#fr

B.
C D

be structurally similar between amino acids


through #34 compared to human-pTH.

. .

have a different
human-PTH.

N terminus and C terminus

rlmn
*-

be structurally diverse between amino acids


through #84 compared to human-pTH.

Copyright @ by The Berkeley Review@

216

GO ON TO THE NEXT PA

73

What can be concluded from the following data?

Passage

Xl

(Questions 74 - 81)
sequence

Fragment 1-84

Vo

Activity in
I00Vo

vitro

Vo

Activity in vivo
l00Vo
32Vo

A researcher was interested in determining the

l-34
1 - 31

llVo
l0Vo
5Vo

of two polypeptide fragments she isolated from

partial

6lVo
0.3Vo

r -28

A. The active site probably contains B. The active site probably contains C. The active site probably contains D. The active site probably contains

amino acid # 17. amino acid # 30. amino acid # 66. amino acid # 83.

hydrolysis of an enzyme. The first fragment (Fragment I) is a six amino acid fragment which cannot be deciphered using molecular weight studies of the component amino acids. The primary structure of the second fragment (Fragment II) was determined using an amino acid sequencer. Fragment II has the following amino acid sequence from N-to-C terminals:

His-Ser-Val-Phe-Ile-Tyr-Phe.

An

automated protein

sequencer cleaves, isolates, and identifies the component amino acids one at a time from the amino terminal to the carboxyl terminal using phenylisothiocyanate. The researcher used the enzymes listed in the Table I for analysis.

Enzyme
Chymotrypsin Clostripain
Pepsin

Amino acid
Phe, Trp, Tyr

Cleavage side
Carboxyl Carboxyl Carboxvl

Arg
Phe, Trp, Tyr Asp, Glu, Leu Leu, Ile, Val

Thermolysin Trypsin

Amino
Carboxyl

Lys, Arg

Table I
The term "cleavage side" indicates the point at which the enzyme breaks the peptide bond. For instance, thermolysin cleaves the peptide bond on the amino (left) side of leucine,

isoleucine, or valine. It should be noted that the enzyme itself is a chain of amino acids, but it is present in such a small concentration that it does not cleave itself. The
enzyme concentration is low because it is a catalyst with a substantial turnover rate, where a "substantial" turnover rate is defined to be greater than 10O/second. Some small polypeptides can be analyzed according to

their isoelectric point or migration rate in an electrophoresis gel. If the isoelectric point of an amino acid is greater than the pH, then that amino acid carries a partial positive charge

and exhibits cationic behavior. Cations migrate to the


cathode in gel electrophoresis. Figure 1 shows histidine in its fully protonated state.

PKat

1'8

CHz
+\

pKo2

HO

PKaz= 9.2 Figure


7

H:N

{J

--

6.1

1 Fully protonated

structure of histidine

4.

What is the approximate isoelectric point of histidine?

A. B. D.
Copyright @ by The Berkeley Review@

4.0
6.1

c.'7.6
9.2

2t7

GO ON TO THE NEXT PAGE.

5. In what direction

would histidine migrate

if it were

L.

added to an electrophoresis gel buffered at pH = 7.0?

A. B. C. D.
7

It would migrate to the anode. It would migrate to the cathode. It would not migrate.
It would denature.

Treatment of Fragment I with chymotrypsin yields the following residues: Ser-Val-Phe and Gly-Cys-Gly. Treatment of the six amino acid Fragment I with
Sanger's reagent (4-dinitrofluorobenzene) binds serine. Which of the following is the sequence for Fragment I?

6. If treating

a polypeptide with z, -dinitrofluorobenzene followed by 6 M HCl(aq) results in an alanine residue bound to a dinitrobenzene moiety, then what can be

A. Gly-Cys-Gly-Phe-Val-Ser B. Gly-Cys-Gly-Ser-Val-Phe C. Ser-Val-Phe-Gly-Cys-Gly D. Phe-Va1-Ser-Gly-Cys-Gly

tr

iS

f,

ffi

concluded from this information?

. B.
A

The first amino acid in the polypeptide is alanine.

mfi

The second amino acid in the polypeptide is


alanine. The fifth amino acid in the polypeptide is alanine. The last amino acid in the polypeptide is alanine.

({ffi

. D.
C
77

ffi

flil

Treatment of Fragment

II with thermolysin would yield:

fri

A. B. C. D.
78.

fragment.

2 fragments.
3 fragments.

4 fragments.

What is the side group on rhe amino acid that weighs


89 grams/moles?

A. B. -H C. -CH2OH D. -CH3
-SCH3
7

9.

Which of the following polypeptides would be observed after Fragment II is treated with clostripain?

. B. C. D.
A
8

His-Ser-Va1-Phe-Ile-Tyr-Phe

His-Ser-Val-Phe Ile-Tyr-Phe His-Ser-Arg

0. If the separate treatment of a protein first with pepsin and then with thermolysin generated the same
polypeptides, then which

of the following

peptide

linkages is present in the protein?

A. B. C. D.

Ile-Phe Leu-Leu

Tyr-Val Val-Met

Copyright O by The Berkeley Review@

218

GO ON TO TIIE NBXT PA

Passage Xlt (Questions 82 - 88)

83. What is the role of


Experiment I?

2,4-d,initrofluorobenzene in

primary sequence of the protein by carrying out four experiments. In Experiment I, the full protein is treated with
2,4-dinitrofluorobenzene (2,4-DFNB) followed by hydrolysis

A protein isolated from the saliva of the Northeastern Radish Spider is found to break down muscle fiber in the Tuscaloosa Spitting Lizard. Researchers determined the

A. B.

DNFB serves to bind the first amino acid in the


protein. protein.

DNFB serves to bind the last amino acid in the following the

C. DNFB serves to cleave the protein


cysteine residues.

of the protein using an acid catalyst. The 2,4-DNFB is a strong electrophile that accepts electrons from the nitrogen of
the first amino acid, and in doing so labels it.

D.

DNFB serves to cleave any disulfide bridges present

in the protein.

The amino acids present in the protein are two alanines, two cysteines, and one each of arginine, glycine, histidine ifound to be the first amino acid), isoleucine, leucine, lysine, methionine, phenylalanine, serine, and tyrosine. Following the hydrolysis experiment, the researcher treated separate samples of the protein with the following reagents;

84

Based on the results of Experiments II, [I, and IV, what is the LAST amino acid in the protein?

Experiment

II:

The protein is first treated with

B_

mercaptoethanol (breaking any disulfide linkages) followed by treatment with trypsin yields these fragments:

A. Alanine B. Arginine C. Histidine D. Serine


85.
Based on the results of Experimenrs II, III, and of the following are valid conclusions EXCEpT:

N-Ser-Ile-Tyr-Ala-C N-His-Lys-C
N-Met-Cys-Leu-Gly-Ala-phe-Cys-Arg-C

IV, all

A.
B-

chymotrypsin cleaves on the carboxyl side of


tyrosine and phenylalanine.

Experiment

III:

The protein is first treated with

mercaptoethanol (breaking any disulfide linkages) followed by treatment with thermolysin yields these fragments:

. . .

C D

trypsin cleaves on the carboxyl side of arginine and lysine. thermolysin cleaves on the amino side of isoleucine
and leucine.

N-Ile-Tyr-Ala-C N-His-Lys-MetCys-C
N-Leu-Gly-Ala-Phe-Cys-Arg-Ser-C

thermolysin cleaves on the amino side ol cysteine


and alanine,

Experiment

IV:

The protein is first treated with

B-

nercaptoethanol (breaking any disulfide linkages) followed by treatment with chymotrypsin yields these fragments: N-Cys-Arg-Ser-Ile-Tyr-C
free alanine

86. Why is it
A

a more valid conclusion that thermolysin

cleaves before leucine and isoleucine, rather than after cysteine and serine?

N-His-Lys-Mer-Cys-Leu-Gly-Ala-phe-C

The researchers were able to use the experimental -:Ibrmation to determine the sequence of the amino acids in :i:e protein. Knowing the first amino acid in the sequence ".ong with any two of the experiments, the sequence can ";curately be deduced. All disulfide linkages must first be :roken in order to isolate the fragments in their correct
r-QUOnC9.

B.

Thermolysin cannot cleave after molecules involved in sulfide bridging. Because serine has a protic side chain, it cannot react with thermolysin.

C. There are two D.

cysteine residues present

protein, so if thermolysin cleaved after cysteine, an additional cut would have been observed.

in

the

There are two cysteine residues present


protein, so

if

in

the

thermolysin cleaved after cysteine,

one less cut would have been observed.

2.

What is the role of B-mercaptoethanol in the first part each experiment?

of

87.

From the information in Experiments

II

and

III,

how

B-Mercaptoethanol serves to bind the first amino acid in the protein. B. B-Mercaptoethanol serves to bind the last amino acid in the protein. C . B-Mercaptoethanol serves to cleave the protein

A.

much of the exact protein sequence is known?

following the cysteine residues. D

A. B. C. D.

It is known only to amino acid two. It is known only to amino acid four. It is known up to amino acid eleven.
The entire sequence of amino acids in the protein is

known.

B-Mercaptoethanol serves to cleave any disulfide bridges present in the protein.

Copyright @ by The Berkeley Review@

219

GO ON TO THE NEXT PAGE.

8.

Based on all of the experimental information, what is the primary sequence of the unknown protein?

Passage

Xlll

(Questions 89 - 96)

A
B C

. N-His-Lys-Met-Cys-Leu-Gly-AlaPhe-Cy s-Arg-S

. N-His-Lys-Met-Cys-Ile-Tyr-Ala-LeuGly-Ala-Phe-Cys-Arg-S
er-

er-Ile-Tyr-Ala-C.
C.
-

When amino acids are synthesized invitro, they form a: mixture of the D- and L-enantiomers. Since the Lenantiomer is desired, resolution is necessary. For resolving the mixture, one of three techniques can be applied.
a racemic

. N-I1e-Tyr-Ala-Leu-Gly-Ala-Phe-CysArg-S er-His-Ly s-Met-Cys


C. C.

. N-Leu-Gly-Ala-Phe-Cys-Arg-SerHis-Lys-Met-Cys-I1e-Tyr-Ala-

Technique l: The racemate is first treated with benzoyl chloride to form an amide at the amino terminal. The racemate of N-benzoyl-Dl-amino acid is treated with a brucine salt causing the D-amino acid to precipitate from solution. The solid is filtered from solution and is treated with acid to dissolve the brucine-amino acid salt into a ne\\' solution. Upon treatment with aqueous hydroxide, a pure sample of the D-amino acid is isolated. Technique 2: The racemate is first treated with benzol'l chloride followed by a strychnine salt, causing the L-amino acid to precipitate from solution. The solid is filtered from solution and is treated with acid to dissolve the strychnineamino acid salt into a new solution. Aqueous hydroxide is added to the new solution to isolate the L-amino acid.
Technique

3:

The racemate is treated with acetic anhydride

to form N-acetyl-Dl-amino acid. This racemic mixture of acylated amino acids is treated with hog renal acylase. which removes the acetyl group from the L-enantiomer.

forming the zwitterion. The D-enantiomer remains acylated at the N-terminal, making it anionic (due to the deprotonation of the C-terminal). The two species are
separated using ether-water extraction.

rNr

A major drawback to the first two techniques is that onl1 one enantiomer is isolated in pure form. The enantiomer tha; does not precipitate from solution is isolated after the solven is evaporated, which does not result in a pure sample. The second drawback is that the success of the techniques varies with different amino acids. For instance, they work weL

with alanine, but are difficult to use with tryptophan Technique 3 has the most universal application. Presenre; below is an experimental application of Technique 3:
Experiment: A suspension of I2.9 grams of N-acetyl-Dlisoleucine in 0.80 liters of water is buffered to 7.0 in an aqueous solution, following which 0.010g hog renal acylase powder is added. The mixture is stirred for l6 hours at 37.0'C (physiological temperature). The mixture is then acidified with 10.0 mL of concentrated acetic acidfiltered through a frit, and evaporated under vacuum to a volume of roughly 50 mL. Ethanol is added to form Lisoleucine crystals. The crude product is recrystallized frorn an ethanol-water mixture to yield roughly 3.5 grams ('71.|Vo yield) of optically pure L-isoleucine.
8

$3.

9.

Why must the first step in Technique 3 be done in al


aprotic solvent?

A. B. C. D.
Coprright
@

A protic solvent is too inert. A protic solvent can react with acetic anhydride.
To ensure that the N-terminal is protonated, To ensure that the C-terminal is deprotonated.

by The Berkeley Review@

220

GO ON TO THE NEXT PAGL

90.

A. B. C. D.

When the brucine salt is used to resolve a racemic mixture of alanine, what is likely to occur? L-alanine is isolated in relatively pure form from precipitate, while D_alanine is^ isolated with impurities from solution. L-alanine is isolated with impurities from precipitate, while D_alanine is isolated in .etatiueif

4, In the experiment, why is the solution stirred


sixteen hours?

for

A . To vaporize the pig renal acylase. B. To agitate the mixture and enhance reactivity. C. To _centrifuge the powder to the bottom of the
,

flask.

pure form from solution.

precipitate, while L_alanine is isolated with impurities from solution. D-alanine is isolated with impurities from precipitate, while L-alanine is isolatid in relativeiy

pure form from solution. D-alanine is isolated in relatively pure form from

D.

reaction rate.

To generate a suspension, and thus decrease the

95

Why is hydroxide anion used at the end of Technique

1?

To remove the benzoyl substituent from the Nterminus.

B. To remove the carboxyl


9

1.

. Separation of enantiomers from a racemic B . Racemization of one enantiomer. C . Reaction of a D-amino acid with a diacid. D. Precipitation of an enantiomeric salt.
A

Which procedure involves an increase in entropy? mixture.

C. D. To racemize the isolated

protons. To remove the cationic brucine salt.

and amino terminal

amino acid.

6.

What precipitates with strychnine in Technique 2?

A.
H3N1
9

B.

*/"
o

2.

Why is the racemate first treated with acetic anhvdride in


Technique 3?

A. B.

To protect the amino terminal.


acylase.

RH
H
I

To form a bond that will react with hog renal

C.
HrN* N

C. To make the amino acid cationic. D. To invert the chiral center.

93.

enantiomer. Why does this process become futile after


three cycles?

solution of the D-amino acid using 1.0 M HCI(aq). The D-enantiomer can be treated with racemase to generate a D-L mixture again. Technique 3 can be applied to this mixture once again to isolate more L_

In Technique 3, the filtrate can be treated with HCI(aq) to a pH of 2.0 to crystallize the N_acetyl_D_isoleucine. The crystals can be dissolved and hydrolyzed into

A. B. C. D.

The percent of the enantiomer obtained from the third cycle is only l2.5%o of the product mixture. The percent of the enantiomer obtained from the third cycle is only 6.25Vo of the product mixture.
renal acylase is consumed completely by the Ygg_ third cycle. The acetic anhydride reacts only when the mixture has a substantial amount of L_enantiomer.

Copyright @ by The Berkeley Review@

221

GO ON TO THE NEXT PAGE.

100. A1l of the following can be reduce to form


Questions 97 - 100 are NOT based on a descriptive passage

aminef,

EXCEPT.

97.

Which of the following graphs BEST represents the rate of the corresponding nucleophilic substitution reaction relative to the pH of the solution?

A. Amides with LiAlH4. B. Nitriles withHCIlZnClZ(aq). C. Imines with H2iPd. D . A11 of the above reactions yield amines.

H3CNH2

+ CF|CI

(l{3C)2NH2+Cl

^1
G

'1
6 z
s

z
.H

&

'1
6
H

--+

"1
G

.'GETINTOCHEM!"

&

pH+

8.

H3CNH2 and H3CCI would show which of


following proton NMR peak patterns? A. Singlet (3H), singlet (3H), septet (lH) B. Singlet (3H), singlet (3H), quartet (1H) C. Singlet (6H), sextet (1H) D. Doublet (6H), septet (lH)

Following neutralization, the product of a reaction of


the

9.

How many mL of 0.010 M NaOH(aq) need to be added to take 10 mL 0.010 M lysine from pH = 5.68 (the first
equivalence point) to pH = 10.80

(pK6 for lysine)?

A. B. C. D.

M NaOH(aq) 10 mL 0.10 M NaOH(aq) 15 mL 0.10 M NaOH(aq) 20 mL 0.10 M NaOH(aq)


5 mL 0.10

1.A 2.C D ',|. B ll. A 12. c 16. A r1. B 21. c 22. A 26. C 21. B 31. B 32. B 36. C 3',7. D 41. D 42. B 46. D 47. A 51. D 52. C 56. C 51. D 61. C 62. C 66. B 67. B 11. A 12. B 16. A 71. C 81. C 82. D 86. C 87. D 91. B 92. B 96. D g',t. C
6.

3.D 4.C 5.A 8.D 9.C l0.A 13. A 14. C 15. C 18. D 19. C 20. D 23. A 24. A 25. D 28. B 29. C 30. A 33. D 34. D 35. D 38. B 39. C 40. D 43. D 44. D 45. B 48. C 49. C 50. B 53. D 54. A 55. B 58. A 59. C 60. A 63. B 64. D 65. D 68. B 69. D 70. D 13. B 74. C 15. B 78. C 79. A 80. C 83. A 84. A 85. D 88. A 89. B 90. c 93. A 94. B 95. A 98. D 99. C 100. B

Copyright @ by The Berkeley Review@

Amines and Amino Acids passage Answers


to the passage, the most basic amine compound in the gas phase is the tertiary amine' The weakest -According gas phase is the base with the least electron donating base in the aliyl groups. This makes ammonia the weakest base. Because ammonia is the weakest base, ammonium .u.-tior-r'(th-e conjugate acid of ammonia) must be the strongest acid in the gas phase. This makes choice A correct.
Choice C is correct. If the pKu for CH3CH2NH g+ is 10.71., then the pK6 for CH3CH2NH2 must be 3.29. Because CH3CH2NH2 is more basic than CF3CH2NH 2, due to the electron wiihdrawirig ,-rut,rr" of fluorine, the pK6 for CF3CH2NH2 must be greater than 3.29. The pK5 for CF3CH2NH2 should be Jnly a point or two greater than the pK6 for CH3CH2NH2, making choice C coirect.
3.

Choice A is correct'

Choice D is correct. In both the gas phase and in water, primary amines are stronger bases than ammonia. This means that the stronger base is on the product side of the equation. A favorable ieaction has the stronger base reacting to yield the weaker base, which is the reverse reaction as written. This means that reactants are more abundant than products as written. This makes the equilibrium constant (Keq) less than one in both the gas phase and in water. The best answer is choice D.
1, the highest pKa is associated with the dimethylammonium cation, so it is ihe weakest acid. As an*acid gets weaker, its conjugate base becomes stronger, so ttre secondary amine is the strongest base. This is choice C.

Choice C is correct. The donating nature of methyl groups increase the basicity of an amine. According to Table

Choice A is correct- An amine of any substitution, tertiary included, will exist in its protonated state in solutions a pH value less than its pKu value. When it is protonated, it is cationic and thus more soluble in water than it is in its neutral state. This implies that as the pH of the solution is lowered, the amine becomes more soluble. The lowest pH of the answer choices is choice A, 3, so choice A is the best choice.

with

6.

pKu for the amino terminal and the amino side chain, so both the amino terminal and the amino side chain are protonated. When the amino terminal and the amino side chain are protonated, they each carry a positive charge. This eliminates choices B and C. The best answer is choice D.
7.

for the carboxyl terminal so the carboxyl terminal is deprotonated. When the clrboxyl terminal is deprotonated, it carries a negative charge. This eliminates choice A. The pH is seven, which is less than the

Choice D is correct- When the pH is less than the pKu, the site exists in its protonated state. When the pH is greater than the pKu, the site exists in its deprotonated state. The pH is seven, which is greater than the pKu

Choice B is correct. The passage discusses the N-H absorbance of a secondary amine, but does not mention a tertiary amine. This is because all of the hydrogens in a tertiary amine are bonded to carbon. The nitrogen has three bonds to carbon, so there is no N-H bond, making choice g the best answer. There is no resonance to speak of, given that there is no n-bond, so choice C is eliminaied. The tertiary amine may or may not be symmetric, but that is irrelevant, so choice D is eliminated. Steric hindrance t""- tempting, because a tertiiry amine has more steric hindrance than less substituted amines. But such ^uy an answer is designed to make you jump at steric hindrance without thinking through the question completely, because it is not the reason for no absorbance.

8'

groups are better electron donating substituents than methyl groups, so the most basic .o--po.rna cannot be determined using background inform.ation. According to the data ln table 1, (CH3)2NH g"rl"*t", a K"O of 30.9 when reacting with ammonium, while (CH3CH2)2NH generates a K"o of 50.1 when reac"ting with amnionium. This means that (CH3CH2)2NH is a stronger base than (CH3)2NH, so cioice D is the best unilr"r.
Copyright @ by The Berkeley Review@

Choice D is correct. The donating nature of alkyl groups increases the basicity of an amine, although it also creates steric hindrance. In this case, the amines are primary and secondary. Secondary amines are more basic than primary amines, so choices A and B are eliminited. the data in Tabie 1 confirmi that secondary amines are more basic than primary amines, because secondary amines yield a greater equilibrium constant when reacting with ammonium than primary amines. Methyl groups are less butky than ethyt groups, but ethyl

223

AMINO ACIDS

&

AMINES EXPLANATIONS

9.

Choice C is correct. Table 1 iists the reactions of neutral amines with ammonium, so for each entry, NH+* shoul: be found on the reactant side. All of the entries in Table t have equilibrium constants greater than 1, ,o to hu.'. an equilibrium constant of 0.03, it must be the reverse reaction of what is shown in Table 1. This means tha: \Hi must be found on the reactant side. This eliminates choices B and D. According to the data in Table ,
amine that results in an equilibrium constant of 0.03 (about 1,/30), so choice C is the best answer.

dimethyl amine ((H3C)2NH) is associated with an equilibrium constant of 50 *nl" trimethyl amrr-.r ItH3C)3N) is associated with an equilibrium constant of roughly 30. It is in the reverse reaction of trimeth'.-

10.

alnLre can be protonated to form a cationic compound (its ammonium conjugate acid), which is more water solui:,. the amide. The extraction therefore will be most successful at a 1olr,' pH (acidic conditions). The lowest p- ot the choices is 3, which makes choice A the best answer.
il'-e-n

Choice A is correct. Because an amine is a weak base while an amide exhibits no acid-base properties l,r,he: ai:led to water, the two compounds can be separated using acid-base extraction. This eliminates choice D. T:.

17. Choice

A is correct. Because of the electron withdrawing nature of the carbonyl group through resonance, --:. ele:tron pair on the nitrogen in an amide is less available for donating than the electron pair on nitrogen in .-alr-lie' The difference in basicity is attributed to resonance and not steric hindrun.e, so choices C and D a:: el-minated. An amine is more basic than an amide, so choice A is the best answer.

12. Choice C is correct. The boiling point of a compound is influenced by its molecular mass and its intermolecu-: forces. The highest boiling point is associated with the compound with both hydrogen bonding and the grea:a;; r,olecular mass. Ammonia has hydrogen bonding, but it is very light. Choice A is eliminated. Trimeth/iut

--* erhibits no hydrogen bonding, so it does not have the highest boiling point of the choices. This elimina:=s ci.oice D. Methyl amine and propyl amine have roughly the same degree of hydrogen bondrng, but propyl ar':,r is heavier. As such, the highest boiling point is observed with propyl amine. Choice C is the best answer.

t3. Choice A is correct. Given that the pKu of (H3C)3NHC1 is 9.8, the pK6 of (H3C)3N: must be 4.2. The elec:::r donating nature of methyl groups increases the basicity of an amine while the electron withdrawing natur. :r benzene decreases the basicity of an amine. Aniline is therefore less basic than trimethyl amine. Becaus- , higher pK6 value is associated with weaker bases, the pK6 of aniline can be no less than 4.2. This makes ch---:*
A not possible. Choice A is your choice for correctness and subsequent happiness.
1.4.

oxygen would convert the compound to an amide and thus reduce its basicity, because oxygen is elec::r r,vithdrawing. Choice B is thus eliminated. Replacing one hydrogen with an alkyl group -orrta increase -:* basicitv, because alkyl groups are electron donating. Choice C is the best answer. Replacing an alkyl group-,l--iT hydrogen would reduce the basicity, because alkyl groups are electron donating. Choice D is thus eliminatec

Choice C is correct. A decrease in the K"o value for a proton transfer reaction with ammonium indicates tha: --:c of Ittgtyresultthe compound is reduced. Thi question is thus "Which of the following changes to an amine \\-. ri NOT in a weaker base?" or better yet, "Which change would result in u.o*po.tnd MORE basic th-- ; primary amine?" Replacing hydrogen with fluorine would reduce the basicity, because fluorines are elec:::,nr n'ithdrawing. Choice A is thus eliminated. Replacing the adjacent (alpha) hydrogens with a double bor.; ir

E ::

15.

Choice C is correct. Method 2 in Figure 1 shows the reduction of a nitrile, a primary imine, a primary amide ,:,; an azide into a primary amine, so a nitrile and primary imine, choices A and B, can definitely be reduced ir,:: , primary amine. Choices A and B are eliminated. A secondary amide can be reduced, but R-groups rer---.flr constant, resulting in the formation of a secondary amine. This means that a secondary amide, choice C, ca:::ri be reduced into a primary amine, so choice C is the best answer. An oxime, R2C=NOH, can be reduced mucL :.* an irnine can be reduced. The resuit is a primary amine and water. Choice D is eliminated. Choice A is correct. The N-H bond, like the O-H bond, has an infrared absorbance value above 3000 cm-1. ---e absorbance is broadened due to hydrogen bonding, although not as broad as a hydroxyl absorbance, be.a-:,"r arnines do not form as strong of hydrogen bonds with amines as alcohols form with alcohols, Broadening r-. :;r'', associated with the question, so we address it for edification purposes only. You simply need to know that 1,.--i bonds are simiiar to O-H bonds, so their IR absorbances are close in value. The best choice is answer A.

16.

Copyright O by The Berkeley Review@

224

AMINO ACIDS

&

AMINES EXPLANATIO\:

17. Choice B is correct' The methyl group usually donates electrons through what is called hyperconjugation. The donating form of the structure is nof truly a resonance form in the traditional sense, so the methyl group is considered to be electron donating through the inductive effect. This is another case of controversy, so when you are uncertain of what-information they are basing their answer on, consult the passage. In this case, which will be true on occasion, the information is not there. So, from background information, resonance is defined as the delocalization of electrons through the n-bonding network. The inductive effect is defined as the delocalization of electrons through the o-bonding network. Theie are no n-bonds in an amine, so the best answer cannot involve resonance' This eliminates choices C and D. The best answer must therefore involve the inductive effect. An electron donating group would make an amine a better nucleophile, so choice B is the best answer.
18.

Choice D is correct. The real question here is whether the reaction proceeds by an 51111 mechanism or an 5512 mechanism. The electrophile is secondary, so there is no easy way to tell. However, on a multiple choice exam, you should let the answers dictate the choices you make. Tire Sry1 reaction would result in a racemic mixture, which is not listed as a choice. The answer choices tell us to decide between an 5612 reaction and an E2 reaction. Methyl amine is not a strong enough base to cause elimination, so the reaction must follow an S1'tr2 m-echanism, eliminating choices A and B. An 5512 reaction results in an inversion of the chiral center. The reactant electrophile has R-chirality, so the product shoulcl have S-chirality. The best answer is choice D.
Choice C is correct. The most nucleophilic nitrogen is the one with a lone pair that is most readily available for sharing' Electron pair a is the lone piir of ut't u-id" nitrogen, so it is tied up in resonance and not available for use as a nucleophilic lone pair. Choice A is eliminated. Electron pair b is ieing shared with the benzene (and the amide carbonyl on the para position of benzene) through resonance, so it is not as available for nucleophilic attack as a standard amine lone pair. Choice B is eliminated. Electron pair c is free from any conjugation (because it has sp2-hybridization ,^and sp2-1-rrbridization has the electrons in an orbital perpendicular to the porbital of the n-network). Because of the resonance from nitrogen d, nitrogen c carries a partially negative charge making it more nucleophilic. Choice C is the best answer.'Electron pal d is involved ln the aromaticity of the five-membered ring, so it is not readily available for nucleophilic atiack. Choice D is eliminated. This question is analogous to what is observed with histidine. The best choice is electron pair c, so choose C. Choice D is correct. An amine can react with both an acid halide (choice A) and an ester (choice B) to form an amide, so choices A and B are eliminated. It can undergo an Sp2-reaction with an alkyl halide (ciloice C), so choice C is eliminated. Ethers are inert and react with very few compounds. Choice D is the best answer.

19.

Choice C is correct- Using the enamine synthesis route, alkyl groups can be added to the alpha carbon. The synthesis that cannot be carried out via the enamine procedure laid out in Figure 2 is the one that forms a new carbon-carbon bond to a carbon other than the alphi carbon. Choice A add"s a methyl to carbon 2 (the alpha carbon of 3-pentanone), choice B adds a butyl g.o.tp to carbon 2 (the alpha carbon of buianal), and choice D adds a pentyl group to carbon 3 (the atpha carbon of acetone). It is in choiie C that the new carbon-carbon bond is formed with a non-alpha carbon (carbon 6 of 3-hexanone is not an alpha carbon). pick choice C for the warm, fuzzy feeTing of correct answer picking.
Choice A is correct. Water is the side product that is generated in the formation of an imine from a ketone and an amine. The amine reactant is a primary amine, so it wiII lose both hydrogens and form a bond to the carbonyl carbon. The best answer is choice A.

Choice A is correct. The treatment of an imine with acidic water results in the hydrolysis of the imine and subsequent formation of an amine and a carbonyl. The cyclic nature of the reactant causes the two functional groups to be present on the same molecule. By cleaving the double bond and adding two hydrogens to the nitrogen and an oxygen (carbonyl oxygen) to the carbon, the pioduct is determined rather easily. This"shown below. The best answer is choice A.

*'o -> "-QO


H

-rpyright @ by The Berkeley Review@

22s

AMINO ACIDS & AMINES EXPLANATIONS

24' Choice A is correct.

The imine bond, when cleaved with acidic water, forms a ketone compound and a prima:amine' A quick pictorial route for determining the hydrolysis products formed from the reaction is shown belor.,

+"+1"{# }*n':O
Primary amine
Ketone

The best answer is choice A.

25'

Choice D is correct. From 3-pentanone to 4-methyl-3-heptanone, the structure has increased its number of carbc:, by three. This means that a three carbon chain has been added to carbon 2 (which is equivalent to carbon -1 :, symmetry) of 3-pentanone. This eliminates choices A and B. To carry out this synthesis, i secondary amine rr.,-,,: be added to the carbonyl reactant (3-pentanone) to form an enamine. This eliminates choice C and makes cho::. D the best answer. In the second step of the synthesis, the alkyl halide compound (RX) is a three-carbon ch: with the halide on the terminal carbon. The final step in the synthesis in all four choices is the addition ., acidic water, so it must be true. The last step converts the nitrogen compound (enamine) irrto an oxygen compo.i: (carbonyl).

HrcH2cAcH2cH3

ll

l{g

R:..

HrcH2cA.HCFL lr.nr.AcHCFb rircHrcA.HCH J L -' '


| I
H3CF{2CF{2C H3CFI2CFI2C

R:p,7n ^,--RH3ccH2cH4 -il-

t'r':,-

fr

26.

choice C is correct. The imine that is formed from a primary amine and a ketone is the imine shown in ,-:: second example in Figure 1. The nitrogen of the imine his a single bond to one carbon and a double bond to :: = other carbon. Upon reduction of the imine, the nitrogen will gain a new o-bond to hydrogen and lose the n-bo:: to the carbon (as the carbonyl oxygen does when the ketone is reduced to a secondary atcJnot;. The final produ:: is a nitrogen with two alkyl substituents and a hydrogen, which describes a secondaiy amine. The best dfls\4,r : choice C.
Choice B is correct. Questions of this type are best solved by drawing the reaction out. Acidic water causes hydrolysis of N-ethyl-3-pentimine, as shown below:

)n

i=

HrcH.c--+1"<:*^.:'
cF{2cFI3

H _____-l> Hzo

CFI"CH" H3CFI2C-

NF!
cFI2CF{3

Ethyl amine
The products are ethyl amine and 3-pentanone, so the best answer is choice B.

3-Pentanone

28.

Choice B is correct. D-Serine has two hydroxyl absorbances, because it has two non-equivalent O-H bonds. l; IR absorbance for the O-H group of a carboxylic acid is broad and it comes between 2500 cm-1 and 3000 cm-1. l:= IR absorbance for the O-H group of a standard alcohol comes between 3200 cm-l and 3600 cm-1 and is also broa: although not as broad the carboxylic acid O-H absorbance. The carboxylic acid hydroxyl and alcohol hydror. are not equivalent rnJR spectroscopy. Whether it be by memory or inductive reasoning, pick B. A carbonyl bon: C=o, absorbs around 1700 cm-1, so choices C and D aie eliminit"d. Urirrg inductive r6aioning, you can conciuj= Weaker bonds have lower bond energies (weaker spring constants, if you wili), so they are easier to stretch. ,: the bond is easier to stretch, then less energy is required to stretch the bond. Less eneigy can be seen in the j: absorbance as a lower frequency (lower wave number). This means that the absorbance foi an acidic O-H must't= less than a standard alcohol, around 3300 cm-1, so choice A cannot be possible.

thai the o-H bond of an acid is weaker than the o-H bond of an alclhoi (which is *h/ii

is more acidic

Copyright O by The Berkeley Review@

226

AMINO ACIDS

&

AMINES EXPLANATIONS

29. Choice C

is correct. There are three synthetic routes presented in the passage. This particular synthesis begins with an aldehyde, therefore it is the Strecker synthesis, as shown in Figure sI upon matching the reagents fiom Figure 3 with the reagents in the answer choices, it is choice C that is a match. If you choie to eva'iuate each step in each answer choice, the sequence of products from the steps in choice C are an imide, cyano amine and amino carboxylic acid. Following the reaction step by step also leads to the correct choice. In choice A, the compound would be oxidized into a carboxylic acid and then deprotonated. In choice B, the cyano group would add to the carbonyl to form a cyano alcohol which can then form an acid with acidic *orknp, but iot an amino acid' Without adding nitrogen to the compound, it is not possible to form an amino acid. Choice D has too many carbons in the reactant, so it cannot form tie product. The best (and correct) choice is answer C.

30.

Choice A is correct. Every amino acid in a hydrophobic solvent has a broad signal between 10 and 12 ppm in the 1HruvR due to the carboxylic acid hydrogen uni u luiwtn signat of twice tte intensity between 2.5 ppm and 4'5 ppm due to the two hydrogens on nitrogen. Deuterochlorofoim, CDCI3, is an aprotic, hydrophobic solvent, so the signal just above 10 ppm is attributed to the carboxylic acid proton ur-rd tl'," signal around 3-pp* is attributed to the two amino terminal hydrogens. the 1HNMR has three iignals, in a 1 : 2 : 2 ratio, so the amino acid has three types of hydrogens. Because there is only one other sigial (the triplet around 2.1 ppm for the alpha hydrogens), there is only one other type of hydrogen present ot'r ih" amino acld molecule. This means that the RgrouP must be a hydrogen, because any other R-group would introduce additional signals into the 1gruUn spectrum. The only amino acid with two equivalent hydrogens in the side chain is glyciie (the side chain is H). Glycine has three types of hydrogens, so choice A is the besianswer.

31.

Choice B is correct. the Gabriel synthesis, the amino acid is released when the phthalimide complex is _In hydrolyzed under acidic aqueous conditions. This means that the side product results from the hydrolysis of the cyclic imide. This makes the side product a diacid, making choice B the best answer. Choice A"is noi possible, because hydrolysis would cleave the amide bonds. Choice i is too reduced and choice D cannot happen, because hydrolysis would likely cleave the cyano groups if the pH was correct.
Choice B is correct- An amino acid itself has one unit of unsaturation due to the n-bond of the carbonyl in the carboxylic acid. The questions centers around the units of unsaturation in the side chain. Aliphatic, by definition, refers to an alkyl grouP with no rings and no n-bonds. Thus, there is no unit of unsaturation present in an aliphatic side chain. The units of unsaturation for the side chain is zero so the total units of unsatu'ration for the molecule is one. Pick B, and you can proudly consider yourself a correct answer picker.

?)

33.

the unique hydrogens labeled.

Choice D is correct. When L-cysteine is completely uncharged, there are five unique hydrogens present on cysteine- When cysteine exists as a zwitterion, there are only four unique hydrogens pr"rilt, because the carboxylic acid proton has been lost. The solvent is deuterochloroform, a hyirophobic solvent because the presence of deuterium on carbon is aprotic, so cysteine exists in its uncharged state. ^Consequently, there are five different hydrogen signals in the iHNtvtR. Pick D for optimal results. i-Cysteine is shown below with all of

HzN\

,/'n"t

L-Cysteine

tX"u
34'
Choice D is correct. Histidine has a positively charged side chain when the pH of the aqueous solution is less than the pK^ value of the side chain This means thit the histidine side chain goes from positively charged to neutral upon deprotonation' Because a hydrophobic environment cannot Jabilize a positive charg-e, the histidine side chain prefers to be neutral in a hydrophobic environment. Histidine will therefore lose the H+ more readily in a hydrophobic environment than a hydrophilic environment. Losing H+ more readily results in greater acidity and thus a lower value for the pKu. Choice A is eliminated, becauie log functions do change when the number changes. The pKu decreases, not increases, when it becomes more acidicl so choice B cannot be correct. Upon deprotonation of the side chain, the side chain becomes neutral, not negatively charged, so choice D is a better answer than choice C.

Copyright @ by The Berkeley Review@

227

AMINO ACIDS

&

AMINES EXPLANATIONS

35. Choice D is correct. In order for an amino acid to have

three equivalence points, when starting in its fullr protonated state and being titrated by a strong base, the amino acid must have three acidic protons. In order icr an amino acid to have three acidic protons, the amino acid must have an active proton on the side chain at lo-,tr pH (i.e., have a side chain that is either acidic or basic). Tyrosine has an acidic side group (a phenol proton ha'r a PKa around 10) while the R-group for the other three choices (glycine, alanine, and leucine) are al1 allogroups (none of which have an acidic proton). The best answer is therefore tyrosine, choice D.

36.

Choice C is correct. According to the passage, there are three aromatic amino acids, phenylalanrne tryptophan, and histidine. This eliminates choice A. According to the passage, threonine is one of the essenti.d amino acids. it has a hydroxyl group on its side chain, so there is at least one essential amino acid that ls hydrophilic. This eliminates choice B. Essential amino acids are natural amino acids, so they must be L-aminc, acids. With the exception of cysteine, L-amino acids have an S-stereocenter at the cr-carbon. Considering cysteine is not one of the essential amino acids listed in the passage, there are no essential amino acids with a: R-stereocenter at the d-carbon, so choice C is the best answer. At a pH of 6, arginine carries a positive charge, sl choice D is eliminated. Histidine has a side chain pKu around 6.05, so it will exist in a state where slightr over half is protonated and cationic.
Choice D is correct. Whenever a protein is removed from a hydrophobic environment and introduced into a hydrophilic environment, the secondary structure of the polypeptide backbone will be altered, because the, hydrogen bonds that are intramolecular in the lipid environment can now be formed with the surrounding wate in the hydrophilic environment. The result is that the protein will turn itself inside-out as it passes from orre environment into another. Pick D, and get another one correct. Choice B is correct. Enzymes such as chymotrypsin, trypsin, and thermolysin break the peptide linkages (amiae bonds) of specific amino acid residues within a protein. This breaks the protein into smaller fragments, whic: can then be evaluated with greater ease. This is done with large proteins, because sequencing techniques hane an upper limit to the number of amino acids they accurately determine. Before using Edman's reagent, a prote:! is denatured using B-mercaptoethanol and urea, to denature and elongate the protein for easier analvsh However, it is not an enzyme that does this, so choice A is eliminated. Enzymes do not cause a protein to coupLe with another protein, and transcription is not associated with protein sequencing, so choice C can be eliminatE;" Choice D is a throw away answer, because there is no mounting of a protein onto slides in sequencing. The be* answer is choice B, because a large protein can be accurately sequenced after it has been cleaved into srnal
pieces.

J/.

38.

39.

Choice C is correct. H is in the front position at both chiral centers, so whatever arc is drawn when prioritizing the substituents from highest through lowest must be reversed to determine the chirality. In this case carbon l gets a clockwise arc and carbon 3 gets a counterclockwise arc. Because the molecule should really be viewed fror the backside, reversing these arcs correctiy identifies carbon 2 as an S-center and carbon 3 as an R-center. The compound has chirality of 25, 3R, so choice C is the best answer. The determination is shown below.

HOH
2

o
cFI3

HO/

-f4 ./t\
n

,"c\
HzN H
it

HO(

A,

HO H, ,t " "o,. |

t),.*

HzN H
For carbon-3, the arc must be reversed for

J\

For carbon-2, the arc must be reversed for

correct viewing, so

has S-chirality.

correct viewing, so

it has R-chirality.

Copyright @ by The Berkeley Review@

228

AMINO ACIDS

&

AMINES EXPLANATIONS

{0.

the pH is greater than the pKu and therefore the carboxyl terminal of is anionic' The amino terminal has a pKu between 9 and"1.0, so at a pH of fnenytatanineless deprotonated and 7.4thepH is than the pKu and therefore the amino lelmilal of phenylalanine is protonated and cationic. There are no acidlbase properties associated with its side chain, so phenylalanine will always have a neutral side group. The result is that phenylalanine has one cationic site ind one anionic site, so it exists primarily as a zwitterion. While it has no net charge, because it has charged sites, it cannot be considered uncharged. Choice A is eliminated. It has no overall charge, so it cannot be considered to be an anion or a cation, so cioices B and C are eliminated. The best description is a zwitterion, so choice D is the best answer.
{1.

Choice D is correct. Phenylalanine has an alkyl side chain, so the only active protons it possesses are for the amino terminal and the carboxyl terminal. The carboxyl terminal has a pKu between 2 ani g,so at a pH of 7.4,

alanine titrated by NaOH is shown below:

Choice D is correct. This question requires that you understand a titration curve in a conceptual manner. Because the base is 1'5 times more concentrated than the alanine solution and an equal volume of each solution is mixed, the amount of base that has been added is one-and-one-half equivalents. Starting at the fully protonated state, addition of 1.5 equivalents of titrant base takes the alanine ,tl,rtior, past its firit equivalencl point and half *-uy the second equivalence point of alanine. If you observe the titraiion curve for ilanine, this makes the pH !o of equal to pKu2, which lies exactly halfway between the first and second equivalence points. The -solution value of pKu2 lies between 9 and 10 for most amino acids, so choice D is the best answer. The titration curve for

Isoelectric Point PH = PKaz

Alanine starts

fully

protonated.HzA*

PH = PKur

nine finishes
the two eq. points.

.1, | /2
Equivalents base added

12' Choice B is correct. For an amino

acid with a hydrophobic, alkyl side group, there are only two pKu values. To determine the isoelectric point, you must urr"rug" the two pKu values" thai are associated with the zwitterion, in this case the only pKu values associated withlhe amino icid. For isoleucine, the values for pKul and pKu2 are 2'3 and 9.7 , respectively. The average of the pKu values is 6, so choice B is the best answer.

{3'

Choice D is correct. Because there are three equivalence points (vertical inflection points) on the titration curve, the amino acid that the titration curve represents must have a side chain with an actlve proton. An active proton is associated with either an acidic or basic side group. This eliminates choices A and i], which both have side chains that contain no active protons. Because both of ine first two plateaus in the graph occur below a pH of 7 '0, the first two pKu values for the amino acid must be less than 7.0. The middle oi tn" plateau represents the point at which the pK^ is equal to the pH of the solution. So, from the graph, a qualitative determination of the pKu values can be made. Because the value of pKu2 is less than 7.0, ihe side chain is a carboxylic acid group rather than an amine group. Aspartic acid has a iarboxylic acid group on the side chain, which has a pKu value less than 7.0, so choice D is the best answer. The side chain of ffrin" is an amine, so its pKu is above 2.0. Choice C is eliminated for this reason.

Copyright @ by The Berkeley Review@

229

AMINO ACIDS

&

AMINES EXPLANATIONS

44.

Choice D is correct. To make the pKu lower, something must be done to the compound to make the proton mc:g acidic' In choice A, the oxidation of carbon 1 by four electrons results in the conversion of a primary alcohol ir:r a carboxylic acid, which increases the acidity of the compound through electron withdrawal via resonarceThis results in a lower pKu, so choice A is eliminated. Because sulfur is Targer than oxygen, exchanging ox\.ge1l for a sulfur allows the proton to be lost more easily. If the proton is lost moL easily, tnei tne.orr1pJ,r1j is mo:e acidic and thus has a lower pKu value. This can be seen *h"n serine (primary alcohol side group I :o "ornparing cysteine (primary thiol side group), where the side chain pKu foi cystline is about 6 less than the side cha-r pKu for serine. This eliminates choice B. Replacing the propyi grorrp with a phenyl group has the same effej as oxidizing carbon from a primary alcohol into a .uiUo*yti. acii, althougn th" resonance effect is not a$ -1 strong. Regardless, the compound becomes more acidic, so the pKu decreas"r. thi, can be seen when comparrjg: serine (primary alcohol side group) to tyrosine (phenol side group), where the side chain pKu for tyrosine 15 about 4 less than the side chain pKu for serine. Choice C is lhus eliminated. The only u.,r-"," choice left u choice D By replacing the oxygen with nitrogen, the compound becomes less acidic. Amines are less acidic tha: alcohols. Nitrogen is roughly the same size as o"yg".r, ro the effect is not the same as choice B with sultu Because nitrogen is less_electronegative than oxygen, it "pulls" electrons less than oxygen does. This makes rre nitrogen more basic and less acidic than the oxygen. This raises the pKu value making choice D the choice tl-a: would nof decrease the pKu. Choice B is correct. When the pH of the solution is greater than the pKu for a proton, that proton is removed br the solution and the compound exists in its deprotonated state. wne" ine pu l, hlgn, the solution t ruri. u.; thus deprotonates the proton. This same conch-rsion can be formed using the Henderson-Hasselbalch equatio:Yh:" the pH is greater than the pKu, the log term in the H-H equation niust be positive, so the concentration irr the base (deprotonated form) must be greater than the concentrition of the acidlprotonated form). This mears that when the pH is less than the pKu, the site retains its proton. Taking the siLs one at a time, the carbor;terminal has a pKu of 2.0, so at pH = 5.0, it is deprotonated and carries i -1 .hurg". The carboxylic acid siie chain on the aspartic acid (right amino acid) his a pKu of 3.9 which is also le"ss than the pFi, so it too s deprotonated and thus carries a -1 charge. The amino terminal has a pKu value of 9.2, whichis greater than itu pH, so it remains protonated and thus carries a +1 charge. The amine side chain on the lysine (ieft amino aciC has a pKu of 10.8, which is also greater than the pH, so if is protonated and thus carries a +1 charge. The sum ol the charges is zero, so the best answer is choice S. The structure at a pH of 5.0 is drawn below.

45.

oYotf

.,

= r.n

I
46.

Choice D is correct. In order for the pH to be greater than the pKu, the conjugate base (deprotonated form) mr:-.: be in higher concentration than the conjugate acid (protonated fot-;. The quistion asks for the deprotonated-t"-* protonated ratio, which is greater than 1. This eliminates choices A and B. B".urse the pH is one unit abore pKu, the log of the base to acid ratio must equal 1, according to the Henderson-Hasselbalch equation. The ]he log of 10 is equal to 1, so the ratio of conjugate basJto conjugate acid must be 10 : 1. The best answer is choice D.

5'r

17.

Choice A is correct. Because the electron withdrawal associated with resonance is greater than the electror. withdrawal associated with the inductive effect, a carboxylic acid is more acidic thai the alcohol group on a carbon with one fluorine atom bonded to it. From the puriug", we know that the pKu for the carboiytic acid = lower than the pKu for the protonated amine site, so the caiboxylic acid (site a) must be the most acidic site. This means that proton a is the most acidic proton on the molecule and only .hoi"" A lists proton a as most acidic so choice A is the best answer.

C:plrighr O by The Berkeley Review@

230

AMINO ACIDS

&

AMINES EXPLANATIONS

;rr

48.

Choice C is correct. Because the sulfur is still protonated at pH = 7.4, the pKu for the proton on sulfur must be greater than7.4. Because the proton on sulfur is the second proton to be lost (it is lostbefore the proton on the amino terminal), it must be more acidic than the amino proton. The amino proton has a pKu around 9.5, so the H on sulfur must have a pKu less than 9.5. The pKu for the proton on sulfur lies somewhere between 7.4 and 9.5, so the best answer is choice C.

49. Choice C

is correct. The fully protonated form of glutamic acid starts with a positive one charge, because the amino terminal is protonated carrying a positive one charge and the carboxyl terminal and carboxylic acid side chain are each protonated and neutral. The compound monosodium glutamate has one sodium cation, so therefore the glutamate portion must carry a negative one charge. To be converted from a positive one charge into a negative one charge, glutamic acid must be deprotonated twice. The best answer is therefore choice C. Drawn below is the step by step conversion from the fully protonated form of glutamic acid to the monoanion form of glutamic acid (glutamate).

o
1st equivalent r

H"*- Jt'
r

oH X H gHz

"'fr#oH
CH,

2nd equivalent

.ll o t'^foH
CH?

OH_

""y?I
O
50.

l-

oFIcharge

"ttY""Tr
"nu,.gu

HzcY

t-

+1

oo

?", .n",r"
-1

Choice B is correct. As drawn, the dipeptide molecule carries a +3 charge when fully protonated. When the molecule is at a pH of 1.8, the two forms of the molecule present in solution are the fully protonated form (carrying a + 3 charge) and the form with the carboxyl terminal deprotonated (carrying a +2 charge). \Atrhen the pH is 1.8, equal to pKu1, these two forms are present in a fifty-fifty ratio. When the molecule is at a pH of 6.1, the two forms of the molecule present in solution are the form with the carboxyl terminal deprotonated (carrying a +2 charge) and the form with both the carboxyl terminal and histidine side chain deprotonated (carrying a +1 charge). VVhen the pH is 6.1, equal to pKu2, these two forms are present in a fifty-fifty ratio. In order to have a ten fold excess of the +2 species, the pH must be less than pKaZby a factor of log 10, which equals pKa2 - 1. This eliminates choices C and D. The pH must be one unit less than 6.1 (pKuz), making choice 8,5.7, the best answer.
Choice D is correct. The most basic compound has the lowest pK5 value and the weakest conjugate acid. The pK6 value can be found by subtracting the pKu for the conjugate acid from 14. Choice A is eliminated, because it is the conjugate acid of choice D. The conjugate acid is always less basic than its conjugate base. The pK6 values for the three remaining choices can be found from the pKu values given for the sample compounds. For choice B, the pK6 value is 74 - 8.4 = 5.6. For choice C, the pK6 value is 14 - 5.0 = 9.0. For choice D, the pK6 value is 14 10.0 = 4.0. The lowest pK6 value is found with choice D, therefore the most basic compound is choice D.

51.

52. Choice C is correct.

At a pH of 1.5, the tripeptide is fully protonated. The protonated amino terminal carries a positive 1 charge and the protonated carboxyl terminal carries no charge. If none of the side chains carry a positive charge, the overall tripeptide carries a positive 1 charge. Only the basic amino acids (histidine, lysine, and arginine) have a positively charged side chain when protonated. This means that the correct answer choice shall not contain histidine, lysine, or arginine. Only choice C fits this criterion, so the best
answer is therefore choice C. Choice D is correct. Hydroxyproline has the normal chiral center associated with an amino acid plus it has an additional chiral center associated with the hydroxyl group of the side chain. This eliminates choice A. Just as proline induces deviations in the secondary structure of a protein (turns and kinks), hydroxyproline causes structural deviations as well. This makes choice B valid, thus it is eliminated. Alcohols are hydrophilic, so choice C is valid and thus can be eliminated. The side chain of hydroxyproline is an alcohol. The pKu of an alcohol is approximately that of water (15.5), therefore the pKu cannot be around 4.0. If it were around 4.0, then serine would be expected to have an active side chain proton as well. The best answer is choice D.

53.

Copyright @ by The Berkeley Review@

23I

AMINO ACIDS

&

AMINES EXPLANATIONS

54'

Choice A is correct. The highest isoelectric pH is found with the tripeptide with the greatest number of bas:; side chains' This is true because the basic side chains retain positive .nurg" at higher pH values than the othe: side chains' Histidine, lysine, and arginine are the three basic amino acidi, and only choice A contains at lea:: one of the three. Because choice A contains two basic side chains, it has an isoeiectric pH that is an average .,: pKu3 and PKa4' The four pKu values associated with Met-Lys-Arg are 2.2 (for the carboxyl terminal .: arginine), 9'3 (for the amino terminai of methionine), 10.8 (for tire sid"e chain of lysine), and 13b (for the sic; chain of arginine). The isoelectric pH is an average of 10.8 and 13.0, which is 11.9. This makes choice A tr; tripeptide with the highest isoelectric point. Choice B is correct. Glycine has no chiral center, so choice A is eliminated. Isoleucine and threoni'e each ha..: a chiral center in their side chain resulting in two chiral centers overall, so choices C and D are eliminate: Histidine has all tp2-hybridized carbons and nitrogens in its ring, so there are no chiral centers on its side cha-:
Because histidine has only one chiral center, choice B is the best answer.

55'

56'

Choice C is correct. This question is a unique way of asking "which dipeptide has the lowest isoelectric poir,: The lowest isoelectric point is found wittr the dipeptide cirrying no-basic side chains and preferentiallr- :*acidic side chain (such as aspartic acid or glutamic acid). Choice A is eiiminated, because histidine l;1"r,which increases^the isoelectric pH. Choice B is eliminated, because lysine is basic, which increases::: isoelectric pH. Choice D is eliminated, because arginine is basic, which inlreases the isoelectric pH. Chorce C has no basic side chain. The isoelectric point is an iverage of the carboxyl terminal pKu of cysteine (1.g) ancl :--: side chain pKu of aspartic acid (3.9) wnlch results in an [oelectric point of 2.85.

57'

Choice D is correct. Cations migrate to the cathode, therefore this question is asking for the amino acid -,: exists as a cation, or carries a partially positive charge, at a pH of z.o. To carry a positive charge, partiar _r full, the isoelectric pH (pI value) must be greater than"the soluiion pH. This question is therefore uit ir,g for ,:'i amino acid with the highest pI value (only one can be the best urlri"r, ,o orliy one can have a pI value grea:--,: than 7'0, which makes it the greatest pI value). In this case, the only basic urr-ri"o acid (which has the high:s; pI value) is lysine, so choice D is the best answer. Choice A is correct. The isoelectric pH is an average of the pKu value leading to the zwitterion and the :!.,, value going from the zwitterion. To hive a pI less th; 5.0, the pl(u values beinjaveraged must tre low num6.:1 For amino acids with side chains having no active protons, tnl pr is found by"uverugir-,g the carboxyl terrn ::i pKu and the amino terminal pKu. This means thai the pI is an average of atout 2.5 u.,d 9.5, resulting in a :around 6'0' This eliminates choices C and D. For amino acids witl basic side chains, the pI is founc --," averaging the side chain pKu and the amino terminal pKu. This means that the pI is an u.r"rug" of a nLrn:*: between 6 and 72 and a number around 9.5, resulting in a pI around 9.0. This eliminates choice B. For amino a..:: with acidic side chains, the pI is found by averaging the carboxyl terminal pKu and the side chain pKu. i:* means that the pI is an average of about 2.5 and a number between 3 ancl 10, resulting in a pI urorr.d 3.0. I:* makes choice A the best answer. To summarize, acidic amino acids have pI values less"than b, basic amino a;.ri have pI values greater than 7, and amino acids with neutral side chains have pI values around 6.

58'

E
llhr

59.

Choice C is correct. The side chain that is most likely to be founcl in a hydrophilic pocket is the side chai: :r, the most water soluble amino acid. The side chains in the answer choicei are methionine, tyrosine, glutan--- * and cysteine. According to the data in Table 1, the most water soluble amino acid of the choices is glitami:- m 3'7 grams per 100 mL water. This is predictable, because of the hydrogen bonding of the side chain. The :,:sl
answer is choice C.

60"

Choice A is correct. Choices B and C should be eliminated immediately, because the alkyl side chain of leu:rnr does not form hydrogen bonds and the charged side chain of lysine is polar. Choice D is true, but the faci -:,ad

leucine has a nonpolar side chain does not enhance the waier solubility of leucine. Choice D is true :ui irrelevant, which eliminates it. Because lysine can form hydrogen bonds *ith th" hydrogens on nitrogen, ci.:,1,o* A is valid.

Copyright @ by The Berkeley Review@

232

AMINO ACIDS

&

AMINES EXPLANATIO\S

61. Choice

C is correct. For amino acids with both neutral and acidic side chains, the isoelectric pH is found by averaging pKul and pKa2. The pI values for the amino acids with both neutral and acidic side chains are consequently less than 6.0. For basic amino acids, because the pI is an average of pKu2 and pKu3, the value of pI is greater than 7.0. The only basic amino acid in the answer choices is histidine, choice C.

62. Choice C is

correct. For cysteine, the amino terminal is protonated and carries a positive charge. The carboxyl terminal is deprotonated and carries a negative charge. This eliminates choices A and B. Because the pH of the solution is less than the pKu of the side chain, it is protonated and carries no charge. This makes choice C the best answer and eliminates choice D.
Choice B is correct. The acidity of a compound is measured by its pKn, so the lowest pKu value is associated with the most acidic compound. The carboxyl terminal proton is the first to be lost, so it is represented as pKa1, meaning that the question is comparing the relative acidity of the carboxyl terminal protons of arginine and alanine, The carboxyl terminal pKu for arginine is lower than carboxyl terminal pKu alanine (according to the values in Table 1), so choices A and C are eliminated. Choice B is the better answer, because it is the side chain, not the amino terminal, that differs between arginine and alanine.

63.

64. Choice

D is correct. To separate two compounds using extraction in an ether/water biphasic solution, it is best when one is charged and the other is neutral. It is at the isoelectric pH that the species carries no net charge, thus when the pH is equal to the pI of lysine, then lysine is neutral and therefore more soluble in ether than in water. At this same pH, tyrosine is partially negative and thus water-soluble. \.Alhen the pH is equal to the pI of tyrosine, then tyrosine is neutral and therefore more soluble in ether than in water. At this same pH, lysine is positive and thus water-soluble. At all other pH values, both lysine and tyrosine are charged. The best answer is choice D. Because of the high water-solubility of lysine in water, the extraction is best carried out at the isoelectric pH of tyrosine.

55.

Choice D is correct. At a pH of 4.0, the carboxyl terminal is predominantly deprotonated, because the pH is greater than the pKu of the carboxyl terminal. This makes choice A false. The isoelectric pH for histidine is 7.60, so at a pH of 4.20 (a value less than 6.05), the side chain carries a positive charge, so far less than 90% exists as a zwitterion. This makes choice B false. The pKu of the amino terminal is greater than the pH, so the amino terminal is predominantly protonated. This makes choice C false. Because the pH is far less than the pKu of the side chain, the side chain is mostly protonated (more than ten percent). The best answer is choice D.

66. Choice

B is correct. The background information that you have to know is that DEAE-cellulose carries a positive charge at neutral pH, so at a pH of 6.20, the column must be positively charged. Because the column carries a positive charge, negatively charged amino acids are the ones that bind to the column. The lower the value of the pI, the more anionic the amino acid. This means that the amino acid with the lowest pI value is the one that is most hindered by the DEAE-cellulose column. Glutamic acid has the lowest pI, because the side chain has a pKu value of only 4.32. This makes choice B the best answer.

67.

Choice B is correct. The side chain of alanine (CHe) is bulkier than the side chain of glycine (H), therefore human PTH has a bulkier R group than bovine PTH. This elirninates choices A and C. The secondary structure is more affected by the steric hindrance of the side chain than the tertiary structure, so the best answer is choice B.
Choice B is correct. All of the amino acids from number 35, valine, to number 43, proline, have alkyl side chains, and thus are hydrophobic and uncharged. This eliminates choices A, C, and D. The best answer is choice B.

68.

69. Choice

D is correct. The polypeptide pre-pro-PTH is 115 amino acids in length. After losing the twenty-three amino acid segment and the two methionine residues (twenty-five amino acids in total), the resulting pro-PTH contains ninety amino acids. In the Golgi region, pro-PTH (ninety AA) is converted into PTH (eighty-four AA), so six amino acids must be lost. This means that a hexapeptide is lost, making choice D the best answer.
PTH, there is a tryptophan, Trp, a phenylalanine, Phe, and no tyrosine residues,

70, Choice D is correct. Within

Tyr. The Trp is in position 23 and the Phe is in position 34. This means that PTH chymotrypsin cleaves after AA #23 and afler #34, resulting in three fragments consisting of 23 amino acids (AA #1 through AA#23),11 amino acids (AA #24 through AA#34), and 50 amino acids (AA #35 through AA #84). The best answer is choice D.
copyright
@

by The Berkeley

Review@

233

AMINO ACIDS & AMINES EXPLANATIONS

71.

Choice A is correct' Choices B and D can be eliminated immediately, because there is no nitrogen in the sii; chain of methionine' Replacing the side chain of methionine with an'aliphatic alkyl group does not increase r: decrease the number of nitrogen atoms in the molecule. Choice C can be eliminated, tecause no change reactivity is observed. This makes the best answer choice A. The side chain listed is similar to methionr:: except that the suifur has been replaced by a methylene group (CHZ).
Choice B is correct. The active region of human-PTH is given in the passage as being amino acids #1 throus: #34 the analogs to PTH should be similar in this samJregion (the active"region). This eliminates choices -r1o and D and makes choice B the best answer so far. The C-terminal and N-terminal do no affect the active s--: reactivity, so choice C is eliminated. The best answer is choice B. Choice B is correct. Choice A can be eliminated, because reactivity is eliminated when the structure is reduct,: to twenty-eight amino acids. Choices C and D are poor choices, te.u.rs" both are found in the 35 through >= segment that does not directly affect reactivity, accolding to the passage. Based on the in aiuo reactivity da:: the reactivity drops severely when going from 31 to 28 imino u"idr, so the active site includes an amino al: between 29 and 31. Choose B for best t"rrlltr.

n.,

73.

74'

Choice C is correct. Because basic amino acids, like histidine, have a +Z charge when fully protonated, th.; must be deprotonated twice to reach their neutral state (as a zwitterion). Th-e isoelectric'point is found:, averaging the two pKu values that involve the zwitterion. For an amino acid with a basic ria* gro"p, if.r" pl ;.. an average of the last two pKu values (PKaz and pKu3). For histidine, the isoelectric point is"(6.1 + 9.2)1) = 7'65, maktns choice C the best answer. Basic amino acids all have a pI greater thanT.0, so choices A anc : should be eliminated immediately.
Choice B is correct. To determine the migration direction of an amino acid in a buffered gel, the isoelectric pc::: must first be determined. Histidine is a basic amino acid, so the isoelectric point oihirtidir," is found:; averaging its last two pK2 values. For all basic amino acids (histidine, lysine, and arginine), the iro"i.lo, point (pI) is the average of pK^2 and pKu3. For all other amino acids, the isoelectric point is the average :: pKul and pK22. The pI for histidine is (6.1 + 9.2)/2 = 7.65. This value is greater than the pH of the solution, I so histidine is partially positive (cationic) in a solution with a pH of 2.0." Cations migrate to the cathode in g., o----- --

75'

E
il_

electrophoresis, so choice B is the best

answer

76'

Choice A is correct. Sanger's reagent, 2,4-dinitrofluorobenzene, binds the first amino acid in a polypeptiit fragment at its amino terminal. The amino terminal nitrogen substitutes on the aromatic ring for atom' This means that the amino acid that binds 2,4-dinitiofluorobenzene must be the first aminothe fluori:: acid in ...r polypeptide. Because 2,4-dinitrofluorobenzene binds serine in the unknown polypeptide, the first amino acicl : the polypeptide must be serine. Choose A if youknow what's good for you.

77

'

that neither valine, leucine, nor isoleucine are the first amino *la itr the sequence when using thermolysin. tl,r break for Fragment II caused by thermolysin is as follows: His-Ser I Val-phe I Iso-Tyr-ph". -pi.t C to score b:;
on this question.

II contains one valine (amino acid #3) and one isoleucine (amino acid #5), neither of which is the first aminr acid' Hence, thermolysin cleaves Fragment II twice, breaking it into three separate pieces. you always chec1,

Choice C is correct. Thermolysin cleaves a protein on the amino side of leucine, isoleucine and valine. Fragme:.:

78'

Choice C is correct. Without a side group, a generic amino acid weighs 17 grarns (for NH3) + 13 grams (for CH * 44gtams (forCo2) =74grams/moleplusthemassof theRgroup. if th"to"tulweightisae*gramimole,thema=. of the side group must be 15 grams/mole. of the answer"selections, only the riethyl girp has a mass oi ,l grams/mole' For this reason, you just have to pick C. To determine the answe r,youcould*also have gone throu= each answer choice and systematically.o*pnt"d each of their molecutu, *"ijnt, if you know the"st;;:;;;.:,

the amino acid.

79'

Choice A is correct. According to Table 1, clostripain cleaves at the carboxyl side of arginine. Fragmen: contains no arginine, so it will not be cleaved by clostripain, leavirrg the fragment in one piec"e. rragment II sta""..: as His-ser-val-Phe-Ile-Tyr-phe, so choice A is the besJ answer.

Copyright O by The Berkeley Review@

234

AMINO ACIDS

&

AMINES EXPLANATIO\s

s0.

g1utamicacidandleucine.Thermolysinc1eavesattheamino.sideofleucine,l,ot",.l,.'",andvaline.^Thi,-"u.',

Choice C is correct. Pepsin cleaves at the carboxyl side of phenylalanine, tryptophan, tyrosine, aspartic

acid,

that if the carboxyl side of either phenylalanine, tryptophan, tyrosine, aspartic acid, glutamic acid or leucine is bonded to the amino side of leucine, isoleucine, or valine, then a peptideiinkage exists that will be cleaved by either pepsin or thermolysin' In such a case, the same fragments would result whether pepsin or thermolysin were employed. In choice A, the carboxyl side of isoleucine is linked to the amino side oiphenylalanine, which is the opposite of what is needed to lead to equal cleavage by pepsin and thermolysin. Choice A is eliminated. In choice B, the carboxyl side of leucine is linked to the amino iiae of leucine, and both pepsin and thermolysin react with proteins containing leucine. The problem with this answer choice is that tirey cleave on different sides, so pepsin will generate a fragment ending in leucine and a free leucine while thermoiysin generates a free leucine and a fragment starting with leucine. Choice B is eliminated. Pepsin does not react with either valine or methionine, so no cleavage occurs with choice D when using pepsin. Beiause thermolysin reacts with proteins containing valine, a break will occur before the Val-Met linkage, resulting in fragmentation. This eliminates choice D. Choice C is the best answer, because pepsin cleaves after tyroJin" ur-rJ thermolysin cleaves before valine, so the same break occurs for the Try-Val linkige whether pepsin or thermolysin is used.
Choice C is correct. As given in Table 1, chymotrypsin cleaves the carboxyl side of phenylalanine, tryptophan and tyrosine. There are two tripeptide fragments foimed when using chymotrypsin, Lne ending in either Ser or Phe (although by convention we assume it ends in Phe as written) ur-rd tn" other ending in Gly [th"r" is no issue with the order of the second fragment, because the sequence is the same either backwaris or forwards.) The SerVal-Phe fragment must be first and it must start with Ser, because it is the only one with either phe, Trp, or Tyr present at the end. This means that the Gly-Cys-Gly fragment is the secona of *re two fragments. puiting the two fragments back together yields the hexapeptide sei-val-Phe-Gly-Cys-Gly. This makes choice C the best answer' If you didn't solve it that way, there is an easier way. Only .noi." C has serine as the first amino acid in the sequence. You know serine is the first amino acid in Fragment I from the results of the treatment with

-l

Sanger's reagent.

i2.

best answer is choice D.

Choice D is correct. The role of the G-mercaptoethanol is to cleave any disulfide linkages formed between the side chain residues of cysteine. This is statedseveral times in the passage, at the start oJ each experiment. The

Choice A is correct. As stated in the passage, the purpose for adding 2,4-dinitrofluorobenzene (2,4-DNFB) was to bind the amino terminal of the first amino acid in the polypeptide"chain. The 2,4-dinitrofluorobenzene only binds the first amino acid making it serve as a marker foi the-fiist amino acid. This means that the role ol 2,4dinitrofluorobenzene is to identify the first amino acid in the sequence. This makes choice A correct.

Choice A is correct. Only histidine appears at the N-terminal in one of the fragments in each of the three IV. Using the data from Experiment II and k]-rowing that histidine is the first amino acid (implying that Lys is the second amino acid), the possibilities for the last amino acid in the polypeptide are either alanine or arginine. The fragments of Experiment III show that the last amino acid is either serine or alanine (because His is first, and therefore Cys must be fourth). Combining this with the information from Experiment II and Experiment III, the last amino acid in the sequence must be alanine. This makes choice A your best choice.
experiments, Experiments II, III, and

]). Choice D is correct. The fragments formed in Experiment IV show that chymotrypsin either cleaves after Tyr and Phe or before Cys and Ala. Because the last fragment (N-His-Lys-Met-bys-L""-Cty-afa-phe-C) shows a p_eptide linkage to the amino terminal of alanine still in tack, chymotrypsin must cleave after Tyr and phe. Choice A is valid. The fragments formed in Experiment II show that trypsin either cleaves after Lys and Arg or before Ser and Met. There is no way to determine which is actually thl-case, so as it is, choice B is not invalid. The fragments formed in Experiment iII show that thermolysin eithlr cleaves after Cys and Ser or before Ile and Leu. Because the last fragment listed from the thermolyiin cleavage (N-Leu-GlyiAlu-ph"-Cys-Arg-Ser-C) shows a peptide linkage to the carboxyl terminal of cysteine still in tick, thermolysin must cleave before Ile and Leu. This makes choice D your best choice, because thermolysin does not cleave the amino terminal of alanine
and cysteine. This also eliminates choice C.

,opyright

by The Berkeley Review@

235

AMINO ACIDS

&

AMINES EXPLANATIONS

86.

Choice C is correct- Because the last fragment in Experiment III (resulting from the thermolysin cleavage), NLeu-Gly-Ala-Phe-Cys-Arg-Ser-C, shows a peptide linkage to the carbo"xyl terminal ,tJril" tu.t, thermolysin cannot cleave following cysteine. if it did "i;yJ,;;;" after cysteine and serine, rather than before "i'uur," isoleucine and leucine, then the Cys-Arg linkage in fragment 3 would have been broken, resulting in an additional fragment. The cleavage has nothi.tg t do witl disulfide linkages, so choice A is eliminated. We have no information to assess whether thermolysin has any interactions witi protic side chains, so while choice B may be true, we cannot choose it. Based on the third fragment, thermolysin'must cleave before isoleucine (Ile) and leucine (Leu)' As it is, thermolysin also cleaves beforeialine (Val). This makes choice C your best choice.

87.

Experiment II and Experim-ent III, the fragments cat-t 6e orretlappeJand matched to sequence the eitire peptide. For instance, Experiment III gives the first four amino acids as His-Lys-Met-Cys. Knowing the third and fburth 11ino acids in the polypeptide identifies the first two fragments in Experiment II (N-HIs-Lys-Met-Cys-LeuGly-Ala-Phe-Cys-Arg-C), which defines the third fragmeniin Experiment II by default. The cumulative resul: gives the entire poiypeptide, so choice D is the best answer.
88.

that histidine is the first amino acid in the polypeptide aiong with comparison of the f.afm"its f.o-.

Choice D is correct. Histidine is the only amino acid that appears at the N-terminal of one of the three fragments in both Expe-riment II and Experiment III, so histidine must be the first amino acid. Coupling the fact

Choice A is correct. Histidine is the only amino acid at the start of a fragment in at least one fragment ir. Experiments II, III, and IV. This means that the polypeptide must begin'with histidine, which eliminate: choices C and D. Alanine is the only amino acid at the end of u f.ug*"nt ii at least one fragment in Experimenis II, III, and IV. This means that the polypeptide must end with alanine, which eliminates choices B and C. Th; only- choice that begins with histidine and ends with alanine is choice A, making it the best answer. This can be verified by matching the fragments from the three experiments, which shows"that the correct sequence mu:: start with His-Lys-Met-Cys and end with Ser-I1e-Tyr-Ala.

89.

Choice B is correct. Choices C and D are eliminated, because in an organic solvent, the N-terminal -. deprotonated and the Cterminal is protonated, resulting in both sites being tincharged. For the experiment t. work, the amino terminal must be neutral and nucleophiti.. choice A is JtiminatJ, because u prttl. soh.e..: would be reactive, not inert. A protic solvent (such is ethanol) can attack acetic anhydride and undergc . substitution reaction at the carbonyl carbon. The best answer is choice B.
Choice C is correct. Technique 1 involves the use of a brucine salt, which forms a precipitate with a D-amL-: acid' The precipitate is what is isolated from the mixture in solution in relatively pure form, not the species '.. solution, so the best answer is choice C. Because of the precipitation of the O-amino acid, rather than the -amino acid, choices A and B are eliminated.

90.

91. Choice B is correct. An increase in entropy involves an ilcrease in randomness. This occurs with a phase char,:. that increases the volume occupied by ihe material or a reaction that generates an increase in the number :: molecules. Choice A is eliminated, because it involves purification Jna separation of stereoisomers, r,r,hi:: increases the order. The reaction of a D-amino acid with an achiral species r-rlith"t. increases nor decreases f entropy, so choice C is eliminated. In choice D, a solute is converted lnto a soiid, which involves a decrease _ entropy of the system. This eliminates choice D. The only increase in entropy occurs when one enantiomel --. racemized into a mixture of two enantiomers, because they are no longer separate and ordered. This ,,-ldri,: choice B the best answer.

Choice B is correct. The enzyme acylase cleaves an acyl bond (found in an amide). Individual amino acids ha'" no peptide linkages (amide bonds), so amino acids must first be acylated to form a bond that can react r.,.-:-= acylase' Acetic anhydride will acylate aI1 of the amino acids, but acylase will only remove the acyl group frr: molecules with the correct stereochemistry. This describes choice B. Because the amino terminal is tied ur the amide bond, it cannot be protonated, and thus the acylated amino acid is only charged at the terminal. This makes the species anionic, not cationic, which eliminates choice C. The ihiral center is :.. : "urUo.l affected, given that the priorities remain the same, so choice D is eliminated. The amino terminal is : :: protected by the acyl group, because there is no reason to protect it. This eliminates choice A, and makes chc :-.
B the best answer.

Copyright O by The Berkeley Review@

236

AMINO ACIDS

&

AMINES EXPLANATIO\S

93.

removal of one isomer and racemization, twenty-five percent of the originat product is D and twenty-five percent of the original product is L. After the removal of one isomer and iacemization again (the third purification), twelve and one half percent of the original product is D and twelve and one half"percent of the original product is L. This means that only twelve and one half percent of the desired enantiomer is isolated fr.om the third cycle. After this point, the percent isolated is less ihan ten percent, and is not worth the effort. The best answer is choice A.
94. Choice B

Choice A is correct- Choice C should be ruled out, because hog renal acylase is an enzyme, and thus it is not consumed in the process. Choice D can be eliminated, because acetic anhydride is achiral, so it shows no preference for one enantiomer over another. The original mixture is fifty percent D and fifty percent L. After

is correct. .stirring will not vaporize (sublime) a solid unless it results in extreme heating. Even with sixteen hours of stirring, not enough energy will be introduced into the system to get a notable amount of phase change. This eliminates choice A. Stirring implies that the solid does not dissolv! weil into the solvent, so the role of agitation is not to centrifuge the solid to the bottom, but to increase the collision frequency and thus increase the reactivity. This eliminates choice C and makes choice B the best answer. Generating a suspension increases the surface area for collision (and thus increases the reaction surface), causing the riaction rate to increase, not decrease. This eliminates choice D.

95.

the benzoyi group, it never gains the proton that an amine group *o.rid gain, so there is no hydrogen to be deprotonated. This eliminates choice B. The brucine salt was broten apart uling a strong acid, noia strlng base, so choice C is eliminated' Treatment with strong base in an aqueous hydrolyzes the amide bond and "nrriron-un"t thereby removes the benzoyl group from the N-terminal. This makes choice A thl best answer, an answer you should pick if you know what is best.
95. Choice

Choice A is correct. The addition of strong base removes the carboxyl terminal and amino terminal protons, if there are protons on the amino terminal to be removed. Because the nitrogen is involved in the amide bond with

D is correct. In technique 2, the racemate is first treated with benzoyl chloride, which N-acylates all of the amino acids in solution, both D and L. The strychnine salt is formed from the selective precipitation of the L-amino acid after it has been N-acylated. Choices A and C can be eliminated, because they contain no benzoyl group attached to the amino terminal nitrogen. Choice B has an R-stereocenter at the c,-caibon, while choice D has an S-stereocenter at the c-carbon. Choice D is the correct answer choice, because S-stereochemistry is associated with an "L" amino acid. You should recall the mnemonic, "you're either a DoctoR or you LoSe," where D and R go together and L and S go together.

97.

concentration of neutral methyl amine. Methyl amine must have a lone pair available to attack the electrophile, so if it is protonated, it cannot be a nucleophile. The implies that at lower pH values there is less neutral amine, so the reaction is slower at lower pH values and fastei at higher pH values. The correct graph should show greater rates at higher pH values, so choices A and B are eliminited. Because pH is a log scalL and it is located on the x-axis, the relationship of the rate and pH is not linear. Choice C is i better answer than
choice D.
93.

Choice C is correct. The real question is "what effect does H+ have on the reaction rate when methyl amine is added to methyl chloride?" The nucleophile is a good nucleophile and the electrophile is small, so tire reaction is an S52 nucleophilic substitution. In S1g2 reactions, the rate of the reaction depends on both the electrophile concentration and the nucleophile concentration. This means that the rate oi the reaction varies with the

Choice D is correct. The final product, after it has been neutralized (deprotonated and returned to an uncharged (H3C)2NH. molecule ?lil",l,: spectrum. This eliminates has two unique types of hydrogeni, so it should show only two peaks in its _This choices A and B, which show three signals rather than just two. The two 'HNMR signals are in a 6:1, ratio, but that is observed in both choice C and choice D, so we are no further along than before. The six like hydrogens split the neighboring signal into a septet (6 neighboring Hs + 1. 7 peaks) uia tn" = one hydrogen on nitrogen splits the neighboring signal into a doublet (t neighb'oring H + t 2 peaks). This means = iUNltR should hive a 1H seltet unJ u ZFI doublet, which confirms that choice D is that the a better answer than choice C, if correct answers are in fact better than wrong answers. Choose D for the warmth and glow of wisdom.

lopyright O by The Berkeley Review@

237

AMINO ACIDS

&

AMINES EXPLANATIONS

99' choice C is correct'- According to the question, the addition of base takes the amino acid from its first

equivalence point to the point at which the pH equals the value ot'-pKu3. rnis process can be observed using a titration curve, as drawn below. To carry oni thi, .or,version-lvoutarequire one and a half equivalents of strong base' when dealing with equivalents, the conversion is best be viewed conc"pi.rutty from a titration curve. 1.5 equivalents of strong base is equal to 15 mL given that the concentrations oi lysine and NaoH are equal and there are 10 mL of the lysine solution present itritiutty. select choic" c f*;;itml experience.

pH
PH = PKug PH = PKaz

10.80

l: i.ii t i

PH = pKu
2.1

HrAt
mL 0.10 M NaOH(aq) added

100'

Choice B is the best answer.

Hz/Pd should be familiar reducing agents. In choice B, there is notiing to provide electrons to nitrogen (reduction is the gain of electrons)'- znc has an oxidation state of +2 and Jhlorine will not give up electron*,

Choice B is correct' This is trivial knowledge, so based on memory, you can say A and C are valid methods" Should your memory fail you, the reaction must be reduction of Ure nitrogen containing compound. LiAlHa ard

Copyright @ by The Berkeley Review@

23a

AMINO ACIDS

&

AMINES EXPLANATI

Separation Technigues

Section Vm
Organic Chemistry Laboratory Techniques
by Todd tsennett

a)

Distillation

i. ii. v.
iii. iv.

Vapor Pressure Distillation Apparatus Simple Distillation Fractional Distillation Vacuum Distitlation

b)

Chromatography i. Mobite Phase and Adsorbents ii. Thin Layer Chromatography iii. Ri values iv. Column Chromatography v. Designer Columns vi. Gas Chromatography (OC)

c)

trxtraction i. AcidlBase trxtraction ii. Analyzing Flow Charts

Purifi cation Techniqueg


Mixture: Amine, Carboxylic acid, Fhenol, and Hydrocarbon Organic solvent I Weak base(aq) Organic

a)

layer

Aqueous laYer

Recrystallization i. Solvent choice and Kefluxing ii. flot and Cold Filtration iii. Solvent Washes

Identifi cation Techniques


Organic

layer

Aqueous laYer

a)

Physical Properlies i. Melting and tsoiling Points

Organic layer
I

Aqueous layer

b) Chemical Tests

to"a{nontact

Hvdrbcarbon

Color of Phase Changes Limiting Reagents c) Derivative Formation i. Melting Point Evidence
d) Mass Spectroscopy i. Base and Parent Peaks

i. ii.

ii.

Fragmentation

REKI{EMY
Speci aLtztng

l)R-.E,.v.l.E.wt
in MCAT Preparation

Laboratory Techniques Section Goals


You must be familiar *itn tn sure liauid chromatography, column chromatography, gel c"hrdmitdgraphy, unO 6"rlif.'r.tfi"t6*rr"[". Ali;f ll: T:h.tgle:.111Yglvlng chromatography have basically-the s5me funcrion. They ail dip6nd on a compounds.atftnity for a mobile phase versus its affiriity for the stationary phise as if migrates
dor,r"n the column.

understand and be able to applv the different forms of chromatographr

underqtand and be able to appty the different forms of distillation.


the difference in the apparatus and set-up for'each of the four. [n addition, vo, *"rt know the purpose.of each, the sifriation where each is best apptied, una lr'" ;;;1;;;;"ri;;i;;;""nrages of each of the techniques.
There are four forms of distillation to know: simple, fractional, steam, and vacuum. you must know

understand and be able to apply the different types of extraction. Extractionbasicallyinvol.ygsthgseparationofcompouna' Denavlor ln two tmmtsctble solvents. You must recognize common solvent mixfures that will b'e biphasic, and their solubility properties. Be able to diaw a flow.nrriloittu"J.iJ **t.r.tion and acid/base extraction, which simpty alters the pH of the aqueous pn"i" l" ili"JaiJ extraction.

"

solid and form nice long crystals. The crystal form of a compound is assumed'to be prirer than the powder form. The technique is a multi-step process whose details vou should understand.
a

Understand and be able to Recrystallization is applied to purify

lization.

Chemicaltestsinvolvereagentsthatselectivelyreactwithasma1lnumb";m a.ph.ase .lt.u"gg or color chanfe upon.doing so. You to ,nJ";;i;tiJ1h[l;;l; ,r11$ergo,either """a behind
chemical tests and be able to interpretlhe iesults.

Be able to deduce structural features

chemical tests.

Be able to deduce structural features using mass spectroscopy. \bu must have a fundamental idea of how the. inrtrur*r"'t op*rut"s in terms of ion p.odu.ti,or., i* separation, and particle detection. You must know what a base peak and parent rleak are and be able to identify them on a typical mass spectroscopygraph. YoJ mustbe'able to'identifv tvoical isotopic d!*tributions for chlorine, bromine, hydrolen]arid carbon. You must be able to intl'ipret graphs Td it doing so identify not onJy the original ipecies, but also be able to identify the fragmbnts that are formed during the process.

1l

il

rfi

il fr

Organic Chemistry

Lab Techniques

Introduction

once a compound has been synthesized, it must be separated, purified, and identified. But let's be pragmatic about things. The aveiage student taking the MCAT doesn't care much about organic chemistry lab techniques, otherwis" tn"y would be aiming for graduate school in chemistry and not -edicitt". So our goal is to present organic chemistry lab techniques in enough depth to ansia,,er multiple choice questions, and no more. Although there are rnu.ry techniques for separation and purification, we shall limit the techniques to distiilation, chromatography, extraction, and crystallization. once purified and isolated, next you must identify what you have collected. Taking melting points, boiling points, and doing identification tests goes only so far in identifying a compoundl It requires much more once you start dealing with rarger molecules. But on a multiple choice exam such as the MCAT, there is a great deal of narrowing down already done for you, As far as studying for the MCAT goes, our concern is with the simple compounds, logical analysis, and the process of eli-itratio.,. Lab procedures are listed as a topic in the MCAT student Manual, they appear in the. AAMC sample passages and questions, and have appeared frequently on different versions of the MCAT over the past few years. We will rnr,r"y a few lab techniques, the rationale behind each technique, and their effectiveness under various conditions. This section is written from a perspective that assumes you did only enough work in your organic chemistry rib courses to turn in yourlab write-ups and didn't spend much energy to learn the theory. The important information to extract here is the theory behind each technique. For instance, the MCAT Student Manual lists boiling points and distillation aslopics, so distillation is a probable passage topic that can be coupled with theoretical boiling point questions. Lab techniques can be broken down into three categories' sepirition, purification, and identification.
To separate anything, one component must move away from the other, whether it be in different directions or at different speeds. There are four separations performed in the lab: 1) solid from liquid, 2) solute from liquid, 3) liquid from liquid, and 4) solid from solid. For each case, there is rnor" thutr one technique possible. For solid from a liquid, there is filtration or centrifugation couplea wiUr

decantation. For solute from liquid, there is distillation, precipitation coupled with filtration, extraction, chromatography, or ion exchange. For liquid from liquid, there is distillation. For solid from solid, there is recrystallization, density
gradient columns, molecular sieves, acid-base extraction, sublimation, or column chromatography. we shall address some of these techniques in this section.
To purify a solid, recrystallization is the most common method. The crystalline form of a solid is the most pure. The purification of a liquid is most commonly

carried out via distillation. For purification of hygroscopic organic liquidi, distillation from a drying agent (such as magnesium sulfate) is used to rehove yatel from the organic solvent. The drying agent binds water and the organic liquid can be boiled away free of any water azeotrope that might be pr6sent otherwise. Specific examples include aromatic solventJsuch as toluene. once a compound has been isolated, it must be identified by techniques such as IR, UV, or NMR spectroscopy, gas chromatography, thin layer chromatography, chemical tests (reactions with reagents that identify specific functional gio"pr;, and physical properties (optical rotation, melting point, boiling point, ind solubilities). This section will address each of these techniquei -except the spectroscopic techniques, which were addressed in section 2.
Copyright
@

by The Berkeley Review

24t

Exclusive MCAT Preparation

Organic Chemistry

Lab Techniques

Separation Techniques

$GH uili,ffiGH

e5

When two materials are in different phases, they can flow in different directions and therefore can be separated. A common sense view of separation techniques dictates that if you have a mixture of two more components, you must convert the system into a state where the components are in diffeient phases. For instance, distillation can be used to separate a tiquid from another liquid by heating the system until one component boils and consequently flows Lp ut i

To separate molecules requires that at least one of the molecules be in motion. We shall use the word flow to describe the motion of bulk material. The ability to flow depends on the state of matter, where solids cannot flow, liquids flow down, gases flow in all directions (including up), and solutes flow witha solvent.

away from the component that remains as a

separation techniques as an effort to get the materials to flow in different ways.

liquid. w" shall look ut utt

although there is a small amount of a less volatile cornponent that vaporlzes too. In distillation, after the vapor escapes from the surface of a liquid it cin travel up the distilling column. The vapor collides with the inner wills of the distilling column, where it condenses. The amount of condensation depends on the wall temperature. Some of the condensed liquid drips into the distilling flask and some reevaporates. Elery time the vapor condenses and reevaporates, it goes through a cycle of purification. The design of the distilling apparatus is such that by the time it reaches the top of the distitling column, it hai gone through enough purification cycles that nearly 100% of the vapor is the more volatile .o*pon"r,I. Vapor Pressure Diagram and Vapor Ratio The separation of two liquids via distillation is based on the vapor pressure ratio of the components in solution. The vapor pressure of a material dlpends on its heat of vaporization, LHvaporizalior-r, and its percent composition in Jolution. As AHvaporizatlorl increases, if is harder for the molecules to escape solution, so the vapor pressure is reduced. As the species' mole fraction in solution decreases, it takes up a smaller percentage of the surface area, and therefore cannot have as many molecules escape per unit area. This too reduces the vapor pressure. To address this phenomenon, Figure 8-1 shows the vapor p."srrr" asi function o{ temperature for two hypothetical components in a mixture of two liquids.
-----

Distillation Distillation removes a liquid from either another liquid or from solute by exploiting their boiling point differences. Upon healing, the most volatile component converts to a gas more readily than the less volatile components,

More volatile species


Less volatile species

Vapor Ratio (More : Less)

3:1

9:7

27 :7

-t *l
nl
6l
init.
Temperature

rl
9:1
27:1

Sol'n

1st

Condensate 2td Condensate 3td Condensate


Solution Ratio (More : less) Figure 8-1

Copyright O by The Berkeley Review

242

The Berkeley Review

Organic Chemistry

Lab Techniques

Separation Techniques

What this graph and calculation demonstrate is that over three evaporationcondensation cycles, the system becomes richer and richer in the more volatile component until the ratio is 27 : 1, which equates to a mixture that has 96.4% of the more volatile component. This values gets larger and larger, approaching 100%, with each additional evaporation-condensation cycle. As the surface area in the distilling column increases, the number of cycles increases, and thus the purity increases. For a mixture where the ratio of vapor pressure of the more volatile component to the vapor pressure of the less volatile component is small, more surface area is required to get good separation. This is what is referred to as fr a c ti o nal dis tillation.

Distillation Apparatus
The heat is added at the lowest point, because heat rises through the solution and

homogenizes the system through thermal circulation. As liquid at the surface of

the solution in the distilling pot evaporates, the vapor rises up the distilling column. At the top of the distilling column, the vapor can flow down the side arm, which is enveloped by a cooling jacket, resulting in condensation of the vapor. The condensed liquid drips from the side arm into the collection flask. Figure 8-2 shows the basic components of a distillation apparatus.
The set up shows an opening between the side tube and the collection flask, which allows the system to vent any build up of pressure that takes place as the liquid vaporizes. As the vapor forms, it pushes the air that is initiatly present in the distilling apparatus out through the venting hole. The system can be sealed and a drying tube can be fitted to the collection flask if it is iced down, so that moisture from the air does not enter the collection flask and condense with the distilled product. An ice bath around the collection flask is used when the distillate is highly volatile.

Thermometer

Heating

Mantle

Collection flask

Figure 8-2 Simple Distillation versus Fractional Distillation Distillation can be carried out under conditions that are referred to as either simple distillation or fractional distillation, depending on the amount of surface area in the distilling column. Fractional distillation employs a distilling column with more surface area than simple distillation. Simple distillation is employed with liquids that have a large difference in boiling points, while fractional distillation is employed with liquids that have a small difference in boiling points (generally less than 30'C in difference, although that is not an absolute number.) Simple distillation may also be employed to remove a solvent from a solute. Simple distillation is faster than fractional distillation and it generates a higher yield, while fractional distillation leads to greater distillate purity. Copyright
@

by The Berkeiey Review

243

Exclusive MCAT Preparation

Organic Chemistry

Lab Techniques

Separation Techniques

Fractional distillation is the same as simple distillation for the most part with the exception being that the distillation column is filled with an inert solid to provide more surface area on which the vapors can condense and then re-enaporaie. This serves to re-distill the mixture as the vapor climbs up the distillation column. Most often, the column is packed with glass beads, because glass is both inert and cheap. The column must be fitted with a trap in the inner core so that the glass beads do not fall through the column into the distillation flask. Fractionai distillation allows the liquids to be separated more efficiently and thus in a purer fashion. The apparatus in Figure 8-2 shows the set up for simple distillition, because the distillation column is empty.
Example 8.1

which of the following mixtures can be separated using simple distillation?

A. B. C. D.

Alpha-D-glucopyranose frombeta-D-glucopyranose Butanol from diethyl ether 1-Pentanol from 2-methyl-1-butanol Acetone from propanal

Solution
Simple distillation separates a solvent from a solute or a liquid from a liquid when they have a large difference in boiling point. The largest difference in boiling point is found with choice B, an alcohol and ether of equal mass. choice A is two diastereomeric solids and choices c and D are of two liquids with roughly equal boiling points. They cannot be separated by simple distillation.
Example 8.2

fractional distillation of acetic acid from trichioroacetic acid? A. Paraffin wax B. Magnesium turnings

which of the following materials could be used to pack the column in

the

C. D.

Aluminum mesh
Glass beads

Solution The material chosen must provide surface area, remain a solid at elevatec temperatures, and be inert. Paraffin wax has a iow melting point, so choice A is eliminated, because the wax will melt and drop into the distillation flask. Metacan oxidize in the presence of acids, resulting in metal oxides that can neutralize the acids, so choices B and C are eliminated. The best answer is choice D because the glass beads are inert and will remain solid at the elevatec temperatures used in distillation.
Vacuum Distillation Vacuum distillation involves attaching the distillation apparatus to a vacuur. pump at a point after the condensate has been collected. The purpose is lc reduce the pressure in the apparatus and in doing so lower the boiling poin: You should recall that the boiling point is defined as the temperature at whicr the vapor pressure equals the atmospheric pressure, so lowering atmosphen: pressure lowers the vapor pressure required to boil, which results in a lorr'e: temperature needed to boil. This is done when the compounds have a hig:. boiling point, higher than the flash point or decomposition point.

Copyright

by The Berkeley Review

244

The Berkeley Relier*

Organic Chemistry

Lab Techniques

Separation Techniques

Chromatography Chromatography is the separation of two or more components in a mixture by exploiting their differences in solubility in a migrating solvent and their affinity for a polymer. Both factors play a significant role in the separation process, so they must both be considered. The various chromatography techniques are all centered around a sort of swimming race between the components in the mixture to some arbitrary finish line. What is meant by "race to the finish" is that different components exhibit different rates of migration across the surface of a stationary phase, referred to as the adsorbent, depending on their solubility in the mobile phase, (the solvent), and their affinity for the stationary phase. The solvent is migrating through the adsorbent and the components are traveling with the solvent to a varying extent. As the solvent moves, the components are towed along with it in most cases. The more soluble the component is in the solvent,

the further

it is pulled by the solvent through the adsorbent. Likewise,

the

greater the affinity for the polymer, the slower the component migrates.

Mobile Phase (Solvent) Properties versus Stationary Phase Properties Because separation in chromatography depends on relative affinities, we should consider the nature of typical materials. The mobile phase is one of several possible solvents, ranging from nonpolar (hexane) to polar/protic (ethanol), or in some instances a mixture of two solvents. When mixtures are used in column chromatography, it usually starts with one pure solvent followed by different mixtures which gradually become richer in a second solvent. Typical solvent
pairs include ether-hexane and methanol-methylene chloride.
The stationary phase is one of two materials, alumina or silica gel. Both of these polymers are polar, so in the absence of any other information, we know that polar materials have a higher affinity for the stationary phase than nonpolar materials. As a consequence, polar species have slower migration rates than nonpolar species in chromatography. How much slower depends on the material and solvent. In addition, the activity of a polymer can be affected by the solvent, where the solvent binds the silica gel or alumina and thereby reduces the interaction of the migrating solute with the polymer. For instance, as alumina gains moisture, it is less able to bind other materials, so it is said to be less active. Alumina comes with a Brockmann activity rating from I to V, where V represents highly moist alumina which exhibits low activity. Anhydrous alumina is more polar than anhydrous silica gel.

The only thing that can be said with certainty in chromatography is that polar species in nonpolar solvents have slow migration rates while nonpolar species in nonpolar solvents have a fast migration rate. The questions on the MCAT will likely stress this point, where you can simply refer to the idea "like dissolves like"
to answer them.

Thin Layer Chromatography (TLC) Thin layer chromatography, TLC, is carried out on a small scale to identify the number of components or type of components in a mixture. It involves spotting small aliquots of sample in a line parallel to the base of the plate near the bottom of a vertical, rectangular plate with either silica gel or alumina on its surface. This plate is then placed into a container that can be fitted with a lid and soivent is added to the container until it is just below the levei of the spots. It is important to not have the solvent level in the container above the spots, otherwise they can dissolve into solution. Due to capillary action, the solvent slowly migrates up the plate, interacting with each of the spots as it travels. In order to prevent the solvent from evaporating from the surface of the plate, a lid is employed to maintain a closed system. Some chemists place a piece of filter
Copyright O by The Berkeley Review

245

Exclusive MCAT Preparation

Organic Chemistry

Lab Techniques

Separation Techniques

pencil.

paper on the inside wall of the jar, not touching the plate, to provide additional surface area from which the solvent can evaporate. This keeps the chamber saturated with vapor. once the solvent is neariy to the top of the plate, the plate is removed from the container and the top of the solvent front is marked ,rrir,g u

The plate can be deaeloped in one of several ways, such as spraying ninhydrin on the surface, adding it to a closed chamber with iodine ..yrtalr, or shining UV light on the plate. In some way, the spots must be made visible for evaluation. Once it is determined where the spots ended, one can ascertain such information as how many components were in the mixture and whether the components were nonpolar, semi-polar, or highly polar. By initially spotting a plate with a know. material you suspect to be the unknowr .o-po.r'rd -in ihe original sample, the spots of the unknown composition can be compared to the knlu,n spot to determine identity of one of the unknown compounds in the original sample. A TLC set trp is shown in Figure g-3.

Lid-----+
lar

Thin Layer Chromatography

TLC plate (with siiica gel or alumina)

Sample (spotted onto plate)

Solvent top

Example 8.3

Thin layer chromatography can be used to do all of the following EXCEpT: A. determine the number of components in a mixture. B. distinguish the relative abundance of enantiomers. C. determine the best solvent for column chromatography. D. monitor the progress of a reaction.

Solution TLC separates the components in a mixture by their relative affinities for the solvent and the adsorbent. As a result, the number of components in the mixture can be ascertained from the number of spots on the plate. This eliminates choice A. Because column chromatography uses the same solvent and adsorbent as TLC, the results of TLC can serve as a preview of column chromatography, and thus be used to test potential solvents. This eliminates choice c. Samples of a reaction can be taken and analyzed by TLC to determine the appearance or disappearance of either a reactant or product. This eliminates choice D. There is no quantitative capability with TLC and enantiomers travel at the same rate, so

r" t"t"t"t"a,*

-heycann",
Copyright
@

by The Berkeley Review

246

The Berkeley Review

Organic Chemistn"y
Example 8.4

Lab Techniques

Separation Techniques

If the sample on the plate were submerged below the top of the solvent, then the sample would:

A. B. C. D.

migrate more rapidly than expected. dissolve into solution rather than climb the plate.
react with the plate. cause the plate to dissolve into solution.

Solution If the sampie on the piate is placed into a flask at a level that is lower than the top of the solvent, then the cornpound is free to dissolve into the solution, and thus it will not migrate up the plate. This eliminates choice A and makes choice ts the best answer. The plate has an inert adsorbent, so no reaction takes piace. This eliminates choice C. The plate does not dissolve into solvent, and even if it did, the height of the plate wouldn't matter. Choice D is eliminated.
R1 Values

In TLC, we often measure an R1 value for the compounds. We can think of an ftg value as a "ratio of the fronts," where the distance a spot travels is compared to the distance that the solvent travels. The distance that the solvent travels is measured from the line on which the spots start, where the solvent first interacts with the sample, to the line where the solvent front finishes. The distance the spot traveis is measured from the center of the spot when it starts to the center of the spot when it ftnishes. Some spots deform and spread out as they climb, so it is best to use the center of the spot as reference. The distance the component travels is divided by the distance the solvent travels to yield its Rl value. The component with the greatest R1 value is the component that has the least affinity for the stationary phase (adsorbent) and is most soluble in that solvent. The range for R; vah-res is: 0 < R; < 1, because there is a chance the spot does not travel at ali or that it travels almost at the solvent front. The R1 value is a physical constant associated with a solute,/solvent/adsorbent combination. The basic idea of all forms of chromatography is separation by relative affinity.
Example E.5

What is NOT true about the Rlvalue for a compound? A. It cannot be greater than 1.0" B. It has no units. C. It is different in different solvents. D. it is always the same on the same adsorbent.

Solution
The spot cannot travel faster or farther than the solrrent carrying it, so the lli value cannot be greater than 1.0, eliminating choice A. Because it is calculated by dividing a distance by a distance, the units cancel out. This means that R1 values have no units, which eliminates choice B. Because solubility is different in each solvent, the impact of each solvent varies from solvent to solvent. As a result, the R1 vaiue differs with each solvent, which eliminates choice C. Because the migration rate depends on the affinity of the compound for both the adsorbent and the solvent, the distance it travels in a given amount of time depends on the adsorbent and solvent. The R1 value varies with adsorbent, making choice D not true about the It1 value for a compound and therefore the best answer.

Copyright

by The Berkelev Review

247

Exclusive MCAT Preparation

Organic Chemistry

Lab Techniques

Separation Techniques

case the solvent is nonpolar while in the other case, the solvent is polar. The nature of the compound can be inferred from the relative R6 values obiained for a series of solvents. Because a solute dissolves best into a solvent of "like" nature, the higher the value of Ri, the more that the solute is like that solvent.
Solvent front stopped Solvent front stopped

Figure 8-4 shows the TLC plates at the start and after completion of the migration and development with iodine crystals for two separate Tr,C experiments. In one

dsolvent

After Nonpoiar Solvent

d.po,
dsolvent

il
Before

dsolte.t

,dspot

o'=*
After
Polar Solvent

Figure 8-4

for column chromatography. The best separation occurs when the R6 values

Based on R1 values, you should be able to make predictions as to the best solvent

have the greatest multiplicative (not additive) difference. For instance R6 values of 0.2 and 0.1 separate better than Rlvalues of 0.3 and 0.2.

Column Chromatography Column chromatography is applied to separate bulk quantities of product. It operates on the same principles as thin iayer chromatography: the solubility of a compound in the solvent as it flows down the column versus the affinity of the compound for the adsorbent in the column. If the compound migrates quickly down the column, we can conclude that it must experience a minimum attiaction to the silica gel or alumina (stationary phase) in the column. In column chromatography, the "race to the finish line" is to the bottom of the column and the components in the mixture separate according to how quickly they finish. To set up a column, it is first packed with a plug (glass wool works well) followed by a thin layer of sand (to form a flat surface), then a large layer of silica gel, followed again by a thin layer of sand, to protect the top of the silica gel from disruptions in its flatness. The top and bottom of the silica gel must be flat and parallel to one another, so that components all travel the exact same distance through the silica gel no matter where they start. we assume that the compounds have no affinity for the sand or glass. solvent is added to the column to saturate it. Once the top of the solvent is flush with the top of the
sand, the sample can be added.

t
,l@

IM

d
ifr6

The sample is dissolved into minimal solvent, which is poured gently into the top of the column. The solvent slowly flows down the silica filled glass column to an opening at the bottom, taking the compounds at different rates as it goes. Samples of the solution are then collected in increments that are analyzed foi anv components tirey may contain. Faster migrating compounds reach the bottom of the column first, so they have shorter elution times. EIuion time is definecl as the time it takes a component to travel the length of the column and drip out. Like the Ri value, elution time is a measurement from the experiment. A large R+ value corresponds to a low elution time, given that the compound has i fast migration rate in each case. Each aliquot of solvent collected from the column L. analyzed by sorne technique (usually spotting on a TLC plate and then either illumination with a UV lamp or treatment with 12 crystals) for the presence of Copyright
@

by The Berkeley Revierv

24a

The Berkeley Revien

Organic Chemistry

Lab Techniques

Separation Techniques

components. The aliquots containing the component are consolidated and the solvent is driven off via evaporation. Figure 8-5 shows a column at different stages, where the bands separate as they migrate down the column.
Solvent is added so it will flolv down the column
Sand layer

Less soluble component More soluble component

f---l
t-....'.._...-t
lrIlt[ililr4

f-_-t

hF# t.:.:.;.:.:.:'t
L.-.-.-".._'_.1

hFFrC
[;:':':;:':':']
t-..._.-._.-.-l

t..-.-..'....-t

Iii,:.:ii.:':.1

[.'i......',..j
[.:.:.:.:.:.:.] l:.:.:.:.:.:.:{ Second

Silica gel

[;,,i,i'i'i,i'i
t.............t
l[[ilil[ilra

ltttrIruilrl

component

l:.:.:.:':.:.:l

still migrating

[.:::':t:::.:i]
[:::::::::::::] [.:.:.:.:.:.:-l t.:.:.:.:.:.:.1

[::i:i:::::i:ij

Sand layer

l.:.:.:.:.:.:.1 l:.:.:.:.:.:.:{

t-'-'........,t

First component has reached the bottom

-t|
6

t=0
Figure 8-5
Example 8.6

t-?

6 t=3

ry
6 ,_ /

Column chromatography relies on which force to separate compounds?

A. B. C. D.

Gravity Gas flow Magnetic attraction Centrifugal

Solution
The solvent migrates down the column, because gravity is pulting the solvent down. The compounds are pulled along by the solvent as it migrates down the column. The driving force behind the overall flow of solution is gravity. This

makes choice

A the best answer. Gas flow is associated with

gas

chromatography, not column chromatography, so choice B is eliminated. There is no magnetic field or circular motion, so choices C and D cannot be true. you should make the correct choice and select choice A. Designer Polymers (Chiral Columns, DEAE-Cellulose, and Affinity Columns) Given that the migration rate through a column depends on the affinity of a compound for the polyrner, there are some columns designed with specific molecules in mind. while the application of such columns is more common irr biochernistry than organic chemistry, the basic idea is still rooted in the fundamentals of coiumn chromatography. An extreme case of designing a polymer is observed in biochemistry, where an antigen column is used to bind and isoiate antibodies from a solution. one problem encountered in such an experiment involves horv to release the antibody from the column. once everything else has eluted from the column, the column can be treated with a

Copyright

by The Berkelev Review

249

Exclusive MCAT Preparation

Organic Chemistry

Lab Techniques

Separation Techniques

varying pH buffer until the antigen and antibody denature enough to no longer bind one another. one problem with changing the pFi of the column is the
chance that the proteins are irreversibly denatured. In such cases, the antibody may instead be released by adding a salt solution of gradually increasing ion concentration or by adding free antigen to compete with the bound antigen for the antibodies.

In organic chemistry, chirsl columns have become popular in recent years, because they are capable of selecting for one enantiomer over another. If the desired product is a compound with a hydroxyl group on a stereogenic carbon with S-chirality, then a column with R-alcohois on the polymer will bind and thereby hinder the flow of the S-enantiomer through the column. As a result, the S-enantiomer has a greater elution time than the R-enantiomer, so they separate as they traverse the length of the column. The fundamental difference between column chromatography in biochemistry and organic chemistry is that in organic chemistry, the aim is to have all of the compounds elute at different times where in biochemistry the goal is to have all but one compound elute.
The most common designer polymers on the MCAT to date are the adsorbents used in ion exchange chromatography. Three with which you should be familiar

are sulfonated-polystyrene, carboxymethyl-cellulose (CM-cellulose), and diethylaminoethoxy-cellulose (DEAE-cellulose). The first two carry negative
Proteins with high pI 'u'alues, ones that are rich in lysine, arginine, or histidine, will bind CM-cellulose columns, ailowing them to be isolated. Proteins with iow pI values, ones that are rich in aspartic acid or glutamic acid, will bind

pH.

charges at neutral pH, while DEAE-cellulose carries a positive charge at neutral

DEAE-celluiose columns, allowing them to be isolated. Typicaily with sulfonated-polystyrene columns, the protein mixture is added at low pH so all of the amino acids and small peptide residues bind the column. The pH of the coiumn is gradually increased, releasing amino acids according to increasing pI values. Figure 8-6 shows DEAE-cellulose and sulfonated-polystyrene.
DEAE-Cellulose

OH

OH

o
CHt

H-O
o
OH CH,

t-

t-

OH

"t'

Sulf onated-polystyrene

i-= \.4
OH
OH Figure 8-6 Copyright O by
Ti-re Berkeley Review

250

The Berkeley R.eview

L--:-

Organic Chemistry

Lab Techniques

Separation Technigues

of three components.

Gas Chromatography Apparatus Cas chromatography, GC, works by first vaporizing a sample (mixture o{ components) into the gas phase and then forcing that organic ,rupo, through a packed column using elevated pressure from a ryti.ra"t o"f an ir-,eit gas lusriaity helium and sometimes argon). The machine measures retention time on the column by recording collisions as gas molecules leave the end of the long, coiled column and strike the detector. The collisions are converted to peaks (like a bar graph), which can be integrated to determine their relative abundance. A greater area for the signal indicates a greater quantity of material presentl The compounds in the samplemust be highly volatile at 200"C and inert with regards to silica gel or wax for GC analysis to work. Figure g-7 shows the design"for a standard gas chromatography machine and a standard chart output for a mixture

Injection port, through which a sample may be added by syringe

Packed

column

"eri"J-iim". identification can be obtained directly from gas chromatography. The retention time depends on the nature of the co]umn and the"compound.
Figure 8-7

column and into the dete-ctor. The detectbr reads Iolisions. The chart *=14.,iJ*;";;, -ih;i';; graphs intensity of collisions

",,nt, gas at this point forces the vaporized sample

Carrier.gas is added

oJ^,. rn.

r"r,*H the packed coiled tir".i!n

are actually read from right to left, where Jignals on the right rep"resent components with smaller eiution times than components on the left. To determine the purity of a product mixture (and relative abundance of components), the graph can be read directly. The purity of the product mixture can be observed directly in the total number of peaks that are ,"L. o., the graph. To ascertain information about one of the components in a mixture, u- p.r.u sample of that component can be added to a gas chromatography apparatus and the retention time for that particular component can -be determined. An alternative way to identify a component involves the addition of a small sample of that compound to the mixture to enrich the sample in that component. That component corresponds to the signal that increases its relative size. This technique is referred to as spikirzg the sample with a component. Gas chromatography is used as both an identification tool for the reaction mixture and" as a tooi for measuring the kinetics of a reaction. By measuring the sample mixture at given intervals, the growth of a peak can be monitored lo determine the rate of formation of a product. Figure g-g shows a typical graph obtained from the GC apparatus. GC peaks are relatively sharp as the graph rho*r.

The relative quantities of the components can be determined by integrating the signals associated with the compounds. GC output is continuorrr, ,oih" gr"aphs

Copyright

by The Berkeley Review

251

Dxclusive MCAT Preparation

Organic Chemistry

Lab Techniques

Separation Techniques

Retention time

ttt*
Example 8.7
Gas chromatography does NOT work with:

A. B. C. D.

alcohols. aldehydes.
ketones. fatty acids.

Solution
case, the component that cannot work is the component that cannot vaporize in the gas chromatographer. The temperature of the vaporizing chamber is often higher than 200"C, a high enough temperature to vaporize most organic liquids, but a fatty acid cannot vaporize at that temperature. The fatty

In this

acid is likely to melt and decompose at this temperature, so it cannot be identified using gas chromatography techniques. Choice D is the best answer.
Analysis of GC Data The MCAT is apt to ask data analysis questions rather than details of operation for the GC apparatus. It is imperative that you be able to interpret graphs such as the one in Figure 8-8. GC analysis focuses on quantitative aspects of the mixture rather than specifics about the structures. Given that the compounds vaporize instantaneously upon addition to a GC machine, the boiling point does not affect the elution time. However, because many columns are often slightly polar, affinity for polar species is greater than affinity for nonpolar species. A second factor that dictates the migration rate is mass. Fleavier gases move slower and have longer elution times. Heavier gases and polar gases typically have high boiling points, so although boiling point has nothing to do with elution time, the factors that raise boiling point also increase the elution time in a GC. This is why many chemists say that molecules with lower boiling points come off of a GC column first.
Extraction

it is typically employed to take advantage of drastic differences in the solubilities of components in two different (immiscible) solvents. To start, we first need to define partitioning. A compound has a different solubility in every solvent. The ratio of its maximum solubility in one solvent compared to its maximum solubility in another solvent is known as the partition coefficient. In cases where the compound can go into either solvent, it will select between the solvents in a ratio equal to the partition coefficient. When
Extraction works based on solubility and
the solvent system involves two immiscible liquids, then the solute must split Copyright
by The Berkeley Review

252

The Berkeley Review

Organic Chemistry
between the two solvents. some components

Lab Techniques

Separation Techniques

the solvents, while the remaini.g will dissolve primarily into the other solvent. This means. that th-e "o*po.,ents of a mixture components &n be separated if they have differing partition coefficienis. By decanting one solution from the other,_the components are then separated from one ur,oih", in their solute form according to their relative solubilities in the two different solvents. From this point, you must remove the component from the solvent in which it is now dissolved. This can be done via disiillation or controlled evaporation.

wil

dissolve primarily into one of

Extraction does not always involve two immiscible solvents, although that is most often the case. A sotid component can be removed from a mixture of solids by adding a solvent r: whif it is highly soluble and the other compounds are relatively insoluble. This is how many nitural products are isolated from plants.
Acid-Base Extraction
V\4ren one of the two immiscible solvents is water, the properties of the solutes can change when the solution pH changes. Acid-base extraction in essence separates compounds by their pKu values, By using
a biphasic solvent

(heterogeneous system),-selective protonation or dJprotonation (depending on the compound) will either increise or decrease tire water soiuuitity of"the compound, depending on whether an ion is generated. Loss of charge results in an increase in preference for the organic layeiwhile a gain of charge results in an increase in the preference for the iqn"o.,s layer. onJe extracted into the water layer, the species can be returned to a neutral state by adjusting the pH, and in most cases it will crash out of solution. Figure B-9 showi the f]owchart for the butanoic acid (CH3Cltrr2CH2CO2H), and ethyl benzene 1bU3ICH2C6H5;.
50.0

mixture

acid-base extraction of a mixture of n-iutylamine (CH3CH2CH2CH2NH2), Mixture: Butylamine, Butanoic acid, Ethylbenzene

mL ether

50.0

mL

10% NaOH(aq)

Layer I (organic)

Layer

II

(aqueous)

toz lHcrluq;
Layer

III
I

(organic)

Layer

IV

(aqueous)

Butanoic acid

evapprate Ethylbenzene

rozlNaoHlaql
Butylamine Figure 8-9

the carboxylic acid. The carboxylic acid form is .,ot .r"ry ivater soluble, so it precipitates out of the water. Butyl amine is removed from the ether layer by extracting it with acidic water, which forms butyl ammonium cation, u i,ignry water soluble species. Butyl ammonium cation can be "crashed out" of solution by adding strong base to the aqueous rayer. The ether layer contains the remaining ethylbenzene, which can be isolated by evaporating tire ether away.

In the first extraction, aqueous sodium hydroxide solution deprotonates butanoic acid to form butanoate, a carboxylate anion (which is highly water soluble due to its negative charge). Brrtylamine and ethylbenzene rerialn in the organic layer. The butanoic acid can be "crashed out" oi solution by adding acid tJregeneiate

Copyright

by The Berkeley Review

253

Exclusive MCAT preparation

Organic Chemistry

Lab Techniques

Separation Techniques

Example 8.8 To separate benzoic acid (C6H5CO2H) from toluene (C6H5CH3), it would be best to use a mixture of an:

A. B. C. D.

alcohol and water at pH = 11. alcohol and water at pH = 3. ether and water at pH = 11. ether and water at pH = 3.

Solution
To carry out an extraction, the two solvents cannot be miscible in one another. Alcohols are typically miscible in water, so choices A and B are eliminated. Benzoic acid can be deprotonated in basic solution, where it becomes its anionic conjugate base (benzoate) which is readily soluble in water. Toluene dissolves into the ether rather than water, so the best answer is choice C.

more soluble in

more soluble in

ether than water

water than ether

insoluble in water

Separatory Funnel and Partitioning into the Biphasic Layer The physical act of separation is most often carried out in a separatory funnel. A separatory funnel is an upside-down, pear-shaped flask with a fritted glass cap on top and a stopcock controlled valve on the bottom. The two layers naturallr' separate, with the less dense layer assuming the top. The system is mixed and shaken to ensure complete interaction of the two layers. One thing to note is that the aqueous layer has a meniscus, so the layer with the most contact against the glass walls is the aqueous layer. To increase the differences between the tn o layers, the aqueous layer is often saturated with salt. Such a solution is referrec to as a brine solution.

Copyright

by The Berkeley Review

254

The Berkeley Revierr

Organic Chemistry

Lab Techniques

Separation Techniques

shows a typical four-compound flow chart. It is a gtod general rule that a weak base should be added beiore- a strong base, so as to isolite the stronger acid. If you were to add the sodium hydroxide first, it would extract both the"phenoxide and the carboxylate into the aqueous layer.

Flow Chart with an Ether, Amine, phenol, and Carboxylic Acid On the MCAT, perhaps the most difficult task when it ctmes to extraction will be analyzing a flow chart. Due to the time limitations of the exam, the chart cannot be too exhaustive, so we shall consider a four-compound mixtrrre. Figure g-10

HeC

dd
0.10

NHC H3

cH2cH3

0.10

M NaHCO3(at)

y"
M NaOH(aq) lacidify and

{ruter

",'d'd",

cH2cH3

d" #""
t
I

lacidify and filter

cH2cH3

distill away from residue

d""
Figure 8-10

As is true with all lab techniques, the conditions must be inert for the technique to be effective. Because extraction involves both acidic water and basic water, there is the potential for hydrolysis. As such, extraction cannot be used to isolate water-sensitive compounds such as acid anhydrides, acid halides, and esters. These compounds can be extracted from a pu$ into an organic sorvent, but they cannot undergo acid-base extraction. Copyright O by The Berkeley Review

Special Cases including Esters, Anhydrides, and Acid Halides

255

Exclusive MCAT preparation

Organic Chemistry

Lab Techniques

Purification Techniques

tdbhilifi$d5

Manl of the separation techniques we have discussed so far can also be used as purification techniques. Column chromatography is useful for purification, because the bands, separates nicely from one another. Depending on the compounds, the isolation of a few grams of highty pure material is common in :?ty1- chromatography. Distillation, when ao"neLultiple times, can generate a highly pure liquid. But rather than describe chromatography and distillation again with the attention to detail that makes fot pntili.aiion, we shall onlrdiscuss, recrystallization. your number one goal is to zero ir-, on topics ,n"i n"r. a good probability of showing up on the MCAT, so we shall consider recrystallization in the purification section.

solution, to avoid any reduction in the yield.

out. upon cooling the solution, the product precipitates out of solution in a " crystalline form free of the impurities from be?ore. The crystals are filtered at low temperature, to prevent any of the material from dissolving back into

first dissolve a product mixture leaving solid impurities in ,olltior-,

the purified crystals. By adding a minimal amount of-solvent in which the solute is saturated at high temperature but insoluble at a lower temperature, one ma'

the insoluble solid impurities, and then slowly cooling the solution to preciiitate

A solid product may be separated from solid impurities through selecti'e precipitation, which organic chemists refer io as recryslallizotion. Recrystallization involves dissolvjng the solid into hot solvent, htt filtering out

Recrystallization of a Crude product Mixture

i,

f ?iit"r"a

solvent Choice (Dissolving sorute into Minimar Refluxing solvent) In recrystallization, solvent choice is essential. Because the Jolid is dissolved into refluxing soivent, it is ideal that the desired material is highly soluble at elevated temperatures. The term refluxing describes a system" where the solvent is boiling, but it is condensing on the walls of the fiask and dripping back into solution. You may recall that acid reflux is a condition where your gastric fluids up your esophagus only to fall back into your stomach. Reflux is the "Ji-P climbing and returning process. In order to maximize the amount of crystal collected, the desired material should be insoluble (or minimally soluble) in the solvent at lower temperatures. The solvent can be a homogeneous mixture of two solvents to meet desired solubility properties. ny mixing solvents, it is po-ss_ible to change the overall polarity of the solvent and thus chinge the overall solubility of the solute at both high temperatures and low temperatures.
Example 8.9 if a solid is too soluble in water at room temperature, which of the following solvents?

-"!

ffi

I
5
_
t! 5

it would be best to add

A. B. C. D.

Hexane Ethanol

Diethyl ether Tetrahyclrofuran

Solution
ov-e1a,ll polarity and hydrogen bonding capacity of the solvent mixture and be soluble in water. All of the choices areless polar and less protic than water, but of the choices, only ethanol is soruble in water. Choice s is ihe best answer.

The key to this question is that the solvent you mix with water must reduce the

::
:_l

-,rvright

by The Berkeley Review

256

The Berkeley Review

Organic Chemistry

Lab Techniques

Purifi cation Techniques

Decolorizing (Adding Charcoal to solution to Bind Colored Impurities) In some instances, the purification can be enhanced by adding a material to preferentially bind some or a1l of the impurities. one such .ur" oJ".r6 when you h.ave colored impurities and a colorleis target crystal. By adding activated charcoal, charcoal that is free of impurities, the colored species will be preferentially bound by the surface of the charcoal and can thus be filtered out. Charcoal binds all organic materials, but because of its n-network and the pl-esencg of conjugated n-networks in colored species, charcoal has a higher affinity for colored organic molecules than colorlesi organic molecules. Hot Filtering (to Remove Insoluble Impurities, i.e., boiling chips) Filtration is employed to remove solids from liquids. It is i laboratory technique patterned after everyday activities, such as collecting spaghetti in a colander while running water over it. In organic chemistry, a fine mi&oscopic net is used to separate the solid from the liquid by allowing the tiquid to flow through while hindering the flow of solids, which are larger than the pore size of the filter. The solids are thus collected by the filter. Filtration can also be used to separate large solid particles from small solid particles by setting up two filtering sites if different size. The first filtration shouid be of a largei pore size than the second filtration so that the larger solid may be collected firit and thereby separated from the smaller solid, which will then be collected in the second fiiter. This is sometimes referred to as sieae filtering.

filtration allows large insoluble solid particles (crystals) to be removed from the solvent (mother liquor). As the solution flowi through the filter, the solid particles are collected in the pores of the filter. Vacuum iiltration uses a filter in conjunction with a vacuum, so that the liquid flows faster. As with all types of filtration, insoluble solid particl"s ate .errroved from a solvent. The liquii flows through the filter due to both gravity and the pressure difference between above t"t"- ,^" tttr. ^"a
Example 8.10

Filtering typically depends on gravity to force the liquid to flow through the filter, although it can be expedited by a pressure difference between the atmosphere above the filter and the atmosphere below the filter. Gravity

B. C. D.

which of the foilowing mixtures can be separated using gravity filtration? A. Ethanol from diethyl ether.
Sodium chloride from water.
Butane from dichloromethane. Oxalic acid from hexane.

Solution In this question, you must rely on experience and memory. In choice A, both compounds are liquids, so filtering is pointless. In choice B, the salt is completely soluble in water, so the salt is actually in solute form. It must be a solid, not solute, to be filtered, so choice B is eliminated. Butane is a liquid as is dichloromethane, as you may recall from lab experiences. This eliminites choice C, ieaving choice D as the oniy remaining choice, so it better be correct. Hexane is a liquid and all carboxylic acids, besides formic acid and acetic acid, are solids at room temperature. Acids do not readily dissolve into hydrocarbon solvents, so oxalic acid exists as a solid and not as a solute in hexane. choice D is comprised of a solid in a liquid, so it is the best answer.

Copyright

by The Berkeley Review

257

Exclusive MCAT Preparation

Organic Chemistry

Lab Techniques

Purification Techniques

crystals must lose this "dirty solvent" to be pure. The dirty solvent can be removed by washing the crystals with a solvent in which the impurities are soluble, but the target compound is not. It is optimal to use a solvent with a lort boiling point, so it can readily evaporate away from the solid. The rinsing solvent dissolves the impurities as it passes across the crystals. A common example is the removal of water from glass by rinsing the glass with acetone. You no doubt have done this in lab a thousand times (okay, maybe only three or four times, but 1000 is so much more impressive). The acetone does not dissoh-e the glass, but it does dissolve the water and rinses it away. The residual amount of acetone left behind on the glass can evaporate away leaving behind drv (anhydrous) glassware. solvent washes are often carried out on the solic collected from filtration. It's similar to use rinsing off with water once we have lathered up in the shower. If we didn't rinse the lather away, the water would evaporate and leave a soap film on our skin. We opt to rinse in a pure, volatile solvent in which we are insoluble, at least most of us do.
Example 8.11
The best choice with which to rinse sodium acetate in order to remove propanoresidue is:

Crystallization, Cold Filtering (to Collect Crystals), and Solvent Wash The final step in a recrystallization procedure is to collect and isolate the target crystals. If the solution is slowly cooled from its refluxing state, it will precipitate long, pure crystals. These crystals are collected using filtration, but with cold solvent, so as to prevent the loss of product to dissolving. The crystals at thrs point still have a residue of solvent with soluble impurities on their surface. The

A. B. C. D.

hydrochloric acid. propanol.

diethyl ether.
water.

Solution The goal of rinsing a product is to remove the dirty solvent (in this case propanol) from the surface of the desired product (in this case H3CCO2Na without reacting with the product nor dissolving the product. Choice A i; eliminated, because HCI will protonate the sodium acetate to form acetic acic Choice B is out, because you cannot add propanol to remove propanol (tha: would be the same as washing a dirty shirt in mud to remove the dirt). Choice D is eliminated, because sodium acetate can dissolve into the water. The bes: choice is the only choice left, choice C. Diethyl ether will dissolve the propancwithout dissolving the sodium acetate. The propanol can be rinsed away u'if. the ether and only residual ether will be left behind in the sodium acetate. Th.e ether can easily evaporate away from the sodium acetate crystals leaving behint
pure sodium acetate.

T],

s
D
b|:

-J

$iir

lfftg

Copyright O by The Berkeley Review

2Sa

The Berkeley Revien

Li

Organic Chemistry

Lab Techniques

Identification Techniques

K
isolated, it is important to be able to characterize the material. compounds are most easily idfntified by physical properties. IR, uv-visible, and NMR spectroscopl hiyl arready r"ur, alr"Lrsed, so we shall consider mostly chemical diagnostics in this sectioi. nf-rtu-,gerprinting .o-porr'a, with their physical properties, it is possibre to distinguish or," frorriur,other. Chemicar tests for specific functional groups car, urro"b" employed. In some instances, a compound can be converted into a new compound (ierivative), and the physical properties of the derivative can be used to help identify the or(inat compound.
Physical Properties It is common to verify a compound or ascertain its purity from its melting point. A rapid change from a solid into a liquid (occurring orrJ, t*o J"grees or less) is indicative of a pure compound. sharp boiling po"int, ur" ,",oi as obvious. To identify a solid, one can mix an unknown with the compound they suspect it is. A,sharp melting point confirms that the unknown is in ti"t tnuicompound, while a broad melting range indicates that the compound is something else. This is the mixed melting point tesi. The name comes from mixing the unknown and known to,f:tr a homogeneous mixture._ other physicar properties to compare include solubilities and optical activity. \Arhatevir physical diff"r"r,." io*porr.d, *uy have can be exploited to distinguish one unknown from either a second or a standaid (known). Table 8_1 shows rhe :19:f :lmfarabte "o*po,r.,d
The last aspect of lab we shall consider are the identification techniques. once a compound is

,t.;;il;i;
-95'C 48'C -64'C

Substance Acetone Benzophenone

Density (g/ml)
0.79 1.1s
1.49

Melting Point Boiling Point


56'C 306'C
61.'C

Chloroform
Cyclohexane

0.78 0.79 1.15 0.85

6'C
-98'C 94'C

Methanol Naphthalene
Stearic acid

81"C

65'C 288'C 297'C

70'c

Table 8-1

C. D.

A. Cyclohexane B. Chloroform
Stearic acid

Example 8.12 According to the data in Table g-1, which of the foilowing compo'nds is a solid at room temperature and sinks to the bottom of the frask #n"', uaa"a to water? Naphthalene

Solution
The compound is insoluble in water and d.enser than 1.00 grams/mL,because it sinks, and doesn't dissolve, in water. Incorrect density er#inates choices A and D' Chloroform has a merting point of -64'C, soit is a tiquid at room temperature. This eliminates choice B. Niphtharene, choice C, is a ,oua ut room temperature, because it has a melting poinf of 94'C. Choice C is the best answer.

Copyright

by The Berkelev Review

259

Exclusive MCAT preparation

Organic Chemistry
Example 8.13

Lab Techniques

Identification Techniques

If a student suspects that the compound they have is L-tyrosine, what should
they add to the sample for A. L-Tyrosine B. D-Tyrosine
a

mixed melting point?

C. D.

Xylose

Glycine

Solution
To support the notion that an unknown compound is L-tyrosine using a mixed melting point study requires mixing the unknown with a pure sample of Ltyrosine. If the unknown compound is in fact L-tyrosine, then the mixture is in fact pure L-tyrosine. This would result in a sharp melting point at 114'c, the melting point for L-tyrosine. If the unknown compound were not L-tyrosine, the melting point range would be broad and would fall outside of 114"C. The best answer is choice A. D-tyrosine, although having the same melting point as Ltyrosine, won't work, because the enantiomers interact with each other differently than they do with themselves. Pure enantiomers typically do not pack into crystals as well as racemic mixtures, so they have lower melting points.
Example 8.14

A melting point range from 31.5"C to 38.5"C supports which conclusion? A. The compound is very pure. B. The compound is highly reactive at low temperatures. C. The compound will boil at a temperature less than 138.5'C. D. The compound has impurities. Solution
The melting point range does not indicate anything about the reactivity of

reactivity, so choice B is eliminated. Knowing a compound's melting point does not give any hint as to its boiling point. This eliminates choice C. A range of 7'C is broad in terms of melting point, and a broad melting point range is associated with an impure compound. The only reasonable answer is choice D.
Example 8.15 To distinguish D-leucine from L-leucine, it is best to compare their: A. specific rotations. B. solubility in ethanol.

compound. As such, no conclusion can be made about the compound =

C. D.

melting points.
odors.

Solution
D-leucine and L-leucine are enantiomers, so they have opposite optical rotatiorr. of equal magnitude. To distinguish the two enantiomers from one another, it :s best to compare their specific rotations. Their solubiiity and melting points should be identical, because of their structural similarities (enantiomers have the same physical properties.) Their odors may or may not be different, because the olfactory lobes are chirally sensitive, but we cannot determine for sure that the',' have different odors. we know for sure that they rotate plane-polarized light i:. opposite directions, so the best answer is choice A.

Copyright O by The Berkeley Review

260

The Berkeley Revien

Organic Chemistry
Example 8.16

Lab Techniques

Identification Techniques

A. benzene. B. ether. C. 5% acid solution. D. 5% base solution.

The difference in solubility between 4-nitrobenzoic acid and glyceraldehyde (2,3dihydroxypropanal) in water is most dissimilar with their rotnUitltv difference in:

Solution Nitrobenzoic acid gets deprotonated in a basic solution and the anion that is formed is more soluble in water than the original acid. Glyceraldehyde has no acidic or basic sites, so its solubility is not affected by the pli of the soiution. For this reason, nitrobenzoic acid shows greater solubility in tne 5% base solution than pure water while glyceraldehyde shows roughiy the same solubility in water as 5% base solution. The best answer is choice D. The important point on this is that you are looking for differences in solubility in witer and .question another solvent, not differences between 4-nitrobenzoic acid and glyceraldehyde in their solubility in benzene, ether, acidic solution or basic solution, This wai in fact a really poorly worded question, which happens from time to time on the MCAT.
Example 8.17

Butanol can be distinguished from butanone by comparing all of the following EXCEPT their:

A. B. C. D.

boiling points. freezing points.


densities.

infrared spectra.

Solution
Butanol and butanone have different boiling points and melting points, because an alcohol is polar and protic while a ketone is simply polar. This eliminates choices A and B. The infrared spectra for both comporrt ds differ greatly, because butanol has a broad peak around 3500 cm-l that butanone does not have and butanone has a sharp peak aroun d 1700 cm-1 that butanol does not have. These two peaks can be used as features that will differentiate the two compounds. This eliminates choice D. The two compounds have different densities, but the question does not allow for all four choices to be eliminated. You are looking for the "best" answer. Both compounds have roughly equal masses and are tiqirids

at room temperature, so their densities are not going to be that diffeient. Although all of the choices work, the best answer is choice C.

Copyright

by The Berkeiey Review

261

Exclusive MCAT Preparation

Organic Chemistry

Lab Techniques

Identification Techniques

Chemical Tests Chemical tests involve reactions that are selective for very few functional groups. The ideal chemical test is positive for only one functional group, but thisls rare. Most chemical tests have multiple functional groups that"give positive results. For a chemical test to be effective, a few things iustte true."

1) 2)

3)

The reagent must react with very few functional groups. The chemical testing reagent must undergo some visible change, such as a color change or phase change, so it can be detected. The testing reagent must be the limiting reagent, so that any excess of the testing reagent will not interfere with the reading.

We shall focus on the tests associated with compounds other than carbohydrates. Table 8-2 shows four common chemical tests used in organic chemistry.

Compound Tested For

Reagent

Positive Yellow precipitate Color change: orange to green Color change: brown to clear Color change: purple to clear

Methyl ketones Iz(s)/OH-(aq) 1'and 2'alcohols, aldehydes KzCrO+/HzSOq Alkenes Brz/CCL+


Unsaturated fats

IBr/CClt
Table 8-2

Do not memorize any of these tests. Know the concepts behind a chemical test, but expect that the passage will provide the details about a specific chemical test.
Example 8.18 1-Iodohexane is distinguished from 3-iodo-3-methylpentane A. reaction with ammonia.

Mosr readily

by:

B. C. D.

molecular mass. water-solubility. the iodoform test.

Solution
1-Iodohexane and 3-iodo-3-methylpentane have the same molecular weight, so choice B is eliminated. Alkyl halides are not very water soluble, so choiie C is eliminated. The iodoform test is for methyl ketones, not alkyl iodides, so choice D is eliminated. The difference between them is that 1-iodohexane is a primanalkyl iodide and 3-iodo-3-methylpentane is a tertiary alkyl iodide. rhe prlmari.

_kyY,"@""
Example 8.19 makes KMnoa in basic water not useful as a chemical test agent? A. It reacts with too many compounds. l. It undergoes both a phase change and color change. C. It is too inert and requires a high concentration. D. It is a gas with low solubility in water.
\21'/hat

Solution
Permanganate can oxide many compounds, including alkenes, alcohols, and aldehydes. Although it has a distinct color change, lt reacts with too manrcompounds for a conclusion to be drawn. Choice A ii the best answer

Copyright

by The Berkeley Review

262

The Berkeley Review

Organic Chemistry

Lab Techniques

Identification Technigues

Derivative Formation Derivatives are products formed from the reaction of a given reagent with an unknown compound. Derivatives are made to be used is a diagnostic for an unknown. The additional physical measurements associated wlth the newly formed derivative can be.used to identify both the derivative and indirectly the lnkqown' They are used to support and confirm an identity for an unknown. The.functional group on the unknbwn compound must be kno*n to decide what
dinitrophenylhydrazones for ketones or aldehydes, benzylesters for alcohols, and osazones for sugars. The ideal derivative is a solid thai can easily be identified by its melting point. An example of a 2,4-dinitrophenylhydrazine derivative is shown in Figure 8-11.

derivative should be synthesized. co*,'o.

derivatives incrude

2,4-

H
I

Nq

*AH
Aldehvde

..N- N
I

o.Y*-ig.
Phenylhydrazine derivative

Ketone

Phenylhydrazine derivative
Figure 8-11

Example 8.20
The limiting reagent of a reaction has a molecular mass of r2g grams per mole and the product has a molecular mass of 160 grams per mole. If 0.20 grams of the limiting reagent generates 0.20 grams of prodirct, then what is the percent yietd?

A. r00% B. 89% c. 80"/" D. 75%

Solution lercent yield is found by taking the actual yield (in either grams or moles) and dividing it by the theoreticai yield (in either grams or molJs). In this case, it is easier to solve using moles rather grams. Either way you choose, you must be certain that the units cancer out. The mathematics is as follows:
0.2

%yietd

ufrl?lTol", ='-7teo9l^orc ='l* = rheorericat ; motes 0 2


Sl.

gl

The yield is 80%, choice C. The percent yield for a reaction must be calculated using the limiting reagent. Most numbers should result from easy calculations.

^^ l12gol^ob

s,t

lrZ, ,Lv

-728 -64- =

160

= -'J 80 10 s.6

Copyright O by The Berkeley Review

263

Exclusive MCAT preparation

Organic Chemistry

Lab Techniques

Identification Techniques

B. C. D.

Example 8.21 To distinguish 2-hexanone from 3-hexanone, it is best to compare: A. their boiling points.

their reactivity with an oxidizing agent. their alcohol solubility. the melting points of their phenylhydrazine derivatives.

Solution
The two compounds (2-hexanone and 3-hexanone) have different boiling points, but only slightly different, because they are both six carbon ketones. This eliminates choice A. Ketones are inert with most oxidizing agents, so choice B is eliminated. Ketones have similar alcohol solubilities, ro C is invalid. To "hoi." distinguish compounds, derivatives are made. Phenylhydrazine derivatives of 2hexanone ar-rd 3-hexanone have different melting points, because they do not pack the same in the solid phase. This makes choice D the best answer. As a point of interest, the iodoform test could work too, but it is not a choice. Example 8.22

which of the following physical properties of the 2,4-drnitrophenylhydrazine


derivative of a ketone is best to use as an identity diagnostic? A. Density B. Molecular weight C. Boiling point D. Melting point

Solution
Phenylhydrazones are solids, so it is not feasible to compare their boiling points. This elimrnates choice C. Their densities and moleculaimasses are not going to be as useful in distinguishing, because most will have similar values foideniitrand molecular mass, or close enough that no strong conclusion can be drawn. This eliminates choices A and B. Melting point is ih" dirting,rishing physical e best answer choiie D. tr"t"ttt r",-""" a
Example 8.23

C. rate of evaporation. D. index of refraction.

To distinguish an alcohol from an ether of approximately the same molecular mass, all of the following can be employed EXCEPT comparing their: A. boiling points. B. water solubility.

Solution An alcohol exhibits hydrogen bonding while ether does not. This increases the boiling point of the alcohol, which also makes it harder to evaporate than an ether. An alcohol has a higher boiling point and slower rate of evaporation than an ether of roughly equal mass, which eliminates choices A and -. Hydrog"n bonding also increases the water solubility of an alcohol of roughly tire sJn" number of carbons as an ether. This eliminates choice B. The index of refraction may very well be different for an ether and an alcohol, but it is not predictable like boiling point and water solubility. This makes it hard to distinglish which compound is the ether and which is the alcohol, so choice D is the beJt answer.

Copyright O by The Berkeley Review

26'4

The Berkeley Review

Organic Chemistry
Mass Spectroscopy

Lab Techniques

Identification Techniques

The last laboratory technique we shall address is mass spectroscopy. Because the

mechanics of the device are covered in physics, we shall focus on the data and interpretation only. At the level of the MCAT, there are only three things we should understand about the data. First, we can ascertain the molecular mass of a compound with great accuracy from the parent peak (peak of highest mass). This can help us to distinguish alkanes from alkenes and alkynes of equal carbon count. Second, we can determine if the compound has either chlorine or bromine (from the 4istribution of their isotopes). Chlorine has two abundant isotopes, 35Cl and 37C1, ir-a ratio of approximately 3 : 1. Therefore, when we see some of the higher mass peaks in a 3 : 1 ratio for the M and M+2 signals in a grouping, we can conclude that the peak combination must be due to chlorine. Bromine has two abundant isotopes, 79Br and 81Br, in a ratio of approximately 1 : 0.98. A peak ratio of 1 : 1 for the M and M+2 peaks in a group generally indicates the presence of bromine in the compound. Lastly, we can determine the mass of the alkyl groups attached to either a carbonyl carbon or a heteroatom. From the difference is mass between the parent peak and the base peak (the tallest peak, which is assigned an intensity of 100 on the y-axis of mass spectroscopy graphs), we can ascertain the alkyl groups that have fragmented off of the original cation.

The one fact we need from physics is that the machine detects ions, and in our case, cations. In some cases, the cation is also a free radical, but that is irrelevant to the operation of the machine. The basic ideas behind the device are that charged particles move in a circular path when traveling perpendicular to linear magnetic fields and that heavier particles traverse a circular path of greater radius than lighter particles. The machine accelerates the cations and then causes a 90' turn, which allows particles of different mass to spread out before they strike a detector. Figure 8-12 shows the mass spectroscopy graph for butanone.
100

Base
80

U]

CJ bU

CJ

t40
../
20

38 40 42 44 46 48 50 52 Molecular Mass (amu)

Figure 8-12
The mass of the most abundant molecule of butanone is 72.0575 g/ rnole, so the peak at 72 is d:ue to the radical cation formed when a nonbonding (ione pair) electron was ionized from oxygen. In ionizing a hydrocarbon, a bonding electron is lost from a carbon-hydrogen bond. An electron is ionized when a molecule is

bombarded with high energy incident particles, usually electrons. Because cations and free radicals are unstable, the species rearranges and fragments,
Copyright
@

by The Berkeley Review

265

Exclusive MCAT Preparation

Organic Chemistry

Lab Techniques

Identification Techniques

sheering off pieces of the molecule. The mass spectrophotometer detects any charged fragments that splinter off, so the peaks that are observed correspond tb the most stable cationic species that rearrangement and fragmentation can form. The peaks themselves represent fragments and the difference between peaks correspond to uncharged fragments that sheered off. In Figure g-r2, the peiks of interest are found at 72, 57 , 43, and 29 . The peak at 72 is the parent peali, so it is attributed to the cationic compound. The peaks at 57 and 43 correspond to the acylium ions formed when the parent compound loses a methyl group and ethyi group respectively. The peak at29 is possibly due to an ethyl cation.
Example 8.24

In mass spectroscopy, the material being analyzed should be: A. at low pressure and in the gas phase. B. at high pressure and in the gas phase. C. at low concentration and in the liquid phase. D. at high concentration and in the liquid phase.

Solution they must be in the gas phase. This eliminates choices c and D. In order to minimize the number of peaks, the material is at low pressure, so that the highly reactive free radical and cationic particles do not collide and undergo reactions. If there are too many molecules, they can combine, which would lead to more complicated data. The pressure should be low, so choice A is the best answer.
Example 8.25 present on the molecule?
Because the particles must travel independently of one another in the apparatus,

which of the following peak combinations would imply that chlorine


A.
B.

.L'"LL
131 132 133 134 135

was

137 732 133 134 135

737 732 733 134 135

737132 133 134 135

Solution
As mentioned earlier, chlorine has two abundant isotopes that come in a ratio of approximately 3 : 1 and differ in mass by 2. we shall ignore the peak at 135 to interpret the ratios, because that peak does not vary from choice to choice. Choice A shows peaks in a 2 :7 : 2: 7 ratio, which in no way can be interpreted as 3 : 1. This eliminates choice A. Because the peaks that differ in mass by 2 are of roughly comparable height, it is more likely that the species contains bromine. Choice B shows peaks in a 1 : 1 : 2:2 rctio, which in no way can be interpreted as 3 : 1 in favor of the lighter peak. This eliminates choice B. Choice c shows peaks in a 9 : 4 : 3 : l..3ratio which shows a 3 : 1 ratj.o for the 133-to-131 peaks and a 3 : 1 ratio for the 134-to-132 peaks. This makes choice C the best answer. Choice D shows peaks in a 9 : 3 :2:8 ratio, which in no way can be interpreted as 3 : 1. This eliminates choice D. Although it may seem arbitrary as to how the height ratio was determined, only choice C shows peaks differing by 2 n mass with a height ratio of 3 : 1 in favor of the lighter one.

Copyright

by The Berkeley Review

266

The Berkeley Review

Organic Chemistry
1,.

Lab Techniques
g)

Section Summary

Key Points for Lab Techniques (Section Separation Techniques

a)

Based on different flow directions or different flow speeds

b)

Distillation i. Converts the most volatile component into a gas so it can flow up the distilling column and away from the mixture ii. Vapor pressure differences are maximized at the boiling point of the least volatile component iii. The distillation apparatus is comprised of a distillin g pot, a distilling column, a thermometer, a condensing side arm, and a colection frask simple distillation has minimal surface area in the distilling column i. Maximizes yield at the expense of purity ii. It is employed when the boiling points of the components differ by a
large amount.

c)

Fractional distillation has significant surface area in the distilling column i. Maximizes purity at the expense of yield ii. It is employed when the boiling points of the components differ by a small amount (less than 30'C). Vacuum distillation is employed when boiling points are high i. Carried out in a closed system where the internal pressure is actively reduced

d) e)

Chromatography separates by relative affinities for a mobile versus


stationary phase i. Mobile phase is the solvent which travels across the adsorbent ii. Adsorbents (the stationary phase) are polar polymers through which or across which migration occurs iii. Thin layer chromatography provides a flat surface across which the solvent migrates, taking solute with it iv. Ri values are the ratio of the solute migration distance to the soivent migration distance in thin layer chromatography

v. column chromatography is a separation and/or purification vi.

f)

technique where a mixture separates as it travels down a column Elution time is defined as the time it takes from when the solutes first interact with the adsorbent to when they exit the base of the column vii' Gas chromatography (GC) involves the vaporization of a mixture which migrates through a hot column and is pushed along by an inert carrier gas Extraction involves the removal of a component from a mixture by selective solubility in a new solvent i. Partitioning describes the distribution of a compound between two solvents

ii. Partition

iii. Acid/base extraction involves partitioning between an organic iv'


solvent and water, where the pH is varied to enhance the solubility of compounds that can form ions when protonated or deprotonated The yield is better when carried out in three consecutive extractions of small volume rather than one extraction of large volume

compound in one solvent versus another solvent

coefficient is the numerical ratio of the solubility of

v. Extraction flow charts involve tracing the pathway of various


components in a mixture when they travel through the steps

Copyright O by The Berkeley Review

267

Exclusive MCAT Preparation

Organic Chemistry
a)

Lab Techniques

Section Summary

Purification Techniques Recrystallizationpurifies a solid by dissolving it and then crystallizing it in a slow, methodical fashion i. The compound _must be highly soluble at high temperatures and

ii. iii. iv. v.

minimally soluble at low teniperatures The dissolving step is carried out under refluxing conditions to optimize the solubility A decolorizing step is carried out when there are colored" impurities and a colorless compound. Activated charcoal is typically used Hot and cold filtration are used depending on whether the compound must stay in solution or remain crystallized solvent washes are employed to rinse away residual solvent that may contain impurities

Identif ication Techniques

a) b)

i. Melting points and boiling points are the common


properties used
as a diagnostic

Physical properties are a quick way to verify the identity of a compound

physical

for a compound's identity

ii. c) i. d)

Chemical tests involve reagents that react with a minimal number of functional groups i, Color or phase changes indicate a positive test
The test agent must be the limiting reagent in the reaction

of a compound

Derivatives are formed to provide additionar evidence as to the identitv Melting points are the most common physical property measured for a derivative

from fragmentation Base and parent peaks represent the most stable molecular ion and the ionic form of the original compound respectively ii' Fragmentation occurs when the unstable species formed upon ionization undergoes chemical processes in an effort to form a more

Mass spectroscopy converts a molecule into a cationic radical and then accelerates, deflects, and separates the cationic species that are formed

i.

stable species

iii.

Bromine on compound results in a 1 : 1 peak ratio for the high mass fragments while chlorine on compound results in a 3 : 1 peik ratio for the high mass fragments

Copyright

by The Berkeley Review

26a

The Berkeley Revieu

Laboratory Techniques
Passages
l5 Passages
lOO Questions

r I

Suggested schedule: r' After reading this section and attending lecture: passages I, IV, vl, & x Qrade passages immediately after completion and log your mistakes. II: Following Task I: Passages II, VII, x, & xr (2g questions in sz minutes) Time yourself accurately, grade your answers, and review mistakes. III: Review: passages III, v, VIII, xII, XIII, & euestions 94 - loo Focus on reviewing the concepts. Do not worry about timing.

R. E. V. I . b.

\llT@

altztng in MCAT Preparation

$,fi.

I. Distillation and Separation I - Z) II. Fractional versus simpre Distillation @ - 14) III. steam Distillation of Natural products es - zl) IV. Caffeine Extraction Experiment e2 - 2g) V. Partition Coefficient Experiment eg _ bS) VI. Acid/Base Extraction Experiment 66 - 42) vll. Extraction and Thin Layer chromatography Experiment (4b _ 49) vlll. Thin Layer and corumn chromatography (5o - 56) IX. Column Chromatography 6Z - 6Z) X. Gas Chromatography rc4 - Zl) XI. Recrystaltization e2 - Zg) xll. Qualitative Anarysis (Bo - 86) XIII. Synthesis and Extraction @Z - gJ) Questions not Based on a Descriptive passage (g4 - l oo)
llL,;

Lab Techniques and Spectroscopy Scoring Scale Raw Score


MCAT Score

lli
"ir

rr
i.

ivl

1l:.tn

84 - loo

15-15

66-85
47 -65

lo-t2
7 -9

['nel

nt;

34-46 1-33

4-6
I -5

Passage

(Questions 1 - 7)

2.

A student is given a mixture of two liquids to separate. There are three possibilities for the composition of the mixture: 1) equal parts by volume of Z-1,2-dichloroethene (b.p. 60'C) and E-1,2-dichloroerhene (b.p. a8'C), 2) equat
parts by volume of 3-pentanone (b.p. 102"C) and 2-hexanone ft.p. 128'C), and 3) equal parts by mass of diethyl ether (b.p.

For the experiment. the thermometer read 49.2"C asthe first aliquot was collected. The compound being collected is most likely which of the following?

A. B. C. D.
3.

E-1.2-dichloroethene Z-1.2-dichloroethene

Diethyi ether
3-pentanone

34"C) and di-n-propyl ether (b.p. 90'C). The student is uncertain as to which of the three mixtures he may have, so he uses fractional distillation to separate the mixture. Fractional distillation is employed when the boiling points of the components are less than 30'C apart fiom one another. The distillation apparatus is fitted with a collection vial that ;an collect the distillate in 10-mL aliquots. The apparatus ;ontains the following components: boiling flask, distilling ;olumn (with optional packing), thermometer, condenser *'ith cooling sleeve, and collection flask. Figure 1 shows the
distillation apparatus used by the student.

A boiling chip. when

added to the distillation flask,

prevents bumping (the sudden pop solution) by:

of vapor in

the

A. B. C. D.
4.

providing surface area fiom which to boil.


causing even distribution ofthe heat. reducing intermolecular forces. increasing intermolecular forces.

The majority of the vapor from the boiled liquid


condenses on which of the following areas?

Thermometer

Distillation
Column

Condenser with cooling sleeve

A. B. C. D.
5.

Cold glass in the condenser Cold glass in the collection flask Warm glass in the condenser Warm glass in the collection flask

Optional
Packing

Collection flask

To separate a dissolved solid (solute) from a liquid solvent in which it is completely soluble, it is best to
use:

Boiling Flask Figure

1 Distillation

apparatus used in the experiment

After approximately thirty minutes, the first drop of vial. The boiling flask is held .t a constant temperature and the distilling column is ,:lsulated with glass wool. A total of three samples (of
.rquid appears in the collection

A. gravity filtration. B. vacuum filtration. C. simple distillation. D. fractional distillation.


6.
Simple distillation would work best with which following mixtures?

:oughly 8 mL each) were collected before the distilling flask .,, as removed from the heat source. A small portion of tiquid :emained in the distilling flask after the heat source was :emoved. Once the system cooled back to room temperature, .!ere were approximately 10 mL of solution still remaining :r the distilling flask.
1

of

the

A. H3CH2COCH2CH3 and H3CH2CCOCH2CH3 B. H3CH2COCH2CH3 and E-i,2-dichloroethene C. H3CH2CH2COCH2CH2CH3 and


H3CH2CCOCH2CH3

When distilling a liquid with a boiling poinr of 56.6"C, you should use an apparatus with which of the

D.

H3CH2CH2CH2CCOCH3 and H3CH2CCOCH2CH3

following?

Two openings to the atmosphere, one before the


collection flask and one after the collection flask.

7.

Fractional distillation is employed to separate:

B. One opening to the


condenser.

atmosphere before the

One opening to the atmosphere after the collection

flask.

A. B. C. D.

agasfromaliquid.
a liquid from a gas.

asolidfromaliquid.
a liquid from a liquid.

D.
*opyright

No opening to the atmosphere, so no vapor


escape.

can

by The Berkeley Review@

211

GO ON TO THE NEXT PAGE.

Passage

ll

(Questions 8 - 14)

10. Which of the following is NOT an advantage of


fractional distillation over simple distillation?

A student intended to separate 10.0 mL of toluene from 10.0 mL of heptane. The boiling poinr of heprane is 9g.4.C and the boiling point of toluene is 110.6"C. To separate the
two liquids, the student could use either simple or fractional distillation. Simple distillation involves distillation through a short, hollow distilling column and it is chosen when the substance to be separated has a boiling point that is at least thirty degrees lower than any other component in the mixture. Simple distillation takes less time than fractional distillation and generates a higher yield. In the separation of heptane from toluene, it would be impractical to use simple distillation, because their boiling points are too close. Fractional distillation diff'ers fiorn simple distillation in that the distillation column has more surface area. This is accomplished either by using a longer column or by packing the column with an inert material onto which the vapors can condense. The advantages of fiactional distillation are that

. B.
A
C

The components that are collected are purer.

The fractional distillation procedure is faster than


simple disrillarion.

Fractional distillation allows for the mixture to continually re-distill throughour the distillation
column.

D.

Fractional distillation provides a more constani


vapor, thus reducing the tendency of the solution to

"bump".

11. Why is distillation bulb (at the base) placecl in a sanc


bath in a heating mantle rather than placed over a flame

which reduces bumping in the solution. Fractional distillation does not recover all of the component, however, and it can take up to five times as long as simple distillation. Table I lists the boiling points of some organic liquids.
Liquid

the product is purer and the vapor pressure is rnore constant,

. B. C.
A

The sand serves as a themal insulator.

D.

The sand is less reactive with the glass than the air. The sand bath keeps the vapor pressure of warer low by ahsorbing moisturc. The sand bath allows for uniform distribution oheat around the distillation bulb.

Boiling Point
34.6"C

Liquid
Octane

Boiling
Point

t2-

What is the role of copper mesh when it is inserted inl a distilling column?

Diethyl ether
Pentane

l25.l"c
138.0'C

36.1'C
65.4'C

n-Pentanol
Nonane

The copper mesh serves as a filter to collect solid particles that have become airborne.
vapor that forms.

ai;

THF
Hexane

150.8'c
158.3'C

B.
C

The copper mesh serves to neutralize any acii.:


The copper mesh allows for an even

68.9'C
82.3"C

Anisole
Decane

Isopropanol

t74.t'c

distribution

Table I
8

heat due to the electrical resistance of the copper.

D.

The copper mesh serves to provide addition.


surface area onto which vapors may condense a: then re-evaporate in fractional distillation.
_

Over time, the effectiveness of {iactional distillation is diminished. How can this best be explained?

the temperature increases, the amount of


condensation and reevaporation decreases.

Over time the distilling column heats up, and

as

13. Which of the following pairs of compounds is b.separated using simple distillation?

!t

B.
C

the temperature increases, the amount of


condensation and reevaporation increases.

Over time the distilling column heats up, and

as

Over time, the solution becomes poor in the more


volatile component, so azeotropes increase. Over time, the solution becomes poor in the more volatile component, so azeotropes decrease.

A. B. C. D.

Diethyl ether and heptane.


Tetrahydrofuran (THF) and hexane.
Octane and pentanol.

lfm'
i[it

r.

Nonane and anisole.

D.

14. Which stereoisomers would MOST likely shou


same boiling point?

--,:

When separating the following moiecule pairs using distillation, which pair requires the longesr clistilling
column?

. B. C. D.
A

Diethyl ether and pentanol.


Tetrahydrofuran (THF) and anisole.
Octane and pentane.

. B. C. D.
A

2R,3S-dichloropentane and 2R,3R-dichloropenr.,. 3R-bromocyclopentene and 35-bromocyclopen:: . D-Glucose and D-mannose 3R-ethy1-trans-decalin and 3S-ethyl-trans-decali:

Isopropanol and toluene.


212

Copyright @ by The Berkeley Review@

GO ON TO THE NEXT PAG[-

Passage

lll

(Questions 15 - 21)

17.

Because molecules can exist in the vapor state at temperatures below the compound's boiling point, it is possible to carry out distillation at temperatures below the boiling point of the components in the mixture when the

Standard distillation of lemon grass oil at 229"C could result in all of the following EXCEPT:

mixture does not form an ideal solution. A common technique for carrying this out is steam distillation. Steam is directed to the distilling flask, resulting in a temperature of 100'C during the course of distillation. The solution slowly evaporates, and is collected in the same fashion as standard distillation. Because of the presence of water vapor in the immediate atmosphere, the distillate is rich in water. Unlike the distillate in simple or fractional distillation, the distillate in steam distillation must be further purified. To simplify
further separation, steam distillation is often employed with oils that are immiscible in water.

. decomposition of citral. B. hydrogenation of citral. C. oxidation ofcitral. D . polymerization of citral.


A

18.

When ether is added to the distillate, what is observed?

A. Water dissolves into the ether layer while citral


sinks to the bottom of the flask.

B. Water dissolves into the ether layer while citral floats to the top of the solution.
C

A student was assigned the task of separating citral (b.p.


=229'C) from lemon
grass

An ether layer forms on top of the water layer, and citral dissolves into it.

oil. Citral

is a l0-carbon terpene

D.

An aqueous layer forms on top of the ether layer,


and citral dissolves into it.

that is a precursor in the commercial synthesis of Vitamin A. It is isolated in conjunction with neral, which varies at the double bond between carbons 2 and 3. Citral and neral are

shown in Figure 1 below.

19. What is the purpose of adding anhydrous


sulfate to the ether extract?

magnesium

Citral

. . C. D.
A
B

To isomerize neral into citral To reduce the aldehyde into


a

primary alcohol

To remove any residual water in the ether layer

To selectively bind neral, leaving behind pure citral

0.

What is the approximate boiling point of neral?

CH:
Neral

CH3

Figure

1 Citral and neral, components of lemon

grass oil

A. 78"C B. 100'c c. t41'c D. 211"C


.

After steam distillation, the mixture is approximately


90Vo water,9Vo cttral, and l7o neral. To isolate the organic products, the mixture is extracted using diethyl ether and dried

using anhydrous magnesium sulfate.

21

What property is expected for Vitamin A?

15. What

compounds are best separated using steam

distillation?

. B. C. D.
A
1

Organic liquids with boiling points over 100'C Organic liquids with boiling points under 100'C Organic solids with melting points over 100'C Organic solids with melting points under 100'C

A. B. C. D.

A lower melting point than citral A high water solubility


A high vapor pressure at room temperature A high affinity lbr lipids

6.

Neral and citral are best described as:

A. B. C. D.

conformational isomers. geometrical isomers.

optical isomers.
structural isomers.
213

Copyright O by The Berkeley Review@

GO ON TO THE NEXT PAGE.

Passage

lV

(Questions 22 - 28)

24.

The flow of nitrogen gas serves to:

A student attempts to remove caffeine from tea leaves by a series of extraction procedures. Initially, 2.0 grams of tea leaves are added to 10 mL of boiling water for 20 minutes. The aqueous solution is then extracted three times with exactly 5.0 mL of merhylene chloride (CH2C!) during each of the three extractions. The 15.0 mL of methvlene chloride solution is collected and consolidated into one flask. The solvent is removed through evaporation by flushing vapor from the flask using a constant stream of nitrogen gas. The
temperature of the solution is maintained at 60'C by partially immersing the flask in a water bath.

A. B.

reduce the partial pressure of the solvent vapor so more solvent can evaporate. increase the partial pressure of the solvent vapor so more solvent can evaporate.

C. reduce the partial pressure


less solvent can evaporate.

of the solvent vapor

so

increase the partial pressure of the solvenl vapor so less solvent can evaporate.

t<

After the solvent is completely removed from the flask, a white residue remains at the bottom. This white residue is sublimed under vacuum and 28 milligrams of a white powder
are isolated. The melting point and infrared spectroscopy data of the white powder marches that of caff'eine (m.p. = 238 'C). The student concludes that the isolated compound is in fact

Which procedure would you use to determine the purit.. of the caffeine?

A. Adding it to B. C. D.

methylene chloride

to test

its

pure caffeine. Following sublimation, the bottom of the flask has no white powder remaining, just a waxy residue.
The structure of caffeine is shown in Figure 1 below.

solubility. Adding it to water to test its solubility. Mixing the compound with caffeine and observins the melting point for a sharp range.

Mixing the compound with an organic solid othe:


than caffeine and observing the melting point for
sharp range.
a

":I5
I

I
N
N

CHr

26- Why is the methylene chloride added in three 5-mL


portions rather than one 15-mL portions?

Methylene chloride is too unstable to be added in big portion.

CH:

B.
1

15 mL of methylene chloride weighs more than 10mL of water, so the organic layer would sink to the

Figure

C The caifeine could have been removed from the organic solvent by treatment with a strong acicl such as hydrochloric acid. The acid can protonate the caf'fcine to form the ionic

. .

ol the flask. Less caffeine can be extracted by three smalborrom extractions than one large extraction.

More caffeine can be extracted by three smal1


extractions than one large extraction.

salt complex, which is relatively insoluble in organic


solvent.

:ti

22.

From the values presented in the passage, what is the


approximate percentage by mass of caffeine in tea?

27. Methylene chloride


[o:

was added to the aqueous solution


!-

A. B" C.

1.470

2.87n
11Vo

B.
C

extract all of the caffeine while leaving behind th; water-so1ub1e impurities. extract most of the caffeine while leaving behinc the water-soluhle impurities.

D.28Vo

extract all of the caffeine along with the watersoluble impurities. extract some of the caffeine along with the watersoluble impurities.

D.
23.
Based on the information in the passage, caffeine has which of the following solubility properties?

A. It is soluble in both water and CH2CI2,


B

but more soluble in CH2C12. It is soluble in both water and CH2C|2, but more
soluble in water.

28.

Sublimation is employed to purify

a:

. It is soluble in water, but insoluble in CH2C|2. D. It is soluble in CH2C12, but insoluble in warer.
C Copyright @ by The Berkeley Review@
214

A. B. C. D.

solid from other solids. solid from liquids. solid from gases.

liquid from solids.

GO ON TO THE NEXT PAGE.

Passage

(Questions 29 - 35)

31

A student wishes to determine the partition coefficient of 4-methylaniline between the water layer and diethyl ether layer in a biphasic mixture. The student decides to dissolve a known mass of 4-methylaniline into a l:l mixture by volume of water and anhydrous diethyl ether. In the first trial, the student uses l0 mL of water, l0 mL of anhydrous diethyl ether, and 1.0 gram of 4-methylaniline. This mixture is placed into a separatory funnel and shaken. The lower layer is removed, without removing any of the upper layer. A drop of water is added to the lower layer, and it is found to dissolve completely into the solution. The student concludes from this that the upper layer, still in the separatory funnel, is the organic (diethyl ether) layer. The student removes the upper layer into a 25 mL Erlenmeyer flask and adds to this
0.5 grams of anhydrous calcium chloride, which forms a layer of powder on the bottom of the flask.

A second student attempted this same experiment using 3.0 grams 4-methylaniline in a mixture of l0 mL each of water and diethyl ether. Which of the following
statements predicts the results?

A. B. C. D.

The partition coefficient was found to be the same as with the 1.O-gram experiment. The partition coefficient increased from the value determined in the 1.O-gram experiment. The partition coefficient decreased from the value determined in the 1.O-gram experiment. The partition coefficient could not be determined

because

the 3-methylaniline didn't

dissolve

completely.

2.

How can the 4-methylaniline that adhered to the pipette


wall during transfer be recovered?

for five minutes. The ether layer is removed from the flask
gram

The flask is swirled gently for two minutes and then sits

completely. The mass of the vial and dried 4-methylaniline is 11.25 grams. The student assumes that the missing 4methylaniline from the original 1.0 g sample must have dissolved into the water layer. By dividing the grams which

by decantation using a warm pipette and is added to a 10.50 vial. The calcium chloride remaining in the Erlenmeyer flask is rinsed with 5.0 mL of anhydrous diethyl ether. This 5.0 mL portion is removed by decantation using a warm pipette and added to the vial. The vial is heated mildly under an exhaust fan until the diethyl ether evaporates away

. The pipette could be rinsed with water. B. The piperte could be rinsed with diethyl ether. C . The pipette could be heated to a high temperature. D . The pipette could be cooled to a low temperature.
A
33

. If the ether used was NOT anhydrous, how would this


affect the calculated partition coefficient?

A. It would lower the value, because initially there


would be more than 10 mL of water and less than 10 mL of ether.

dissolved into the ether layer by the grams that dissolved into the water layer, a partition coefficient is calculated.

B. It

To verify that the answer is valid, the student consults a book of physical constants and finds that the solubility at 25'C for 4-methylaniline in water is 3.8 grams/100 mL and the solubility at25'C for 4-methylaniline in diethyl ether is 22.8 grams/100 mL. From these values, the student concludes that the experimental partition coefficient is too low. This deviation is most likely attributed to the loss of solid in the transfer steps.

would raise the value, because initially there would be more than l0 mL of water and less than 10 mL of ether.

C. It would lower the value, because initially there D.


would be more than l0 mL of ether and less than 10 mL of water. It would raise the value, because initially there would be more than 10 mL of ether and less than 10 mL of water.

29.

From the experimental values using 1.0 gram in a mixture of l0 mL each of water and diethyl ether, what
is the partition coefficient?

34

. All of the following


ether EXCEPT:

solvents be used in place of diethyl

A. B.

2.5

3.0 4.0
6.0
3

c.

D.

A. tetrahydrofuran, THF. B. ethanol. C. cyclohexane. D . methylene chloride.


5.
Which of the following is a desired property of the
organic solvent?

30" From the literature values for solubility,


partition coefficient?

whar is rhe

A. 2.5 B. 3.0 c. 4.0 D. 6.0


Copyright @ by The Berkeley Review@
215

A. Highly miscible in warer B. Highly dense C. Solute is highly soluble in the solvent D. Its boiling point is less than room temperature
GO ON TO THE NEXT PAGE.

Passage

Vl

(Questions 36 - 42)

6.

Which of the following organic compounds could


used in place ofether in this acid/base extraction?

be

sodium hydroxide solution and ether

sodium bicarbonate solution, 5% hydrochloric acid solution, l)Vo acetic acid solution , and 5Va sodium hydroxide solution. Each student mapped out a scheme in the form of a flow chart to separate the three compounds. Student I chose first to add

help in separating the three compounds into their pure individual components, the students had availabl e 70Vo

Two students were each presented with a mixture of equal parts benzoic acid, resorcinol, and meta dinitrobenzene. To

A. Ethanol (b.p. = 79'C) . 1,4-Dichlorobenzene (m.p. = 53"C) C. Naphthalene (m.p. =94"C) D. Chloroform (b.p. = 6l'C)
B

to

separate the

compounds. Student II chose first to add sodium bicarbonate solution and ether to separate the compounds. Each scheme, along with the structures of each compound and their pKu values, are shown in Figure I below.

7.

What solution should be added to Tube 4 to cause precipitate to fall out of solution?

oY?l=

Benzoic Acid

OG*
!H
Resorcinol

,r

nK" = o se

NO"

A. B. C. D.

5Vo

solution. hydrochloric acid solution. I}Vo acetic acid solution. hydroxide solution.

10Vo sodium bicarbonate

5Vo sodium

8.

What can be said about Tube 2 and Tube 5?

meta-Dinitrobenzene

Tube 2 contains dinitrobenzene, while Tube


contains resorcinol.

-<

Proposed Scheme

for Student I
Aqueous sodium hydroxide Iayer

B.

Tube 2 contains dinitrobenzene, while Tube


contains both resorcinol and benzoic acid.

_<

Ether layer

C. Tube 2 contains resorcinol,


dinitrobenzene.

while Tube 5 contains

D.
Aqueous sodium bicarbonate layer Tuhe
I

Tube 2 and Tube 5 both contain dinitrobenzene.

39.
Tube
2

What is true about the pK values for the conjugat:


bases

Tube

Proposed Scheme

for Student II
Aqueous sodium bicarbonate layer

of the benzoic acid and resorcinol? pKb(sodium benzoate) > pKb2(sodium resorcinoxid: and pKullr"rorcinol) > pKa2(resorcinol)

B.
C D

Ether layer

. .

and PKal (resorcinol) > pKa2(resorcinol) PKI(soaium benzoate) > pKb2(sodium resorcinoxi:: and pKu2lt.rorcinol) ) PKal(resorcinol)
pKUZ(soaium resorcinoxide) > pKb(sodium benzoa:: and pKu2lr.rorcinol) > PKai (resorcinol)

pKUZ(soOium resorcinoxide) > pKb(sodium benzoa::

Ether layer

Aqueous sodium roxide layer Tube 4

Tube 5

0.

Tube 6

What can be concluded about the two schemes preseni3: in the passage?

Figure

Reaction schemes lor Students

and

II

The ether layer and aqueous layer are immiscible in one another, so they can be separated using decanting techniques. To isolate the solute from the ether layer, the ether solvent is

B. C. D.

allowed to evaporate" To isolate the components from the aqueous layers, the soiutions are neutralized and filtered. The two students did not get the same resuits. The melting point ranges for the three solids isolated by Student II are sharper than the meiting point ranges for the three solids isolated by
Student L

The scheme for Student II worked while the scher:tbr Student I did not work. The scheme for Student I worked while the scher. for Student II did not work. Both the scheme for Student I and the scheme i:: Student II worked.

Both the scheme for Student


Student

and the scheme:.

II did not work.

Copyright @ by The Berkeley Review@

216

GO ON TO THE NBXT PAGL

41. If 3-methylaniline were present in the mixture for


Student

Passage

Vll

(Questions 43 - 49)

II, where it be found?


Organic compounds can be isolated from plants through
a process known as extraction, a common industrial practice

A. B. C. D.

Exclusively in Tube 4 Exclusively in Tube 6 Evenly split between Tubes 5 and 6 Predominantly in Tube 5

used to isolate natural products for medicine, preservatives, dyes, and flavoring agents. Trimyristin, for instance, can be isolated from nutmeg beans by grinding the beans into a pulp and then adding diethyl ether to the ground pulp. Trimyristin

2.

Which of the following compounds cannot be isolated


using acid-base extraction techniques?

A. 4-Ethylcyclohexanone B. 3-Bromophenol C. 3-Nitrobenzoic acid D. Ethyl benzoate

dissolves readily into diethyl ether. In ordinary extraction procedures, the solution is isolated by either decanting it away from the suspension or by filtering the solid impurities out of the suspension by passing it through cotton. In both procedures, the desired compound remains dissolved in rhe

organic solvent. When left to sit, the solvent

will slowly

evaporate, leaving behind residue of the compound. In the case of trimyristin, precipitation can be expedited by adding methanol to the solution. The purity of the extract may be tested by one of several spectroscopic techniques (including IIINMR and IR), by thin layer chromatography, or by evaluating the melting point of the powder.

Trimyristin is a fatty acid triglyceride, derived from

it is sensitive to acid-base extraction techniques. In an


experiment, students attempt to extract trimyristin from nutmeg beans. The solvent chosen for extraction was tetrahydrofuran,

glycerol and three fourteen-carbon fatty acids. Like all esters,

THF. After filtering

through cotton and

adding methanol to solution, a white powder formed on the bottom of the flask. It was isolated by passing it through a Biichner funnel fitted with filter paper. A small sample of the powder was dissolved into three drops of dichloromethane, and the new solution was spotted onto a TLC (thin layer chromatography) plate in three places. The plate was placed into a beaker with minimal dichloromethane at the bottom. The beaker was sealed and the solvent proceeded to climb the plate. Before the solvent reached the top of the plate, the plate was removed from the solvent and the top of the solvent front was marked. The plate was set into an iodine chamber for developing. Spots appeared and were analyzed. Figure I shows a rough sketch of what appeared: Solvent front
stopped

dsolvent

Before
Figure
Copyright @ by The Berkeley Review@
217

After

Dichloromethane Solvent

*r=;*

Thin layer chromatography results of extract

GO ON TO THE NEXT PAGE.

0.83. The compound with an R1 = 0.59 (referred to as Compound II) ri'as the most abundant species of the three
compounds rhat were isolated from the crude product mixture producrs

The rhree spots resulted in R1 values of 0.19, 0.59, and

7.

Which of the following solvents would show a similar result as a TLC solvent as carbon tetrachloride (CCl+)?

using column chromatography techniques. The other two \', ere present in smaller quantities.
an R1 value of 0.j2 in a nonpolar solr'ent like hexane will likely show what R1 value in a poiar solvent like ethyl acetate?
1.84 0.12

{3. -\ spot that shows . B.


A

A. Acetone (CH3COCH3) B. Propanal (CH3CH2CH=O) C. Ethanol (CH3CH2OH) D. Pentane (CH3CH2CH2CH2CH3)

48.

R1 value

Which of the following values is NOT possible for in a TLC analysis?

rhe

. 0.13 D. -0.21
c

A. 0.00 B. 0.35 c. 0.70 D. 1.05


9.

44.

group

What solvent should be added to the beans to remove any naturally occurring sugar? A typical sugar has a carbonyl group and all other carbons have an alcohol

(cH2oHCH(oH)cH(oH)cH(oH)cH(oH)cHo)

How can THF and trimyristin best be described?

A. A long-chain alkane. B. A cyclic ether. C. A short-chain alcohol. D. A long-chain ketone.

THF is a nonpolar solvent and trimyristin is


hydrophobic compound.

B.
C

THF is a nonpolar solvent and trimyristin is


hydrophilic compound.

. THF is a polar

solvent and trimyristin is solvent and trimyristin is

hydrophobic compound.

D. THF is a polar
45. Why
should the initial height of the solvenr in
rhe

hydrophilic compound.

beaker NOT be above the initial height of the spots on the TLC plare?

A. B. C.

The spots would not migrate up as fast

if they

are

submerged below the level of the solvent. The spots would become saturated and thus migrate too rapidly up the plate.

The spots would migrate in a radial manner rather than a linear manner, if submerged below the top of the solvent.
submerged below the top of the solvent.

D. The spots would dissolve into solution if

46. If two

compounds

in a mixture have respective

R1

values of 0.58 and 0.29 in hexane, and 0.07 and 0.21 in ethanol, how are they best separated?

A. By column chromatography,
short column.

using hexane in using hexane in using ethanol in using ethanol in

B. By coiumn chromatography,
long column.
C

. By column chromatography,
short column.

. By column chromatography,
long column.

Copyright O by The Berkeley Review@

218

GO ON TO THE NEXT PAGE"

Passage

Vllt (euestions 50 - 56)

0.

Chromatography operates based on the partitioning of a ;ompound between a mobile phase and a stationary phase. If :he compound has a higher affinity for the eluting solvent mobile phase) than the adsorbent (stationary phase), then it '.i ill travel quickly. One type of chromatography is thin layer chromatography, which is done on a small scale to analyze the components of a mixture. It can also be done on a larger scale in a column, where the length of the column is at least ten times the diameter of the column. Columns are capable of separating up to approximately l0 grams of material, rihether it is a liquid or solid.

Which combination has the fastest elution time through a column filled with alumina?

A. Benzoic acid solute in n-pentane B. Glycerol solute in petroleum ether C. o-Xylene solute in ligroin D. Glycine solute in ethanol
51

For two compounds with R1 values that are different by a factor of 2, what is true in column chromatography?

plate and solvent is allowed to climb up the plate through capillary action. The solutes in the spots migrate with the solvent up the plate, but at varying rates depending on their relative partitioning between eluting solvent and adsorbent.
In column chromatography, the same material is added to the top of a column packed with adsorbent and a thin layer of clean sand at the top. The sand prevents the adsorbent from distorting when solvent is poured into the column.

In thin layer chromatography, the adsorbent is bound to a rectangular plate made of either glass or plastic, depending on the adsorbent. The sample is spotted near the bottom of the

A. The elution time is 4 times greater for the compound with the larger R1 value than the
compound with the smaller Rp value.

B. The elution time is 2 times greater for the compound with the larger R1 value than the
compound with the smaller R1 value.

C. The elution time is 4 times greater for the compound with the smaller R1 value than the
compound with the larger R1 value.

D. The elution time is 2 times greater fbr the compound with the smaller Rp value than the
compound with the larger Rp value.

Brockmann Activity rating of


r.vater by mass.

terms of binding a solute. The activity can be reduced by the addition of water, because water binds the alumina, thereby reducing the surf'ace area to which the solute can bind. A

silica gel (SiOz). Alumina comes is three forms, acidic, basic, and neutral. The neutral form is given a Brockmann Activity rating from I to V, where I is the most active in

There are two common adsorbents, alumina (Al2O3) and

52.

greatest R1 value?

Using TLC, which solvent would give salicylic acid the

III refers to alumina that is 6Vo

A. n-Hexane B. Ligroin C. Methanol D. 2-Pentanol


53. What is true of alumina with a Brockmann Activitv
rating of IV?

Solvents are rated according to their polarity, from least polar to most polar. As a solvent becomes more polar, it has a greater affinity for a polar solute. Table 1 shows solvents according to elution rate for a nonpolar solute from fastest to slowest across alumina. Often, the affinity of polar solutes for the adsorbent exceeds all other affinities.
Rank
1

. B.
A

It It
a

has 4Vo water by mass. has a greater

affinity fbr solute than alumina with

Solvent
n-Hexane Petroleum ether Cyclohexane

Rank
8

Solvenl
Tetrahydrofuran Dioxane

C. D. It can be formed by adding more than 6 grams of


water to 94 grams of alumina with a Brockmann

Brockmann Activity rating of III. It is rich in silicon aroms.

2 .l

9
10
11

Activiry raring ol

I.

Ethyl acetate

4
5

Ligroin
Carbon disuiflde

Dimethyl sulfoxide
2-Propanol

l2
t3

54.

6
1

Ethyl ether
Dichloromethane

Ethanol Methanol

How does a sudden change in solvent from n-pentane to methanol in an alumina column harm the ei'fectiveness

t4
1

of the column?

A. B.

Table

phenols > carboxylic acids. R1 values can be predicted from the relative elution rate and rhe solvent rankings by applying the simple principle: "like dissolves like." Copyright @ by The Berkelev Review@

is: alkanes > alkenes > dienes > aromatic hydrocarbons> ethers > esters > ketones > aldehydes > amines > alcohols >

The general elution order for solutes in nonpolar solvents

The methanol endothermically binds the alumina, forming vapor pockets in the column. The methanol exothermically binds the alumina, forming vapor pockets in the column. drops drops through the column too fast.

C. Methanol is immiscible in hexane, so it D. Methanol is immiscible in hexane, so it


through the column too slow.
279

GO ON TO THE NEXT PAGE.

55

Why does adding water to alumina raise its Brockmann

Passage

Activity rating?
^4.

lX

(Question 57 - 63)

. ,

After water binds the surface of alumina, there

are

fewer sites on the alumina for solute to bind. Water causes the alumina to contract, reducing its surface area for the binding of solute.
Water makes the nonpolar alumina more polar.

Benzylic hydrogens are unusually reactive for hydrogens on an spj-hybridized carbon. They have pKus around 23-25. considerably lower than 45-50 for alkyl hydrogens. They are also more susceptible to oxidation than alkyl hydrogens. Figure 1 shows the oxidation of toluene into benzoic acid. a typical oxidation reaction for benzylic protons.

. D.
C

Water oxidizes the alumina into a new species that is more hydrophobic than alumina.

CH: [o] Figure

56. .\iumina would likely


',',

have the highest affinity for

hich of the following solutes?


1,3-Cyclohexadiene

Oxidation of the benzylic hydrogens of toluene

\, B. C. D.

Cyclohexanol

Ethyl butanoate
3-Methylpentanal

A chemist decided to oxidize fluorene into fluorenone br adding sodium dichromate, Na2Cr2O7, in acetic acid solvenr The reaction is shown in Figure 2 below.

tol +

Figure

Oxidation of fluorene into fluorenone

w
o
R1

Because both fluorene and fluorenone are solids at room temperature, separation cannot be done by distillation. Foi separating solids, extraction or chromatography are typically employed. If the sample is 10 grams or less, then columr chromatography works well. The chemist chose to separare the product mixture using column chromatography. prior to carrying out the column chromatography experiment, she ran a series of thin layer chromatography trials to determine which elution solvent and adsorbent to use in the column.

The adsorbent in each trial is silica gel. Two spots are observed in every trial. One spot is yellow in color, and both are UV active. Table I shows the R1 values for each of the

two compounds in various solvents used in the TLC trials.


Solvent
R1

for Yellow
0.18 0.24 0.19
0.41

Compound

for Colorless compound


0.31
0.33 0.51 0.55

Liquid I

II Liquid III Liquid IV


Liquid

THF

0.31

0.42
1

Table
5

7.

Fluorenone should have the greatest affinity for which

of the following solvents?

A. Acetone B. Cyclohexane C. Dichloromethane D. Methanol


Copyright @ by The Berkeley Review@ 280

GO ON TO THE NEXT PAGE.

if,iE

58. Why does fluorene elute from the column before


fluorenone when cyclohexane is the eluting solvent?

63. Why

do nonpolar solutes elute before polar solutes from

A. B.

columns containing silica gel, even when a polar


solvent is used?

Fluorene has a higher affinity for silica gel than


fluorenone.

A.
it
travels

Fluorene is lighter than fluorenone, so


faster.

Polar compounds do not follow the ',like dissolves like" rule.

C. Fluorene has a higher affinity for cyclohexane than


fluorenone.

B.
C

The silica gel is highly polar, which hinders rhe migration of polar solutes.

D.

Fluorene is smailer than fluorenone, so it can pass more easily through the pores in silica gel.

Nonpolar species become more polar when the


solvent is polar.

D. Polar

compounds aggregate and form large molecules that do not migrate quickly because of
steric hindrance.

59. According to the data in Table

1, what eluting solvent is best for separating fluorenone from fluorene?

. B. C. D.
A

Liquid

II

Liquid Itr Liquid IV

THF

60. When eluting a mixture of biphenyl, ethyl

benzoate, and toluic acid from an alumina column using ligroin as the eluting solvent, the sequence off the column is:

A.

biphenyl first, ethyl benzoate second, and toluic


acid last.

B. toluic acid first, ethyl benzoate


biphenyl last.

second, and

C.

ethyl benzoate first, biphenyl second, and toluic


acid last.

D. biphenyl first, toluic acid


benzoate last.

second, and ethyl

61.

Once the silica gel is set in the column, why should you wait to add the sample until the solvent is flush

with the top of rhe silica gel?

A. B. C. D.

So that the sample does not float on the solvent


and never get to the silica gel

So that the sample does not adhere to the glass of


the column

So that all of the sample starts traveling down the column at the same time So that top of the column can dry out before the
sample interacts with the silica gel

2.

Why is a thin layer of sand added to the top of the silica


gel?

A
B

. . C.

To prevent the sample from reaching the silica gel To reduce the poiarity of the gel

To keep the top of the silica gel flat when more


solvent is poured into the column To bind sandphilic solures

D.

Copyright @ by The Berkele-v- Rer.iew@

281

GO ON TO THE NEXT PAGE.

Passage

tQuestions 64 - Z1)

-{ researcher carried out a two-step reaction using the ketone 3,3-dimethylcyclopentanone as the reactant. The reaction sequence is shown in Figure 1 below.

ooo

Q.f-q;,ft.,
CH: CH:
CHr

Reaction

Figure

Gas chromatography results for Trial

III

Gas chromatography can be used

A*
Figure

to

quantitatively

H-.",
CHr

Reaction

R-x
cHs
CH:
2

ana\yze the composition of mixtures by evaluating the area under each peak. The relatives areas under each curve can be used to determine the percent composition of the mixture.

64. If

the product mixture from Trial III were spiked with the reactant, which of the peaks in the example would
1

grow?

Two-step reaction of 3,3-dimethylcyclopentanone

The reaction sequence involves deprotonating an alpha hydrogen and subsequently adding an electrophile to form a bond to the alpha carbon. Because the reactant contains two

A. Peak B. Peak 2 C. Peak 3 D. Without knowing

products. The two methyl groups on carbon three offer steric hindrance, so one alpha carbon is less sterically hindered than the other alpha carbon.
The reaction was carried out at different temperatures.

alpha carbons, there are two possible structural isomer

the composition of the column, it cannot be determined.

5. If the column in the gas chromatography machine were


packed with a nonpolar polymer, what would be true

lower temperatures, the product mixture favored the less sterically hindered product. This is often referred to as the kinetic product of the reaction. The thermodynamic product results from the formation of the more stable bond. In the reaction of Figure 1, there is no distinct thermodynamic product. Table I shows the percentage of each species in
soiution following the reaction at various temperatures.

At

about the retention times of polar and nonpolar


compounds?

A. B.
C

Nonpolar compounds would have longer retention times, because they bind the column less tightl1,
than polar compounds.

Temp
Trial

7o

Reactant

Vo

Product

I: -33"C TrialII: 0"C Trial III: 15'C Trial IV: 30'C Trial V: 50'C Trial Vl: 78"C

Z, p.oduct B g

Nonpolar compounds would have longer retention times, because they bind the column more tightll, than polar compounds. Nonpolar compounds would have shorter retention times, because they bind the column less tightl;.
than polar compounds.

25 20 16 t3 l0 j
Table
1

66 60 56 53 50 48

20
28 34

D.

40
4.5

Nonpolar compounds would have shorter retention times, because they bind the column more tightil. than polar compounds.

6.

components. Spiking is the term given to adding a small portion of one of the components to the product mixture.
The peak that increases is deducecl to beiong to the compound that has been spiked.

Figure 2 is the gas chromatography chart produced when III is analyzed. The compounds were passed through a Carbowax_ 12 column that is partially polar. The identity of each peak was obtained by spiking the product mixture with a sample of one of the pure

Which of the following relationships accurately depicts the relative retention times of the components?

the product mixture tiom Trial

A. Product A > ProductB > Reactant B. Reactant > Product A > product B C. Reactant > Product B > productA D. Product B > Product A > Reactant

Copyright O by The Berkeley Review@

282

GO ON TO THE NEXT PAGE.

.ir

7.

increasing the temperature of the reaction?

According to Table 1, what can be concluded about

Passage

Xl

(euestions 72 - 79)
a

. It makes the reaction more favorable. B. It favors the formation of the kinetic product. C. It favors the retention ofreactants. D . It makes the reaction less favorable.
A

solid into a minimal amount of solvent, and then precipitating the solute from solution in a highly pure
charcoal that may have been added) and ultimately to iiolate the recrystallization process is listed below:

Recrystallization is the laboratory process ofdissolving

to remove solid impurities (such as a boiling ctrips fr

crystalline form. The procedure includes some filtering steps

the desired crystals. The general procedure for

8.

mixture from Trial IV, gas chromatography separates primarily by;

For the compounds associated with the crude product

I. II.

Add the impure solid mixture into a minimal


the compound partially.

. B. C. D.
A

amount of solvent at room temperature to dissolve

the size of molecule. the mass of the heaviest atom in the molecule. ofthe compound. the affinity for the column.
the charge

s-olvent drop by drop dissolved.

Heat the solvent- to its boiling point in a refluxing system. If not all of the solid has dissolved, add hol

until the solid is completely

III.
69.
The best gas to use as a carrier gas (to propel the vaporized components of the mixture thiough the
carbon dioxide. hydrogen.

Once the solid is completely dissolved, hot_filter the solution, avoiding any precipitation of the solute.

IV.

column) is:

A. B. C. D.

room temperature, and then place the container into an ice bath. Crystals should form slowly during this step of the procedure.
reduced temperature.

Slowly cool the filtered solution until

it

reaches

V. Filter the collected crystals from the solvent at VI. _


Rinse the crystals with a volatile solvent in which they are insoluble and allow them to air dry.

helium.
water.

0.

The major product from the reaction in Trial

isolated using which of the following techniques?

II

can be

A. Column chromatography B. Simple distillation C. Vacuum sublimation D . Acid-base extraction

does not evaporate away.

solid barely soluble at room temperature and fully soluble at the boiling point of the solvent. The solvent is refluxed so that it exists at its highest temperature (boiling point), but

enough in the solvent, the quantity of solvent is so large that impurities are dissolved as well. The ideal solvent has the

If the solid is too soluble in the solvent, the quantity of solute that is recrystallized is minimized. If the solid is not soluble

that is soluble in an organic solvent.

The procedure is generic and can be applied to any solid

71. Why must a strong base be used in Reaction I


Figure
1?

in

B_ecause alpha hydrogens are highly acidic, with pKu values around 2 _ 5.

B. C. D.

B,ecause alpha hydrogens are highly acidic, with pKu values around lj _20.

Charcoal is a decolorizing agent that is sometimes added to the first solution. Charcoal has a high affinity for colored organic molecules, because both charcoal and organic chromophores are rich in n-bonds. Charcoal is only added to the solution when the desired compound is colorless and the solution is colored.

pK. values around 2 _ 5.

B-ecause alpha hydrogens are weakly acidic, with

B_e-cause alpha hydrogens are weakly acidic, with pKu values around 17 _ 20.

are purer than smaller crystals.


72

corresponds to pure crystals. As a general rule, larger crystJs

The purity of the crystals can be quickly tested by looking at the melting point range. A narrow range

B. Small rectangular crystals formed by slow cooling. C. Long cubic crystals formed by slow cooling. D. Long rectangular crystals formed by rapid cooling.
Copyright @ by The Berkeley Review@ 283

Which of the following crystals are the purest? A. Small cubic crystals formed by rapid cooling.

GO ON TO THE NEXT PAGE.

73.

The ideal solvent for recrystallization should have which of the following properties?

8.

What is NOT true about recrystallization?

I. II. m. A. B. C. D.

A low affinity for the solid lattice


The solid should be highly soluble in the solvent at all temperatures

A. B.
D

The final crystals are purer than the starting solid.

The process requires knowing the boiling point of the solvent.

Impurities should be highly soluble in the solvent

C. It is best carried out in a highly volatile solvent. . It results in less than 100Vo recovery of the original
sample.

I only II only I and II only I and III only

79

To which solution should activated charcoal be


during a recrystallization experiment?

added

A.
7

A solution with colorless impurities and


target crystal

a colorless

4,

The purity of the crystal CANNOT be verified by which of the following?

B. A solution
C

with colorless impurities and a colored


impurities and a colorless
and a colored

. B. C. D.
A

Density of the crystal

target crystal

Melting point of the crystal Mass of the crystal Index ofrefraction ofthe crystal

. A solution with colored


target crystal target crystal

D. A solution with colored impurities

5.

What is the role of horfiltering the solution in Step III?

. B. C. D.

To filter out any soluble impurities To filter out any insoluble impurities To filter out the compound To reduce the amount of solvent in the flask

6.

What is the purpose of using the ice bath in Step V?

A. To increase the amount of crystals formed


increasing the solubility of the solid

by

B. To decrease
C

the amount of crystals fbrmed by


by

increasing the solubility of the solid

. To increase the amount of crystals formed


decreasing the solubility ofthe solid decreasing the solubility ofthe solid

D. To decrease

the amount of crystals formed by

7.

What is a potential problem


too rapidly?

if

the crystals are formed

A. B.
C

They grow to be too large.

They trap impurities from solution in their lattice


structure.

. It does not allow enough time for impurities to


form in the lattice. It prevents solvent from being incorporated into the
lattice structure.

D.

Copyright @ by The Berkeley Review@

284

GO ON TO THE NEXT PAGE.

Passage

Xll

(Questions B0 - 86)

81. A corrpound

that turns clear with Br2lCCl4 and has one

A researcher has chemicals organized according to molecular mass. In one section there are three compounds which say only 100 grams/mole on their respective bottles. In an attempt to identify the three unknowns, the researcher
labels the bottles Compound
Compound purified.

degree of unsaturation CANNOT be:

C.

A, Compound B, and 1.00 gram of each compound is isolated and

A. B. C. D.

a ketone.

an alkene. an ether. an alcohol.

A 1.00 mg sample of each compound was completely oxidized. For all three compounds, 2.64 mg CO2 gas and 1.08 mg H2O liquid was collected after complete oxidation. No nitrogen, sulfur or halides were found in any of the compounds. No analysis for the oxygen content was conducted due to the difficult nature of analyzing oxygen
content. Calculations show that there are exactly six carbons and twelve hydrogens in each of the three compounds, implying that the three unknowns are isomers. After the

82. Which of the lollowing types of compounds would


a

lose

peak in its 1HNMR when D2O is added?

A. Alcohol (ROH) B. Ketone (RCOR) C. Ether (ROR) D. Aldehyde (RCHO)


83. Which of the three compounds would undergo a color
change when treated with KMn04. considering KMnO4 can oxidize alkenes, selected alcohols, and selected carbonyl compounds'/

of the three compounds to


summarized in Table
1.

formula analysis was complete, the researcher subjected each

standard chemical tests.

as

CrO3/H+
Compound

B12(CCla) remained

(NO2)2C6H3COCl
precipitate formed
no precipitate

turned green remained ofange remained orange

brown
remained

Compound B

A. Compound A B. Compound C C. Compounds A and C D. Compounds B and C


84.
(NO2)2C6H3COC1 is NOT a good reactant to tesr fbr which of the following?

brown
turned clear

formed no precipitate
fbrmed

Compound C

Table I

It

should be noted that the organic product from the

reaction of Compound A with CrO3/H+ does not turn litmus paper red. Compound B, when treated with 12 and KOH,

A. Alcohol (ROH) B. Amine (RNHz) C. Ether (ROR) D. Thiol (RSH)


85. Compound A, by turning green with CrO3/H+ and not forming a compound that turns litmus red, is most
probably:

forms a yellow precipitate and acidic solution (a positive result for the iodoform test for methyl ketones). From the
molecular mass and carbon count, it can be inferred that the compounds contain one oxygen. The chemical tests indicate

that each compound is either an alcohol, an ether or a carbonyl compound. A positive test with bromine in carbon tetrachloride indicates the presence of an alkene. Each of these three types of functional group reacts differently, so
predictions can then be made about further reactions that the isomers can undergo, in order to further support their identity.
8

A. B. C. f).

aprimaryalcohol.
a secondary alcohol.

a tertiary alcohol. an ether.

0.

Which of the following IR absorbances would be the key peaks in Compound A and Compound B?

86. A positive iodoform test can be associated with which


of the following?

A. B, C. D.

Compound Compound Compound

A:

broad absorbance at 3500 cm-1; sharp absorbance

Compound B: sharp absorbance at 1710 cm-l

A:

at

1710 cm-i;

Compound B: broad absorbance at 3500 cm-1

A: sharp absorbance at 2980 cm-i; Compound B: sharp absorbance at 1710 cm-1

A. B. C. D.

A yellow precipitate confirming

methyl ketone.

A yellow precipitate contirming an aldehyde. A black precipitate confirming a methyl ketone. A black precipitate confirming an aldehyde.

Compound A: sharp absorbance at 1710 cm-l; Compound B: sharp absorbance at 2980 cm-1
28s

Copyright @ by The Berkeley Review@

GO ON TO THE NEXT PAGE.

Passage

Xlll

(Question 87 - 93)

0.

A student starts with 138 mg of 1,4-dimethoxybenzene (138.166 grams/mole) and treats it with 1.32 mL anhydrous HNO3 (1.50 g/ml, 63.012 g/mole) dissolved into 2.61 mL anhydrous H2SOa. This mixture is then heated to 35'C for twenty minutes and then left to cool to room temperature. As the mixture is being heated, a brown gas appears in the reaction flask. To this mixture is added 5.0 mL of water previously cooled to 0'C. This results in a white crystalline precipitate forming instantly in the solution upon addition of
the water.

What is the identity of the brown gas formed in the first part of the reaction?

A. Nz B. CO
C. NO2 D. SOo

91.

Which of the following is a possible cause for


thermometer error?

rhe

The white solid is filtered from solution using a Hirsch funnel and washed with three aliquots of 1.0 mL of cold water. The product is then isolated, dried and weighed. The mass collected for the isolated crude product (assumed to be 2,S-dimethoxynitrobenzene (183 g/mole)) is 91.5 mg. The melting point range is measured to be from 12.5'C to i4'C.

I. Air has leaked into the thermometer

resulting

il

reduced vacuum within the thermometer.

II. A divot exists on the inside wall of


thermometer between the 45 and 55 'C mark.

the
the

m. A divot exists on the inside wall of


thermometer between the 85 and 95 'C mark.

A small sample of the product is then pulverized and a drop of mineral oil is added to the pulverized sample. The mull (mixture of the mineral oil drop and pulverized solid) is stirred and added to the face of a salt plate for analysis using infiared spectroscopy. Key peaks in the IR show that the compound contains a nitro group, a C-O single bond, and a benzene ring. Based on the infrared data and the melting point of the crude product, the student concludes that the crude product is 2,5-dimethoxynitrobenzene. The literature value for the melting point of 2,5-dimethoxynitrobenzene is 74 - 15"C. The error in the observed melting point is attributed to thermometer error and not an error in the
reaction.
8

A. B. C. D.

I only I and II only I and III only I, II, and III all explain the themometer error.

92. To further purify


A

the product mixture, what type tr:

laboratory technique should be carried out?

The crude product mixture could be dissolved inlr hot solvent and then recrystallized from the sol\eias

it cools.

7.

B.
What is the percent yield for this reaction?

The crude product mixture could be filtered a secon:

time with the same solvent.


C

A. C. D.

25Vo

B.50Vo
15Vo

The crude product mixture could be heated to liqur* form and then distilled via fractional distillation.

D.

1007o

The crude product mixture could be treated rl;r: nitric acid and sulfuric acid again to react a:-i
unreacted starting material.

8.

What can be deduced from the melting point range and the IR data for the crude product?

93. Why is the addition of water carried out at a lc';


temperature?

A. B. C. D.

The product is still wet with solvent. The product is free of any significant impurities. The product is impure.

The starting material was isolated rather than the


product.

. B. C. D.
A

To increase the solubility of the product. To decrease the solubility ofthe product. To increase the acidity of the solution. To terminate the reaction by protonating the
group.

nir:

89.

Filtering is used to separate

a:

A. B. C. D.

solid from other solids. solid from liquids. solid from gases. liquid from other liquids.

Copyright @ by The Berkeley Review@

GO ON TO THE NEXT PAGL

Questions

descriptive passage.

94 through 100 are NOT

based on

7.

Which of the following compounds, during an acid_base extraction, would be extracted into 0. l0 M NaOH(aq),?

94

. If a pure sample of a compound with S-stereochemistry


has
composition
enantiomer that exhibits a specific rotation of +22.?

a specific rotation of -55", then what is the of a mixture of that compound and its

A. 4-Methyl benzoic acid B. 3-Ethyl anisole C. Ethyl benzoate D. N-Merhylanilinc

, 50Vo S-enantiomer with S\Ea R-enantiomer . 33Vo S-enantiomer with 6lVo R-enantiomer C. 30Vo S-enantiomer wtth70% R-enantiomer D, 20Vo S-enantiomer withS}Vo R-enantiomer
A
B

98. In which of the following solvents or solutions is 4_


hydroxybenzoic acid MOST soluble?

5.

The following TLC plate and column correspond to the same components within the mixture and the same mobile phase (solvent). Column

A. 0.10 M HCI(aq) B. 0.10 M KOH(aq) C. Water D. Diethyl ether

TLC Plate

9.

Which of the following lab techniques will NOT work


for the task listed?

A. Using distillation ro separare two liquids. B. Using chromatography to separate two solutes. C. Using extraction to separate two gases. D. Using crystallization to separate two solids.

100.

Butanone shows all of the fbllowing EXCEpT:

Which correctly correlates the spot to the band?

A. BandA = o; BandB =0 ; Band C = o B. BandA = o; BandB = $; Bandc = o C. BandR = 0; BandB = o; Band C = o


D. BandA

. high solubility in acetone. B. a negative Jones test for oxidation. C. a mass spcctroscopy peak at 57 amu. D . a specific rotation of op = {1.2'.
A

= o; BandB = O; Band

96.

Fractional distillation differs from simple distillation in all of the following ways EXCEpT:

The distilling column in fractional distillation has more surface area than the distilling column in
simple distillation.

B.
C

Fractional distillation is chosen over simple distillation when the boiling points are close for the components in the mixture.
Fractional distillation generates a higher purity than simple distillarion.

D. Fractional

distillation generates a greater yield than

simple distillation.

l.c 1.D 13. A 19. C 25. C 31. D 31. B 43. C 49. A s5. A 6r. c 61. A 13. D '19. c 85. B 91. B 91. A
281

2.A 8.A 14. B 20. D 26. D 32. B 38. D 44. C -50. C s6. B 62. C 68. D 14. C 80. A 86. A 92. A 98. B

3.A 4.A 5.C 9.D l0.B l1.D 15. A 16. B 11. B 21. D 22. A 23. A 21. B 28. A 29. B 33. A 34. B 35. C 39. C 40. A 41. D 45. D 46. C 41. D 51. D 52, C 53. D 51. A 58. C s9. B 63. B 64. A 65. B 69. C 70. A 11. D 15. B 16. C 71. B 81. A 82. A 83. C 87. B 88. B 89. B 93. B 94. C 95. A 99. C 100. D

6.4
12.D 18. C 24. A 30. D 36. D
42. D

48. D 54. B 60. A 66. D 12. C 78. C 84. C 90. C 96. D

Copyright O by The Berkeley Review@

STOP! START GRADING.

Laboratory Techniques Passage Answers


Choice C is correct. No matter what the boiling point of the liquid, no distillation should ever be done in a will build up (as T increases, so does P if there is constant V) and the system will er-entually explode. This eliminates choice D. The venting of the system must occur after the vapor has been condensed and collected, otherwise the vapor will escape and nothing will be collected. This eliminates choices A and B and makes choice C the best answer.
closed svstem, otherwise pressure
2.

Choice A is correct. A temperature of 49.2"C indicates that the boiling point of the compound being collected is near 49.2"C. The compound has to be one of the six given in the passage, so you must scan the first paragraph for the compound with a boiling point closes t to 49.2 "C. The best choice for the identity of the comporlnd is t-t,Zdichloroethene, which has a boiling point of 48"C. This makes choice A correct. This question might seem so straight forward that you get nervous and think you're missing something. Keep in mind that the MCAT has a range of question difficulties and that straight forward questions appear. Choice A is correct. When a boiling chip is added to the distillation flask, it sinks to the bottom, which is the hottest point in the solution (heat is added at the bottom of the solution.) Because vapor must escape from a surface and there is minimal surface area at the point where the solution is hottest, the solution can iuperheat (reach a temperature above its boiling point). Because heat rises (hotter solutions are less dense and therefore more buoyant than colder solutions), the superheated solution will migrate to the top of the solution, at which poht it has surface area from which to evaporate. Because the solution is beyond i1s boiling point, it rapidlr' evaporates (bumps), causing a splash. To avoid this problem, the system needs more surfac" ur"u at the bottom of the boiling flask. A boiling chip is added to provide a rough surface on which vapor can collect and build up near the bottom of the flask. This is the kinetic theory explanation of bumping. The best explanation is choice A. Choices C and D should have been eliminated, because the boiling chip does not interact with the molecules irr solution other than providing a surface area for them to collect on. Intermolecular forces are neither enhanced nor diminished by a boiling chip. The boiling chip is stationary, so it does not help in the homogenization of the solution. This eliminates choice B. Choice A is correct. The majority of the vapor collects on cold glass, as opposed to warm glass, which eliminates D. The condenser is cooled so as to condense the vapor. The condensed vapor then collects and trickles into the collection flask in droplets. The actual condensation occurs in the condenser, however, which
choices C and

3.

4.

Itl

makes choice A the best answer.


5.

Choice C is correct. Filtration can not be used when there is a solute, because the compound is fully dissoived into solution and cannot be collected in the filter. This eliminates choices A and B. A liquid may be separated from a dissolve solute by distilling (evaporating) the liquid away. This makes choice C or choice D the best answer. Because the boiling points are very different (the solute is normally a solid at room temperature, so it has to melt before it can boil), there is no need to employ fractional distillation. The sample is best distilled using simple distillation, because the boiling points are drastically different. Choice C is your choice.

It:

6.

Choice A is correct. Simple distillation works best when the two compounds have the greatest difference rn boiling points. Choice A is composed of two compounds which have boiling points of 34'C and 102"C, a difference of 68'C. Choice B is composed of two compounds which have boiling points of 34"C and 48"C, a difference o.' 14'C. Choice C is composed of two compounds which have boiling points of 90"C and 102"C, a difference of 12"C. Choice D is comprised of two compounds which have boiling points of 128"C and 102'C, a difference of 26"C. The largest difference in boiling points between the components is observed with the compounds in choice A. This question is reaily a nomenclature question. You must translate from formula in the question into the IUPAC name and boiling point from the text of paragraph 1. Choice D is correct. Fractional distillation is employed to separate two liquids with relatively close boiling points. This means that it is employed to separate a liquid from a liquid. This eliminates choices A, B, and C and makes choice D the best answer.

13

7.

Copyright O by The Berkeley Review@

LAB & SPECTROSCOPY EXPLANATIONS

8'

Choice A is correct' Fractional distillation depends on the condensation and re-evaporation of the more volatile component in the mixture' Each cycle of condensation/re-evaporation further purifies the vapor so that the first c ond ens a te o rr o r th;' " EX"'J ;,1'J 1t'1 ","1T; ;; ;" ff surface of the distillation apparatus heats up so that the amount of condensation'on the inner walls is reduced, and the components condenie and re-evaporate less. This does not segregate the less volatile component as well as when the walls are cooler. The net result is that over time, the distil'iate becomes less pure in terms of the more volatile component present in the vapor. The best answer is choice A. Azeotropes should have an equal bearing on the mixture whether the distillation is simple or fractional. The surface area does not affect the azeotropic mixture.

;;i;;; ilh *;

#;il:ii

il:TT;T#ll": ffi ili

9.

Choice D is correct. A long distilling column is used in fractional distillation, which is chosen when the boiling points of the components in the mixtlre are close in value. Close is defined as boiling points within 30.C of one another' In this question, you are asked for the best choice, which for fractionui dirtillution is the pair of components with the closest boiling points. In choice A, diethyl ether (b.p. = 34.6.C) and pentanol (b.p. = 138'0"c) have a difference in boiling points of over 100'c. simple distillation'works with choice A. In choice B, tetrahydrofuran (b'p' = 65'4'c) and anisole (b.p. 158.3'c) have a difference in boiling points of nearly 100"c. = Simple distillation will work fine for choice B. In choice C, octane (b.p. = 725.7"C) and pentane (b.p. = 36.1.C) have a difference in boiling points of nearly 90"C. Simple distillatiorr *itt work j# fine in choice C as well, In choice D, isopropanol (b.p, = 82.3"C) and ioluene (b.p. 110.6"C) have a difference in boiling points of under = 30"C' simple distillation will not work well, so fractional distillation should be used with choice D.

10.

Choice B is correct' The major advantage of fractional distillation over simple distillation is that the fractional distillation procedure allows components to be distilled and collected in a purer manner. Fractional distillation can always be used no matter how close the boiling points of two valid' Fractional distillation works by providing surface area from which the substances may be. choice A is vapor can continuously condense and re-evaporate' Choice C is thus vaiid. The drawback to fractional distillation is the time required to carry out the distillation and the loss of the vapor that condenses throughout th; appurutrr. Fractional distillation takes a larger amount of time than simple distillation, thus choice B i, un incorrect statement. The best answer is choice B.
Choice D is correct' Just as a water bath is used for better heat transfer (in both cooling and heating), a sand bath is chosen to transfer heat into the flask most efficiently. The sand bath covers the outside surface of the glass' and transfers heat-into the system through the glass. The sand serves to conduct heat (not insulate), so choice A is eliminated. sand is not reactirr" *ith glasl, but neither is air. Choice B may or may not be a true statement' but either way, it is not the best answer. sand is not particularly hygroscopic, so it will not affect the vapor pressure of water, eliminating choice C. The best answer involves heai t"ransfer, so choose D for optimal performance.
The role of the copper mesh in the distilling column is to increase the surface area onto which the vapor may condense so as to collect more condensate in ttre coiumn. The condensate can evaporate once again from the copper mesh, further distilling the condensed liquidThe copper mesh does not filter anything, it is inert, and it does not distribute heat anyLetter than the vaptr does. copper mesh just allows for fractional distillation to take place. The best answer is choice D.

11.

'l'2' Choice D is correct'

13'

choice B, thus fractional distillation must be used. Ctoice B is eliminated. In choice C, octane (b.p. = 725'7'c) and pentanol (b'p' = 138.0"C) have a difference in boiling points or auo.rt 12"C. Simple distillation will not work with choice C. Choice C is thus eliminated. In choice"d,lonlne 6; = 1S0.g"C) and anisole (b.p. = 158'3"c) have a difference in boiling points of only 7.5"C. simple distillation witt not work in this case, thus choice D is eliminated. Choice A is lhe best answei.
Copyright @ by The Berkeley Review@

Choice A is correct' Simple distillation is chosen when the boiling points of the components in the mixture are far apart' Far apart is defined as boiling points that are *oru Ihun 30"C apart from one another. For this question, you are asked for the best choice. The best choice for simple distillition will be the two components with the greatest boiling point difference. In choice A, diethyl ether'(b.p. gi..e"C) and heptane (b.p. = 9g.4.C) = have a difference in boiling points of over 60"C. simple distiilation will work fine with choice A, but you must look at the other choices to iee if there is a difference greater than 63.g'c. In choice B, tetrahydrofuran (b.p. = 65'4'c) and hexane (b'P' = 68'9"c) have a difference in"boiling points or or"rty sl'c. simple distillation will not work with

2a9

LAB & SPECTROSCOPY EXPLANATIONS

'l'4'

epimers of one another, which is another way of saying that they are diastereomers. The only pair oi enantiomers in the choices are 3R-bromocyclopentene and 3S-bromocyclopentene, making choice B the best answer. When a compound has only one chiral center, it cannot have a diastereomer, beciuse diastereomers require at least two chiral centers. Choices A, C, and D have multiple chiral centers.

Choice B is correct. The same boiling point for two stereoisomers is observed when the two stereoisomers are enantiomers, as they have the same functional groups and the same steric hindrance. They show the same intermolecular forces, which results in the same boiling point. The two molecules in both choice A and choice D are diastereomeric pairs, therefore both choice A and choice D can be eliminated. Diastereomers do not sho*the same intermolecular forces, so they likely show different boiling points. Glucose and mannose are C-l

15.

Choice A is correct. Steam distillation involves boiling a mixture of components in the presence of water, so the boiling point is more important than the melting point. This eliminates ihoices C and b. Water boils at 100"C. so adding steam to help distill a mixture of organic compounds is best done when the boiling point of each component is greater than 100'C. This makes choice A the best answer.

16.

Choice B is correct. Both compounds contain ten carbons and have three units of unsaturation, so they are isomers. All of the answer choices are isomers of some kind, so that didn't help much. They have the san.e connectivity (and therefore the same root for their IUPAC names), so they are not structural isomers. This eliminates choice D. Neither citral nor neral has a chiral center, so they are not optical isomers. Thi: eliminates choice C. They cannot readily contort or rotate about a bond to inter-convert between one another, sc choice A is eliminated. Conformational isomers must be able to rotate or flip about sigma bonds to interchange between one another, like the gauche and anti conformations of a compound lik" brtur-r". Citral and neral var' ai the positioning around the double bond between carbons 2 and3, so they are cis-trans isomers. Cis-trans isomer-.
are more correctly defined as geometrical isomers, so the best answer is choice B.

17.

Choice B is correct. Standard distillation of lemon grass oil at 229' exposes the compounds in lemon grass oil tc extreme heat, which can decompose, destroy, or denature a compound. Decomposition can certainly oc-cur at hig: temperatures, as is seen with most biological molecules, so choice A is eliminated. High heat in the pr"r"rrc" ot oxygen gas results in the oxidation of most organic compounds, so choice C is valid, ind therefore Lhminatec Citrai can react with itself at the n-bond carbons, so at high temperature, polymerization is a possibility. ThL. eliminates choice D. Without a source of hydrogen, hydrogenation is not a vlible possibility, so hydrogenatior is not a iikely result when heating a citral-neral mixture. This makes choice B the best answer.

18.

than 1.00

Choice C is correct. Ether is chosen to extract the organic components from the aqueous layer, because it i. immiscible in water and it has a high affinity for organic molecules. The immiscibility of water in ethe: eiiminates choices A and B. Citral dissolves into the ether layer, which favors choice C over choice D. The wording is not perfect, so we must also consider that ethers float on water, given that their densities are les.

g/nrL. This supports choice C as the best

answer.

'19. Choice C

is correct. The term anhydrous provides a big clue here, because it refers to something without wate: which implies that water must have been removed from it, otherwise the fact that it is anhydrous is irrelevant. The best answer is that an anhydrous salt is added to the ether layer to absorb, and thereby remove watei Choice C is the best answer. Isomerization of neral into citral is done with either light or heat, but an insoluble salt has no bearing on the isomerization. Choice A is eliminated. Sulfate, SO42-, is rich in oxygen an; magnesium dication, Mg2*, is poor in electrons, so neither could be a reducing agent. Neither i"drr." *, "u. aldehyde into an alcohol, so choice B is eliminated. Selectively binding a compound such as neral might souni iike something a chemist would want to do, but there is no reason to believe that magnesium sulfate could di that. Choice D is eliminated.

20. Choice D is correct. Neral only varies from citral in the positioning of one double bond, so in solution, it exhibisimilar intermolecular forces as citrai. \A4rile the melting point may be impacted by the cis versus trans positior of the double bond (given that packing depends on steric hindrance), the boiling point is less impact"a. Th. boiling point of citral is 229"C, so the boiling point of neral should be close to thit value. Choice b is the best answer. Given that steam distiilation is employed, the boiling point of neral is above 100"C, so at the t,er.,least, choices A and B should have been eliminated.
Copyright @ by The Berkeley Review@

29o

LAB

&

SPECTROSCOPY BXPLANATIO\S

21' Choice D is correct. Vitamin A is made from citral, so it is likely a hydrophobic aldehyde too. It is larger than citral, so it will have a higher melting point, eliminating choice A. " It d hydrophobic, so it has a low water solubility, eliminating choice B. Citral is a liquid with i high boiling point, ,o it hu, a low vapor pressure.
vitamin A is likely the same, so choice C is eliminated. Being a tl*pia, Vitamin A has a higir afiinity for lipids, which makes choice D the best answer. Vitamin A is a lipil-soluble drug.

22' Choice A is correct._

The percentage of caffeine in the tea leaf is the mass of the extracted caffeine divided by the mass of the tea leaves multiplied by 100%. The amount extracted is 0.028 grams (28 milligrams) and the original sample of tea leaves weighed 2.0 grams, so the value is 0.028/2x 100% 0.014 xI0O"/, \.4"h. Choice A = = is the best answer.

23, Choice A is correct.

Because caffeine is extracted with both hot water and methylene chloride, it must be soluble in both hot water and methylene chloride. This eliminates choices C and o. tn the second extraction, methylene chloride is used to remove the caffeine from the water, so caffeine must be more soluble in methylene chloride than water. This eliminates choice B and makes choice A the best al1swer.

24'

Choice A is correct. A stream of nitrogen, when flowing across the surface of a liquid, will force the gases present above the surface away' thus reducing the partial pt"s.rr" of the vapor above soiution. If the air above the solution is saturated with vapor before the nitrogen gas stream, lt witl not be saturated afterwards, aliowing new vapor to form' The solution continues to vaporize (evaporate) to equilibrate with the vapor present above solution, but because the vapor is continualty being ,e*ou"d, it can never reach equilibrium. Eventually, the liquid evaporates away completely. This is best explained as choice A.
and easy test for the identity of a solid is the mixed melting point test. By adding a small amount of the compound you believe the sample to be, it is possible to observe lhl melting point for the mixture of the known and unknown compounds. If they are the ru-" .o-pound, then the meltin! point will be sharp and exact. If_the two compounds are not the same, then the melting point wili be broad. TIii means that to prove that an unknown is caffeine, the unknown is best mixed with cafieine for a melting point measurement. This makes choice C the best answer.

25' Choice C is correct. A quick

26'

Choice D is correct. Methylene chloride is common as a solvent, so it must be a stable compound. Even if it were unstable, the quantity should not be the factor that triggers its instability. Choice A is eliminated. Choice B aims for the test taker who is asleep at the wheel, so io speak. The top and bottom layers have to do with relative densities, not relative masses. Less dense species float in more dense mediums, no matter how massive they are. Choice B is eliminated. The goal of extriction is to isolate the product, so hopefully any method or procedure would aim to maximize what is isolated. This makes choice D a better ir-rr*"r" thin choice C, although that logic does not explain why it is true. This is where you have to keep in mind that it's a standardized exam, and not an exam of what you know. Using test logic to choose D in this case is perfect, because you save time' A mathematical example is shown here Io r.rppo"rt the answer, but it is unnecessary to
consider during the actual exam.

Consider a solute that splits evenly between two solvents. In one case, 15 mL of Solvent A is used with 10 mL solvent B. This would result in 60% of the solute being extracted into Solvent A. In a second approach, the 15 mL of Solvent A is added in three 5-mL portions. In portion 7,33.3"/' goes into Solvent A and 66.6% remains in Solvent B. In portion 2, based on the percentage oi th" original matIrial, the partitioning ratio of Solvent A to Solvent B is 22.2"h,to 44'4"/,. In portiorr 3, based on the percentage of the original material] the partitioning ratio of Solvent A to Solvent B is 14.8% to29.6"k. Adding up the percentages extracted into Solvent A from the three separate extractions, we get: g3.g% + 22.2% + 74.g% = 70.3"/".

Three separate smaller extractions yields 70.3% while one larger extraction yields 60%. Three small extractions do in fact generate a better result than one big extraction, so choice D is the best answer.
27

'

Choice B is correct. Initial treatment of the tea leaves with hot water dissolved the water-soluble products, including caffeine' Methylene chloride is added to this solution to remove organic substances thai are only partially soluble in water at lower temperatures. Because caffeine is water solui'le at elevated temperatures, it is still partially soluble at a lower temperature, so not all of the caffeine can be extracted into the methvlene chloride layer. The best answer is choice B.

Copyright O by The Berkeley Review@

29I

LAB & SPECTROSCOPY EXPLANATIONS

28' Choice A is correct' Sublimation

a gas, so choices C and D are not valid answers. As a laboratory technique, it is used to convert one solid into a gas, while leaving behind the other solids' The gas can migrate u*uy f.o- the other solids and then be collected on a cold surface as a pure sample of the solid. Purifying a solid from a liquid does not work, because the liquid can vaporize,

is the physical process of converting a solid directty into

resulting in more than one compound ln the gas phase. ThiJ ehminates choice B. The best answer choice is A.

29'

Choice B is correct.- Exactly 0'75 grams of the original 1.0 gram sample was isolated from the organic layer. It is assumed that the other 0.25 grams of sample were dissolrrld ir-rto thl aqueous layer. The partition coefficient is the amount that dissolves into the organic layer divided by the u-or;r,t that dissolves into the aqueous layer, which in this case is O.zs/0.2s = 3.0. This makes choice B the best answer. Choice D is correct. As stated in the passage, the solubility of 4-methylaniline in water at ZS"Cis 3.g grams/100 mL and in diethyl ether at 25'C is 22.8 grams/ 100 mL. The partition coefficient is the solubility of the sample in diethyl ether divided by the solubility or tn" sampie in water. The partition coefficient is found as follows:
2? Rl

30'

Choice D is the best answer.

f;=H=#=#=#=u

31'

assuming any errors remains the same in magnitude.

Choice D is correct. As stated in the passage, the solubility of 4-methylaniline at 25'C in water is 3.g grams/100 mL and the solubility of 4-methylaniline itzs'c in diethyl ether is )z.s grams1100 mL. This means that the maximum amount of 4-methylaniline that can dissolve inio 10 mL diethyi ether is 2.2g grams. The maximum amount of 4-methylaniline that can dissolve into 10 mL water is 0.38 grams. The maximum total mass between the two solutions is 2.66 grams. Adding 3.00 grams exceeds this alount, so not all of the 3.00 grams of .1methylaniline can dissolve ilto the two solutions combined. The undissolved portion will not partition between the two solvents, but instead forms a precipitate at the bottom of the flask. This precipitate makes the lower layer appear to ha_ve_more 4-methylaniline than it actually has dissolved into solution. This makes choice D the best choice' Had the compound been fuliy soiuble, ihe partition coefficient should have been the same.

32' Choice

B is correct. Because 4-methylaniline is more soiuble in the diethyl ether than water, the pipette coulci with diethyl ether to wash off any residue of 4-methylaniline. Heating and cooling the pipette is pointless once the compound has adhered to the walls of the pipette. Water is J poor choice"to remove the adhered soiid, because 4-methylaniline is relatively insoluble irrwater. The residue on the glass was lost frorr. the diethyl ether layer, so if it is recovered, it is best recovered into the diethyl ether layerf pick choice B for
be rinsed
best resuits.

-i3'

Choice A is correct. If the ether were not anhydrous, it would contain some water impurity. This implies that irr the 10 mL of ether, there is actually a smali portion of water and therefore less than 10 mL of ether. Thi-. eliminates choices C and D, because they indiiate an excess (a value greater than 10 mL) of ether. With less than 10 mL of ether, less 4-methylaniline than expected wouid dissolve" into the ether layer. This means that a smaller amount of 4-methylaniline would be isoiated from the ether layer. This would lower the numerator and thus lower the value for the calculated partition coefficient. This makes choice A the best answer.
Choice B is correct. Diethyl ether is used because it is immiscibte in water and the organic compounds are solubl. 'n it' Any solvent that_is used in place of diethyt ether must have the same properties. tetrihydrofuran, THf :s a cyclic ether, so it has the same properties as diethyl ether. This eliminates choice A. iyclohexane an; nethylene chloride are both organic solvents that are immiscible in water, so choices C and D are eliminatei Ethanol is miscible in water, so it cannot be used in lieu of diethyl ether. This makes choice B the best answer.

Sl

Choice C is correct. For two layers to {orm, the organic solvent must be immiscibie in water, eiiminating choi;e \' The density is not important, as long as its not the same density as water. As long as it has a densi:.., lifferent than water, higher or lower, the system will have a layer on top and on bottom. rnls eiimrr1ate. :hoice B. The solute should be soluble in the organic layer, so that it can be removed from the aqueous lar-e: Choice C is the best answer. If its boiling point iJ less than room temperature, then it will exist u, u gu5, at roor:-, :emperature. To be useful as a solvent, it needs to be a liquid at room temperature. This eliminates choice D. : .:ieht @ by The Berkeley Revicw@ 292 LAB & SPECTROSCOPY EXPLANATIO\S

I
f:

36. Choice

D is correct. Ether is chosen as a solvent, because it is immiscible in water and it is a liquid at room temperature. The organic solvent used in lieu of ether must also be immiscible in water and a liquid at room temperature. Choices B and C are eliminated, because they are not liquids. Choice A (ethanol) dissolves into water, so choice A is eliminated. Only choice D remains. Chloroform is an immiscible liquid, so choose D.
Choice B is correct. To extract the contents of Tube 4 from the organic layer, a basic aqueous solution was used, so Tube 4 must be at a high pH. This means that the solute is likely in its anionic form. To remove the solute (dissolved anion) from solution, the solution must be neutralized. To neutralize a basic solution, an acid is added. This eliminates choices A and D. The acid should be strong, so it can fully react with the base in

J/.

solution. The best answer is choice B, hydrochloric acid.


38.

Choice D is correct. Tubes 2 and 5 contain the component that does not react with either the strong base or the weak base to become more water soluble. Dinitrobenzene has no acidic protons, given that all of its hydrogens are bonded to carbon, so it will remain neutral and organic soluble during the entire separation. It is not possible to convert dinitrobenzene into a charged species with just acid and base reactions. The best answer is dinitrobenzene for both Tube 2 and Tube 5, so choice D is the best answer.

39. Choice C

is correct. The second proton is by definition less acidic than the first proton, so the second pKu is greater than the first pKu. Choices A and B are eliminated. The value of pK6 can be found by subtracting the pKu for the conjugate acid from 14. Because the pKu for benzoic acid is lower than the pKu for the first proton of resorcinol, the pK6 for the conjugate base of benzoic acid (benzoate) must be greater than the pK6 for the conjugate base of the first acidic proton of resorcinot (pK62). The first proton lost by a diprotic acid is the second one gained by the fully deprotonated form of the conjugate base. This makes choice C the best answer.

40. Choice

A is correct.

Because the melting points

for the compounds isolated by Student II showed a narrower

range than the compounds isolated by Student I, the products from the scheme of Student II must have been purer than the compounds from the scheme of Student I. Thus, the scheme for Student II must have worked better than the scheme for Student I. The scheme for Student II isolated the stronger acid first (Tube 4), the weaker acid second (Tube 6), and the neutral species in the organic layer (Tube 5). It can be concluded that the scheme for Student II worked while the scheme for Student I did not. The best answer is choice A.
4't,

Choice D is correct. 3-Methylaniline is an amine, making it a basic compound. Basic compounds dissolve into water under acidic conditions. F{owever, in the reaction scheme for Student II, only base is added to the water and not acid. As such, the amine is not protonated, and remains in the organic layer for both extraction steps. The result is that it starts and finishes in the organic layer, which finishes in Tube 5. A small amount may dissolve into water, so the best answer is choice D, predominantly in Tube 5.

42. Choice D is correct. A compound can be separated using extraction techniques as long as it is not reactive in any of the mediums. 4-Ethylcyclohexanone is a large ketone, which is not reactive with the ether layer, acidic water,

or basic water. 4-Ethylcyclohexanone can be isolated using acid-base extraction techniques, so choice A is eliminated. 3-Bromophenol is a mild organic acid, which is not reactive with the ether layer or acidic watet, and undergoes reversible deprotonation in basic water. 3-Bromophenol can be isolated using acid-base extraction techniques, so choice B is elimirated. 3-Nitrobenzoic acid is a mild organic acid, which is not reactive with the ether layer or acidic water, and undergoes reversible deprotonation in basic water. 3-Nitrobenzoic acid can be isolated using acid-base extraction techniques, so choice C is eliminated. Ethyl benzoate is an ester. Esters can undergo hydrolysis under acidic aqueous conditions or saponification (base catalyzed hydrolysis) under basic aqueous conditions. Because esters react under the conditions used in acid-base extraction, ethyl benzoate cannot be isolated using acid-base extraction techniques. Choice D is the best answer.

43.

Choice C is correct. An Ri value of 0.92 implies that the compound is highly soluble in the solvent and that the compound has a very low affinity for the surface of the TLC plate. A change in solvent from nonpolar to polar would show a reduction in solubility, and thus a reduction in the Rp value. The Ri cannot be negative, so choice D is eliminated. The R6 vaiue cannot be greater than 1, so choice A is eliminated. The only value greater than zero but less than 0.92 is choice C, making it the best answer.

Copyright O by The Berkeley Review@

293

LAB & SPECTROSCOPY EXPLANATIONS

++.

choice C is correct. Because a sugar contains a large number of hydroxyl groups, it will be most soluble in a solvent with a hydroxyl group, such as an alcohol. This is baseclon the idea'that,,like dissolves like.,, An alkane, ether, and ketone do not have a hydroxyl group, so the best answer is choice C.
Choice D is correct. The basic principle behind thin layer chromatography is having the spots migrate up the riate at different rates by exploiting their differences in attraction to the solvent and affinity to the TLC plate. Spotting should be done in a wav that allows the spots to migrate. If the spots are placed Uetow the top of tn" 'oh ent (submerged into the solvent), they can dissolve into the solvent. This makes choice D the best answer. Choice A should be eliminated, because tire rate of migration does not depend on the placing of the :t'ots' The spots are not going to become_ saturated, * tn"y will not rise up the plate too rapidly. solvent a.d Choice B is t-imrnated' The spots will dissolve in al1 directions, so radial migration ii an ii-rcor.ect answer. Choice C is rliminated. Choice D is better than the other poor answers.

45.

Choice C is correct. For column chromatography, the best separation results from the solvent that shows the i:eatest difference in ft1 values in terms of ratio. In hexane, the R1 values are different by a factor of 2, while in e:hanoi, the R6 values are different by a factor of 3. Do not subtrict the R1 values to deiermine the separation, 're R1 values must be divided. A better separation is observed with ethanol than hexane, so choices A and B srould be eliminated. Because the R1 valnes are small, the column should be short, so that less solvent is :equired and the comPonents do not remain on the column for any unnecessary time after separation has been =stablished' A mixed-solvent of hexane and ethanol wouid not work well, because the solutes will travel faster ::rd the ratio of the R1 values will decrease only slightly. Choice C is the best answer. Choice D is correct. Carbon tetrachloride is a nonpolar solvent, so it shows similar solubility properties (and ::'lus similar TLC results) as other nonpolar solvents. Acetone and propanal both contain the carbonyl functional -B. :roL1p/ which makes them polar. This eliminates both choices a ana Ethanol is not only polar, tut it also is 'Because :ap.able of forming hydrogen bonds with solute particles. Choice C can thus be eliminated. pentane is a 1''-''drocarbon, it is nonpolar. Pentane will behavl most like carbon tetrachloride, so the best answer is choice D. Choice D is correct. The Ri value is defined as the distance that a spot travels up a TLC plate (droo1) divided by :he distance that the solvent travels up a TLC plate (ds61rr"rl1). Because the spoi migrates due to its affinity for lne solvent, the spot can never migrate a greater aiiiance'ttran the solveni migrates. This means that the ;5oh'sn1-wi11 always be a greater value than the droo1. This consequently means ttat the R; value can never be {reater than or equal to 1.00. Choice D, 1.05, is nof'possible. An R1'value of 0.0 simply that the spot does not dissolve into that particular solvent, and therefore it does not move. An R1 ,rat.re -"urls of 0.0 is possible.

t-

15.

Choice A is correct- Trimyristin is a fatty acid triglyceride, meaning it is a triester. Long chain esters are nvdrophobic, because they cannot form hydrogen 6onds and they aie reiatively nonpolar. This eliminates ihoices B and D. THF is an ether, making it nydropnobic. Because of the oryg"n in the ring, THF is a mildly p'o1ar solvent, although it is aprotic. The best answer is choice A.

50' Choice C is correct. Alumina is a polar adsorbent,

so it hinders polar solutes more than nonpolar solutes. \onpolar solutes come off the column first, resulting in a fast elution time. Benzoic acid is hyirophilic and

colar, eliminating choice A. Glycerol is also polar, so choice B is eliminated. Glycine is polar and hydrophilic, so despite the presence of ethanol as the eiuting solvent, it is stilt hindered by the alumina. Choice D is elimi-r-rated. A nonpolar compound like o-xylene exhibits affinity for the alumina, so it travels quickly _little i-lorvn the column. Using a nonpolar solvent liice ligroin speeds it up even"more, so choice C is the best answer.

51' Choice D is correct. The compound with

the iarger R1 value_ climbs a TLC plate faster, so it travels through a coiumn faster as well' This means that it should nu,r" i smaller elution tlme. This eliminates choices A and B. The R1 values differ by a factor of two,, so the relative migration distance in a given time differs by a factor of trr-o This means that their rates also differ by a factor oi t-o, meaning that their times for traveiing a given distance shall also differ by a factor of two. This makes choice D the besianswer.

52'

Choice C is correct. Salicylic acid is highly polar, as stated in the passage. The greatest R1 value corresponds to the greatest migration, which is observed inlhe most polar solveni. H"iun" andhgroin are nonpolar, so choices A and B are eliminated' Methanol is more polar than pentanol, so choice C is the best answer.

Copyright O by The Berkeley Review@

294

LAB & SPECTROSCOPY EXPLANATIONS

lLfiWr

5J.

column, so choice D is the best answer.


54- Choice

Choice D is correct. The BrockmarLn Activity rating for alumina has to do with the amount of water bound to the column. According to the passage, a rating of III refers to a column that is 6ok water by mass. A rating of IV has more water than a column with a rating of III, so choice A is eliminated. Because there is so much watei bound to alumina, there is less binding of solute to the column. Having more water bound with a rating of IV than a rating of III, a rating of IV represents a lower affinity than a rating of III. This eliminates choice B. Alumina is made of aluminum oxide, so there is no silicon present. This elimrnates choice C. A Brockmann Activity rating of III is caused by 6% water by mass, so a rating greater than III results from more than six percent water by *uri. Gi,r"r-, that a rating of I is perfectly anhydrous, a rating of IV requires more than 6 grams of water p"i f OO grams of

eliminates choices C and D.


55.

B is correct. Methanol has a high affinity for alumina, so the binding of methanol to the alumina is exothermic. This eliminates choice A. When methanol is added to the column, heat pockets can form, which results in the formation of vapor pockets in the column. These vapor pockets can ctaik the gel, resulting in a break in uniformity for the pathway of solute through the column. Choices B is the best answer. Methanol is miscible in hexane, but even without knowing such a fact, there is no reason to assume that rate at which methanol migrates through the column has any dependence on the miscibility of the two solvents. This

Choice A is correct. A higher Brockmann Activity rating represents a greater amount of water on the alumina, which corresponds to a lower affinity for polar molecules. The lower affinity is due to the fact that water is taking up space that the polar molecule would bind if it was open. This makes choice A the best answer. Water does not cause the contraction of alumina. If anything, it may dissolve alumina with such a high affinity for water. Choice B is eliminated. Alumina is polar, so although the addition of water may affect the polirity, choice C is incorrect, because alumina is polar to begin with, not nonpolar. Water does not oxidize ilumina, given that aluminum is fully oxidized in At2o3. This eliminates choice D.

56.

Choice B is correct. Alumina is polar, so the solute for which it has the highest affinity must also be polar. Choice A, 1,3-cyclohexadiene, is nonpolar and hydrophobic, so it is eliminated. Choice C, ethyl butanoite, is an ester, so while it has a dipole, it is classified as nonpolar species and hydrophobic. Choice C is eliminated. Choice B, cyclohexanol, is polar and able to form hydrogen bonds, so it has a high affinity for alumina. Choice D, 3-methylpentanal, is an aldehyde, so it polar and should also exhibit an affinity for alumina. A protic species has a greater affinity for alumina than an aprotic species, so choice B is a better answer than choice D.

5/

Choice A is correct. Because like dissolves like, fluorenone should be most soluble in another ketone. As solubility increases in the solvent, the affinity is said to be higher. The only ketone in the answer choices is acetone, so choice A is the best answer.
Choice C is correct. Fluorenone is more polar than fluorene, so independent of solvent, fluorene should elute from the column first. The fact that a nonpolar, hydrophobic solvent is being used makes the difference in migration rate even greater. Fluorene is a nonpolar species, so it has a low affinity for silica gel. This eliminates choice A. Fluorene is in fact lighter than fluorenone, but in a column, much like free fall, the mass of the species does not factor into its migration rate except for the steric hindrance associated with larger, more massive molecules. Choice B is eliminated. Fluorene does in fact have a higher affinity for cyclohexane then fluorenone does, so choice C is a valid statement. Fluorene is smaller than fluorenone, but there are no pores in silica gel that separate by molecular size (molecular sieves do that), so choice D is eliminated. Choice C is the best answer.

58.

59. Choice B is correct.

The best solvent is the one that allows the two species to separate by the most. This is determined by comparing the R1 values of fluorene and fluorenone in each of the iolvents. fne greatest ratio represents the best solvent. For Liquid II, the ratio is 0.33 to 0.24, which is less than 2. For Liquid III, the ratio is 0.51 to 0.19, which is slightly greater than 2.5. For Liquid IV, the ratio is 0.55 to 0.41, whiah is less than 2. For THF, the ratio is 0.42 to 0.31, which is slightly over 1. Liquid II is best, so choice B is the best answer. Choice A is correct. Aiumina is a polar adsorbent and ligroin is a nonpolar solvent, so nonpolar species will exit the column before polar species. Of the three compounds, biphenyl is nonpolar and hydrophoblc, so it will elute from the column first. This eliminates choices B and C. Toluic acid is polar and hydrophilic, so it will elute from the column last. This makes choice A the best answer.

60.

Copyright @ by The Berkeley Review@

295

LAB & SPECTROSCOPY EXPLANATIONS

61.

Choice c is correct' \4rhen the solvent is flush with the top of the silica gel, all of the solute is aiso flush with the top of the siiica gel. This means that the solute will enter the gel ui th" same time, resulting in sharper bands gorng down the column. Choice C is the best answer. The sJmple must dissolve into the solvent to be effective, so it cannot float, making choice A incorrect. Whether the solvent is flush with the top of the silica gel does not impact whether or not the solute adheres to the glass. If that is an issue, it will result in problems as.the sample migrates down the column, no matter where the solvent level starts. choice B is eliminated. The silica ge1 should not dry out, so choice D is incorrect. The best answer is in fact choice C.
Choice C is correct. Although the world miglt be a better place with words like "sandophilic,, in it, there is no such term' This unfortunately makes choice D incorrect. tfr" sample must reach the silica gel, so choice A is eliminated' The sand does not interact with the silica gel, so it has no impact on the polarity of the gel. Choice B is eliminated. The role of the sand at the top of the;ilica gel is ensure that the top of the silica gi stays flat, ^Without so all solute travels the same distance through the column no*matter where it starts. the sXnd, pouring solvent into the system could disturb the flat surface. Choice C is the best answer.

62.

63. Choice B

dorvn

is correct. Silica gel is a polar adsorbent, so its affinity for polar species is high while its affinity for nonpolar species is low. The question states that when using siiica git, ,"ronpotar species elute first, no matter what solvent is used. This simply means that the affinity of"the adsorbent *.rrt ort*"igh the affinity for the solvent' Polar compounds do follow the "like dissolves like" rule, so choice A is eliminated. Nonpolar species remain nonpolar when polar solvent is added, so choice C is eliminated. polar compounds do not form large molecular networks any than nonpolar molecules would, so choice D is eliminated. Choice B is the 1nor9 best answer/ because silica gel is highly polar and it thereby hinders polar species by binding them as the travel
a

I
72

column.

64. Choice A is correct. According to the data in Table 1, the reactant is in the lowest concentration in Trial III (76?''), so Peak 1 must be due to reactant. In GC graphs, the peak with the smallest area correlates to the lowest

concentration. Upon adding reactant, spiking the milture wiih reactant, peak 1 should be the one to is done to confirm the identity of a peak. The best answer is choice A.

grow. This

b5.

Choice B is correct. If-the column is packed with a nonpolar material, then nonpolar compounds will bind to the column more so than the polar compounds. This allows the polar compounds to travel ttuough the column with less resistance. Polar compounds will th,rs have shorter elutiln times. The best answer
is choice B.

66.

time. The best answer is choice D.


67.

Choice D is correct._ Figure 1 represents the output for Trail III at 15'C. According to Table 1, the relative concentrations at 15"C are Product A > Product B > Reactant. Peak 2 is the largest of area, so peak 2 must be associated with Product A. Peak 3 is the second largest, so Peak 3 is associat"a #itn product B. peak 1 must be due to the reactant. The first peak to appear haslhe shortest retention time, so the reactant has the lowest retention time' This eliminates choices B and C. Product B comes off last, so Product B has the longest retention

Choice A is correct. By definition, an increase in temperature favors the formation of the thermodynamic product, so choice B is eliminated. Choices C and D are the same answer, but worded differently. There cannot be two best answer choices, so choices C and D are both eliminated. The MCAT writers love to place the same answer twice with slightly different wording. From Table 1 in the passage, it can be seen that the amount of reactant decreases with increasing temperature, so the reaction ir proceJding more towards products. This implies that the reaction is more favorable with increasing temperature. The best answer is choice A. Choice D is correct. Gas chromatography separates by differences in migratory rates of vaporized organic compounds' Although size and mass are variables ihat affect the migiato ry rcte of a gas, the primary iifferences in retention times in this example can be attributed to the atlractivl forces between the packing ::rateriai in the column and the compounds. If mass and size were the primary factors, then good separation " ' ould not be possible for the two products from the reaction in the passage. the'charge of the compo'nd woutd - ave an effect if charged .o^porrt ds could easily vaporize. Because charf"d .o-poun"ds do not reaiily vaporize, :j i gas chromatography apparatus does not distinguish charged species.- atthough the passage doesn,t mention ': :ackground knowledge should tell you that ionic compoirnds (1ike NaCl) do"not vaporize easily. The best
;:-i'.\-er is choice D.

t'\

i'

-,:

^:--: 3 by The Berkeley Review@

296

"r

LAB & SPECTROSCOPY EXPLANATIONS

69. Choice C is correct. The best gas to propel through the gas chromatography apparatus is an inert gas that does

not react with any of the compounds. Nucleophiles can react with carbon dioxide, so choice A is eliminated. Hydrogen gas can react with air, so hydrogen gas should not be used. This eliminates choice B. Helium is the only inert gas of the choices, so helium should be chosen. The best answer is choice C. Water is too reactive.
70. Choice

A is correct. The major product from Trial II is the less hindered compound. It has no acid-base properties, so acid-base extraction is not viable. Choice D is eliminated. The boiling point of the major product is likely to be close to the boiling point of the minor product, so only fractional distillation would work, not simple or vacuum distillation. Choices B and C are eliminated. Choice A is the best answer by default.

7'1,. Choice D is correct. A strong base is used to deprotonate a very weak acid. This eliminates choices A and B. The pKu for a hydrogen on a carbon that is alpha to a carbonyl is around 19, so the best answer is choice D. Sometimes trivia is necessary to solve a problem.

72.

Choice C is correct. Longer crystals are indicative of higher purity. Impurities disrupt the crystal lattice, so crystals cannot grow as large. This eliminates choices A and B. Slow gradual formation of crystals yields the finest, purest crystals. This makes choice C the best answer.
Choice D is correct. The ideal solvent should not bind to the lattice structure of the crystals, otherwise it could incorporate as an impurity in the crystal lattice as it precipitates from solution. This makes Statement I valid. The ideal solvent for recrystallization should have the solid insoluble at low temperatures and highly soluble at elevated temperatures. This eliminates Statement IL The impurities should be highly soluble in the solvent so that they do not precipitate out and possibly get trapped in the crystals. This makes Statement III valid. Statements I and III are both valid, so choice D is the best answer.

73.

74. Choice C is correct. Physical properties can be used to determine the purity of a compound. If the density of the

crystal is precise, it implies that there are no impurities in the crystal. The melting point of the compound (crystalline or not) can be used to determine the purity. A broad melting point range implies that the compound is not pure. The mass of the crystal has nothing to do with its shape and purity. This makes choice C an invalid statement, and therefore the best answer. The index of refraction is another of the physical properties that depends on the lattice structure. The index of refraction varies with impurities. The best answer is choice C.
/5.

Choice B is correct. By keeping the solvent and glassware hot, the original solid remains in solution during the filtration. The purpose of filtering at this point is to remove any solid (insoluble) impurities that are present, so the target compound should remain in solution for highest yield. This makes choice B the best answer choice.

/o. Choice C is correct. The ice bath serves to cool the solution to 0'C. The solubility of a solute decreases as the temperature of the solution decreases, causing more solid to crystallize (precipitate) out of solution. Overall, the purpose of the ice bath is to cool the solvent to decrease the solubility and thus increase the amount of crystals formed. Pick C for correctness.
77.

Choice B is correct. If crystals form too rapidly, then impurities (such as solvent molecules) can get trapped in the crystal lattice. If an impurity happens to collide with the surface of the crystal, it has time to escape if the crystals are formed slowly. However, if the next layer of the crystal forms too quickly, the impurity cannot
escape. The best answer is choice B.

78. Choice C is correct. Ideally, the crystals formed from recrystallization are purer than the original solid, otherwise recrystallization would be pointless. Choice A is a valid statement. Considering that the solvent must be heated to its boiling point to maximize solubility (and thus minimize the amount of solvent required), it is important to know the boiling point of the solvent. This makes choice B valid. If the solvent is highly volatile, then it cannot be raised to a very high temperature (the more volatile the liquid, the lower its boiling point), thus more solvent is required to fully dissolve the solid. The more solvent required at maximum temperature, the more solvent present when the solution is cooled. The more solvent present when recrystallizing, the less solid that will crystallize out from solution. This reduces the percent recovery, making choice C the incorrect statement. It is never possible to recover 100% of a material after it has been dissolved
into solution. A small portion always remains soluble. The best answer is choice C.
copyright
@

by The Berkeley

Review@

297

LAB & SPECTROSCOPY EXPLANATIONS

79. Choice C is correct.

Charcoal binds colored organic molecules. If impurities are colored, they can bind the charcoal and the charcoal can be filtered awayt taking the bound impurities with it. This means that charcoal is employed when there are colored impurities. This eliminates choices A and B. If the target molecule is colored, then charcoal can bind it, and using charcoal would decrease the yield. This means that the desired compound must be colorless, making choice C the best answer. To be a chemistry warrior supreme, pick C.

80.

Choice A is correct. Because Compound A is oxidized by chromic acid, it must be either an alcohol or an aldehyde' Dinitrobenzoylchloride reacts readily with nucleophiles (such as alcohols) to form a derivative that is a solid at room temperature. The formation of a precipitate therefore confirms that Compound A is a nucleophile, so the aldehyde choice is not possible. Although you may not know what these reagents do specifically, you should be able to deduce the purpose of each test. The molecular formula for Compound A (C6H12O) indicates that the compound must be either a carbonyl, ether, or alcohol. The ether ihoice is eliminated, because it would not be oxidized by the chromic acid. The carbonyl choice is eliminated for the reasons stated above. The compound must be an alcohol. An O-H stretch is found at 3500 cm-1. The peak is broad due to hydrogen bonding. Because Compound B does not change color in CrOg/H+, it must either be an ether or ketone. Because it shows a positive iodoform test, it is a methyl ketone. If you did not know that, you can deduce that the ether possibility is out, because ethers are inert and will not test positive with 12 and KOH. The key peak in a ketone would be the C=O peak at 17L0 cm-1. This makes choice A the best answer.
Choice A is correct. This question has more depth than meets the eye. According to the formula, the compound has one degree of unsaturation, which can be either a ketone n-bond, an alkene n-bond, or a ring. The reagent added (Br2 in CCl4) reacts with an alkene n-bond, therefore an alkene functionality must be present in the molecule. The ketone is not possible for the structure, because the C=O of a ketone would require aiecond degree of unsaturation and given that the one unit of unsaturation present in the compound has already been used (by

81.

the alkene). There can be no carbonyl present in the compound. The positive test confirms that there is an alkene present, so it does not show that it cannot be an alkene. Choice B is thus eliminated. The ether and alcohol functionalities are possible, because the molecule could be an alkene (for which the bromine in CC14 test is positive) with an ether or alcohol functionality. The positive test only shows that the compound cannot be a ketone, thus you must pick A you correctness trooper youl
82. Choice A is correct. An acidic hydrogen (protic hydrogen) will exchange a proton for deuterium in the presence of D2O. This results in the loss of one signal (broad peak) in the proton NMR spectrum. The only compound with a protic hydrogen is the alcohol, choice A. An ether should have been eliminated immediately, because ethers have no acidic protons. A ketone and aldehyde have alpha hydrogens that can be removed by strong base, but they do not readily exchange in water without the presence of a strong base or strong acid. This question is best solved by selecting the most acidic compound of the choices. A proton on the alcohol oxygen is more acidic than the hydrogen on an alpha carbon. No matter what your path to wisdom, finish off beautifully by choosing answer choice A.
83.

well. Both chromic acid and potassium

Choice C is correct. Because Compound A was oxidized by the chromic acid, it can be oxidized by the KMnO4 as permanganate are strong oxidizing agents. The other compound of interest is Compound C, which turned the bromine solution clear (confirming the presence an alkene double bond). Alkenes also react with permanganate solution, forming a vicinal diol with syn orientation. This is stated tn the passage. Both Compound A and Compound C can react with aqueous potassium permanganate (KMnO4(aq)), resulting in the purple permanganate solution changing color upon reaction. This makes choice C the best
answer.

84.

Choice C is correct" Acid chlorides are highly reactive electrophiles (because the chloride is a good leaving group and the carbonyl carbon is electron poor). Acid chlorides can react with even the weakest of nucleophiles. Alcohols, amines, and thiols are all nucleophiles, so they all wiII react with the dinitrobenzoyl chioride ((NO2)2C6H3COC1) kicking out the chloride leaving group. Thus the best choice for a compound that will rzr: react with (NO2)2C6H3COCI is the ether. The best answer is choice C. Based on the inert nature of ethers rr, general, it is safe to pick the ether, even if you know nothing about acid halide reactivity. In other words, fo: questions that ask "Which will not react?", they are really asking 'Which is the least reactive species?"

Copyright @ by The Berkeley Review@

294

LAB & SPECTROSCOPY EXPLANATIONS

85.

Choice B is correct. Choices C and D can be eliminated, because tertiary alcohols and ethers cannot be oxidized by chromic acid (CrO3 in H+). The color change observed from the treatment of Compound A with chromic acid shows that the compound can be oxidized. A primary alcohol (choice A) can be eliminated, because when primary alcohols are oxidized, they form carboxylic acids which would turn litmus paper red when added to the paper. For this reason, the secondary alcohol is the best choice. FeeI correctness euphoria with choice B. Be euphoric and pick it. Choice A is correct. This is perhaps a question of trivial knowledge to a point. If you have your chemical tests memorized, then this question is a no brainer. The MCAT provides a great deal of information, therefore we recommend that you focus on extracting information via critical analysis of the passages and tables, rather than memorization. The color of the precipitate is given in the passage as yellow. This eliminates the choices C and D which list the precipitate as being black. The passage also states that the test confirms the presence of a methyl ketone, which outright gives choice A as the correct answer. This is a free question for just reading the passage. If you did not read the passage, the aldehyde should be eliminated, because aldehydes can be oxidized, and Compound B did not react with chromic acid. If Compound B were an aldehyde, it would have showed a positive test with the chromic acid (a color change from orange to green). From the information listed in Table 1 and knowledge that the precipitate is yellow, the best choice is answer A.

86.

87.

Choice B is correct. The limiting reagent is 1,4-dimethoxybenzene, not nitric acid, so the percent yield should be calculated based on 1,4-dimethoxybenzene. The percent yield for the reaction is the actual yield (97.5 mg 2,5dimethoxynitrobenzene) divided by the theoretical yield (183 mg 2,5-dimethoxynitrobenzene) based on the 1.0 mmole of the limiting reagent (1,4-dimethoxybenzene). Because the actual yield is only half of the theoretical yield, the percent yield is fifty percent. The best answer is thus choice B. The math is shown below.

%yield

moles limiting reagent

molesproduct -91"5*glfigglmole 1s-g/rss


g/mole

0.5 mmoles

product

1.0 mmoles

limiting reagent

= 50o/"

88.

Choice B is correct. The melting point of the product has a range of 1.5 "C, which is a small enough range to deduce that the product is relatively pure. Because the melting point is sharp (a sharp melting point range is generally regarded as two degrees or less) and close to that of the literature value for the product, the best answer is choice B. Be a B supporter! There is no mention of a broad OH peak in the passage, so it is unlikely that it is wet. Choice A is eliminated.
Choice B is correct. Filtering collects (and thus removes) the solid particles from a solution or mixture. Filtering is most commonly used in organic lab for the separation of solids from Liquids. The best answer is choice B. Choice C is correct. Both nitrogen gas and carbon monoxide gas are colorless, so choices A and B are eliminated. You should know the color of these two gases from your everyday experiences in life. Nitrogen gas makes up the majority of our air, and air is transparent. Carbon monoxide is given off with the exhaust from combustion engines. If you are aware of your trivia, sulfur dioxide is colorless to faint yellow in color, which eliminates choice D. If you didn't know this, you hopefully thought of smog, a brown gas containing a nitrogen/oxygen molecule of some kind (nitrogen dioxide). Because smog is comprised of a nitrogen / oxygen compound, and smog is brown, it is a logical conclusion to select NO2, choice C. Choice C is the best answer.

89.

90.

9't. Choice B is correct. The error in the thermometer resulted in a reading that was too low (the experimental value of 72.5 to 74 is less than the literature value of 74 to 75). This means that the mercury did not climb as high as it should have in the inner column. The presence of some air above the mercury column (resulting from a reduced vacuum) hinders the ability of the mercury column to rise, thus Statement I results in a reading lower than expected. A divot present between 45'C and 55'C results in the mercury column filling the divot at temperatures where the top of the mercury column is above the divot (temperatures greater than 55'C in Statement II). This causes the mercury column to not rise as high. This results in a reading that is too low. If the divot is above the mercury level, then it will have no effect, because the mercury has not risen high enough to fill the divot. This means that Statement II is valid but Statement III is invalid. Choice B is the best answer.

Copyright

by The Berkeley Review@

299

LAB & SPECTROSCOPY EXPLANATIONS

92. Choice

A is correct. Filtering the compound again would not purify the crystals, it would simply reduce the yield, so choice B is eliminated. Distilling the liquid (formed frorn melting) would be impraciical given the high temperature and.probability of deposition of ihe gas into solid throughJut the distilling.oi.r-r,,"ro choice C is eliminated' Addition of more sulfuric acid and niiric acid could furtlier react with the iroduct rather than the starting material, so choice D is eliminated. Recrystallization is the ideal laboratory technique to employ in the purification of a solid. The best answer is choice A, so you better pick that A.

93.

Choice B is correct. The addition of cold water causes the crashing out (rapid crystailization) of product from solution. This means that solubility decreased, not increased, eliminiting cnoic" A and making choice B the best answer. The solution already has two strong acids, a mixture of nitric and sulfuric, so the additlon of water will not increase its acidity of the solution. This eliminates choice C. Water will not protonate a species that is deprotonated in the presence of strong acid, so choice D is eliminated. you should ,""uil thut lower temperatures result in reduced solubility, so choice B is the best answer choice.

94. Choice C is correct. If the S-enantiomer

has a specific rotation of -55', then the R-enantiomer has a specific rotation of +55'. The net rotation is positive, so there must be more of the R-enantiomer than the S-enantiomer. This eliminates choice A. Soiving this is going to require math, so it is easier to start with an answer choice and work back to the specific rotation. For plugging in the numbers, choice C is best, because only one calculation needs to be done. The value is either going to be too high (implying that choice B is correct), iigt-,t on the mark (making choice C correct), or too low (implying that choice D is coriect). A mixture of 30% of the S-enantiomer and 70"/, of the R-enantiomer has a 40"h excess of the R-enantiomer. A 100% R-enantiomer solution has a specific rotation of +55', so 40"k excess of R yields a rotation of 40"/o of +55", which is 22". Choice C is the best answer.

95'

Choice A is correct. The solute that elutes first from the column is the fastest. In a TLC experiment, the fastest sol-rte climbs the plate the most, resulting in the highest spot. This means that the dark clrcle corresponds to Band A. This eliminates choices B and C. By the same reasoning used to ascertain the dark spot, thl hollon, spot corresponds to Band C, making choice A the best answer and thus, the most pickable answer. Choice D is correct. Fractional distillation is used to separate liquids with boiling points that are close to one another. This is achieved by increasing the surface area in the distllling column. This makes choice A a valid statement, thereby eliminating it. The boiling points are close to one inother in fractional distillation, which makes choice B a valid statement, thereby eliminating it. Fractional distiliation is more selective, so the purity is high, but a great deal of residue is left behind in the distilling column, resulting in a lower yield with fractional distillation. This makes choice D the best answer. Choice A is correct. The compound that gets extracted into sodium hydroxide solution is the organic compound that becomes an ion when treated with hydroxide. Neutral acids become anions when they are ieprotonated, so the answer is an organic acid. The only organic acid is choice A, a carboxylic acid. Choice A is the^best answer.

96,

97,

98. Choice B is correct. Carboxylic acids are hydrophilic, although the aromatic ring helps with its lipid solubility. It should be somewhat solubie in diethyi ether. Despite the two nJ,atopniUc groups on hydroxybenzoic acid, it is only slightly soluble in water, because of the benzene rlng. CaiUoxylic acids are
rather easily deprotonated, so they can be converted into an anion and made to be m-ore water soluble. This makes a compound like benzoic acid highly soluble in a basic environment. The best answer is choice B.

99' Choice C is correct. Distillation

D. Extraction

involves boiling a liquid away from a mixture, so it works with two liquids that have different boiling points and don't form an-azeotrope. Choice A is eliminated. Chromatographv involves soiutes rnigrating down a column at different rates, so it works for solutes. Choice B is eliminated. Crl'siallization dissolves and selectively precipitates a solid from a mixture of solids. This eliminates choice
separates solids and liquids, but not gases. Choice C is the best answer.

100' Choice D is correct. Butanone should be highiy soluble in other ketones such as acetone, so choice A is eliminated. Ketones don't oxidize with chromium (VI), so they do show a negative Jone's test. Choice B is eliminated. Butanone has a molecular mass of T2 grams per mole, so if it losesl methyl group during, it can form a fragment with a molecular mass of 57 amu. Choice C is eliminated. There rr" ,ro chirat centers in butanone, so it cannot have optical activity. This makes choice D the best answer.
Copvright @ by The Berkeley Review@

500

LAB & SPECTROSCOPY EXPLANATIONS

a book of this nature ended up being a more difficult project than I first imagined back in 1990. For every one hundred questions you see in this book, about three hundred had to be lr-ritten. A g:reat deal of time went into selecting which questions served the purpose of conceptual analysis, developrn*

Writing

test-taking skills, honing table-reading skills, testing memory, and building confidence. But as challenging as it was at times, it has been a wonderful experience. This particular edition is the culmination of fourteen years of writing and rewriting. Along the path I have incorporated the feedback and input of roughly 5000 students. I have tried my best to rnclude many of the examples that have proven useful in lecture. I have tried to keep in mind the sole purpose of this book in your life: to help you achieve a higher MCAT score in the biological sciences section and offer test-taking tips that are universally applicable. It is imperative that you keep that in mind while working through our materials. Your goal is to learn information and strategies that allow you to quickly pick the best of four choices when asked a multiple-choice question. Completing this book would have never been possible without the support of many special people io -y life. I would like to thank:
Cecile Santos: You have been an inspiration to my teaching. I see the love you have in your eyes for the students you teach and it is awe-inspiring. You have been my sounding board for teaching ideas and much of the text in this book. Without you, I could never have done this.

Cambria and Madison: You fill my heart with so much love. You keep me balanced at times I need it most. Your laughter and smiles keep my spirits high and I am so lucky you came into my life. You are the greatest children a father could ever want.

Our Wonderful Teachers: Over the years I have gleaned so many great ideas from you. It has been an honor working alongside so many of you. \Atrhile so many of you have been helpful in my path, I would be remiss if I didn't specifically mention six very special teachers to me (although we have had many more.) Anthony Gopal showed me about the power of enthusiasm and how demonstrations and real life examples can drive classroom learning. Moin Vera showed me how simple genius and fundamental concepts can make even the hardest topic seem easy. Derek Welsbie showed me how simple logic, when applied with a little bit of information, can make every question easier to understand. Sam Biggers showed me how bonding with the class and empathizing with their anxieties can build a bond that helps the entire team do better. Kim Gordy showed me the importance of silliness in gaining the students' interest and trust and how analogies can power learning. Cecile Santos showed me the importance of engagement and open dialogue with her open forum discussion in lecture where every student is involved. You are all incredible teachers and I learn something from you every time I watch you in action. Thank you!
Dale Schmidt: You have been incredibly supportive over the years, picking up the slack when it came. Your work ethic and attention to detail is second to none. I am always sure that if you did it, it was definitely done correctly.

Kimberly Gordy and Doug Coe: You spent so much time combing through my books to help them flow better" I can't thank you enough, because I am not an easy person to work with when it comes to editing. You managed to not only make this book better, but you made me realize the need for a different style
and voice at many junctures.

Our Incredible Students: For the books I have written, I owe my biggest thank you to the thousands of students who have used them to help prepare for their MCAT. Some of you are well into your practice and this is a distant memory. Others of you have just recently finished your exam and are working your butt off to make it into medical school. To each of you who offered some feedback, told me about a typo, asked me why a question was worded a certain way, or just simply said what you thought, I am deeply grateful. This book, and the others I have written, is as much a part of you as it is a part of me. This is our project and I am so happy you were a part of its development.

Copyright @ by The Berkeley Review@

301

LAB

&

SPECTROSCOPY EXPLANATIONS

PERIODIC TABLE OF THE ELEIqENTS

H
1.0
3

I
Ile
9.0
t

I,i
6.9

ll
Na

t2
N,Ig

\
2l
Sc

l.\

.t )

24.3 20

t9

K
39. 31
r

Ca
,10. I

45.0
39

41

38

Rb
85.5
-5-5

Sr
87.6
.16

Y
8rJ.9

9t.2.

.9 40 7,r

Ti

))

?l

2-5

v
50.9

Cr
52.0 42

NIn
54.9

4l
Nb
92.9
73

+)
Tc
(e8 )

NIo
95.9
1,1

l/.
LoT
1-18.9
I

Cs
132.9 87

Ba
137.3

'72 IIf

15

78.-5

Ta I 80 9
105

w
183.9 106

Re il16.2
107

8i
Ra

8eQ

Fr

Ac"

(223) 226.0 221.0

(26t\

104 Rf

Db
(262)

Sg
(263
)

Bh
(262)

: 2lJ lNi Cu Zr. -:-58.1 tir -55.8 -16 :44, -' Pd \s Ll Ru l0l.l l{)2.q lttr' -r -it 7h 16 Ir Pt {u llOs - 190.2 lql.l l'-'.. "iog 109 I llr trlt Iun {r: I-' Hs (265) (166) {16e, -----

2(t

Fe

27 Co 58.9 4-5 Rh

li.r

58 Ce 1'10. I 90 Th
232.0

59

60

6l
Pm

Pr
I

Nd
111.2

62 Sm
150.,1

6i 6l Eu Gd
l-<l.l) li-.r 95 .lr. Anr Cnt

': T:
j' Bi.

-r

40.9

(r45)
93

-'
r-:

9l 92 Pa U (23 r) 238.0

91

Pu (237) (241)

Np

Specializing in MCAT Preparation

Das könnte Ihnen auch gefallen